diff --git "a/data/fr/fr.train_casimedicos.jsonl" "b/data/fr/fr.train_casimedicos.jsonl" new file mode 100644--- /dev/null +++ "b/data/fr/fr.train_casimedicos.jsonl" @@ -0,0 +1,434 @@ +{"id": 402, "year": 2016, "question_id_specific": 183, "full_question": "Juan, interne de deuxième année, s'occupe de Sofia, une jeune fille de 15 ans qui s'est apparemment évanouie à l'école sans perdre connaissance. La patiente dit qu'elle devait passer un examen, ce qui l'angoissait beaucoup. D'après l'interrogatoire, il semble qu'elle soit victime de brimades de la part de ses camarades de classe et qu'elle souffre peut-être d'un trouble de l'alimentation. Les signes vitaux et l'examen neurologique sont normaux. Juan garde Sofia en observation en attendant que ses parents viennent au service, en vérifiant périodiquement l'état de la patiente. Après le choc initial, la patiente semble s'animer de plus en plus et se montre très amicale. À une occasion, Juan la trouve en train de discuter sur son téléphone portable. Juan lui dit qu'il vaudrait mieux qu'elle pose son téléphone portable et qu'elle se repose. Pour la rassurer, il lui dit qu'il utilise aussi beaucoup les réseaux sociaux depuis qu'il est à l'université. Sofia s'excuse de ne pas avoir su qu'elle devait garder son téléphone portable éteint et, après l'avoir éteint, elle lui demande si elle peut lui faire une demande d'ami sur Facebook. Quelle est, selon vous, la meilleure réponse de Juan ?", "full_answer": "Il ne fait aucun doute que la relation médecin-patient transcende aujourd'hui l'environnement physique de la salle de consultation. Mais il ne faut pas oublier que nous devons y maintenir les mêmes codes éthiques et professionnels que dans l'environnement réel. Le code de déontologie Article 26-3. L'exercice clinique de la médecine au moyen de consultations exclusivement épistolaires, téléphoniques, radiophoniques, par voie de presse ou par internet est contraire aux règles de déontologie. Une pratique correcte implique inévitablement un contact personnel direct entre le médecin et le patient.", "type": "SOINS PRIMAIRES ET RÉSEAUX SOCIAUX", "options": {"1": "Dites-lui de faire la demande d'amitié et qu'il l'acceptera, car il est sûr qu'il n'y a pas d'éléments inappropriés sur sa page pour une fille de l'âge de Sofia.", "2": "Parce que vous considérez Sofia comme une patiente vulnérable et que vous craignez qu'elle n'interprète mal un refus, acceptez qu'elle fasse la demande, mais ne l'autorisez à accéder qu'à certains contenus de votre page.", "3": "Il a répondu qu'il est important de maintenir certaines limites professionnelles entre les patients et les praticiens et que, malheureusement, s'il fait la demande, il ne pourra pas l'accepter, et qu'il est donc préférable de ne pas le faire.", "4": "Dites-lui de faire la demande mais de ne pas l'accepter.", "5": null}, "correct_option": 3, "explanations": {"1": {"exist": false, "char_ranges": [], "word_ranges": [], "text": ""}, "2": {"exist": false, "char_ranges": [], "word_ranges": [], "text": ""}, "3": {"exist": true, "char_ranges": [[495, 600]], "word_ranges": [[71, 86]], "text": "Une pratique correcte implique inévitablement un contact personnel direct entre le médecin et le patient."}, "4": {"exist": false, "char_ranges": [], "word_ranges": [], "text": ""}, "5": {"exist": false, "char_ranges": [], "word_ranges": [], "text": ""}}} +{"id": 237, "year": 2014, "question_id_specific": 144, "full_question": "À la suite d'un accident de la route, un patient de 38 ans est admis en réanimation dans le coma. Après plusieurs jours, son état neurologique ne s'améliore pas et un scanner révèle des lésions hémorragiques ponctuelles dans le corps calleux et la jonction cortico-sous-corticale. Quel est le diagnostic ?", "full_answer": "Les lésions axonales diffuses entraînent une détérioration précoce et durable du niveau de conscience (comme mentionné dans l'exposé du cas) sans qu'il y ait de lésion à la tomodensitométrie pour justifier l'image. Parfois, des hémorragies ponctuelles au niveau du corps calleux, de la jonction cortico-sous-corticale et de la partie dorsolatérale du tronc cérébral sont mises en évidence par cet examen d'imagerie.", "type": "NEUROLOGIE", "options": {"1": "Hématome sous-dural aigu.", "2": "Purpura trobocytopénique.", "3": "Contusion hémorragique cérébrale.", "4": "Lésions axonales diffuses graves.", "5": "Encéphalopathie hypoxique ischémique."}, "correct_option": 4, "explanations": {"1": {"exist": false, "char_ranges": [], "word_ranges": [], "text": ""}, "2": {"exist": false, "char_ranges": [], "word_ranges": [], "text": ""}, "3": {"exist": false, "char_ranges": [], "word_ranges": [], "text": ""}, "4": {"exist": true, "char_ranges": [[0, 214]], "word_ranges": [[0, 32]], "text": "Les lésions axonales diffuses entraînent une détérioration précoce et durable du niveau de conscience (comme mentionné dans l'exposé du cas) sans qu'il y ait de lésion à la tomodensitométrie pour justifier l'image."}, "5": {"exist": false, "char_ranges": [], "word_ranges": [], "text": ""}}} +{"id": 620, "year": 2022, "question_id_specific": 121, "full_question": "Une femme de 79 ans est admise pour une fracture ostéoporotique de la hanche. En ce qui concerne la prévention secondaire des fractures de fragilité, cochez la MAUVAISE réponse :", "full_answer": "Dans l'ostéoporose, l'un des principaux risques associés à l'augmentation du risque de fracture est la mauvaise observance du traitement, la réponse 1 est donc correcte. La réponse 2 se trouve dans les lignes directrices du SER, qui confirment que certaines études concluent que les marqueurs du remodelage osseux peuvent être utiles pour le suivi précoce de l'observance et de la réponse au traitement. La réponse 4 est correcte car, toujours dans les lignes directrices du SER 2019, on cite : \"Les preuves scientifiques actuelles nous permettent d'affirmer que ni l'augmentation du calcium alimentaire ni la prise de suppléments de calcium seuls ne protègent contre l'apparition de fractures\". Par conséquent, la réponse correcte à cette question est l'option 3. Les patients sous traitement pharmacologique de la PO devraient utiliser des suppléments de calcium et de vitamine D car pratiquement tous les essais cliniques qui ont démontré l'efficacité des médicaments anti-ostéoporotiques incluent systématiquement des suppléments de calcium et du cholécalciférol (vitamine D3), mais pas en monothérapie.", "type": "TRAUMATOLOGIE", "options": {"1": "Une mauvaise observance du traitement est associée à un risque accru de fracture.", "2": "Les marqueurs du remodelage osseux peuvent être utiles pour le suivi précoce de la réponse au traitement.", "3": "La monothérapie à la vitamine D est efficace pour réduire ces fractures chez les personnes âgées non institutionnalisées.", "4": "L'augmentation du calcium alimentaire ou la prise de suppléments de calcium ne protègent pas à elles seules contre les fractures.", "5": null}, "correct_option": 3, "explanations": {"1": {"exist": true, "char_ranges": [[0, 169]], "word_ranges": [[0, 25]], "text": "Dans l'ostéoporose, l'un des principaux risques associés à l'augmentation du risque de fracture est la mauvaise observance du traitement, la réponse 1 est donc correcte."}, "2": {"exist": true, "char_ranges": [[170, 403]], "word_ranges": [[25, 63]], "text": "La réponse 2 se trouve dans les lignes directrices du SER, qui confirment que certaines études concluent que les marqueurs du remodelage osseux peuvent être utiles pour le suivi précoce de l'observance et de la réponse au traitement."}, "3": {"exist": true, "char_ranges": [[696, 1107]], "word_ranges": [[107, 164]], "text": "Par conséquent, la réponse correcte à cette question est l'option 3. Les patients sous traitement pharmacologique de la PO devraient utiliser des suppléments de calcium et de vitamine D car pratiquement tous les essais cliniques qui ont démontré l'efficacité des médicaments anti-ostéoporotiques incluent systématiquement des suppléments de calcium et du cholécalciférol (vitamine D3), mais pas en monothérapie."}, "4": {"exist": true, "char_ranges": [[404, 695]], "word_ranges": [[63, 107]], "text": "La réponse 4 est correcte car, toujours dans les lignes directrices du SER 2019, on cite : \"Les preuves scientifiques actuelles nous permettent d'affirmer que ni l'augmentation du calcium alimentaire ni la prise de suppléments de calcium seuls ne protègent contre l'apparition de fractures\"."}, "5": {"exist": false, "char_ranges": [], "word_ranges": [], "text": ""}}} +{"id": 396, "year": 2016, "question_id_specific": 136, "full_question": "Un homme de 41 ans se présente aux urgences après trois jours de gonflement et de douleur au genou droit, d'impuissance fonctionnelle et de fièvre. Deux semaines auparavant, il avait eu une diarrhée spontanée. L'examen révèle un épanchement articulaire ; une arthrocentèse est donc pratiquée et 50 cc de liquide trouble sont obtenus, avec une viscosité réduite et les paramètres analytiques suivants : leucocytes 40.000/microL (85% de neutrophiles), glucose 40 mg/dL, absence de cristaux, coloration de Gram : aucun micro-organisme n'a été observé.000/microL (85 % de neutrophiles), glucose 40 mg/dL, absence de cristaux, coloration de Gram : aucun micro-organisme n'est observé. Laquelle des affirmations suivantes concernant ce patient est \"CORRECTE\" ?", "full_answer": "Un Gram négatif n'exclut PAS une infection. Nous marquons quatre points.", "type": "TRAUMATOLOGIE ET ORTHOPÉDIE", "options": {"1": "Un traitement à base de cloxacilline et de ceftriaxone doit être mis en place en attendant le résultat de la culture du liquide.", "2": "Une arthrocentèse quotidienne est conseillée pour soulager les symptômes et prévenir la destruction des articulations.", "3": "Si la culture est négative, il s'agit probablement d'une arthrite réactive.", "4": "Une coloration de Gram négative exclut la possibilité d'une arthrite septique.", "5": null}, "correct_option": 4, "explanations": {"1": {"exist": false, "char_ranges": [], "word_ranges": [], "text": ""}, "2": {"exist": false, "char_ranges": [], "word_ranges": [], "text": ""}, "3": {"exist": false, "char_ranges": [], "word_ranges": [], "text": ""}, "4": {"exist": true, "char_ranges": [[0, 43]], "word_ranges": [[0, 7]], "text": "Un Gram négatif n'exclut PAS une infection."}, "5": {"exist": false, "char_ranges": [], "word_ranges": [], "text": ""}}} +{"id": 253, "year": 2014, "question_id_specific": 86, "full_question": "Une femme de 37 ans atteinte d'une colite ulcéreuse étendue présente une poussée sévère pour laquelle un traitement par prednisone à la dose de 1 mg/kg est instauré. Après une semaine de traitement, la patiente ne présente aucune amélioration. Quelle est la prochaine mesure thérapeutique à prendre ?", "full_answer": "Lorsqu'une poussée sévère ne répond pas aux corticostéroïdes iv à pleine dose (1mg/kg de poids corporel), il faut passer à la ciclosporine ou à l'infliximab. L'étanercept n'est pas approuvé en Europe pour les maladies inflammatoires de l'intestin (pour le moment).", "type": "SYSTÈME DIGESTIF", "options": {"1": "Colectomie subtotale en urgence, suivie d'une prostectomie et d'un réservoir iléoanal.", "2": "Associer un immunosuppresseur tel que l'azathioprine.", "3": "Associer la mésalazine à une dose de 4 g par jour par voie orale et la triamcinolone par voie rectale à raison d'une application toutes les 12 heures.", "4": "Ciclosporine intraveineuse 2mg/kg.", "5": "Un traitement à l'étanercept (un anticorps anti-TNFa) doit être envisagé."}, "correct_option": 4, "explanations": {"1": {"exist": false, "char_ranges": [], "word_ranges": [], "text": ""}, "2": {"exist": false, "char_ranges": [], "word_ranges": [], "text": ""}, "3": {"exist": false, "char_ranges": [], "word_ranges": [], "text": ""}, "4": {"exist": true, "char_ranges": [[0, 157]], "word_ranges": [[0, 25]], "text": "Lorsqu'une poussée sévère ne répond pas aux corticostéroïdes iv à pleine dose (1mg/kg de poids corporel), il faut passer à la ciclosporine ou à l'infliximab."}, "5": {"exist": true, "char_ranges": [[158, 264]], "word_ranges": [[25, 40]], "text": "L'étanercept n'est pas approuvé en Europe pour les maladies inflammatoires de l'intestin (pour le moment)."}}} +{"id": 224, "year": 2014, "question_id_specific": 63, "full_question": "Patient âgé de 52 ans qui se plaint de dyspnée à l'effort depuis 6 mois. Il n'a pas présenté d'angine de poitrine ni de syncope. L'examen physique et les tests diagnostiques ont montré un gradient aortique transvalvulaire moyen de 55 mmHg et une surface calculée de 0,7 cm². La fraction d'éjection est de 65 %. Le traitement indiqué pour ce patient est le suivant :", "full_answer": "Quelque chose d'aortique. On me donne un gradient qui, sans connaître les valeurs, semble un peu élevé (55 mmHg !) et la surface, qui semble minuscule à l'œil nu : ce sera une sténose aortique... Et il a une dyspnée. Sténose aortique symptomatique, l'examinateur veut-il que je vous fasse opérer ? Avec les lignes directrices en main, oui : il s'agit d'une sténose sévère à la fois en termes de surface (<1 cm²) et de gradient moyen (>40 mmHg), avec une fraction d'éjection préservée, et symptomatique. Indication chirurgicale. Si l'on fait 1, on réduit l'espérance de vie, car on permet au ventricule de claudiquer. L'indication 2 est dépassée : valvuloplastie par ballonnet ? Seulement si le risque chirurgical est inabordable, auquel cas nous ferions le 5, l'implantation d'un TAVI (voir [1]). Mais il s'agit d'un homme de 52 ans, dont nous supposons qu'il a une bonne musculature parce qu'il marche pour souffrir de dyspnée, et qui se rend donc au bloc opératoire. Une homogreffe ? C'est la technique de Ross, je prends le pulmonaire et je le mets à la place de l'aortique. Mais c'est une technique rare, complexe et réservée aux patients pédiatriques. La bonne chose à faire est 3, sans aucun doute : remplacer par une prothèse, de préférence mécanique, sauf si le patient a une contre-indication à l'anticoagulation.", "type": "CARDIOLOGIE", "options": {"1": "Diurétiques et contrôles plus fréquents par un spécialiste.", "2": "En cas de dyspnée croissante à l'effort, une dilatation percutanée de la valve aortique à l'aide d'un cathéter à ballonnet doit être effectuée.", "3": "Remplacement de la valve aortique par une prothèse/bioprothèse.", "4": "Remplacement de la valve aortique par une homogreffe.", "5": "Implantation de valves percutanées."}, "correct_option": 3, "explanations": {"1": {"exist": true, "char_ranges": [[528, 616]], "word_ranges": [[88, 104]], "text": "Si l'on fait 1, on réduit l'espérance de vie, car on permet au ventricule de claudiquer."}, "2": {"exist": true, "char_ranges": [[616, 728]], "word_ranges": [[103, 120]], "text": "claudiquer. L'indication 2 est dépassée : valvuloplastie par ballonnet ? Seulement si le risque chirurgical est inabordable,"}, "3": {"exist": true, "char_ranges": [[341, 527]], "word_ranges": [[59, 88]], "text": "il s'agit d'une sténose sévère à la fois en termes de surface (<1 cm²) et de gradient moyen (>40 mmHg), avec une fraction d'éjection préservée, et symptomatique. Indication chirurgicale."}, "4": {"exist": true, "char_ranges": [[1083, 1156]], "word_ranges": [[184, 194]], "text": "c'est une technique rare, complexe et réservée aux patients pédiatriques."}, "5": {"exist": true, "char_ranges": [[802, 927]], "word_ranges": [[132, 154]], "text": "il s'agit d'un homme de 52 ans, dont nous supposons qu'il a une bonne musculature parce qu'il marche pour souffrir de dyspnée,"}}} +{"id": 430, "year": 2018, "question_id_specific": 115, "full_question": "Femme de 65 ans, ayant subi une greffe de précurseurs hématopoïétiques il y a 30 jours et présentant une neutropénie sévère. Elle a commencé par une toux et une expectoration hémoptotique ainsi que de la fièvre et une dyspnée à l'effort modérée qui ne se sont pas améliorées malgré cinq jours de traitement par amoxicilline-acide clavulanique (875/125 mg/8 h) et lévofloxacine (500 mg/12 h). La radiographie du thorax montre de multiples nodules pulmonaires mal définis, dont certains sont cavités. Lequel des diagnostics suivants est le plus probable ?", "full_answer": "Patient au 30ème jour post-transplantation de précurseurs hématopoïétiques, avec une neutropénie sévère, présentant une toux avec expectoration hémoptotique. Ce tableau est généralement observé en cas d'infection par Aspergillus spp. Nous pouvons exclure une infection par Staphylococcus aureus en raison d'un traitement antérieur, bien que nous ne sachions pas si la réponse soulève également la possibilité d'un SARM.", "type": "MALADIES INFECTIEUSES ET MICROBIOLOGIE", "options": {"1": "Embolies septiques.", "2": "Aspergillose pulmonaire invasive.", "3": "Pneumonie à staphylocoque doré.", "4": "Candidose systémique.", "5": null}, "correct_option": 2, "explanations": {"1": {"exist": false, "char_ranges": [], "word_ranges": [], "text": ""}, "2": {"exist": true, "char_ranges": [[0, 233]], "word_ranges": [[0, 29]], "text": "Patient au 30ème jour post-transplantation de précurseurs hématopoïétiques, avec une neutropénie sévère, présentant une toux avec expectoration hémoptotique. Ce tableau est généralement observé en cas d'infection par Aspergillus spp."}, "3": {"exist": true, "char_ranges": [[234, 331]], "word_ranges": [[29, 42]], "text": "Nous pouvons exclure une infection par Staphylococcus aureus en raison d'un traitement antérieur,"}, "4": {"exist": false, "char_ranges": [], "word_ranges": [], "text": ""}, "5": {"exist": false, "char_ranges": [], "word_ranges": [], "text": ""}}} +{"id": 588, "year": 2022, "question_id_specific": 63, "full_question": "Un homme de 36 ans consulte pour une hyperémie conjonctivale et une sensation de corps étranger. A l'examen, il présente une atteinte du ganglion préauriculaire. Parmi les pathologies suivantes, laquelle ne correspond PAS à cet examen ?", "full_answer": "L'atteinte du ganglion lymphatique préauriculaire est la lymphadénopathie typique des affections infectieuses conjonctivales. Il est vrai que la conjonctivite bactérienne typique ne présente généralement pas d'adénopathie. Ces bactéries ont une évolution plus rapide et se résorbent d'elles-mêmes et ne suscitent pas de réponse lymphoïde comme le font généralement les adénovirus. Ou encore certaines bactéries plus atypiques qui produisent une conjonctivite chronique, comme Chlamydia, ou des bactéries beaucoup plus rares comme celles qui sont responsables du syndrome oculoglandulaire de Parinaud (Bartonella henselae, Francisella tularensis, Sporothrix schenckii, etc). L'adénopathie étant très typique de la conjonctivite adénovirale, cette option est la plus facile à éliminer. Étant donné que les chlamydiae suscitent la réponse lymphoïde de la même manière que les adénovirus (ils produisent également des follicules conjonctivaux, par exemple), il s'agirait de la deuxième option la plus facile à exclure. Le syndrome oculoglandulaire de Parinaud est peut-être le plus difficile à exclure, car il s'agit d'une maladie très rare. Si nous ne savons pas qu'il s'agit d'une maladie infectieuse, nous pouvons hésiter. De toute façon, la conjonctivite allergique n'est pas un problème infectieux. Il n'y aura donc pas d'adénopathies. Bien que nous puissions hésiter avec certaines des options précédentes (en particulier avec Parinaud), si nous nous en tenons aux principes de base (la conjonctivite allergique ne provoque pas d'adénopathies), nous pouvons avoir raison. L'option 2 est la bonne.", "type": "OPHTHALMOLOGIE", "options": {"1": "Conjonctivite adénovirale.", "2": "Conjonctivite allergique.", "3": "Syndrome oculoglandulaire de Parinaud.", "4": "Conjonctivite à Chlamydia.", "5": null}, "correct_option": 2, "explanations": {"1": {"exist": true, "char_ranges": [[674, 783]], "word_ranges": [[86, 102]], "text": "L'adénopathie étant très typique de la conjonctivite adénovirale, cette option est la plus facile à éliminer."}, "2": {"exist": true, "char_ranges": [[1337, 1598]], "word_ranges": [[186, 225]], "text": "Bien que nous puissions hésiter avec certaines des options précédentes (en particulier avec Parinaud), si nous nous en tenons aux principes de base (la conjonctivite allergique ne provoque pas d'adénopathies), nous pouvons avoir raison. L'option 2 est la bonne."}, "3": {"exist": true, "char_ranges": [[1222, 1336]], "word_ranges": [[169, 186]], "text": "De toute façon, la conjonctivite allergique n'est pas un problème infectieux. Il n'y aura donc pas d'adénopathies."}, "4": {"exist": true, "char_ranges": [[784, 1014]], "word_ranges": [[102, 137]], "text": "Étant donné que les chlamydiae suscitent la réponse lymphoïde de la même manière que les adénovirus (ils produisent également des follicules conjonctivaux, par exemple), il s'agirait de la deuxième option la plus facile à exclure."}, "5": {"exist": false, "char_ranges": [], "word_ranges": [], "text": ""}}} +{"id": 11, "year": 2011, "question_id_specific": 74, "full_question": "Dix jours après avoir pratiqué une hémithyroïdectomie sur un patient présentant un nodule thyroïdien de 1,5 cm, il reçoit le rapport anatomopathologique suivant : \"modèle folliculaire très cellulaire sans invasion capsulaire et infiltrant les vaisseaux sanguins et les nerfs adjacents\". Cochez la bonne réponse :", "full_answer": "Je pense que la bonne réponse est 3, les réponses 2, 4 et 5 sont clairement fausses. Le doute se pose entre les réponses 1 et 2. L'affirmation de la réponse 1 est correcte : en cas de carcinome folliculaire sans invasion capsulaire, une thyroïdectomie totale peut être pratiquée. Mais s'il existe des signes de mauvais pronostic et la possibilité de métastases - comme dans ce cas - il est préférable de compléter la thyroïdectomie et d'effectuer un contrôle à l'iode radioactif pour exclure les métastases ou les traiter si elles sont présentes.", "type": "CHIRURGIE", "options": {"1": "Comme il n'y a pas d'invasion capsulaire, aucune autre intervention chirurgicale n'est nécessaire.", "2": "La forme folliculaire est une indication pour un curage prophylactique des ganglions lymphatiques cervicaux.", "3": "Il s'agit d'un carcinome folliculaire qui nécessite une thyroïdectomie complète.", "4": "Le rapport permet d'exclure avec certitude la présence d'un carcinome papillaire dans le reste de la thyroïde.", "5": "La mesure de la calcitonine plasmatique permet de distinguer le carcinome papillaire du carcinome folliculaire."}, "correct_option": 3, "explanations": {"1": {"exist": true, "char_ranges": [[129, 546]], "word_ranges": [[27, 93]], "text": "L'affirmation de la réponse 1 est correcte : en cas de carcinome folliculaire sans invasion capsulaire, une thyroïdectomie totale peut être pratiquée. Mais s'il existe des signes de mauvais pronostic et la possibilité de métastases - comme dans ce cas - il est préférable de compléter la thyroïdectomie et d'effectuer un contrôle à l'iode radioactif pour exclure les métastases ou les traiter si elles sont présentes."}, "2": {"exist": false, "char_ranges": [], "word_ranges": [], "text": ""}, "3": {"exist": true, "char_ranges": [[287, 546]], "word_ranges": [[50, 93]], "text": "s'il existe des signes de mauvais pronostic et la possibilité de métastases - comme dans ce cas - il est préférable de compléter la thyroïdectomie et d'effectuer un contrôle à l'iode radioactif pour exclure les métastases ou les traiter si elles sont présentes."}, "4": {"exist": false, "char_ranges": [], "word_ranges": [], "text": ""}, "5": {"exist": false, "char_ranges": [], "word_ranges": [], "text": ""}}} +{"id": 535, "year": 2021, "question_id_specific": 121, "full_question": "Une femme noire de 27 ans consulte pour l'apparition d'œdèmes des membres inférieurs, une diminution de la diurèse, de la fièvre et une éruption malaire en ailes de papillon évoluant depuis 20 jours. Les examens de laboratoire montrent une créatinine à 3 mg/dl, une leucopénie, une hémoglobine à 10,5 g/dl (normochrome normocytaire), une CRP à 9 mg/dl et une ESR à 60 mm. En ce qui concerne la pathologie présentée par ce patient, veuillez indiquer la bonne réponse :", "full_answer": "Une autre question sur les anticorps du LED. Cas clinique de LED avec atteinte rénale, dans lequel la présence d'anticorps anti-DNA double brin augmente le risque de néphrite. Pour le reste, les anticorps anti-Sm sont spécifiques du LED, les ANA sont détectés dans plus de 90 % des cas et les anticorps anti-centromères sont typiques de la sclérodermie.", "type": "RHEUMATOLOGIE", "options": {"1": "L'anti-Sm n'est pas spécifique à cette pathologie.", "2": "Des anticorps anti-centromères sont détectés dans 90 % des cas.", "3": "Les ANA (anticorps antinucléaires) sont détectés dans 50 % des cas.", "4": "L'anti-DNA bicaténaire ou natif est en corrélation avec le risque de néphrite.", "5": null}, "correct_option": 4, "explanations": {"1": {"exist": true, "char_ranges": [[191, 237]], "word_ranges": [[31, 38]], "text": "les anticorps anti-Sm sont spécifiques du LED,"}, "2": {"exist": true, "char_ranges": [[289, 353]], "word_ranges": [[50, 58]], "text": "les anticorps anti-centromères sont typiques de la sclérodermie."}, "3": {"exist": true, "char_ranges": [[238, 285]], "word_ranges": [[38, 49]], "text": "les ANA sont détectés dans plus de 90 % des cas"}, "4": {"exist": true, "char_ranges": [[45, 175]], "word_ranges": [[8, 28]], "text": "Cas clinique de LED avec atteinte rénale, dans lequel la présence d'anticorps anti-DNA double brin augmente le risque de néphrite."}, "5": {"exist": false, "char_ranges": [], "word_ranges": [], "text": ""}}} +{"id": 452, "year": 2018, "question_id_specific": 149, "full_question": "Chez une femme victime d'une crise d'épilepsie présentant les caractéristiques cliniques suivantes : aura épigastrique, odeur désagréable, déconnexion de l'environnement, automatismes moteurs (succion, déglutition, ouverture et fermeture d'une main) et amnésie postcritique, quelle est votre suspicion diagnostique ?", "full_answer": "Il est clair que la réponse est 4, avec une clinique très caractéristique de crises temporaires.", "type": "NEUROLOGIE", "options": {"1": "Crise généralisée non convulsive ou absence typique.", "2": "Épilepsie partielle continue.", "3": "Crise amyotonique.", "4": "Crise partielle complexe du lobe temporal.", "5": null}, "correct_option": 4, "explanations": {"1": {"exist": false, "char_ranges": [], "word_ranges": [], "text": ""}, "2": {"exist": false, "char_ranges": [], "word_ranges": [], "text": ""}, "3": {"exist": false, "char_ranges": [], "word_ranges": [], "text": ""}, "4": {"exist": true, "char_ranges": [[0, 96]], "word_ranges": [[0, 16]], "text": "Il est clair que la réponse est 4, avec une clinique très caractéristique de crises temporaires."}, "5": {"exist": false, "char_ranges": [], "word_ranges": [], "text": ""}}} +{"id": 55, "year": 2011, "question_id_specific": 74, "full_question": "Dix jours après avoir pratiqué une hémithyroïdectomie sur une patiente présentant un nodule thyroïdien de 1,5 cm, elle reçoit du pathologiste le rapport définitif suivant : \"modèle folliculaire très cellulaire sans invasion capsulaire envahissant les vaisseaux sanguins et les nerfs adjacents\". Cochez la bonne réponse :", "full_answer": "Je pense qu'il s'agit d'une question un peu plus difficile, car il est nécessaire de bien comprendre l'anatomie pathologique. En tout état de cause, elle pourrait être écartée par exclusion et par l'expression des réponses : Réponse 2 : la vidange prophylactique est indiquée dans la moelle épinière. Réponse 4 : rien que par la façon dont elle est exprimée, elle aurait dû être exclue. Réponse 5 : vous savez déjà que la calcitonine est utilisée dans le carcinome médullaire et non dans le carcinome différencié. Les réponses 1 et 3 sont contraires, l'une recommandant la thyroïdectomie complète et l'autre non. Les seuls pour lesquels la thyroïdectomie complète n'est pas recommandée sont les microcarcinomes mesurant moins de 1 cm.", "type": "ENDOCRINOLOGIE", "options": {"1": "Comme il n'y a pas d'invasion capsulaire, aucune autre intervention chirurgicale n'est conseillée.", "2": "La forme folliculaire est une indication pour un curage prophylactique des ganglions lymphatiques cervicaux.", "3": "Il s'agit d'un carcinome folliculaire qui nécessite une thyroïdectomie complète.", "4": "Le rapport permet d'exclure avec certitude un carcinome papillaire dans la thyroïde restante.", "5": "La mesure de la calcitonine plasmatique permettra de différencier le carcinome papillaire du carcinome folliculaire."}, "correct_option": 3, "explanations": {"1": {"exist": true, "char_ranges": [[514, 734]], "word_ranges": [[86, 119]], "text": "Les réponses 1 et 3 sont contraires, l'une recommandant la thyroïdectomie complète et l'autre non. Les seuls pour lesquels la thyroïdectomie complète n'est pas recommandée sont les microcarcinomes mesurant moins de 1 cm."}, "2": {"exist": true, "char_ranges": [[237, 300]], "word_ranges": [[39, 48]], "text": "la vidange prophylactique est indiquée dans la moelle épinière."}, "3": {"exist": true, "char_ranges": [[514, 734]], "word_ranges": [[86, 119]], "text": "Les réponses 1 et 3 sont contraires, l'une recommandant la thyroïdectomie complète et l'autre non. Les seuls pour lesquels la thyroïdectomie complète n'est pas recommandée sont les microcarcinomes mesurant moins de 1 cm."}, "4": {"exist": true, "char_ranges": [[313, 386]], "word_ranges": [[51, 65]], "text": "rien que par la façon dont elle est exprimée, elle aurait dû être exclue."}, "5": {"exist": true, "char_ranges": [[419, 513]], "word_ranges": [[72, 86]], "text": "la calcitonine est utilisée dans le carcinome médullaire et non dans le carcinome différencié."}}} +{"id": 281, "year": 2016, "question_id_specific": 33, "full_question": "Une femme de 67 ans a été diagnostiquée avec un carcinome canalaire infiltrant du sein sans antécédents familiaux de néoplasie. Quelles sont les études complémentaires à réaliser sur la tumeur pour ses implications cliniques et thérapeutiques ?", "full_answer": "La bonne réponse serait 2, HER2 and HORMONE RECEPTOR STUDY, car des récepteurs HER2 positifs impliquent une plus grande agressivité de la tumeur mais un traitement au trastuzumab et des récepteurs hormonaux positifs impliqueraient l'utilisation d'une thérapie hormonale. La réponse 1 peut être exclue visu visu, tandis que les réponses 3 et 4 devraient être exclues car, comme il s'agit de la première femme de la famille atteinte d'un carcinome mammaire, une étude des parents de la patiente ne serait pas obligatoire. De même, l'e-cadhérine est importante pour le diagnostic différentiel du carcinome lobulaire du sein, et non du carcinome canalaire.", "type": "ANATOMIE PATHOLOGIQUE", "options": {"1": "Étude phénotypique complète par cytométrie de flux.", "2": "Étude des récepteurs hormonaux et de HER2.", "3": "Étude des récepteurs hormonaux, de l'e-cadhérine et étude des parents au premier degré.", "4": "Dépistage BRCA 1-2 et dépistage des parents au premier degré.", "5": null}, "correct_option": 2, "explanations": {"1": {"exist": true, "char_ranges": [[271, 310]], "word_ranges": [[38, 46]], "text": "La réponse 1 peut être exclue visu visu,"}, "2": {"exist": true, "char_ranges": [[64, 270]], "word_ranges": [[11, 38]], "text": "des récepteurs HER2 positifs impliquent une plus grande agressivité de la tumeur mais un traitement au trastuzumab et des récepteurs hormonaux positifs impliqueraient l'utilisation d'une thérapie hormonale."}, "3": {"exist": true, "char_ranges": [[323, 519]], "word_ranges": [[48, 82]], "text": "les réponses 3 et 4 devraient être exclues car, comme il s'agit de la première femme de la famille atteinte d'un carcinome mammaire, une étude des parents de la patiente ne serait pas obligatoire."}, "4": {"exist": true, "char_ranges": [[323, 519]], "word_ranges": [[48, 82]], "text": "les réponses 3 et 4 devraient être exclues car, comme il s'agit de la première femme de la famille atteinte d'un carcinome mammaire, une étude des parents de la patiente ne serait pas obligatoire."}, "5": {"exist": false, "char_ranges": [], "word_ranges": [], "text": ""}}} +{"id": 310, "year": 2016, "question_id_specific": 216, "full_question": "Un parent se présente aux urgences avec son fils de deux ans car il rapporte qu'il s'est cogné près de l'œil droit en jouant. En effet, un hématome a été observé sur la paupière droite, apparemment sans importance. Des hémorragies intrarétiniennes ont été observées dans le fond de l'œil, non seulement dans l'œil mentionné par les parents, mais aussi dans l'autre œil. Il est frappant de constater que l'enfant semble somnolent et peu tonique. Parmi les affirmations suivantes, laquelle doit être prise en compte au regard de ce tableau clinique ?", "full_answer": "Une question facile en principe, mais avec un petit piège. Si vous avez la bonne intuition lorsque vous la lisez et que vous allez directement à ce que vous pensez, vous aurez probablement raison. Le problème, c'est l'option 4, qui peut nous embrouiller. Voyons : nous avons un coup au niveau de l'œil droit, avec une ecchymose sur la paupière. Jusque-là, tout est normal. Le problème est que des hémorragies rétiniennes apparaissent non seulement dans cet œil (ce qui est rare en soi, car un coup externe à l'œil chez un enfant provoque rarement une hémorragie dans la rétine), mais aussi dans l'autre œil. Par conséquent, quelque chose d'autre que le coup a provoqué des hémorragies dans les deux rétines. Ensuite, on nous dit que l'enfant semble somnolent et peu tonique. Nous avons ici les deux indices ; nous devons soupçonner ce qui n'est pas mentionné dans les réponses, mais dont l'enfant est atteint : le syndrome du bébé secoué. Ce syndrome se caractérise par des hémorragies rétiniennes bilatérales, un œdème cérébral et un hématome sous-dural. Des fractures des os longs et d'autres problèmes peuvent également survenir. La somnolence et l'hypotonie sont des causes de complications intracrâniennes présumées. La cause du syndrome du bébé secoué est presque toujours la maltraitance, la réponse est donc 1. Les options 2 et 3 sont faciles à écarter : rien n'indique un traumatisme perforant et cela n'explique pas les hémorragies dans l'autre œil. Et l'œdème de Berlin est un œdème de la macula qui se présente comme une lésion blanchâtre, ce qui n'est pas décrit dans la déclaration. La rétinopathie de Purstcher peut également apparaître dans le contexte de l'enfant secoué, et donc se produire en cas de maltraitance. En voyant ces deux questions partiellement liées, on peut hésiter. Cependant, si nous lisons attentivement la réponse 4, elle ne dit pas que Purstcher peut survenir en cas de maltraitance de l'enfant. Elle affirme que le tableau décrit ci-dessus (hémorragies intra-rétiniennes, sans autres signes) est caractéristique de l'angiopathie de Purstcher. Ce n'est pas le cas. Dans l'angiopathie de Purstcher, il y a des hémorragies, mais aussi des exsudats cotonneux et des œdèmes. S'il n'y a que des hémorragies, ce n'est pas une angiopathie de Purstcher. Voici la partie la plus compliquée, mais elle est très bien formulée. S'il avait un Purstcher, ce pourrait être aussi pour maltraitance d'enfant, mais il n'en a pas.", "type": "OPHTHALMOLOGIE", "options": {"1": "Il s'agit d'une histoire très suggestive d'abus d'enfants.", "2": "Il s'agit de l'évolution normale d'un traumatisme intraoculaire non perforant.", "3": "Le diagnostic le plus probable est l'œdème de Berlin.", "4": "C'est l'image caractéristique de l'angiopathie rétinienne traumatique de Purstcher.", "5": null}, "correct_option": 1, "explanations": {"1": {"exist": true, "char_ranges": [[1133, 1318]], "word_ranges": [[187, 215]], "text": "La somnolence et l'hypotonie sont des causes de complications intracrâniennes présumées. La cause du syndrome du bébé secoué est presque toujours la maltraitance, la réponse est donc 1."}, "2": {"exist": true, "char_ranges": [[1319, 1459]], "word_ranges": [[215, 239]], "text": "Les options 2 et 3 sont faciles à écarter : rien n'indique un traumatisme perforant et cela n'explique pas les hémorragies dans l'autre œil."}, "3": {"exist": true, "char_ranges": [[1319, 1459]], "word_ranges": [[215, 239]], "text": "Les options 2 et 3 sont faciles à écarter : rien n'indique un traumatisme perforant et cela n'explique pas les hémorragies dans l'autre œil."}, "4": {"exist": true, "char_ranges": [[2103, 2283]], "word_ranges": [[340, 370]], "text": "Dans l'angiopathie de Purstcher, il y a des hémorragies, mais aussi des exsudats cotonneux et des œdèmes. S'il n'y a que des hémorragies, ce n'est pas une angiopathie de Purstcher."}, "5": {"exist": false, "char_ranges": [], "word_ranges": [], "text": ""}}} +{"id": 293, "year": 2016, "question_id_specific": 214, "full_question": "Un patient se présente aux urgences avec une érythrodermie, de la fièvre et un malaise général. À l'examen, les ongles présentent des piqûres et des zones jaunâtres distales sous la forme d'une tache d'huile. Quelle est la principale maladie à l'origine de cette affection ?", "full_answer": "Chacune des quatre options peut être la cause de l'érythrodermie, bien que la description de lésions des ongles fortement évocatrices de psoriasis rende ce diagnostic le plus probable.", "type": "DERMATOLOGIE, VÉNÉRÉOLOGIE ET CHIRURGIE PLASTIQUE", "options": {"1": "Lymphome cutané.", "2": "Dermatite atopique.", "3": "Psoriasis.", "4": "Ichthyose.", "5": null}, "correct_option": 3, "explanations": {"1": {"exist": false, "char_ranges": [], "word_ranges": [], "text": ""}, "2": {"exist": false, "char_ranges": [], "word_ranges": [], "text": ""}, "3": {"exist": true, "char_ranges": [[75, 184]], "word_ranges": [[12, 28]], "text": "la description de lésions des ongles fortement évocatrices de psoriasis rende ce diagnostic le plus probable."}, "4": {"exist": false, "char_ranges": [], "word_ranges": [], "text": ""}, "5": {"exist": false, "char_ranges": [], "word_ranges": [], "text": ""}}} +{"id": 132, "year": 2012, "question_id_specific": 229, "full_question": "Un homme de 54 ans, ayant des antécédents personnels de diabète, d'hypertension et d'insuffisance rénale chronique légère, s'est présenté avec une somnolence et une hémiparésie droite. Examens de laboratoire : créatinine 2,3 mg/dl, Hb 10,3 g/dl et plaquettes 20000 mm3, avec coagulation normale. Tomodensitométrie crânienne : lésions ischémiques avec nécrose microhémorragique. Par la suite, il a commencé à présenter une détérioration progressive du niveau de conscience et une augmentation de la Cr, de la LDH et de la bilirubine non conjuguée. Le test de Coombs direct est négatif. Schistocytes dans les frottis. Compte tenu du diagnostic suspecté, la conduite à tenir recommandée était la suivante :", "full_answer": "Voyons : un patient avec certains antécédents qui présente une anémie microangiopathique : - Schistocytes dans le frottis : indique une anémie microangiopathique, une rupture des globules rouges due à des causes mécaniques, que l'on peut observer principalement dans le PTT, l'anémie hémolytique auto-immune et la CIVD. - Bicytopénie : les causes peuvent être très, très variées. - Élévation de la LDH et de la bilirubine, qui indique une dégradation massive des globules rouges. - Test de Coombs direct négatif : ce test est essentiel, car il indique qu'aucun anticorps ne colle aux globules rouges. Réponse 1 : ce n'est pas possible car une transformation hémorragique ne donne pas une élévation aussi brutale de la LDH et de la Cr, et n'explique pas non plus la schistocytose. Réponse 2 : plasmaphérèse urgente...en cas de purpura thrombocytopénique thrombotique, oui ; J'aime bien cette réponse car elle justifie la bicytopénie, l'anémie hémolytique microangiopathique étant due à la rupture des globules rouges lorsqu'ils sont fragmentés par des multimères ultralongs du facteur von Willebrand en cas de déficit de l'enzyme ADAMST13, ce qui entraîne des agrégats plaquettaires brutaux et le cytomètre de flux ne compte pas 400 plaquettes, mais un molondron de plaquettes formant une masse, d'où la thrombocytopénie. Cela explique aussi le Coombs direct négatif, la LDH et la Cr élevées. Réponse 3 : une situation aussi critique que celle de ce patient indique qu'il se passe quelque chose de grave et je ne resterais pas à attendre que les glucocorticoïdes fassent effet... et je ne sais pas pour l'instant quelle pathologie donne ces symptômes et a ce traitement en standard. Réponse 4 : chercher un déclencheur de CIVD... ce serait logique si ce n'était pas le fait que la CIVD est TRÈS caractéristique de la consommation de facteurs de coagulation, ce qui n'est pas le cas ici. Réponse 5 : le myélome ne provoque pas de schistocytose et n'a pas cette symptomatologie, il faudrait d'autres données pour étayer cette pathologie, comme les douleurs osseuses et l'hypercalcémie....", "type": "NEUROLOGIE ET NEUROCHIRURGIE", "options": {"1": "Nouvelle tomodensitométrie crânienne en cas de suspicion de transformation hémorragique d'un accident vasculaire cérébral ischémique.", "2": "Mettre en place une plasmaphérèse d'urgence.", "3": "Initiation d'un traitement par glucocorticoïdes à la dose de 1 mg/kg/jour.", "4": "Recherche d'une cause de déclenchement du DIC.", "5": "Myélome multiple suspecté : aspiration de la moelle osseuse."}, "correct_option": 2, "explanations": {"1": {"exist": true, "char_ranges": [[601, 779]], "word_ranges": [[95, 127]], "text": "Réponse 1 : ce n'est pas possible car une transformation hémorragique ne donne pas une élévation aussi brutale de la LDH et de la Cr, et n'explique pas non plus la schistocytose."}, "2": {"exist": true, "char_ranges": [[780, 872]], "word_ranges": [[127, 139]], "text": "Réponse 2 : plasmaphérèse urgente...en cas de purpura thrombocytopénique thrombotique, oui ;"}, "3": {"exist": true, "char_ranges": [[1392, 1578]], "word_ranges": [[218, 250]], "text": "Réponse 3 : une situation aussi critique que celle de ce patient indique qu'il se passe quelque chose de grave et je ne resterais pas à attendre que les glucocorticoïdes fassent effet..."}, "4": {"exist": true, "char_ranges": [[1682, 1885]], "word_ranges": [[268, 305]], "text": "Réponse 4 : chercher un déclencheur de CIVD... ce serait logique si ce n'était pas le fait que la CIVD est TRÈS caractéristique de la consommation de facteurs de coagulation, ce qui n'est pas le cas ici."}, "5": {"exist": true, "char_ranges": [[1886, 2085]], "word_ranges": [[305, 334]], "text": "Réponse 5 : le myélome ne provoque pas de schistocytose et n'a pas cette symptomatologie, il faudrait d'autres données pour étayer cette pathologie, comme les douleurs osseuses et l'hypercalcémie...."}}} +{"id": 49, "year": 2011, "question_id_specific": 159, "full_question": "Un garçon de 8 ans se plaint d'une boiterie du membre inférieur droit, de présentation insidieuse et d'un mois d'évolution. Il n'y a pas d'antécédents de traumatisme ni de signes constitutionnels. À l'examen, il existe des signes de restriction de la mobilisation passive du membre, en particulier en abduction et en rotation interne. Parmi les affirmations suivantes, toutes sont vraies SAUF une :", "full_answer": "La bonne réponse est 3. La maladie de Perthes est plus fréquente et a un meilleur pronostic chez les jeunes enfants.", "type": "PÉDIATRIE", "options": {"1": "Une radiographie de la hanche est conseillée.", "2": "Dans le cas de la synovite transitoire, l'apparition est généralement aiguë.", "3": "Dans la maladie de Legg-Calvé-Perthes, plus l'âge d'apparition est jeune, plus le pronostic est mauvais.", "4": "La cause peut être un état d'hypercoagulabilité.", "5": "L'option thérapeutique varie en fonction du degré d'atteinte."}, "correct_option": 3, "explanations": {"1": {"exist": false, "char_ranges": [], "word_ranges": [], "text": ""}, "2": {"exist": false, "char_ranges": [], "word_ranges": [], "text": ""}, "3": {"exist": true, "char_ranges": [[24, 116]], "word_ranges": [[5, 21]], "text": "La maladie de Perthes est plus fréquente et a un meilleur pronostic chez les jeunes enfants."}, "4": {"exist": false, "char_ranges": [], "word_ranges": [], "text": ""}, "5": {"exist": false, "char_ranges": [], "word_ranges": [], "text": ""}}} +{"id": 120, "year": 2012, "question_id_specific": 192, "full_question": "Un chirurgien orthopédiste de 40 ans qui a détecté des taux d'anti-HBs de 30 UI/l lors d'un test sérologique de routine. Il déclare avoir été complètement vacciné contre l'hépatite B selon le calendrier de vaccination standard il y a 4 ans. Il serait souhaitable :", "full_answer": "Tant que les taux sérologiques d'anti-HBs sont supérieurs à 10 UI/l, la revaccination n'est pas nécessaire.", "type": "MÉDECINE PRÉVENTIVE ET ÉPIDÉMIOLOGIE", "options": {"1": "Reprendre le calendrier des vaccinations (0-1-6 mois).", "2": "Reprendre le calendrier des vaccinations (0-1-2-12 mois).", "3": "Ne pas revacciner.", "4": "Donner une dose de rappel.", "5": "Administrer des immunoglobulines anti-hépatite B et reprendre le calendrier de vaccination (0-1-2-12 mois)."}, "correct_option": 3, "explanations": {"1": {"exist": false, "char_ranges": [], "word_ranges": [], "text": ""}, "2": {"exist": false, "char_ranges": [], "word_ranges": [], "text": ""}, "3": {"exist": true, "char_ranges": [[0, 107]], "word_ranges": [[0, 16]], "text": "Tant que les taux sérologiques d'anti-HBs sont supérieurs à 10 UI/l, la revaccination n'est pas nécessaire."}, "4": {"exist": false, "char_ranges": [], "word_ranges": [], "text": ""}, "5": {"exist": false, "char_ranges": [], "word_ranges": [], "text": ""}}} +{"id": 304, "year": 2016, "question_id_specific": 174, "full_question": "Homme de 87 ans ayant des antécédents d'hypertension et de gonarthrose. Situation de base avec une autonomie fonctionnelle et cognitive complète qui lui permet de continuer à vivre seul dans la communauté. Il est régulièrement traité au périndopril et aux diurétiques thiazidiques pour contrôler sa tension artérielle et prend régulièrement de l'ibuprofène 1800 mg/jour pour contrôler les symptômes de sa gonarthrose. Après un examen de routine, on a observé une tension artérielle persistante de 190 et un TAD de 80 mmHg. Quelle serait la modification thérapeutique la plus raisonnable pour parvenir à contrôler les chiffres de la tension artérielle ?", "full_answer": "Avant d'intensifier le traitement antihypertenseur, il faut s'attaquer aux causes potentielles. Dans le cas présent, il s'agit d'un patient âgé qui suit un traitement chronique aux AINS, provoquant une hypertension artérielle secondaire à une hypoperfusion rénale. L'option la plus raisonnable serait d'arrêter l'ibuprofène et de le remplacer par un analgésique d'un groupe autre que les AINS (paracétamol, opiacés), en surveillant ensuite l'évolution des niveaux de pression artérielle.", "type": "NEPHROLOGIE", "options": {"1": "J'ajouterais un bloqueur de calcium.", "2": "J'augmenterais la dose d'hydrochlorothiazide à 25 mg/jour.", "3": "Je remplacerais l'ibuprofène par le paracétamol pour éviter l'influence possible du paracétamol sur l'effet des antihypertenseurs.", "4": "J'ajouterais un alpha-bloquant en raison de la forte prévalence du syndrome de la prostate chez les hommes de cet âge.", "5": null}, "correct_option": 3, "explanations": {"1": {"exist": false, "char_ranges": [], "word_ranges": [], "text": ""}, "2": {"exist": false, "char_ranges": [], "word_ranges": [], "text": ""}, "3": {"exist": true, "char_ranges": [[265, 487]], "word_ranges": [[36, 67]], "text": "L'option la plus raisonnable serait d'arrêter l'ibuprofène et de le remplacer par un analgésique d'un groupe autre que les AINS (paracétamol, opiacés), en surveillant ensuite l'évolution des niveaux de pression artérielle."}, "4": {"exist": false, "char_ranges": [], "word_ranges": [], "text": ""}, "5": {"exist": false, "char_ranges": [], "word_ranges": [], "text": ""}}} +{"id": 214, "year": 2014, "question_id_specific": 228, "full_question": "Un homme de 19 ans consulte pour une douleur, un gonflement et une impotence fonctionnelle du genou droit depuis 24 heures, accompagnés d'une fièvre à 38°C. L'examen physique a révélé des signes inflammatoires et un épanchement articulaire au niveau du genou droit. Les examens de laboratoire ont révélé une leucocytose avec neutrophilie et une élévation de la protéine C-réactive. Le diagnostic syndromique de monoarthrite aiguë est posé. Quel est le diagnostic étiologique le plus probable ?", "full_answer": "Une monoarthrite aiguë associée à de la fièvre, une leucocytose avec neutrophilie et une augmentation des réactifs de phase aiguë n'est pas toujours d'origine septique. En l'absence d'informations complémentaires (anamnèse plus complète sur la maladie en cours, facteurs de risque, antécédents personnels et familiaux, symptômes ou signes extra-articulaires, etc.), on peut dire que 1 et 2 (et très exceptionnellement 5) pourraient également débuter avec un tableau clinique et biologique similaire. Avec les données fournies et compte tenu du fait qu'il s'agit d'un jeune homme, l'option la plus probable serait une arthrite infectieuse bactérienne (les arthrites causées par des mycobactéries ont tendance à avoir une évolution chronique). Et surtout, en raison de ses implications, c'est la première option qu'il faut toujours écarter.", "type": "RHEUMATOLOGIE", "options": {"1": "L'arthrite microcristalline.", "2": "Arthrite réactive.", "3": "Arthrite infectieuse bactérienne.", "4": "Arthrite infectieuse d'origine mycobactérienne.", "5": "Polyarthrite rhumatoïde."}, "correct_option": 3, "explanations": {"1": {"exist": true, "char_ranges": [[742, 838]], "word_ranges": [[106, 121]], "text": "Et surtout, en raison de ses implications, c'est la première option qu'il faut toujours écarter."}, "2": {"exist": false, "char_ranges": [], "word_ranges": [], "text": ""}, "3": {"exist": true, "char_ranges": [[529, 649]], "word_ranges": [[75, 93]], "text": "compte tenu du fait qu'il s'agit d'un jeune homme, l'option la plus probable serait une arthrite infectieuse bactérienne"}, "4": {"exist": true, "char_ranges": [[650, 741]], "word_ranges": [[93, 106]], "text": "(les arthrites causées par des mycobactéries ont tendance à avoir une ��volution chronique)."}, "5": {"exist": false, "char_ranges": [], "word_ranges": [], "text": ""}}} +{"id": 249, "year": 2014, "question_id_specific": 120, "full_question": "Une femme de 53 ans consulte pour une fièvre qui dure depuis 15 jours, sans symptôme de foyer infectieux. L'examen révèle une hépatomégalie douloureuse à 5 cm du rebord costal et la rate est palpée à 14 cm du rebord costal gauche. L'hémogramme montre une Hb à 8,5 g/dl, des leucocytes à 630/ml (lymphocytes 63 %, monocytes 20 %, neutrophiles 17 %) et des plaquettes à 35 000/ml. La biochimie montre une élévation modérée de la biochimie hépatique, la LDH est normale et une hypergammaglobulinémie polyclonale (3,5 g/dl) est observée dans le protéinogramme. Il a des antécédents d'infection par le VIH depuis 10 ans et une adhésion irrégulière au traitement antirétroviral, avec une numération des lymphocytes CD4 récente de 350 cellules/mL et une charge virale du VIH de 154 copies/mL. Depuis trois mois, elle est traitée pour une polyarthrite symétrique séronégative par 10 à 20 mg/jour de prednisone. Laquelle des affirmations suivantes est correcte ?", "full_answer": "Une question facile et assez typique. On nous parle d'un patient VIH avec un suivi irrégulier du traitement antirétroviral, qui présente de la fièvre, une hépatosplénomégalie, une pancytopénie et une hypergammaglobulinémie polyclonale. Toutes ces données sont évocatrices d'une leishmaniose viscérale et il convient donc de réaliser une biopsie de l'OM pour rechercher des amastigotes et confirmer le diagnostic.", "type": "LES MALADIES INFECTIEUSES", "options": {"1": "Je ferais une biopsie de la moelle osseuse, car le diagnostic le plus probable est la leishmaniose viscérale.", "2": "La pancytopénie est justifiée par une cirrhose associée au virus C et je ne ferais pas d'autres tests.", "3": "Il s'agit probablement d'une toxicité de la moelle osseuse due à la prednisone, qui serait traitée par l'arrêt du médicament et le filgastrim.", "4": "Je demanderais un test ANA pour exclure un lupus systémique disséminé.", "5": "J'intensifierais le traitement antirétroviral, car toutes les manifestations sont probablement dues au VIH."}, "correct_option": 1, "explanations": {"1": {"exist": true, "char_ranges": [[236, 412]], "word_ranges": [[32, 58]], "text": "Toutes ces données sont évocatrices d'une leishmaniose viscérale et il convient donc de réaliser une biopsie de l'OM pour rechercher des amastigotes et confirmer le diagnostic."}, "2": {"exist": false, "char_ranges": [], "word_ranges": [], "text": ""}, "3": {"exist": false, "char_ranges": [], "word_ranges": [], "text": ""}, "4": {"exist": false, "char_ranges": [], "word_ranges": [], "text": ""}, "5": {"exist": false, "char_ranges": [], "word_ranges": [], "text": ""}}} +{"id": 53, "year": 2011, "question_id_specific": 146, "full_question": "Un homme de 31 ans sans antécédents psychiatriques se présente au service des urgences en signalant... des symptômes d'anxiété)¨. Il y a deux jours, il a connu un nouvel épisode des mêmes symptômes... .... Il y a 1 semaine, il a rompu avec son partenaire... a exclu toute pathologie organique.... :", "full_answer": "Il est bon qu'ils vous parlent de leur histoire (nous avons tous une histoire parce que nous avons tous une vie) et qu'ils excluent toute pathologie organique. C'est fondamental. N'oublions pas que les hypocondriaques ont aussi un corps. On élimine d'abord le trouble \"aigu\" de la personnalité le plus frappant. La personnalité nous accompagne toute notre vie. Il n'y a pas de caractéristiques \"aiguës\" dans notre personnalité. Un trouble dépressif ? Il y a un critère de temps qui n'est pas respecté, je peux être triste, mais triste n'est pas une dépression (1). agoraphobie ? si c'était quelque chose comme ça, on nous aurait dit le contexte réaction de stress post-traumatique ? C'est une bonne question. The correct term would be post-traumatic stress disorder or acute stress reaction, and according to ICD 10: \"The stressor may be a devastating traumatic experience involving a serious threat to the safety or physical integrity of the sufferer or loved one(s) (e.g. natural disasters, accidents, battles, muggings, rapes) or an abrupt and threatening change in the individual's rank or social environment (e.g. loss of several loved ones, house fire, etc.)\" or more succinctly: \"the stressor may be a devastating traumatic experience involving a serious threat to the safety or physical integrity of the sufferer or loved one(s) (e.g. natural disasters, accidents, battles, muggings, rapes) or an abrupt and threatening change in the individual's rank or social environment (e.g. loss of several loved ones, fire in the home, etc.).)\" ou plus brièvement : \"caractère exceptionnellement menaçant ou catastrophique\" Rompre avec un partenaire est difficile, je le comprends, mais pas exceptionnel. Donc par élimination 5. crise de détresse. Il s'agit d'une question utile, courante dans la pratique quotidienne, et nous voyons donc que nous devrions éviter de donner un diagnostic de longue évolution pour une réaction naturelle. Les diagnostics 1 à 4 reviendraient à qualifier de diabétique une personne en hyperglycémie qui aurait mangé trois muffins. Question utile et réaliste.", "type": "PSYCHIATRIE", "options": {"1": "Réaction de stress post-traumatique.", "2": "Trouble de la personnalité dépendante \"aiguë\".", "3": "Trouble dépressif.", "4": "Agoraphobie.", "5": "Crise d'angoisse."}, "correct_option": 5, "explanations": {"1": {"exist": true, "char_ranges": [[1623, 1703]], "word_ranges": [[253, 265]], "text": "Rompre avec un partenaire est difficile, je le comprends, mais pas exceptionnel."}, "2": {"exist": true, "char_ranges": [[238, 427]], "word_ranges": [[38, 67]], "text": "On élimine d'abord le trouble \"aigu\" de la personnalité le plus frappant. La personnalité nous accompagne toute notre vie. Il n'y a pas de caractéristiques \"aiguës\" dans notre personnalité."}, "3": {"exist": true, "char_ranges": [[451, 559]], "word_ranges": [[71, 92]], "text": "Il y a un critère de temps qui n'est pas respecté, je peux être triste, mais triste n'est pas une dépression"}, "4": {"exist": true, "char_ranges": [[579, 644]], "word_ranges": [[95, 107]], "text": "si c'était quelque chose comme ça, on nous aurait dit le contexte"}, "5": {"exist": false, "char_ranges": [], "word_ranges": [], "text": ""}}} +{"id": 48, "year": 2011, "question_id_specific": 158, "full_question": "Un garçon de 10 ans est amené aux urgences car depuis 2 heures il tourne involontairement son cou vers la droite, associé à une douleur cervicale marquée à chaque tour. La grand-mère nous dit ensuite qu'il vomit depuis hier et qu'elle lui a donné un sirop ; quelle serait l'approche thérapeutique la plus appropriée ?", "full_answer": "La bonne réponse est 1. L'anticholinergique peut être administré par voie intraveineuse et également par voie intramusculaire si je ne me trompe pas. L'apparition d'une dystonie comme effet secondaire est fréquente chez les enfants et c'est l'une des principales raisons pour lesquelles les pédiatres ne prescrivent généralement pas d'antiémétiques aux enfants.", "type": "PÉDIATRIE", "options": {"1": "Injecter un anticholinergique par voie intraveineuse.", "2": "Ne pas traiter jusqu'à ce qu'un électroencéphalogramme ait été effectué dans les prochains jours.", "3": "Ne pas traiter jusqu'à ce que les résultats de la culture du liquide céphalo-rachidien soient connus dans les prochains jours.", "4": "Appelez le psychiatre de garde.", "5": "Administrer un antibiotique par voie intraveineuse après avoir effectué un prélèvement pharyngé."}, "correct_option": 1, "explanations": {"1": {"exist": false, "char_ranges": [], "word_ranges": [], "text": ""}, "2": {"exist": false, "char_ranges": [], "word_ranges": [], "text": ""}, "3": {"exist": false, "char_ranges": [], "word_ranges": [], "text": ""}, "4": {"exist": false, "char_ranges": [], "word_ranges": [], "text": ""}, "5": {"exist": false, "char_ranges": [], "word_ranges": [], "text": ""}}} +{"id": 490, "year": 2020, "question_id_specific": 106, "full_question": "Une femme de 65 ans a été adressée au service des urgences pour de la fièvre et des altérations de ses analyses sanguines : hémoglobine 11,4 g/dL, leucocytes 0,86 x103/μL (neutrophiles 41,9 %, lymphocytes 55,8 %), plaquettes 48,0 x103/μL, fibrinogène 118 mg/dL, D-dimères 20,2 μg/mL. Un examen de la moelle osseuse a été effectué et le patient a été diagnostiqué avec une leucémie aiguë avec t(15;17) dans 60 % des cellules. Laquelle des réponses suivantes est correcte ?", "full_answer": "Il s'agit d'une leucémie promyélocytaire définitive avec t(15;17), et comme vous le savez tous, le traitement est le trioxyde d'arsenic + ATRA. L'option 1 laisse le traitement incomplet, il n'y a que l'ATRA et il doit être traité tant que l'état du patient le permet, qu'il y ait ou non des symptômes. L'option 3 aurait été la bonne il y a trois ans, mais aujourd'hui on n'utilise ni chimiothérapie ni héparine. Et l'option 4 est fausse car la fièvre est due à la leucémie elle-même et la priorité est de mettre en place un traitement spécifique de la leucémie.", "type": "HÉMATOLOGIE", "options": {"1": "Si elle est asymptomatique, l'acide transrétinoïque (ATRA) sera mis en place et une surveillance en hôpital de jour sera recommandée.", "2": "Un traitement au trioxyde d'arsenic, à l'ATRA et une thérapie de soutien seront mis en place.", "3": "Il s'agit d'une leucémie myéloblastique de type M3. Une chimiothérapie et de l'héparine seront donc mises en place pour contrôler la coagulation intravasculaire disséminée.", "4": "Un traitement antibiotique doit être mis en place. Une fois la fièvre disparue, le traitement de la leucémie doit être mis en place.", "5": null}, "correct_option": 2, "explanations": {"1": {"exist": true, "char_ranges": [[144, 301]], "word_ranges": [[22, 52]], "text": "L'option 1 laisse le traitement incomplet, il n'y a que l'ATRA et il doit être traité tant que l'état du patient le permet, qu'il y ait ou non des symptômes."}, "2": {"exist": true, "char_ranges": [[0, 143]], "word_ranges": [[0, 22]], "text": "Il s'agit d'une leucémie promyélocytaire définitive avec t(15;17), et comme vous le savez tous, le traitement est le trioxyde d'arsenic + ATRA."}, "3": {"exist": true, "char_ranges": [[302, 411]], "word_ranges": [[52, 71]], "text": "L'option 3 aurait été la bonne il y a trois ans, mais aujourd'hui on n'utilise ni chimiothérapie ni héparine."}, "4": {"exist": true, "char_ranges": [[415, 561]], "word_ranges": [[72, 99]], "text": "l'option 4 est fausse car la fièvre est due à la leucémie elle-même et la priorité est de mettre en place un traitement spécifique de la leucémie."}, "5": {"exist": false, "char_ranges": [], "word_ranges": [], "text": ""}}} +{"id": 416, "year": 2018, "question_id_specific": 76, "full_question": "Une femme de 78 ans atteinte de démence et institutionnalisée a été amenée par ses soignants pour de fortes douleurs abdominales accompagnées d'une détérioration de l'état général et d'une distension abdominale. Les examens de laboratoire révèlent une leucocytose, un hématocrite élevé, une insuffisance rénale et une acidose métabolique. L'ECG révèle une fibrillation auriculaire. La tomodensitométrie abdominale montre des boucles œdémateuses de l'intestin grêle, avec une accumulation intestinale et portale. Le diagnostic le plus probable est le suivant :", "full_answer": "Il s'agit d'une image classique et l'auteur de la question a pris soin de nous donner suffisamment d'indices pour que nous ne la manquions pas. La lecture de cet énoncé déclenche de multiples sonnettes d'alarme : - Fibrillation auriculaire : dans le mir et à la porte des urgences, toute personne âgée présentant une FA et des douleurs abdominales est, jusqu'à preuve du contraire, une cause emboligène d'ischémie mésentérique. - Douleur abdominale aiguë, SIRS, insuffisance rénale, acidose métabolique (ischémie !), probablement une élévation de l'acide lactique... - CT : La découverte d'un gaz portal est également très indicative d'une ischémie mésentérique, on aurait également pu nous parler d'une pneumatose intestinale. Pourquoi les autres images n'ont-elles pas été prises en compte ? - Ulcère perforé : on nous aurait parlé d'antécédents de prise d'AINS, ou d'antécédents de douleur qui s'améliore avec l'ingestion, de pneumopéritoine à la radiographie thoracique. - Iléus biliaire : il y aurait des antécédents de colique biliaire ou de douleur dans l'hypochondre droit, une aérobilie à l'examen d'imagerie et une dilatation des anses de l'intestin grêle avec une image (ou non) de lithiase biliaire dans l'iléon terminal. - Néoplasme néo-sigmoïdien : dans l'examen d'imagerie, perte de poids, changement dans les habitudes de selles, pneumopéritoine à la radiographie ou au scanner et exploration abdominale péritonitique, et la masse peut même être palpée.", "type": "CHIRURGIE GÉNÉRALE", "options": {"1": "Perforation d'un ulcère gastrique ou duodénal.", "2": "canal biliaire.", "3": "Néoplasie sigmoïdienne obstructive avec perforation.", "4": "Ischémie mésentérique.", "5": null}, "correct_option": 4, "explanations": {"1": {"exist": true, "char_ranges": [[813, 974]], "word_ranges": [[125, 147]], "text": "on nous aurait parlé d'antécédents de prise d'AINS, ou d'antécédents de douleur qui s'améliore avec l'ingestion, de pneumopéritoine à la radiographie thoracique."}, "2": {"exist": true, "char_ranges": [[994, 1233]], "word_ranges": [[151, 189]], "text": "il y aurait des antécédents de colique biliaire ou de douleur dans l'hypochondre droit, une aérobilie à l'examen d'imagerie et une dilatation des anses de l'intestin grêle avec une image (ou non) de lithiase biliaire dans l'iléon terminal."}, "3": {"exist": true, "char_ranges": [[1263, 1469]], "word_ranges": [[193, 223]], "text": "dans l'examen d'imagerie, perte de poids, changement dans les habitudes de selles, pneumopéritoine à la radiographie ou au scanner et exploration abdominale péritonitique, et la masse peut même être palpée."}, "4": {"exist": true, "char_ranges": [[242, 427]], "word_ranges": [[40, 69]], "text": "dans le mir et à la porte des urgences, toute personne âgée présentant une FA et des douleurs abdominales est, jusqu'à preuve du contraire, une cause emboligène d'ischémie mésentérique."}, "5": {"exist": false, "char_ranges": [], "word_ranges": [], "text": ""}}} +{"id": 45, "year": 2011, "question_id_specific": 154, "full_question": "Un garçon de 9 ans, asymptomatique et présentant un souffle innocent, subit un ECG qui révèle un syndrome de Wolf-Parkinson-White. Laquelle des affirmations suivantes est FAUSSE ?", "full_answer": "La bonne réponse est 1. La réponse est également donnée par l'énoncé qui commence par dire que l'enfant est asymptomatique. L'erreur me semble être une méprise car pour l'étude d'un souffle, on ne demande généralement pas un ECG mais une échocardiographie. L'insuffisance cardiaque est une complication du syndrome de Wolf-Parkinson-White et rarement une forme initiale de la maladie.", "type": "PÉDIATRIE", "options": {"1": "Nécessite un traitement pour l'insuffisance cardiaque.", "2": "Une tachycardie supraventriculaire paroxystique peut survenir.", "3": "L'échographie 2D-Doppler permet d'exclure toute association avec la maladie d'Ebstein.", "4": "Un ECG de 24 heures (Holter) et une ergométrie doivent être réalisés.", "5": "Chez certains patients, l'ablation par radiofréquence de la voie accessoire est l'approche thérapeutique."}, "correct_option": 1, "explanations": {"1": {"exist": true, "char_ranges": [[35, 123]], "word_ranges": [[7, 20]], "text": "est également donnée par l'énoncé qui commence par dire que l'enfant est asymptomatique."}, "2": {"exist": false, "char_ranges": [], "word_ranges": [], "text": ""}, "3": {"exist": false, "char_ranges": [], "word_ranges": [], "text": ""}, "4": {"exist": false, "char_ranges": [], "word_ranges": [], "text": ""}, "5": {"exist": false, "char_ranges": [], "word_ranges": [], "text": ""}}} +{"id": 336, "year": 2016, "question_id_specific": 31, "full_question": "Une femme de 20 ans présentant une tumeur ovarienne kystique solide de 15 cm détectée par échographie après avoir présenté des symptômes abdominaux non spécifiques. Lors de l'étude histopathologique de l'échantillon correspondant, des dents, des poils, des zones d'épithélium intestinal, des zones d'épithélium malpighien (15 %) et d'épithélium bronchique, ainsi que des éléments neuroectodermiques et embryonnaires ont été trouvés dans plusieurs des préparations histologiques. En ce qui concerne ce cas, veuillez indiquer le bon diagnostic :", "full_answer": "Le tératome kystique mature est une tumeur ovarienne bénigne fréquente chez la femme adulte. Les tissus qui composent la tumeur sont bien différenciés (de type mature ou adulte) ; outre les structures cutanées, de nombreux autres tissus sont visibles, en particulier dans un épaississement ou un éperon qui s'émince dans la cavité, dans laquelle on trouve souvent des dents, du cartilage et de l'os.", "type": "GYNÉCOLOGIE ET OBSTÉTRIQUE", "options": {"1": "Tératocarcinome.", "2": "Tératome immature.", "3": "Tératome kystique mature.", "4": "Dysgerminome.", "5": null}, "correct_option": 3, "explanations": {"1": {"exist": false, "char_ranges": [], "word_ranges": [], "text": ""}, "2": {"exist": false, "char_ranges": [], "word_ranges": [], "text": ""}, "3": {"exist": true, "char_ranges": [[0, 399]], "word_ranges": [[0, 64]], "text": "Le tératome kystique mature est une tumeur ovarienne bénigne fréquente chez la femme adulte. Les tissus qui composent la tumeur sont bien différenciés (de type mature ou adulte) ; outre les structures cutanées, de nombreux autres tissus sont visibles, en particulier dans un épaississement ou un éperon qui s'émince dans la cavité, dans laquelle on trouve souvent des dents, du cartilage et de l'os."}, "4": {"exist": false, "char_ranges": [], "word_ranges": [], "text": ""}, "5": {"exist": false, "char_ranges": [], "word_ranges": [], "text": ""}}} +{"id": 609, "year": 2022, "question_id_specific": 115, "full_question": "Un homme de 27 ans, sportif régulier, a signalé une douleur à la jambe droite après une course continue. Il a consulté un kinésithérapeute à plusieurs reprises, qui a diagnostiqué une surcharge du mollet. Plusieurs mois se sont écoulés, son état ne s'est pas amélioré et il signale une douleur intense après l'activité physique, qui s'estompe au repos dans les heures qui suivent l'exercice. Quel test peut aider à établir le diagnostic ?", "full_answer": "Le syndrome des loges induit par l'exercice est une affection de la jambe induite par l'exercice. Il se caractérise par une ischémie réversible des muscles d'un compartiment musculaire. Le diagnostic est posé en mesurant la pression des compartiments au repos, pendant l'exercice et après l'exercice (réponse 2 correcte). Le traitement consiste généralement en une fasciotomie des compartiments touchés. Bien que l'IRM ne soit pas très utile pour établir le diagnostic, elle peut aider au diagnostic différentiel.", "type": "TRAUMATOLOGIE", "options": {"1": "Tomographie par émission de positons au 18 FDG.", "2": "Détermination de la pression du compartiment aval immédiatement après l'activité.", "3": "Échographie Doppler pour exclure un trouble circulatoire du membre inférieur.", "4": "Spectrométrie par résonance magnétique.", "5": null}, "correct_option": 2, "explanations": {"1": {"exist": false, "char_ranges": [], "word_ranges": [], "text": ""}, "2": {"exist": true, "char_ranges": [[0, 321]], "word_ranges": [[0, 48]], "text": "Le syndrome des loges induit par l'exercice est une affection de la jambe induite par l'exercice. Il se caractérise par une ischémie réversible des muscles d'un compartiment musculaire. Le diagnostic est posé en mesurant la pression des compartiments au repos, pendant l'exercice et après l'exercice (réponse 2 correcte)."}, "3": {"exist": false, "char_ranges": [], "word_ranges": [], "text": ""}, "4": {"exist": true, "char_ranges": [[322, 513]], "word_ranges": [[48, 76]], "text": "Le traitement consiste généralement en une fasciotomie des compartiments touchés. Bien que l'IRM ne soit pas très utile pour établir le diagnostic, elle peut aider au diagnostic différentiel."}, "5": {"exist": false, "char_ranges": [], "word_ranges": [], "text": ""}}} +{"id": 389, "year": 2016, "question_id_specific": 235, "full_question": "Un nourrisson de 3 mois, correctement vacciné pour son âge, qui, après deux semaines de rhinorrhée, d'éternuements et de toux, est admis en raison de l'intensification des quintes de toux, avec cyanose à la fin de celles-ci, se terminant par une inspiration profonde ou un coq inspiratoire, nécessitant une stimulation, une aspiration des sécrétions et de l'oxygène pour s'en remettre. Selon leur suspicion diagnostique et en ce qui concerne le vaccin contre cette maladie, toutes les réponses sont vraies SAUF :", "full_answer": "On nous interroge sur le vaccin contre la coqueluche. L'option 1 est correcte, il s'agit du vaccin diphtérie-tétanos-coqueluche. L'option 4 est également correcte, car il s'agit de micro-organismes inactivés. Il ne reste plus que le doute entre les options 2 et 3... L'option 3 est celle qui \"n'est pas vraie pour toutes celles qui nous sont proposées\" : l'immunité dure environ 10 ans après la dernière dose.", "type": "PÉDIATRIE", "options": {"1": "En Espagne, le vaccin est administré en association avec les vaccins contre la diphtérie et le tétanos.", "2": "Les adultes qui seront en contact avec des nourrissons de moins de 6 mois doivent être vaccinés.", "3": "L'immunité naturelle et l'immunité vaccinale sont maintenues à vie.", "4": "La vaccination est préparée à partir de micro-organismes tués.", "5": null}, "correct_option": 3, "explanations": {"1": {"exist": true, "char_ranges": [[0, 128]], "word_ranges": [[0, 18]], "text": "On nous interroge sur le vaccin contre la coqueluche. L'option 1 est correcte, il s'agit du vaccin diphtérie-tétanos-coqueluche."}, "2": {"exist": false, "char_ranges": [], "word_ranges": [], "text": ""}, "3": {"exist": true, "char_ranges": [[267, 409]], "word_ranges": [[42, 67]], "text": "L'option 3 est celle qui \"n'est pas vraie pour toutes celles qui nous sont proposées\" : l'immunité dure environ 10 ans après la dernière dose."}, "4": {"exist": true, "char_ranges": [[129, 208]], "word_ranges": [[18, 29]], "text": "L'option 4 est également correcte, car il s'agit de micro-organismes inactivés."}, "5": {"exist": false, "char_ranges": [], "word_ranges": [], "text": ""}}} +{"id": 201, "year": 2013, "question_id_specific": 76, "full_question": "Un patient présentant des problèmes de compréhension du langage parlé et écrit, une incapacité à nommer des objets et à répéter des mots qui lui sont adressés, un discours fluide incompréhensible avec des paraphasies sémantiques et phonémiques. Il s'agit d'un :", "full_answer": "L'aphasie transcorticale a préservé la répétition. L'aphasie de Broca se caractérise par une compréhension préservée et l'absence d'un discours fluide. L'aphasie de Wernicke se caractérise par un discours fluide (même en excès) avec des paraphasies et des problèmes de compréhension, de nomination et de répétition.", "type": "NEUROLOGIE", "options": {"1": "Aphasie globale.", "2": "Aphasie de Wernicke.", "3": "Aphasie de Broca.", "4": "Aphasie transcorticale sensorielle.", "5": "Aphasie motrice transcorticale."}, "correct_option": 2, "explanations": {"1": {"exist": false, "char_ranges": [], "word_ranges": [], "text": ""}, "2": {"exist": true, "char_ranges": [[152, 315]], "word_ranges": [[20, 45]], "text": "L'aphasie de Wernicke se caractérise par un discours fluide (même en excès) avec des paraphasies et des problèmes de compréhension, de nomination et de répétition."}, "3": {"exist": true, "char_ranges": [[51, 151]], "word_ranges": [[6, 20]], "text": "L'aphasie de Broca se caractérise par une compréhension préservée et l'absence d'un discours fluide."}, "4": {"exist": true, "char_ranges": [[0, 50]], "word_ranges": [[0, 6]], "text": "L'aphasie transcorticale a préservé la répétition."}, "5": {"exist": true, "char_ranges": [[0, 50]], "word_ranges": [[0, 6]], "text": "L'aphasie transcorticale a préservé la répétition."}}} +{"id": 345, "year": 2016, "question_id_specific": 159, "full_question": "Une femme de 69 ans se présente à la clinique en se plaignant de saignements génitaux depuis plusieurs mois. Elle refuse le traitement hormonal substitutif et l'anticoagulation. Elle a fourni une cytologie cervicovaginale normale. L'examen physique général et génital n'a rien révélé d'intéressant. IMC de 38 kg/m2. Indiquez l'approche la plus appropriée :", "full_answer": "On nous présente un cas de métrorragie post-ménopausique. Dans un tel cas, il faut exclure une néoplasie endométriale, donc une biopsie de l'endomètre serait indiquée, soit par canule de Cornier si possible, soit par hystéroscopie. Une biopsie cervicale n'a pas d'intérêt dans ce cas puisque la cytologie est normale et que l'évaluation hormonale ne sera pas utile pour le diagnostic de néoplasie endométriale. Enfin, la prescription de progestérone cyclique n'est pas indiquée chez les femmes ménopausées.", "type": "GYNÉCOLOGIE ET OBSTÉTRIQUE", "options": {"1": "Prescrire de la progestérone cyclique.", "2": "Biopsie de l'endomètre.", "3": "Biopsies cervicales aléatoires.", "4": "Évaluation hormonale avec FSH, LH et œstradiol.", "5": null}, "correct_option": 2, "explanations": {"1": {"exist": true, "char_ranges": [[418, 506]], "word_ranges": [[64, 76]], "text": "la prescription de progestérone cyclique n'est pas indiquée chez les femmes ménopausées."}, "2": {"exist": true, "char_ranges": [[0, 118]], "word_ranges": [[0, 18]], "text": "On nous présente un cas de métrorragie post-ménopausique. Dans un tel cas, il faut exclure une néoplasie endométriale,"}, "3": {"exist": true, "char_ranges": [[232, 316]], "word_ranges": [[35, 49]], "text": "Une biopsie cervicale n'a pas d'intérêt dans ce cas puisque la cytologie est normale"}, "4": {"exist": true, "char_ranges": [[324, 410]], "word_ranges": [[51, 63]], "text": "l'évaluation hormonale ne sera pas utile pour le diagnostic de néoplasie endométriale."}, "5": {"exist": false, "char_ranges": [], "word_ranges": [], "text": ""}}} +{"id": 166, "year": 2013, "question_id_specific": 82, "full_question": "Une femme de 45 ans présente une hypertension artérielle (190/120 mmHg) accompagnée d'un taux de K 2,5 mEq/l. Une échographie abdominale a montré une sténose des deux artères rénales. Indiquez quel traitement est contre-indiqué :", "full_answer": "Si les artères rénales sont sténosées, le sang a du mal à atteindre les reins. En d'autres termes, les deux sont mal perfusés et ne peuvent créer le gradient de pression nécessaire à la filtration que par la contraction de l'artériole efférente. L'administration d'un inhibiteur de l'ECA entraîne le relâchement de cette artériole et la diminution du taux de filtration glomérulaire. Et si aucun des deux reins ne filtre, parce qu'ils présentent tous deux une sténose artérielle, nous avons un problème...", "type": "CARDIOLOGIE ET CHIRURGIE CARDIOVASCULAIRE", "options": {"1": "Enalapril.", "2": "Propanolol.", "3": "Amiloride.", "4": "Prazosine.", "5": "Amlodipine."}, "correct_option": 1, "explanations": {"1": {"exist": true, "char_ranges": [[246, 383]], "word_ranges": [[42, 61]], "text": "L'administration d'un inhibiteur de l'ECA entraîne le relâchement de cette artériole et la diminution du taux de filtration glomérulaire."}, "2": {"exist": false, "char_ranges": [], "word_ranges": [], "text": ""}, "3": {"exist": false, "char_ranges": [], "word_ranges": [], "text": ""}, "4": {"exist": false, "char_ranges": [], "word_ranges": [], "text": ""}, "5": {"exist": false, "char_ranges": [], "word_ranges": [], "text": ""}}} +{"id": 481, "year": 2020, "question_id_specific": 171, "full_question": "Une femme de 67 ans ayant des antécédents de dyslipidémie se présente aux urgences avec une dysurie et une pollakiurie suivies de fièvre, de frissons et d'une détérioration de l'état général. À son arrivée, elle semble sévère, tachycarde, tachypnéique, avec une pression artérielle de 60/40 mmHg et une température de 39°C. Laquelle des mesures suivantes ne ferait PAS partie de la prise en charge INITIALE ?", "full_answer": "Patient présentant fièvre+tachypnée+CF>100 = 3 critères de SIRS (Systemic Inflammatory Response Syndrome). Rappel : le sepsis est un SIRS dû à une infection, qui entraîne une diminution du RVS, ce qui se traduit par une diminution de la précharge et du volume systolique conduisant à une hypotension. Les premières mesures thérapeutiques sont l'administration d'antibiotiques empiriques à large spectre et la substitution liquidienne (réponse 4) pour tenter de compenser cette diminution de la précharge. Mais lorsque cela est insuffisant, il faut essayer de corriger la diminution du RVP en utilisant des médicaments vasoactifs [2]. Mais le médicament vasoactif de choix dans le cadre d'un choc septique est la noradrénaline ([3], [4]) (réponse 1 FAUX) Un minimum de deux hémocultures est obligatoire (réponse 3), pour le diagnostic de la bactériémie et pour faciliter une antibiothérapie ciblée.[D'autre part, il est intéressant de déterminer le taux d'acide lactique dans le sang pour deux raisons : il est actuellement considéré comme le meilleur marqueur de l'hypoperfusion/hypoxie tissulaire et permet de prédire la réponse au traitement [2] (réponse 2).", "type": "SOINS INTENSIFS", "options": {"1": "Perfusion intraveineuse de dobutamine.", "2": "Mesure du lactate sérique.", "3": "Extraction d'hémocultures.", "4": "Gestion des fluides.", "5": null}, "correct_option": 1, "explanations": {"1": {"exist": true, "char_ranges": [[639, 753]], "word_ranges": [[94, 113]], "text": "le médicament vasoactif de choix dans le cadre d'un choc septique est la noradrénaline ([3], [4]) (réponse 1 FAUX)"}, "2": {"exist": true, "char_ranges": [[898, 1160]], "word_ranges": [[133, 173]], "text": "ciblée.[D'autre part, il est intéressant de déterminer le taux d'acide lactique dans le sang pour deux raisons : il est actuellement considéré comme le meilleur marqueur de l'hypoperfusion/hypoxie tissulaire et permet de prédire la réponse au traitement [2] (réponse 2)."}, "3": {"exist": true, "char_ranges": [[754, 897]], "word_ranges": [[113, 134]], "text": "Un minimum de deux hémocultures est obligatoire (réponse 3), pour le diagnostic de la bactériémie et pour faciliter une antibiothérapie ciblée.[D'autre"}, "4": {"exist": true, "char_ranges": [[116, 504]], "word_ranges": [[14, 73]], "text": "le sepsis est un SIRS dû à une infection, qui entraîne une diminution du RVS, ce qui se traduit par une diminution de la précharge et du volume systolique conduisant à une hypotension. Les premières mesures thérapeutiques sont l'administration d'antibiotiques empiriques à large spectre et la substitution liquidienne (réponse 4) pour tenter de compenser cette diminution de la précharge."}, "5": {"exist": false, "char_ranges": [], "word_ranges": [], "text": ""}}} +{"id": 612, "year": 2022, "question_id_specific": 116, "full_question": "Une femme de 95 ans vivant dans une maison de retraite, indépendante pour les activités de base de la vie quotidienne, sort se promener dans le jardin. Elle a des antécédents d'hypertension, de dyslipidémie, d'ostéoporose et de troubles cognitifs légers. Elle fait une chute en se levant la nuit pour aller aux toilettes. La radiographie montre une fracture sous-capitale déplacée de la hanche droite. Quel est le traitement recommandé ?", "full_answer": "Réponse correcte 1 : Étant donné qu'il s'agit d'une fracture déplacée de la hanche sous-capitale, le traitement chirurgical à envisager est le remplacement de l'articulation de la hanche, étant donné que la blessure décrite présente une forte probabilité de nécrose de la tête fémorale. Les options sont l'arthroplastie totale ou partielle : chez les patients âgés, l'arthroplastie partielle est préférable car il s'agit d'une chirurgie plus courte et moins agressive que l'arthroplastie totale de la hanche. Réponse incorrecte 2 : nous envisagerions cette technique dans les fractures basicervicales et les lésions de la masse trochantérienne, et non dans les fractures sous-capitales. Réponse incorrecte 3 : nous envisagerions cette technique dans les fractures sous-capitales non déplacées chez les jeunes patients. Réponse incorrecte 4 : nous n'envisagerions un traitement non chirurgical que chez un patient dont l'état général de base est très médiocre : patients alités dont l'espérance de vie est faible.", "type": "TRAUMATOLOGIE", "options": {"1": "Hémiarthroplastie de la hanche.", "2": "Fixation du clou trochantérien.", "3": "Fixation par vis canulées.", "4": "Conservateur : la vie en fauteuil roulant.", "5": null}, "correct_option": 1, "explanations": {"1": {"exist": true, "char_ranges": [[0, 508]], "word_ranges": [[0, 76]], "text": "Réponse correcte 1 : Étant donné qu'il s'agit d'une fracture déplacée de la hanche sous-capitale, le traitement chirurgical à envisager est le remplacement de l'articulation de la hanche, étant donné que la blessure décrite présente une forte probabilité de nécrose de la tête fémorale. Les options sont l'arthroplastie totale ou partielle : chez les patients âgés, l'arthroplastie partielle est préférable car il s'agit d'une chirurgie plus courte et moins agressive que l'arthroplastie totale de la hanche."}, "2": {"exist": true, "char_ranges": [[509, 686]], "word_ranges": [[76, 101]], "text": "Réponse incorrecte 2 : nous envisagerions cette technique dans les fractures basicervicales et les lésions de la masse trochantérienne, et non dans les fractures sous-capitales."}, "3": {"exist": true, "char_ranges": [[687, 818]], "word_ranges": [[101, 119]], "text": "Réponse incorrecte 3 : nous envisagerions cette technique dans les fractures sous-capitales non déplacées chez les jeunes patients."}, "4": {"exist": true, "char_ranges": [[819, 1012]], "word_ranges": [[119, 150]], "text": "Réponse incorrecte 4 : nous n'envisagerions un traitement non chirurgical que chez un patient dont l'état général de base est très médiocre : patients alités dont l'espérance de vie est faible."}, "5": {"exist": false, "char_ranges": [], "word_ranges": [], "text": ""}}} +{"id": 585, "year": 2022, "question_id_specific": 74, "full_question": "Primigestation à 34 semaines, présentant une tension artérielle de 165/95 et des céphalées depuis deux jours. Examens de laboratoire : hémoglobine 10,5 g/dL, plaquettes 98 000/mm3, AST 356 UI/L (0-31), ALT 234 UI/L (0-31), LDH 878 UI/L (125-243). Le laboratoire est appelé en raison de la présence de schistocytes dans le frottis de sang périphérique. Elle a reçu la deuxième dose de corticostéroïdes pour la maturation fœtale il y a 24 heures. Lors de l'échographie obstétricale, le poids fœtal estimé est dans le 1er percentile pour l'âge gestationnel et le Doppler de l'artère ombilicale montre une absence de flux en fin de diastole. Lequel des éléments suivants est l'approche clinique la plus appropriée ?", "full_answer": "Ils décrivent le syndrome HELLP. À ces semaines de gestation et avec un fœtus récemment mature (il a déjà reçu deux doses de corticostéroïdes), l'attitude à adopter est l'interruption immédiate de grossesse.", "type": "OBSTÉTRIQUE ET GYNÉCOLOGIE", "options": {"1": "Attente vigilante avec des médicaments antihypertenseurs et du sulfate de magnésium jusqu'à ce que les plaquettes maternelles s'améliorent.", "2": "Attente vigilante avec traitement antihypertenseur à domicile et contrôles toutes les 48 heures.", "3": "Fin de la gestation lorsque la maturation des poumons du fœtus est terminée.", "4": "Interruption immédiate de la grossesse.", "5": null}, "correct_option": 4, "explanations": {"1": {"exist": false, "char_ranges": [], "word_ranges": [], "text": ""}, "2": {"exist": false, "char_ranges": [], "word_ranges": [], "text": ""}, "3": {"exist": false, "char_ranges": [], "word_ranges": [], "text": ""}, "4": {"exist": true, "char_ranges": [[0, 207]], "word_ranges": [[0, 32]], "text": "Ils décrivent le syndrome HELLP. À ces semaines de gestation et avec un fœtus récemment mature (il a déjà reçu deux doses de corticostéroïdes), l'attitude à adopter est l'interruption immédiate de grossesse."}, "5": {"exist": false, "char_ranges": [], "word_ranges": [], "text": ""}}} +{"id": 3, "year": 2011, "question_id_specific": 37, "full_question": "Une femme de 76 ans, sans antécédents d'hypertension, consulte pour un ictère indolore et un prurit avec anorexie. Les examens de laboratoire ont révélé une bilirubine de 12 mg/dl (9,5 direct). L'échographie montre une dilatation biliaire intrahépatique et extrahépatique ainsi qu'un nodule hépatique unique de moins de 2 cm situé en périphérie sur la face antérieure du lobe gauche. La tomodensitométrie confirme ces résultats et démontre également la présence d'une masse pancréatique de 3,5 cm. Dans la tête du pancréas. La ponction-aspiration du nodule hépatique est concluante en faveur d'un adénocarcinome. Indiquez la meilleure option thérapeutique :", "full_answer": "Ce cas ne me semble pas si simple. Il s'agit d'un adénocarcinome du pancréas avec métastases, le traitement doit donc être palliatif, les options 1 et 5 sont donc éliminées. En principe, il s'agit d'une patiente en moyennement bon état général, l'idéal serait donc de réaliser une dérivation biliaire permanente et une chimiothérapie, l'option 4 étant l'option que je considère comme la plus correcte.", "type": "DIGESTIF", "options": {"1": "Chimiothérapie/radiothérapie néoadjuvante, conditionnant l'option de chirurgie radicale en cas de réponse initiale.", "2": "Drainage biliaire externe percutané à des fins palliatives avec conversion possible en drainage interne en cas d'intolérance ou de complications.", "3": "Pontage biliaire chirurgical palliatif avec ou sans gastrojéjunostomie prophylactique en fonction des résultats peropératoires.", "4": "Prothèse biliaire métallique par cholangiopancréatographie rétrograde endoscopique avec option de chimiothérapie palliative.", "5": "Duodénopancréatectomie céphalique, avec ablation par radiofréquence percutanée ou alcoolisation de la lésion hépatique."}, "correct_option": 4, "explanations": {"1": {"exist": true, "char_ranges": [[35, 173]], "word_ranges": [[8, 30]], "text": "Il s'agit d'un adénocarcinome du pancréas avec métastases, le traitement doit donc être palliatif, les options 1 et 5 sont donc éliminées."}, "2": {"exist": false, "char_ranges": [], "word_ranges": [], "text": ""}, "3": {"exist": false, "char_ranges": [], "word_ranges": [], "text": ""}, "4": {"exist": true, "char_ranges": [[174, 401]], "word_ranges": [[30, 64]], "text": "En principe, il s'agit d'une patiente en moyennement bon état général, l'idéal serait donc de réaliser une dérivation biliaire permanente et une chimiothérapie, l'option 4 étant l'option que je considère comme la plus correcte."}, "5": {"exist": true, "char_ranges": [[35, 173]], "word_ranges": [[8, 30]], "text": "Il s'agit d'un adénocarcinome du pancréas avec métastases, le traitement doit donc être palliatif, les options 1 et 5 sont donc éliminées."}}} +{"id": 439, "year": 2018, "question_id_specific": 96, "full_question": "Femme de 64 ans. Ménopause à 54 ans. Deux grossesses et des accouchements eutociques. Elle ne prend aucun médicament. Elle se présente au service des urgences en signalant des saignements vaginaux depuis deux jours avec une gêne au niveau de l'hypogastre. L'examen clinique du gynécologue ne révèle aucune lésion au niveau des organes génitaux externes, du vagin et du col de l'utérus. L'examen vaginal n'est pas concluant. Lequel des éléments suivants vous semble être la première approche diagnostique la plus appropriée ?", "full_answer": "La cause la plus fréquente des saignements vaginaux chez les femmes ménopausées étant l'atrophie urogénitale, l'existence d'une pathologie endométriale ne peut être exclue. Selon la SEGO : \"Les saignements génitaux anormaux sont le principal signe de suspicion qui doit nous amener à écarter une pathologie néoplasique de l'endomètre, en particulier chez les patientes ménopausées ou celles qui présentent des facteurs de risque associés (...) Compte tenu du signe de suspicion, l'échographie transvaginale, ou à défaut transrectale, permet : d'écarter une pathologie organique (fibromes, polypes) ; de mesurer l'épaisseur de l'endomètre sur une coupe longitudinale. Un seuil de 3 mm est recommandé pour la biopsie de l'endomètre chez les patientes symptomatiques\".", "type": "GYNÉCOLOGIE ET OBSTÉTRIQUE", "options": {"1": "Échographie transvaginale et mesure de l'épaisseur de l'endomètre. En fonction de cette mesure, une biopsie de l'endomètre peut être effectuée ou non.", "2": "La biopsie par aspiration de l'endomètre en clinique. Elle me permet d'obtenir un diagnostic très fiable et elle est peu coûteuse.", "3": "Hystéroscopie avec sédation et biopsie dirigée en cas de lésion.", "4": "J'effectuerais un microbleed, car c'est le test qui me donnerait le diagnostic définitif.", "5": null}, "correct_option": 1, "explanations": {"1": {"exist": true, "char_ranges": [[110, 666]], "word_ranges": [[15, 94]], "text": "l'existence d'une pathologie endométriale ne peut être exclue. Selon la SEGO : \"Les saignements génitaux anormaux sont le principal signe de suspicion qui doit nous amener à écarter une pathologie néoplasique de l'endomètre, en particulier chez les patientes ménopausées ou celles qui présentent des facteurs de risque associés (...) Compte tenu du signe de suspicion, l'échographie transvaginale, ou à défaut transrectale, permet : d'écarter une pathologie organique (fibromes, polypes) ; de mesurer l'épaisseur de l'endomètre sur une coupe longitudinale."}, "2": {"exist": false, "char_ranges": [], "word_ranges": [], "text": ""}, "3": {"exist": false, "char_ranges": [], "word_ranges": [], "text": ""}, "4": {"exist": false, "char_ranges": [], "word_ranges": [], "text": ""}, "5": {"exist": false, "char_ranges": [], "word_ranges": [], "text": ""}}} +{"id": 192, "year": 2013, "question_id_specific": 160, "full_question": "Les parents d'un nourrisson de 5 mois viennent parce que leur enfant, qui fréquente la crèche, présente depuis 3 jours une infection des voies respiratoires supérieures, des selles liquides 4 fois par jour, des vomissements alimentaires occasionnels et une température axillaire de 38,3ºC. Il est nourri au sein avec une alimentation mixte. À l'examen, le nourrisson est en bon état général, bien nourri et hydraté et sa respiration est eupnéique ; son poids est de 4 730 g et a diminué de 70 g par rapport à la semaine précédente. L'abdomen est souple et dépressible, sans masses ni mégalithes, et la fontanelle est normotendue. Hormis la présence d'une rhinorrhée aqueuse, le reste de l'examen des organes et appareils est normal. Parmi les affirmations suivantes, cochez la réponse que vous considérez comme CORRECTE :", "full_answer": "Le bébé n'est pas déshydraté et est en bon état général. Il s'agit d'éviter la déshydratation en reconstituant ce qui est perdu, en évitant autant que possible les changements radicaux dans les habitudes alimentaires. Le reste des réponses aborde de nombreux \"mythes\" et coutumes entourant la gastro-entérite. Il n'existe pas non plus de preuves d'intolérance au lactose (irritation périanale, selles mousseuses et acides, etc.) pour recommander le lait sans lactose.", "type": "PÉDIATRIE", "options": {"1": "Il convient d'estimer les pertes, de recommander un jeûne de 4 heures et de procéder à une réhydratation à l'aide d'une solution de réhydratation orale pendant cette période.", "2": "Il est conseillé d'introduire des céréales de riz pour leur effet astringent.", "3": "Une culture de selles doit être effectuée dès que possible pour exclure une origine bactérienne.", "4": "L'utilisation d'un lait maternisé sans lactose doit être recommandée.", "5": "Il est conseillé de remplacer les pertes après chaque selle par une solution de réhydratation orale et de continuer à s'alimenter normalement."}, "correct_option": 5, "explanations": {"1": {"exist": true, "char_ranges": [[218, 309]], "word_ranges": [[34, 47]], "text": "Le reste des réponses aborde de nombreux \"mythes\" et coutumes entourant la gastro-entérite."}, "2": {"exist": true, "char_ranges": [[218, 309]], "word_ranges": [[34, 47]], "text": "Le reste des réponses aborde de nombreux \"mythes\" et coutumes entourant la gastro-entérite."}, "3": {"exist": true, "char_ranges": [[218, 309]], "word_ranges": [[34, 47]], "text": "Le reste des réponses aborde de nombreux \"mythes\" et coutumes entourant la gastro-entérite."}, "4": {"exist": true, "char_ranges": [[310, 467]], "word_ranges": [[47, 70]], "text": "Il n'existe pas non plus de preuves d'intolérance au lactose (irritation périanale, selles mousseuses et acides, etc.) pour recommander le lait sans lactose."}, "5": {"exist": true, "char_ranges": [[0, 217]], "word_ranges": [[0, 34]], "text": "Le bébé n'est pas déshydraté et est en bon état général. Il s'agit d'éviter la déshydratation en reconstituant ce qui est perdu, en évitant autant que possible les changements radicaux dans les habitudes alimentaires."}}} +{"id": 261, "year": 2014, "question_id_specific": 100, "full_question": "Une femme de 63 ans s'est présentée aux urgences pour des céphalées sévères avec des signes d'irritation méningée, des troubles visuels bilatéraux et une ophtalmoplégie. Un scanner a montré une lésion de 2 cm occupant l'espace dans la selle turcique, compatible avec un adénome hypophysaire avec des signes d'hémorragie intratumorale, avec déviation de la tige hypophysaire et compression du tissu glandulaire. Indiquez laquelle des réponses suivantes est FAUSSE :", "full_answer": "Question très difficile sur l'apoplexie hypophysaire. Il s'agit d'un sujet marginal dans le MIR et la réponse est complexe. En éliminant les réponses, il ne reste plus qu'à intervenir (réponse 4) ou à ne pas intervenir (réponse 2). Chez les patients présentant une instabilité hémodynamique, une diminution de l'état de conscience, une baisse de l'acuité visuelle et des défauts importants du champ visuel, une décompression chirurgicale est recommandée au cours de la première semaine suivant l'apparition des symptômes. Je pense que la réponse est 4, bien qu'elle soit étrangement formulée. Je ne sais pas si le neurochirurgien soutiendra ma réponse.", "type": "ENDOCRINOLOGIE", "options": {"1": "Le diagnostic suspecté est l'apoplexie hypophysaire.", "2": "Un traitement par corticostéroïdes à forte dose doit être instauré et l'évolution doit être surveillée, car ce traitement peut réduire le volume de la lésion et éviter une intervention.", "3": "Un traitement par glucocorticoïdes doit être envisagé pour éviter une insuffisance surrénalienne secondaire qui compromettrait le pronostic vital du patient.", "4": "La présence d'une ophtalmoplégie et de défauts visuels sont des indications pour une intervention rapide par décompression chirurgicale urgente.", "5": "Après la disparition des symptômes aigus, un panhypopituitarisme se développe souvent."}, "correct_option": 4, "explanations": {"1": {"exist": false, "char_ranges": [], "word_ranges": [], "text": ""}, "2": {"exist": false, "char_ranges": [], "word_ranges": [], "text": ""}, "3": {"exist": false, "char_ranges": [], "word_ranges": [], "text": ""}, "4": {"exist": true, "char_ranges": [[232, 521]], "word_ranges": [[38, 78]], "text": "Chez les patients présentant une instabilité hémodynamique, une diminution de l'état de conscience, une baisse de l'acuité visuelle et des défauts importants du champ visuel, une décompression chirurgicale est recommandée au cours de la première semaine suivant l'apparition des symptômes."}, "5": {"exist": false, "char_ranges": [], "word_ranges": [], "text": ""}}} +{"id": 289, "year": 2016, "question_id_specific": 65, "full_question": "Veuillez choisir parmi les complications suivantes celle qui est la plus probable chez un patient opéré d'un ulcère gastrique il y a 20 ans par antrectomie et gastrojéjunostomie (Billroth II) et qui se présente à votre cabinet avec des douleurs abdominales postprandiales, une distension abdominale, une diarrhée et des données analytiques de malabsorption des graisses et de la vitamine B12 :", "full_answer": "L'exposé a décrit les caractéristiques cliniques de ce syndrome. Il s'agit généralement d'une complication tardive (le patient a été opéré il y a 20 ans pour un Bilroth II, rappelons qu'il s'agit d'une gastrectomie partielle, d'une fermeture du moignon gastrique et d'une anastomose de la première anse jéjunale au moignon), elle s'accompagne de douleurs postprandiales, d'une distension abdominale. Il s'agit d'une sténose de l'anse afférente, de sorte que la bile et les sécrétions pancréatiques ne se mélangent pas au bol alimentaire, s'accumulent, provoquent des douleurs qui sont soulagées par les vomissements, l'absorption des graisses n'est pas possible (absence de lipase pancréatique), d'où une diarrhée et, si elle est associée à une prolifération bactérienne, une malabsorption de la vitamine B12. S'il s'agissait d'un adénocarcinome gastrique, on aurait parlé de syndrome constitutionnel, de manifestations extradigestives (ganglion de sœur Maria José, tumeur de Krukenberg, ganglion de Virchow). Il ne s'agit pas d'un dumping car il ne mentionne pas de symptômes végétatifs (palpitations, bouffées vasomotrices, etc.). La gastropathie par reflux biliaire est un diagnostic endoscopique et n'est pas associée à une diarrhée ou à une malabsorption.", "type": "CHIRURGIE GÉNÉRALE", "options": {"1": "Gastropathie par reflux biliaire.", "2": "Adénocarcinome gastrique.", "3": "Syndrome de vidange gastrique rapide (dumping).", "4": "Syndrome de l'anse afférente avec surcroissance bactérienne.", "5": null}, "correct_option": 4, "explanations": {"1": {"exist": true, "char_ranges": [[1133, 1260]], "word_ranges": [[162, 182]], "text": "La gastropathie par reflux biliaire est un diagnostic endoscopique et n'est pas associée à une diarrhée ou à une malabsorption."}, "2": {"exist": true, "char_ranges": [[810, 1009]], "word_ranges": [[119, 144]], "text": "S'il s'agissait d'un adénocarcinome gastrique, on aurait parlé de syndrome constitutionnel, de manifestations extradigestives (ganglion de sœur Maria José, tumeur de Krukenberg, ganglion de Virchow)."}, "3": {"exist": true, "char_ranges": [[1010, 1132]], "word_ranges": [[144, 162]], "text": "Il ne s'agit pas d'un dumping car il ne mentionne pas de symptômes végétatifs (palpitations, bouffées vasomotrices, etc.)."}, "4": {"exist": true, "char_ranges": [[400, 809]], "word_ranges": [[58, 119]], "text": "Il s'agit d'une sténose de l'anse afférente, de sorte que la bile et les sécrétions pancréatiques ne se mélangent pas au bol alimentaire, s'accumulent, provoquent des douleurs qui sont soulagées par les vomissements, l'absorption des graisses n'est pas possible (absence de lipase pancréatique), d'où une diarrhée et, si elle est associée à une prolifération bactérienne, une malabsorption de la vitamine B12."}, "5": {"exist": false, "char_ranges": [], "word_ranges": [], "text": ""}}} +{"id": 366, "year": 2016, "question_id_specific": 106, "full_question": "Un homme de 32 ans, originaire du Cameroun, a consulté pour fièvre, toux et douleur dans l'hémithorax gauche depuis un mois. Il avait pris de l'amoxicilline-acide clavulanique pendant une semaine sans amélioration de ses symptômes. Une analyse de sang a révélé un taux de globules blancs de 8000/microL et une hémoglobine de 12,8 g/dL. La radiographie du thorax montre un épanchement pleural gauche localisé occupant un tiers de l'hémithorax. Une thoracentèse montre un liquide jaunâtre avec les caractéristiques suivantes : globules rouges 2000/µL, leucocytes 2500/µL avec 90% de lymphocytes, protéines 4,9 g/dL, lactate déshydrogénase 550 VIL, glucose 67 mg/dL et pas de cellules malignes à la cytologie. Lequel des tests suivants serait le plus utile pour diagnostiquer la cause de l'épanchement pleural ?", "full_answer": "La suspicion est très élevée pour un épanchement pleural tuberculeux, chez un jeune patient provenant d'une région où le taux de tuberculose est élevé, un exsudat lymphocytaire évoquant une tuberculose, un lymphome ou une tumeur. Une tumeur est peu probable en raison de l'âge et d'une cytologie négative. Il pourrait s'agir d'un lymphome, mais il n'y a pas de preuve de pathologie ganglionnaire associée. Un taux élevé d'adénosine désaminase dans la PL indiquerait avec une grande certitude une tuberculose pleurale, bien qu'il soit conseillé de confirmer le diagnostic par une biopsie pleurale, étant donné qu'il peut également se produire dans les lymphomes et les empyèmes d'autre origine.", "type": "PNEUMOLOGIE ET CHIRURGIE THORACIQUE", "options": {"1": "Une tomodensitométrie thoracique (CT scan).", "2": "Mesure du pH du liquide pleural.", "3": "Mesure de l'adénosine désaminase du liquide pleural.", "4": "Test de tuberculine.", "5": null}, "correct_option": 3, "explanations": {"1": {"exist": false, "char_ranges": [], "word_ranges": [], "text": ""}, "2": {"exist": false, "char_ranges": [], "word_ranges": [], "text": ""}, "3": {"exist": true, "char_ranges": [[406, 693]], "word_ranges": [[64, 107]], "text": "Un taux élevé d'adénosine désaminase dans la PL indiquerait avec une grande certitude une tuberculose pleurale, bien qu'il soit conseillé de confirmer le diagnostic par une biopsie pleurale, étant donné qu'il peut également se produire dans les lymphomes et les empyèmes d'autre origine."}, "4": {"exist": false, "char_ranges": [], "word_ranges": [], "text": ""}, "5": {"exist": false, "char_ranges": [], "word_ranges": [], "text": ""}}} +{"id": 544, "year": 2022, "question_id_specific": 40, "full_question": "Une jeune femme de 16 ans souffre d'une rhinoconjonctivite allergique due à une sensibilisation aux pollens de graminées depuis l'âge de 10 ans, dont les symptômes ne sont pas totalement maîtrisés pendant la saison pollinique par des antihistaminiques oraux et des corticostéroïdes topiques par voie nasale. Quelle option de traitement étiologique serait indiquée ?", "full_answer": "Actuellement, le seul traitement efficace capable de modifier l'évolution naturelle de l'allergie respiratoire est l'immunothérapie spécifique à l'allergène pendant au moins trois années consécutives. Chez ce patient, le traitement conventionnel n'ayant pas fonctionné, l'immunothérapie spécifique à l'allergène peut être utilisée. Il est vrai qu'il n'est pas nécessaire d'attendre 10 ans pour la commencer puisqu'elle est indiquée et que son utilisation est autorisée s'il y a une pertinence clinique chez les patients de plus de 5 ans. L'omalizumab est un traitement biologique qui ne serait indiqué qu'en cas d'asthme bronchique modéré sévère secondaire à une allergie respiratoire.", "type": "ALLERGOLOGIE", "options": {"1": "Immunothérapie spécifique par voie sublinguale ou sous-cutanée pendant une période minimale de 3 ans.", "2": "Immunothérapie spécifique par voie sublinguale ou sous-cutanée pendant une période maximale d'un an.", "3": "Omalizumab par voie sous-cutanée toutes les 4 semaines pendant 1 an maximum.", "4": "Omalizumab par voie sous-cutanée toutes les 4 semaines pendant une période minimale de 6 mois.", "5": null}, "correct_option": 1, "explanations": {"1": {"exist": true, "char_ranges": [[0, 200]], "word_ranges": [[0, 24]], "text": "Actuellement, le seul traitement efficace capable de modifier l'évolution naturelle de l'allergie respiratoire est l'immunothérapie spécifique à l'allergène pendant au moins trois années consécutives."}, "2": {"exist": true, "char_ranges": [[0, 200]], "word_ranges": [[0, 24]], "text": "Actuellement, le seul traitement efficace capable de modifier l'évolution naturelle de l'allergie respiratoire est l'immunothérapie spécifique à l'allergène pendant au moins trois années consécutives."}, "3": {"exist": true, "char_ranges": [[538, 685]], "word_ranges": [[76, 96]], "text": "L'omalizumab est un traitement biologique qui ne serait indiqué qu'en cas d'asthme bronchique modéré sévère secondaire à une allergie respiratoire."}, "4": {"exist": true, "char_ranges": [[538, 685]], "word_ranges": [[76, 96]], "text": "L'omalizumab est un traitement biologique qui ne serait indiqué qu'en cas d'asthme bronchique modéré sévère secondaire à une allergie respiratoire."}, "5": {"exist": false, "char_ranges": [], "word_ranges": [], "text": ""}}} +{"id": 533, "year": 2021, "question_id_specific": 179, "full_question": "Une femme de 32 ans atteinte du syndrome de néoplasie endocrinienne multiple de type 2A (MEN-2A) et porteuse d'une mutation RET. Une échographie du cou a permis d'identifier un nodule hypoéchogène de 6 mm avec des calcifications à l'intérieur. Il a été décidé de procéder à une thyroïdectomie totale et à un curage ganglionnaire cervical. L'étude macroscopique a permis d'identifier trois nodules au total, deux dans le lobe droit mesurant 5 et 6 mm, et un dans le lobe gauche mesurant 4 mm. Au microscope, les trois lésions consistent en une prolifération uniforme de cellules arrondies disposées de façon solide et accompagnées de calcifications et de dépôts amyloïdes. Les noyaux ne sont pas clairs et ne présentent pas de fentes ou de pseudo-inclusions. La coloration immunohistochimique de la synaptophysine est positive. Quel est le diagnostic pathologique des lésions identifiées lors de la thyroïdectomie totale ?", "full_answer": "RET typique de la thyroïde médullaire (cellules C, calcitonine élevée, hérédité MEN-2nd, question typique MIR).", "type": "ONCOLOGIE", "options": {"1": "Carcinome multifocal de la moelle épinière.", "2": "Carcinome papillaire multifocal.", "3": "Carcinome folliculaire.", "4": "Hyperplasie des cellules parafolliculaires.", "5": null}, "correct_option": 1, "explanations": {"1": {"exist": true, "char_ranges": [[0, 111]], "word_ranges": [[0, 15]], "text": "RET typique de la thyroïde médullaire (cellules C, calcitonine élevée, hérédité MEN-2nd, question typique MIR)."}, "2": {"exist": false, "char_ranges": [], "word_ranges": [], "text": ""}, "3": {"exist": false, "char_ranges": [], "word_ranges": [], "text": ""}, "4": {"exist": false, "char_ranges": [], "word_ranges": [], "text": ""}, "5": {"exist": false, "char_ranges": [], "word_ranges": [], "text": ""}}} +{"id": 80, "year": 2012, "question_id_specific": 37, "full_question": "Un homme de 52 ans consulte pour obtenir un deuxième avis sur la nécessité d'une biopsie du foie pour étudier une hypertransaminasémie détectée il y a deux ans lors de tests de routine effectués par la société. Ses antécédents familiaux comprennent le décès de son père d'une cirrhose épatique d'étiologie non identifiée. Il est asymptomatique et a une vie sociale et professionnelle sans restriction. Il nie toute consommation d'alcool. L'examen physique a révélé une pigmentation métallique de la peau et une hépatomégalie minime et non douloureuse. Les examens de laboratoire ont donné les résultats suivants : billirubine, albumine, transaminases AST et ALT, hémogramme et temps de prothrombine normaux ; glycémie 150 mg/dl ; ferritine sérique 950 ; saturation de la transferrine >45% ; test de dépistage du virus hépatotrope négatif. L'échographie abdominale est normale. Il a subi une étude génétique HFE et est homozygote pour la mutation C282Y. Quelle serait la meilleure recommandation sur la base des informations disponibles ?", "full_answer": "Ce patient a déjà un diagnostic d'hémochromatose, en raison d'une étude génétique positive et de données de surcharge en fer (TSI >45% et ferritine élevée). Une biopsie serait utile si le diagnostic est incertain ou si l'on veut évaluer une atteinte hépatique qui n'est pas suspectée par cette analyse. Le traitement de choix est la phlébotomie.", "type": "SYSTÈME DIGESTIF", "options": {"1": "Effectuer une biopsie du foie.", "2": "Réaliser une IRM du foie.", "3": "Initier le traitement par des phlébotomies.", "4": "Commencer le traitement par la desferroxamine.", "5": "Commencer le traitement avec de la vitamine E."}, "correct_option": 3, "explanations": {"1": {"exist": true, "char_ranges": [[157, 302]], "word_ranges": [[25, 49]], "text": "Une biopsie serait utile si le diagnostic est incertain ou si l'on veut évaluer une atteinte hépatique qui n'est pas suspectée par cette analyse."}, "2": {"exist": false, "char_ranges": [], "word_ranges": [], "text": ""}, "3": {"exist": false, "char_ranges": [], "word_ranges": [], "text": ""}, "4": {"exist": false, "char_ranges": [], "word_ranges": [], "text": ""}, "5": {"exist": false, "char_ranges": [], "word_ranges": [], "text": ""}}} +{"id": 394, "year": 2016, "question_id_specific": 225, "full_question": "Mariano est un homme de 53 ans qui se présente à votre cabinet en signalant qu'il se sent très mal depuis quelques années. Il relie son malaise à un comportement qu'il trouve absurde mais qu'il ne peut éviter. Ce comportement, qu'elle a toujours lorsqu'elle rentre du travail, consiste à tourner la poignée de la porte de la salle de bains de l'étage avant de faire quoi que ce soit d'autre, même avant de saluer sa famille. Il a parfois essayé de résister à ce comportement, mais n'a réussi qu'à s'angoisser et à le retarder de quelques minutes. Ce comportement, que Mariano exécute de manière incoercible et automatique, est ce que nous appelons :", "full_answer": "Cette question demande le nom du symptôme et non celui de la maladie, la bonne réponse est donc la compulsion. Selon le DSM-5, une compulsion est un comportement répétitif (par exemple, se laver les mains, ranger, vérifier les objets) ou un acte mental (par exemple, prier, compter, répéter des mots en silence) que le sujet exécute en réponse à une obsession ou selon des règles qui doivent être appliquées de manière rigide. Le but de ces comportements ou de ces actes mentaux est de prévenir ou de réduire l'anxiété ou l'inconfort, ou d'éviter un événement ou une situation redoutés ; cependant, ces comportements ou ces actes mentaux n'ont pas de lien réaliste avec ceux qui sont censés être neutralisés ou prévenus, ou sont manifestement excessifs.", "type": "PSYCHIATRIE", "options": {"1": "Troubles obsessionnels compulsifs.", "2": "Obsession.", "3": "Impulsion.", "4": "Compulsion.", "5": null}, "correct_option": 4, "explanations": {"1": {"exist": false, "char_ranges": [], "word_ranges": [], "text": ""}, "2": {"exist": false, "char_ranges": [], "word_ranges": [], "text": ""}, "3": {"exist": false, "char_ranges": [], "word_ranges": [], "text": ""}, "4": {"exist": true, "char_ranges": [[111, 426]], "word_ranges": [[20, 72]], "text": "Selon le DSM-5, une compulsion est un comportement répétitif (par exemple, se laver les mains, ranger, vérifier les objets) ou un acte mental (par exemple, prier, compter, répéter des mots en silence) que le sujet exécute en réponse à une obsession ou selon des règles qui doivent être appliquées de manière rigide."}, "5": {"exist": false, "char_ranges": [], "word_ranges": [], "text": ""}}} +{"id": 229, "year": 2014, "question_id_specific": 178, "full_question": "Nourrisson de 10 mois, précédemment en bonne santé, présentant une gastro-entérite aiguë d'un jour d'évolution et des signes de déshydratation légère, sans sang ni mucus dans les selles et sans tolérance orale. Quel est le traitement initial de choix dans notre contexte ?", "full_answer": "Le nourrisson présente une GEA aiguë sans signes d'alarme ou de déshydratation sévère. La première étape consisterait à essayer la tolérance orale avec une solution saline de réhydratation orale et à poursuivre l'alimentation normale si elle est bien tolérée. La réponse 1 peut susciter des doutes, du fait que l'énoncé précise que le patient \"ne tolère pas la voie orale\". Dans ce cas, une réhydratation intraveineuse pourrait être débutée... mais il ne serait pas souhaitable de laisser le patient à une diète absolue pendant 8 heures ou à une diète astringente ultérieure, c'est pourquoi je préfère la réponse 2. Les autres traitements n'apportent aucun bénéfice.", "type": "PÉDIATRIE", "options": {"1": "Réhydratation intraveineuse, régime absolu de 8 heures et début de l'alimentation avec un régime astringent.", "2": "Réhydratation avec une solution de réhydratation orale à faible osmolalité (sodium 60-75 mEq/L) et poursuite du régime alimentaire habituel.", "3": "Réhydratation avec une solution de réhydratation orale à faible osmolalité (sodium 60-75 mEq/L), maintien de l'alimentation habituelle et amoxicilline orale pendant 10 jours.", "4": "Réhydratation avec une solution de réhydratation orale à faible osmolalité (sodium 60-75 mEq/L) et début de l'alimentation avec des formules sans lactose.", "5": "Réhydratation avec une solution de réhydratation orale à faible osmolalité (sodium 60-75 mEq/L) en maintenant l'alimentation habituelle et le lopéramide pendant 7 jours."}, "correct_option": 2, "explanations": {"1": {"exist": true, "char_ranges": [[87, 615]], "word_ranges": [[13, 99]], "text": "La première étape consisterait à essayer la tolérance orale avec une solution saline de réhydratation orale et à poursuivre l'alimentation normale si elle est bien tolérée. La réponse 1 peut susciter des doutes, du fait que l'énoncé précise que le patient \"ne tolère pas la voie orale\". Dans ce cas, une réhydratation intraveineuse pourrait être débutée... mais il ne serait pas souhaitable de laisser le patient à une diète absolue pendant 8 heures ou à une diète astringente ultérieure, c'est pourquoi je préfère la réponse 2."}, "2": {"exist": true, "char_ranges": [[87, 615]], "word_ranges": [[13, 99]], "text": "La première étape consisterait à essayer la tolérance orale avec une solution saline de réhydratation orale et à poursuivre l'alimentation normale si elle est bien tolérée. La réponse 1 peut susciter des doutes, du fait que l'énoncé précise que le patient \"ne tolère pas la voie orale\". Dans ce cas, une réhydratation intraveineuse pourrait être débutée... mais il ne serait pas souhaitable de laisser le patient à une diète absolue pendant 8 heures ou à une diète astringente ultérieure, c'est pourquoi je préfère la réponse 2."}, "3": {"exist": true, "char_ranges": [[616, 666]], "word_ranges": [[99, 105]], "text": "Les autres traitements n'apportent aucun bénéfice."}, "4": {"exist": true, "char_ranges": [[616, 666]], "word_ranges": [[99, 105]], "text": "Les autres traitements n'apportent aucun bénéfice."}, "5": {"exist": true, "char_ranges": [[616, 666]], "word_ranges": [[99, 105]], "text": "Les autres traitements n'apportent aucun bénéfice."}}} +{"id": 198, "year": 2013, "question_id_specific": 73, "full_question": "Un homme de 77 ans ayant des antécédents de diabète de type 2 a été diagnostiqué il y a deux ans avec une déficience cognitive légère. À cette époque, un test de Minimental (Folstein) 28/30, abrégé test de Yesavage 14/15, un test sanguin avec TSH et facteurs de maturation normaux ainsi que des sérologies négatives pour le LUES et le VIH et une imagerie par résonance magnétique (IRM) montrant une atrophie corticale diffuse prédominant dans la zone postérieure ont été réalisés. Le patient était accompagné de son épouse, qui a signalé une évolution lente et progressive des troubles de la mémoire (par exemple, confusion avec les doses d'insuline). Il n'y a pas d'hallucinations ni de troubles du comportement. Le Minimental actuel est à 24/30 et le test de Yesavage à 14/15, il n'y a pas de focalité neurologique, de tremblements ou de troubles du tonus ou de la marche à l'examen physique. Il y a deux mois, il s'est présenté au service des urgences après un traumatisme cérébral léger dû à une chute accidentelle (la seule en deux ans) et un scanner a été réalisé, qui n'a apporté aucune nouvelle information par rapport à l'IRM précédente. Quel est le diagnostic le plus probable chez ce patient ?", "full_answer": "Ce cas décrit une maladie d'Alzheimer avec apparition insidieuse de troubles de la mémoire à court terme. Sur le plan clinique, cette maladie se distingue des autres options par l'absence de troubles du comportement (typiques de la démence frontotemporale) et d'hallucinations (précoces dans la maladie à corps de Lewy). La démence vasculaire est diagnostiquée sur la base de critères incluant des déficits de la mémoire et de deux autres domaines cognitifs avec une déficience fonctionnelle conséquente, mais la maladie vasculaire cérébrale sur la base de l'histoire, de l'examen et/ou des tests de neuro-imagerie, qui ne sont pas indiqués dans le cas présent. La maladie de Creutzfeldt-Jacob est le plus souvent rapidement progressive et se présente généralement avec d'autres caractéristiques cliniques, un examen neurologique anormal et des altérations de l'IRM.", "type": "NEUROLOGIE", "options": {"1": "Démence frontotemporale.", "2": "La maladie d'Alzheimer.", "3": "Démence vasculaire.", "4": "Maladie à corps de Lewy.", "5": "La maladie de Creutzfeldt-Jakob."}, "correct_option": 2, "explanations": {"1": {"exist": true, "char_ranges": [[0, 320]], "word_ranges": [[0, 49]], "text": "Ce cas décrit une maladie d'Alzheimer avec apparition insidieuse de troubles de la mémoire à court terme. Sur le plan clinique, cette maladie se distingue des autres options par l'absence de troubles du comportement (typiques de la démence frontotemporale) et d'hallucinations (précoces dans la maladie à corps de Lewy)."}, "2": {"exist": true, "char_ranges": [[0, 320]], "word_ranges": [[0, 49]], "text": "Ce cas décrit une maladie d'Alzheimer avec apparition insidieuse de troubles de la mémoire à court terme. Sur le plan clinique, cette maladie se distingue des autres options par l'absence de troubles du comportement (typiques de la démence frontotemporale) et d'hallucinations (précoces dans la maladie à corps de Lewy)."}, "3": {"exist": true, "char_ranges": [[321, 504]], "word_ranges": [[49, 76]], "text": "La démence vasculaire est diagnostiquée sur la base de critères incluant des déficits de la mémoire et de deux autres domaines cognitifs avec une déficience fonctionnelle conséquente,"}, "4": {"exist": true, "char_ranges": [[0, 320]], "word_ranges": [[0, 49]], "text": "Ce cas décrit une maladie d'Alzheimer avec apparition insidieuse de troubles de la mémoire à court terme. Sur le plan clinique, cette maladie se distingue des autres options par l'absence de troubles du comportement (typiques de la démence frontotemporale) et d'hallucinations (précoces dans la maladie à corps de Lewy)."}, "5": {"exist": true, "char_ranges": [[662, 866]], "word_ranges": [[102, 129]], "text": "La maladie de Creutzfeldt-Jacob est le plus souvent rapidement progressive et se présente généralement avec d'autres caractéristiques cliniques, un examen neurologique anormal et des altérations de l'IRM."}}} +{"id": 351, "year": 2016, "question_id_specific": 161, "full_question": "Une femme de 27 ans a été adressée au service de gynécologie pour évaluation, signalant une dyspareunie depuis 8 mois, ainsi qu'une dyschésie et une rectorragie occasionnelle coïncidant avec les menstruations depuis 3-4 mois. Elle signale également une dysménorrhée depuis des années, qu'elle contrôle bien avec de l'ibuprofène. Elle essaie de tomber enceinte depuis 16 mois, mais n'y est pas encore parvenue. Lors de l'examen gynécologique, seule une douleur est notée à la pression du fornix vaginal postérieur. Quel examen vous permettrait, selon vous, de poser un diagnostic de certitude sur sa pathologie ?", "full_answer": "Le cas clinique présenté est celui d'une femme en âge de procréer (27 ans) présentant une dyspareunie + une dysménorrhée + une infertilité, ce qui doit faire penser à une endométriose. L'histoire clinique et l'échographie donnent un diagnostic de suspicion, mais le diagnostic de certitude est donné par l'observation directe (avec ou sans biopsie associée) d'implants ectopiques de tissu endométrial, soit par laparotomie, soit par laparoscopie.", "type": "GYNÉCOLOGIE ET OBSTÉTRIQUE", "options": {"1": "Échographie transvaginale.", "2": "La laparoscopie diagnostique.", "3": "Imagerie par résonance magnétique.", "4": "Coloscopie.", "5": null}, "correct_option": 2, "explanations": {"1": {"exist": false, "char_ranges": [], "word_ranges": [], "text": ""}, "2": {"exist": true, "char_ranges": [[185, 446]], "word_ranges": [[31, 66]], "text": "L'histoire clinique et l'échographie donnent un diagnostic de suspicion, mais le diagnostic de certitude est donné par l'observation directe (avec ou sans biopsie associée) d'implants ectopiques de tissu endométrial, soit par laparotomie, soit par laparoscopie."}, "3": {"exist": false, "char_ranges": [], "word_ranges": [], "text": ""}, "4": {"exist": false, "char_ranges": [], "word_ranges": [], "text": ""}, "5": {"exist": false, "char_ranges": [], "word_ranges": [], "text": ""}}} +{"id": 69, "year": 2012, "question_id_specific": 124, "full_question": "La fille d'une femme de 82 ans nous consulte car elle trouve sa mère plus confuse que d'habitude. La patiente est diagnostiquée comme souffrant d'une maladie d'Alzheimer modérée. Elle souffre également de FA, de dépression et d'arthrose. Elle reçoit un traitement stable depuis 3 ans, comprenant de la digoxine, de l'acénocoumarol, de la fluoxétine depuis 4 mois et de l'ibuprofène depuis un mois pour des douleurs articulaires. A l'examen, sa tension artérielle est de 130/80, sa fréquence cardiaque de 48 bpm et sa fréquence respiratoire de 10 rpm. Lequel des éléments suivants est la cause la plus probable de l'augmentation de la confusion ?", "full_answer": "La cause la plus probable est l'intoxication digitalique, qui produirait ce ralentissement de la conduction, comme le montre la fréquence cardiaque. Probablement secondaire à une insuffisance rénale due à la toxicité de l'ibuprofène (mais c'est un peu exagéré).", "type": "ANESTHÉSIOLOGIE, SOINS INTENSIFS ET MÉDECINE D'URGENCE", "options": {"1": "Progression de la maladie d'Alzheimer.", "2": "Aggravation de la dépression.", "3": "Intoxication à la digitale.", "4": "Démence à corps de Lewy.", "5": "Toxicité de l'ibuprofène."}, "correct_option": 3, "explanations": {"1": {"exist": false, "char_ranges": [], "word_ranges": [], "text": ""}, "2": {"exist": false, "char_ranges": [], "word_ranges": [], "text": ""}, "3": {"exist": true, "char_ranges": [[0, 148]], "word_ranges": [[0, 21]], "text": "La cause la plus probable est l'intoxication digitalique, qui produirait ce ralentissement de la conduction, comme le montre la fréquence cardiaque."}, "4": {"exist": false, "char_ranges": [], "word_ranges": [], "text": ""}, "5": {"exist": false, "char_ranges": [], "word_ranges": [], "text": ""}}} +{"id": 140, "year": 2012, "question_id_specific": 134, "full_question": "Un nouveau-né de 37 semaines d'âge gestationnel, sans aucun signe pathologique à l'échographie prénatale, s'est présenté à l'examen pratiqué en salle d'accouchement avec un \"arrêt\" au passage de la sonde nasogastrique. La radiographie du thorax et de l'abdomen a montré une poche oesophagienne atrésique, avec une pneumatisation gastro-intestinale normale. Après une évaluation diagnostique qui a permis d'exclure d'autres anomalies et de trouver la patiente dans un état respiratoire stable, une intervention chirurgicale a été décidée. Sur quelles bases la priorité d'intervention chez cette patiente a-t-elle été fondée ?", "full_answer": "La bonne réponse est 3. Le fait qu'il y ait une aération intestinale indique l'existence d'une fistule du segment inférieur de l'œsophage avec la trachée ou une bronche. Dans ce cas, les complications pulmonaires sont fréquentes en raison du passage du contenu gastrique dans les voies respiratoires.", "type": "PÉDIATRIE", "options": {"1": "Incapacité à avaler de la salive.", "2": "En raison des malformations cardiaques fréquemment associées.", "3": "En raison du risque de pneumonie d'aspiration.", "4": "En raison de l'impossibilité d'une alimentation entérale.", "5": "En raison de la trachéomalacie associée."}, "correct_option": 3, "explanations": {"1": {"exist": false, "char_ranges": [], "word_ranges": [], "text": ""}, "2": {"exist": false, "char_ranges": [], "word_ranges": [], "text": ""}, "3": {"exist": true, "char_ranges": [[24, 300]], "word_ranges": [[5, 47]], "text": "Le fait qu'il y ait une aération intestinale indique l'existence d'une fistule du segment inférieur de l'œsophage avec la trachée ou une bronche. Dans ce cas, les complications pulmonaires sont fréquentes en raison du passage du contenu gastrique dans les voies respiratoires."}, "4": {"exist": false, "char_ranges": [], "word_ranges": [], "text": ""}, "5": {"exist": false, "char_ranges": [], "word_ranges": [], "text": ""}}} +{"id": 475, "year": 2020, "question_id_specific": 89, "full_question": "Un homme de 46 ans souffrant de troubles bipolaires est amené au service des urgences après une overdose de carbonate de lithium. L'examen révèle des tremblements sévères, une ataxie, une dysarthrie, une myoclonie et des fasciculations. La lithémie est de 4,1 mEq/L (toxicité > 1,6 mEq/L). Laquelle des options thérapeutiques suivantes serait la plus indiquée ?", "full_answer": "Il s'agit d'un cas d'intoxication grave au lithium (une lithémie > 3,5 mEq/L met en jeu le pronostic vital), par conséquent, l'option thérapeutique de choix est de commencer l'hémodialyse de toute urgence. N'oubliez pas que le charbon actif n'est PAS indiqué dans les cas d'intoxication au lithium.", "type": "SOINS INTENSIFS", "options": {"1": "Aminophylline associée à un cathartique.", "2": "Charbon actif.", "3": "Hémodialyse.", "4": "Diurèse forcée.", "5": null}, "correct_option": 3, "explanations": {"1": {"exist": false, "char_ranges": [], "word_ranges": [], "text": ""}, "2": {"exist": true, "char_ranges": [[206, 298]], "word_ranges": [[32, 47]], "text": "N'oubliez pas que le charbon actif n'est PAS indiqué dans les cas d'intoxication au lithium."}, "3": {"exist": true, "char_ranges": [[0, 205]], "word_ranges": [[0, 32]], "text": "Il s'agit d'un cas d'intoxication grave au lithium (une lithémie > 3,5 mEq/L met en jeu le pronostic vital), par conséquent, l'option thérapeutique de choix est de commencer l'hémodialyse de toute urgence."}, "4": {"exist": false, "char_ranges": [], "word_ranges": [], "text": ""}, "5": {"exist": false, "char_ranges": [], "word_ranges": [], "text": ""}}} +{"id": 85, "year": 2012, "question_id_specific": 50, "full_question": "Un patient ayant des antécédents de maladie fébrile et de douleur thoracique se présente à l'hôpital avec une dyspnée et une tachypnée. A l'examen physique, sa tension artérielle est basse, la pression veineuse jugulaire est élevée avec un sinus descendant profond en \"X\" et il a un pouls paradoxal. Quelle pathologie doit être suspectée ?", "full_answer": "Le pouls paradoxal est une diminution de la pression artérielle > 10 mmHg pendant l'inspiration ; il représente une exagération du phénomène physiologique de diminution inspiratoire de la pression artérielle (normale jusqu'à 10 mmHg). Dans la tamponnade cardiaque, l'inspiration, qui provoque une augmentation du flux sanguin vers les cavités droites, augmentant le volume des cavités droites, provoque secondairement un déplacement du septum interventriculaire vers la gauche, de sorte que moins de sang se loge et est éjecté du cœur gauche pendant la systole et que la fréquence du pouls diminue. Dans un cœur normal, ce déplacement exagéré, causé par la pression exercée par la tamponnade sur la paroi libre du VR, ne se produit pas. Le sinus X représente l'effondrement systolique du pouls veineux, c'est-à-dire la chute de pression due à la relaxation auriculaire (également due en partie à un déplacement vers le bas de la base du VR pendant la systole). Le sinus Y représente l'effondrement diastolique du pouls veineux, c'est-à-dire la chute de pression qui se produit à partir du moment où le sang pénètre dans le ventricule par la valve tricuspide. En cas de tamponnade cardiaque, le sinus X profond est caractéristique. Dans la péricardite constrictive, c'est le sinus Y profond. Pour toutes ces raisons, la bonne réponse est 5.", "type": "CARDIOLOGIE ET CHIRURGIE VASCULAIRE", "options": {"1": "Maladie cardiaque ischémique.", "2": "Cardiomyopathie dilatée.", "3": "Sténose aortique sévère.", "4": "Péricardite constrictive.", "5": "Épanchement péricardique avec tamponnade cardiaque."}, "correct_option": 5, "explanations": {"1": {"exist": false, "char_ranges": [], "word_ranges": [], "text": ""}, "2": {"exist": false, "char_ranges": [], "word_ranges": [], "text": ""}, "3": {"exist": false, "char_ranges": [], "word_ranges": [], "text": ""}, "4": {"exist": false, "char_ranges": [], "word_ranges": [], "text": ""}, "5": {"exist": true, "char_ranges": [[737, 1290]], "word_ranges": [[115, 204]], "text": "Le sinus X représente l'effondrement systolique du pouls veineux, c'est-à-dire la chute de pression due à la relaxation auriculaire (également due en partie à un déplacement vers le bas de la base du VR pendant la systole). Le sinus Y représente l'effondrement diastolique du pouls veineux, c'est-à-dire la chute de pression qui se produit à partir du moment où le sang pénètre dans le ventricule par la valve tricuspide. En cas de tamponnade cardiaque, le sinus X profond est caractéristique. Dans la péricardite constrictive, c'est le sinus Y profond."}}} +{"id": 527, "year": 2021, "question_id_specific": 31, "full_question": "Le résultat de l'analyse des gaz du sang artériel d'un patient est le suivant : pH : 7,40, PaCO2 60 mmHg ; bicarbonate 36 mM/L. Quelle est l'altération qu'il présente ?", "full_answer": "Les gaz du sang peuvent être trompeurs car un pH normal peut suggérer qu'il n'y a pas de perturbation de l'équilibre acido-basique, alors qu'il masque une double perturbation dans des directions opposées, comme dans ce cas. Le patient présente une acidose respiratoire (PaCO2 60 mmHg ; normale jusqu'à 50) et une alcalose métabolique compensatoire (bicarbonate 36 mmol/l ; normale jusqu'à 28). Cette situation est typique d'un patient souffrant de BPCO et d'insuffisance respiratoire chronique compensée.", "type": "NEPHROLOGIE", "options": {"1": "Aucun, le pH est normal.", "2": "Acidose respiratoire.", "3": "Acidose respiratoire et alcalose métabolique.", "4": "Alcalose métabolique.", "5": null}, "correct_option": 3, "explanations": {"1": {"exist": false, "char_ranges": [], "word_ranges": [], "text": ""}, "2": {"exist": false, "char_ranges": [], "word_ranges": [], "text": ""}, "3": {"exist": true, "char_ranges": [[224, 504]], "word_ranges": [[36, 75]], "text": "Le patient présente une acidose respiratoire (PaCO2 60 mmHg ; normale jusqu'à 50) et une alcalose métabolique compensatoire (bicarbonate 36 mmol/l ; normale jusqu'à 28). Cette situation est typique d'un patient souffrant de BPCO et d'insuffisance respiratoire chronique compensée."}, "4": {"exist": false, "char_ranges": [], "word_ranges": [], "text": ""}, "5": {"exist": false, "char_ranges": [], "word_ranges": [], "text": ""}}} +{"id": 407, "year": 2016, "question_id_specific": 81, "full_question": "Une femme de 30 ans s'est présentée à la clinique en se plaignant d'anxiété, d'une perte de poids d'environ 6 kg et d'un sentiment de \"nervosité\" au cours des trois derniers mois. L'examen physique a révélé une tachycardie, une hyperréflexie et l'absence de goitre. Les valeurs de TSH sont < 0,01 microU/mL, la T4 est élevée et le taux de thyroglobuline est bas. Une scintigraphie révèle une absence de captation dans la région thyroïdienne. Quel est, selon vous, le diagnostic le plus probable ?", "full_answer": "Les patients atteints de ce trouble ingèrent volontairement ou accidentellement des quantités excessives d'hormones thyroïdiennes, ce qui entraîne une hyperthyroïdie en l'absence de goitre. Contrairement à toutes les autres causes d'hyperthyroïdie, la thyroglobuline sérique n'est pas élevée et est presque toujours faible ou à la limite inférieure de la normale. La question fondamentale que nous nous posions avec le tératome ovarien (struma ovarien) est une variante rare de tératome monodermique mature composé de tissu thyroïdien, décrite au début du 20e siècle. Il est appelé ainsi lorsque le tératome est principalement composé de tissu thyroïdien mature (au moins 50 % pour être classé comme struma ovarii). Il représente 2,5 à 5 % des tératomes ovariens. En soi, il s'agit d'une tumeur bénigne, mais dans 5 % des cas, elle peut développer une transformation maligne de ses éléments, classiquement appelée struma ovarii malin. Le tissu néoplasique qui se développe est principalement un carcinome papillaire, plus rarement un carcinome folliculaire. Cette transformation maligne survient principalement dans les formes classiques de struma, mais elle est extrêmement rare dans les cas de tératomes kystiques matures. Son comportement est similaire à celui d'autres tumeurs ovariennes primaires, tendant à métastaser dans la cavité péritonéale et, par voie hématogène, dans les os, le foie et le cerveau. L'âge moyen de présentation du carcinome thyroïdien sur struma ovarii se situe entre 42,9 et 44 ans et entre 21 et 63 ans pour la variante du carcinome papillaire. Sur le plan clinique, la forme prédominante de présentation est une masse pelvienne ou des douleurs abdominales (45%), moins fréquemment une hyperthyroïdie (5-8%) ou une ascite (17%). On suppose que le tissu thyroïdien ovarien dans le struma est autonome dans la production d'hormones thyroïdiennes, ce qui explique pourquoi la mesure des taux sériques de thyroglobuline chez les patients atteints d'hyperthyroïdie est essentielle, car ils indiquent une production extra-thyroïdienne d'hormones thyroïdiennes.", "type": "ONCOLOGIE (ECTOPIQUE)", "options": {"1": "Thyrotoxicose factice.", "2": "Hyperthyroïdie due à la maladie de Graves.", "3": "Tératome de l'ovaire (stroma ovarien).", "4": "Thyroïdite subaiguë.", "5": null}, "correct_option": 1, "explanations": {"1": {"exist": false, "char_ranges": [], "word_ranges": [], "text": ""}, "2": {"exist": false, "char_ranges": [], "word_ranges": [], "text": ""}, "3": {"exist": false, "char_ranges": [], "word_ranges": [], "text": ""}, "4": {"exist": false, "char_ranges": [], "word_ranges": [], "text": ""}, "5": {"exist": false, "char_ranges": [], "word_ranges": [], "text": ""}}} +{"id": 15, "year": 2011, "question_id_specific": 126, "full_question": "Pour la troisième fois au cours des six derniers mois, un homme de 84 ans atteint de démence avancée est admis à l'hôpital pour une pneumonie d'aspiration. Il a perdu 9,5 kg au cours des 10 derniers mois. Il présente une escarre au niveau du sacrum. Il ne communique pas verbalement, ne se déplace pas et est dépendant pour toutes les activités de la vie quotidienne. Sa femme s'occupe de lui à la maison et ne veut pas le placer dans une maison de retraite. Une étude de la déglutition indique qu'il s'étouffe avec toutes sortes d'aliments. Le médecin de l'hôpital propose de l'alimenter par sonde. Le \"testament de vie\" du patient indique que sa femme est le décideur représentatif pour ses soins médicaux et qu'il ne veut pas de mesures extraordinaires pour prolonger sa vie, y compris la nutrition artificielle. Laquelle des recommandations suivantes est la plus appropriée pour ce patient ?", "full_answer": "C'est peut-être la moins scientifique des questions MIR de cette année, car sa réponse est basée sur l'interprétation d'une loi, qui est également de nature autonome. C'est probablement dans l'esprit de l'intention de l'examinateur de refléter cette loi, dans un cas extrême, je le répète. Voyons, à partir de l'article 11 de la loi 41 /2002, du 14 novembre (1), loi fondamentale réglementant l'autonomie du patient et les droits et obligations en matière d'information et de documentation clinique (B.O.E. n° 274, du 15 novembre) de caractère national, en Castille-La Manche, la loi 6/2005, du 7 juillet (2), de Castille-La Manche, sur la déclaration des directives anticipées en matière de santé, est légiférée. (D.O.C.M. Nº 141, du 15 juillet) et le décret 15/2006, du 21 février (3), sur le registre des directives anticipées de Castilla-La Mancha (D.O.C.M. Nº 42, du 24 février). Le résumé publié par la JCCM (4) est le suivant : \"La déclaration de directives anticipées (également appelée directives préalables ou testament de vie) est la déclaration écrite d'une personne capable qui, agissant librement, exprime les instructions à prendre en compte concernant les soins de santé qu'elle souhaite recevoir dans les situations qui l'empêchent de communiquer personnellement sa volonté, ou concernant la destination de son corps ou de ses organes une fois qu'elle est décédée. Dans notre déclaration de directives anticipées, nous pouvons faire référence à : 1. l'expression de valeurs personnelles, afin d'aider à l'interprétation du document lui-même et de guider les médecins lors de la prise de décisions cliniques ; 2. des instructions concernant les soins et les traitements à prodiguer à la personne décédée. 2. des instructions concernant les soins et les traitements liés à notre santé que nous souhaitons ou non recevoir 3. nous pouvons désigner un représentant qui servira d'interlocuteur au médecin responsable ou à l'équipe soignante pour veiller à ce que les instructions contenues dans la déclaration soient suivies d'effet 4) Nous pouvons également indiquer notre décision concernant le don d'organes. Dans ce cas, l'autorisation de la famille ne sera pas requise pour le prélèvement ou l'utilisation des organes donnés\". Il semble donc évident que la réponse souhaitée est 5.", "type": "SOINS PALLIATIFS", "options": {"1": "Placer une SNG permanente et transférer le patient dans une maison de repos avec des soins spécialisés.", "2": "Placement à moyen terme d'un SLNG et sortie vers une maison de soins infirmiers avec des soins spécialisés jusqu'à ce que les escarres guérissent.", "3": "Décharge dans une maison de repos avec des soins spécialisés jusqu'à ce que les escarres guérissent.", "4": "Poser une sonde de gastrostomie et renvoyer le patient chez lui.", "5": "Renvoi à domicile avec des soins palliatifs dispensés par le service de santé."}, "correct_option": 5, "explanations": {"1": {"exist": false, "char_ranges": [], "word_ranges": [], "text": ""}, "2": {"exist": false, "char_ranges": [], "word_ranges": [], "text": ""}, "3": {"exist": false, "char_ranges": [], "word_ranges": [], "text": ""}, "4": {"exist": false, "char_ranges": [], "word_ranges": [], "text": ""}, "5": {"exist": false, "char_ranges": [], "word_ranges": [], "text": ""}}} +{"id": 322, "year": 2016, "question_id_specific": 85, "full_question": "Un garçon de 22 ans souffrant d'hyposmie présente une absence de développement des caractères sexuels secondaires et une infertilité. Volume testiculaire bilatéral de 4 ml. Sur le plan analytique, EST-I 1,2 U/L (vn 5-15) ; LH 0,6 U/L (vn 3-15) ; testostérone 100 ng/dL (vn 300-1200), prolactine normale. Indiquez le traitement qu'il proposera pour obtenir la fertilité :", "full_answer": "Il s'agit d'un syndrome de Kallman (hypogonadisme hypogonadotrope + troubles olfactifs). La triptoréline est un agoniste de la GnRH qui, par une action continue, réduit la FSH et la LH. Elle est utilisée dans les cancers dépendants des hormones sexuelles ainsi que dans la puberté précoce. La bromocriptine est un agoniste dopaminergique utilisé dans l'hyperPRL (il a une PRL normale), les tumeurs hypophysaires... Nous restons entre 1 et 3. Pour induire la maturation testiculaire, on peut utiliser la GnRH pulsatile avec pompe, l'hCG ou FSH+hCG. La LH n'est pas utilisée comme traitement, elle serait stimulée par la GnRH intermittente.", "type": "UROLOGIE", "options": {"1": "Pompe à perfusion de GnRH.", "2": "Administration intramusculaire mensuelle de triptoréline.", "3": "Administration intramusculaire de FSH et LH une fois par semaine.", "4": "Traitement par la bromocriptine.", "5": null}, "correct_option": 1, "explanations": {"1": {"exist": true, "char_ranges": [[442, 547]], "word_ranges": [[69, 85]], "text": "Pour induire la maturation testiculaire, on peut utiliser la GnRH pulsatile avec pompe, l'hCG ou FSH+hCG."}, "2": {"exist": true, "char_ranges": [[89, 289]], "word_ranges": [[11, 46]], "text": "La triptoréline est un agoniste de la GnRH qui, par une action continue, réduit la FSH et la LH. Elle est utilisée dans les cancers dépendants des hormones sexuelles ainsi que dans la puberté précoce."}, "3": {"exist": true, "char_ranges": [[548, 638]], "word_ranges": [[85, 99]], "text": "La LH n'est pas utilisée comme traitement, elle serait stimulée par la GnRH intermittente."}, "4": {"exist": true, "char_ranges": [[290, 414]], "word_ranges": [[46, 63]], "text": "La bromocriptine est un agoniste dopaminergique utilisé dans l'hyperPRL (il a une PRL normale), les tumeurs hypophysaires..."}, "5": {"exist": false, "char_ranges": [], "word_ranges": [], "text": ""}}} +{"id": 610, "year": 2022, "question_id_specific": 116, "full_question": "Une femme de 95 ans vivant dans une maison de retraite, indépendante pour les activités de base de la vie quotidienne, sort se promener dans le jardin. Elle a des antécédents d'hypertension, de dyslipidémie, d'ostéoporose et de troubles cognitifs légers. Elle fait une chute en se levant la nuit pour aller aux toilettes. La radiographie montre une fracture sous-capitale déplacée de la hanche droite. Quel est le traitement recommandé ?", "full_answer": "Fracture déplacée de la hanche, jardin III-IV. Traitement chirurgical. Hémiarthroplastie de la hanche. Traitement de l'option 2 pour les fractures pertrochantériennes. Option 3 : fractures non déplacées. Option 4 : patients non ambulatoires présentant une douleur minimale et un risque chirurgical élevé.", "type": "TRAUMATOLOGIE", "options": {"1": "Hémiarthroplastie de la hanche.", "2": "Fixation du clou trochantérien.", "3": "Fixation par vis canulées.", "4": "Conservateur : la vie en fauteuil roulant.", "5": null}, "correct_option": 1, "explanations": {"1": {"exist": true, "char_ranges": [[0, 102]], "word_ranges": [[0, 13]], "text": "Fracture déplacée de la hanche, jardin III-IV. Traitement chirurgical. Hémiarthroplastie de la hanche."}, "2": {"exist": true, "char_ranges": [[103, 167]], "word_ranges": [[13, 21]], "text": "Traitement de l'option 2 pour les fractures pertrochantériennes."}, "3": {"exist": true, "char_ranges": [[168, 203]], "word_ranges": [[21, 27]], "text": "Option 3 : fractures non déplacées."}, "4": {"exist": true, "char_ranges": [[204, 304]], "word_ranges": [[27, 42]], "text": "Option 4 : patients non ambulatoires présentant une douleur minimale et un risque chirurgical élevé."}, "5": {"exist": false, "char_ranges": [], "word_ranges": [], "text": ""}}} +{"id": 391, "year": 2016, "question_id_specific": 222, "full_question": "Un homme célibataire de 26 ans est amené à la clinique par sa famille parce qu'il refuse de quitter son domicile depuis trois mois. La raison invoquée par le patient est la conviction qu'il a une mâchoire asymétrique et un visage tordu. Selon le patient, cette situation est progressive et il semble de plus en plus déformé lorsqu'il se regarde dans le miroir. Il a honte de son apparence, ne veut pas sortir, est très angoissé lorsqu'il voit son image et ne peut s'empêcher de penser à sa déformation tout au long de la journée. Il a consulté plusieurs chirurgiens maxillo-faciaux, mais ceux-ci lui ont dit qu'il n'avait pas d'asymétrie faciale et l'ont orienté vers un psychiatre. Le diagnostic du patient est le suivant :", "full_answer": "Encore une question simple avec une réponse immédiate, qui ne laisse aucun doute. Elle décrit un patient préoccupé par un défaut physique inexistant, dont la préoccupation l'angoisse et l'empêche de sortir de chez lui. En tant qu'interne en psychiatrie, j'aurais aimé que les questions MIR de ma spécialité soient un peu plus stimulantes et approfondies, même si je sais que les secondes que vous aurez gagnées en cochant directement la quatrième sont très précieuses.", "type": "PSYCHIATRIE", "options": {"1": "Trouble dépressif majeur avec délire d'incompatibilité d'humeur.", "2": "Troubles obsessionnels compulsifs.", "3": "Schizophrénie paranoïaque.", "4": "Dysmorphie corporelle.", "5": null}, "correct_option": 4, "explanations": {"1": {"exist": false, "char_ranges": [], "word_ranges": [], "text": ""}, "2": {"exist": false, "char_ranges": [], "word_ranges": [], "text": ""}, "3": {"exist": false, "char_ranges": [], "word_ranges": [], "text": ""}, "4": {"exist": true, "char_ranges": [[82, 218]], "word_ranges": [[13, 34]], "text": "Elle décrit un patient préoccupé par un défaut physique inexistant, dont la préoccupation l'angoisse et l'empêche de sortir de chez lui."}, "5": {"exist": false, "char_ranges": [], "word_ranges": [], "text": ""}}} +{"id": 324, "year": 2016, "question_id_specific": 55, "full_question": "Une femme de 70 ans a été admise aux soins intensifs après avoir subi un IAM antérieur traité par angioplastie coronaire et pose d'un stent sur l'artère descendante antérieure. Quatre jours plus tard, elle a soudainement présenté une hypotension nécessitant un soutien volumétrique vigoureux, l'initiation de médicaments vasoactifs, l'intubation orotrachéale et la mise en place d'une ventilation mécanique. L'examen physique a révélé un souffle qui n'était pas présent auparavant. La suspicion d'une complication mécanique de l'infarctus a conduit à une échocardiographie transthoracique montrant un épanchement péricardique. Cochez la bonne réponse :", "full_answer": "La mortalité n'est pas aussi faible, un saut oxymétrique se produit en cas de rupture septale, ainsi qu'une phrémie (la bonne réponse est donc 3) et des complications peuvent survenir dès la première semaine.", "type": "LES SOINS CRITIQUES, PALLIATIFS ET D'URGENCE", "options": {"1": "La mortalité sous traitement médical est de 20 %.", "2": "En cas de rupture de la paroi libre, il y a un ressaut du ventricule droit dans le cathétérisme de Swan-Ganz.", "3": "En cas de rupture de la paroi libre, il n'y a pas de frémito palpable.", "4": "Les complications mécaniques surviennent généralement le premier jour suivant l'infarctus.", "5": null}, "correct_option": 3, "explanations": {"1": {"exist": false, "char_ranges": [], "word_ranges": [], "text": ""}, "2": {"exist": false, "char_ranges": [], "word_ranges": [], "text": ""}, "3": {"exist": true, "char_ranges": [[0, 145]], "word_ranges": [[0, 25]], "text": "La mortalité n'est pas aussi faible, un saut oxymétrique se produit en cas de rupture septale, ainsi qu'une phrémie (la bonne réponse est donc 3)"}, "4": {"exist": false, "char_ranges": [], "word_ranges": [], "text": ""}, "5": {"exist": false, "char_ranges": [], "word_ranges": [], "text": ""}}} +{"id": 240, "year": 2014, "question_id_specific": 150, "full_question": "Un homme de 76 ans consulte pour une détérioration cognitive, une lenteur et une maladresse dans les mouvements, qui durent depuis huit mois. La famille rapporte que le patient a eu des hallucinations visuelles, pour lesquelles son médecin traitant a prescrit de faibles doses de rispéridone, avec une aggravation significative de son état moteur. Au vu de ces données, quel est le diagnostic le plus probable ?", "full_answer": "La bonne réponse est 2 car elle présente le cas avec les symptômes typiques de la démence à corps de Lewy, le symptôme directeur étant les hallucinations visuelles. Des troubles cognitifs progressifs, des signes moteurs parkinsoniens (déduits de la maladresse des mouvements) et une hypersensibilité aux neuroleptiques, même à faibles doses, sont également mentionnés.", "type": "NEUROLOGIE", "options": {"1": "La maladie d'Alzheimer.", "2": "Démence à corps de Lewy.", "3": "Démence frontotemporale.", "4": "Démence vasculaire.", "5": "Maladie de Creutzfeldt-Jakob sporadique."}, "correct_option": 2, "explanations": {"1": {"exist": false, "char_ranges": [], "word_ranges": [], "text": ""}, "2": {"exist": true, "char_ranges": [[0, 164]], "word_ranges": [[0, 28]], "text": "La bonne réponse est 2 car elle présente le cas avec les symptômes typiques de la démence à corps de Lewy, le symptôme directeur étant les hallucinations visuelles."}, "3": {"exist": false, "char_ranges": [], "word_ranges": [], "text": ""}, "4": {"exist": false, "char_ranges": [], "word_ranges": [], "text": ""}, "5": {"exist": false, "char_ranges": [], "word_ranges": [], "text": ""}}} +{"id": 68, "year": 2012, "question_id_specific": 123, "full_question": "Un homme de 64 ans, ayant des antécédents d'hypertension et de cardiopathie ischémique, se présente aux urgences pour une douleur thoracique oppressante ressentie en regardant la télévision. Lors de la consultation de triage aux urgences, des valeurs T. art de 155/95 mmHg et une saturation capillaire en O2 de 95 % sont détectées. Laquelle des mesures suivantes est la plus correcte ?", "full_answer": "Si l'ECG montre des changements ischémiques, vous serez évidemment vu aux urgences et le cardiologue sera prévenu. Dans le cas contraire, vous serez traité comme s'il s'agissait d'une angine de poitrine et mis en observation, avec des enzymes en série et un traitement dans la zone d'observation.", "type": "ANESTHÉSIOLOGIE, SOINS INTENSIFS ET MÉDECINE D'URGENCE", "options": {"1": "Vous demander de vous rendre dans la salle d'attente. Vous serez appelé pour des tests.", "2": "Ce patient doit être pris en charge dans le box d'urgence vitale (salle de choc).", "3": "Nous préviendrons le cardiologue de garde pour qu'il évalue le patient.", "4": "Vous serez orienté vers l'unité coronarienne à partir du triage.", "5": "Nous réalisons un ECG en moins de 10 minutes."}, "correct_option": 5, "explanations": {"1": {"exist": false, "char_ranges": [], "word_ranges": [], "text": ""}, "2": {"exist": false, "char_ranges": [], "word_ranges": [], "text": ""}, "3": {"exist": false, "char_ranges": [], "word_ranges": [], "text": ""}, "4": {"exist": false, "char_ranges": [], "word_ranges": [], "text": ""}, "5": {"exist": false, "char_ranges": [], "word_ranges": [], "text": ""}}} +{"id": 206, "year": 2014, "question_id_specific": 125, "full_question": "Dans le cas d'un homme asymptomatique de 49 ans, avec des antécédents familiaux d'un père décédé d'un cancer de la prostate, un PSA (Prostate Specific Antigen) de 5,9 ng/ml, avec un rapport PSA libre/PSA total de 11 % et un toucher rectal montrant une augmentation de la consistance dans le lobe droit de la prostate, quelle est l'indication clinique suivante ?", "full_answer": "Le patient a un PSA >4 et une palpation avec une consistance accrue, ce qui implique la nécessité d'une biopsie guidée par échographie transrectale, afin de diagnostiquer s'il s'agit d'une hyperplasie bénigne de la prostate ou d'un cancer de la prostate.", "type": "UROLOGIE", "options": {"1": "Demander au patient de subir une échographie transrectale et une biopsie de la prostate.", "2": "Réaliser un scanner abdominopelvien.", "3": "Initier un traitement avec des inhibiteurs de la 5-alpha réductase pour réduire de moitié le taux de PSA.", "4": "Initier un traitement combiné avec des analogues de la LH-RH et des anti-androgènes.", "5": "Effectuer une scintigraphie osseuse."}, "correct_option": 1, "explanations": {"1": {"exist": true, "char_ranges": [[0, 254]], "word_ranges": [[0, 41]], "text": "Le patient a un PSA >4 et une palpation avec une consistance accrue, ce qui implique la nécessité d'une biopsie guidée par échographie transrectale, afin de diagnostiquer s'il s'agit d'une hyperplasie bénigne de la prostate ou d'un cancer de la prostate."}, "2": {"exist": false, "char_ranges": [], "word_ranges": [], "text": ""}, "3": {"exist": false, "char_ranges": [], "word_ranges": [], "text": ""}, "4": {"exist": false, "char_ranges": [], "word_ranges": [], "text": ""}, "5": {"exist": false, "char_ranges": [], "word_ranges": [], "text": ""}}} +{"id": 570, "year": 2022, "question_id_specific": 153, "full_question": "Un homme de 19 ans, sans antécédents particuliers, s'est présenté au service des urgences car, après une infection respiratoire, il présentait une asthénie, un malaise général, une oligurie et des céphalées. Examen physique : tension artérielle 210/120 mmHg, fond d'œil avec rétinopathie hypertensive de grade III. Examens de laboratoire : Hb 7,4 g/dl, plaquettes 85 000/mm3, 2-3 schistocytes dans le frottis de sang périphérique, LDH 950 UI/ml, créatinine sérique 8,75 mg/dl. Système urinaire : protéinurie 300 mg/dl, sédiment 15 globules rouges par champ. Les taux d'ADAMTS-13 sont normaux. Au vu de ces résultats, quelle serait votre principale suspicion diagnostique ?", "full_answer": "Question difficile. Le patient présente une microangiopathie thrombotique (MAT) avec une insuffisance rénale aiguë. Cela indique un SHU, un PTT, une AMT induite par des médicaments ou une AMT médiée par le complément, la coagulation ou le métabolisme. Des taux normaux d'ADAMTS-13 excluent la possibilité d'un PTT (option 1 incorrecte). L'absence de symptômes gastro-intestinaux exclut un SHU typique dû à la toxine de Shiga (option 4 incorrecte). Les infections à pneumocoques peuvent également provoquer un SHU, bien que cela soit rare et plus fréquent chez les adultes. L'antécédent d'infection respiratoire (non précisé, il pourrait s'agir d'une pneumonie à pneumocoque) ainsi que l'absence de données sur la CIVD (pas de mention d'hémorragie ou d'altération des tests de coagulation) me font penser à un SHU atypique (option 2 correcte).", "type": "NEPHROLOGIE", "options": {"1": "Purpura thrombocytopénique thrombotique.", "2": "Syndrome hémolytique et urémique atypique.", "3": "Coagulation intravasculaire disséminée.", "4": "Syndrome hémolytique et urémique typique.", "5": null}, "correct_option": 2, "explanations": {"1": {"exist": true, "char_ranges": [[252, 336]], "word_ranges": [[38, 50]], "text": "Des taux normaux d'ADAMTS-13 excluent la possibilité d'un PTT (option 1 incorrecte)."}, "2": {"exist": true, "char_ranges": [[573, 842]], "word_ranges": [[87, 127]], "text": "L'antécédent d'infection respiratoire (non précisé, il pourrait s'agir d'une pneumonie à pneumocoque) ainsi que l'absence de données sur la CIVD (pas de mention d'hémorragie ou d'altération des tests de coagulation) me font penser à un SHU atypique (option 2 correcte)."}, "3": {"exist": false, "char_ranges": [], "word_ranges": [], "text": ""}, "4": {"exist": true, "char_ranges": [[337, 447]], "word_ranges": [[50, 67]], "text": "L'absence de symptômes gastro-intestinaux exclut un SHU typique dû à la toxine de Shiga (option 4 incorrecte)."}, "5": {"exist": false, "char_ranges": [], "word_ranges": [], "text": ""}}} +{"id": 506, "year": 2020, "question_id_specific": 87, "full_question": "Un trouble obsessionnel-compulsif (TOC) d'apparition soudaine chez un enfant de 9 ans présentant des tics et une chorée comme manifestations neurologiques doit être exclu :", "full_answer": "Il existe une abondante littérature sur le sujet. Nous ne cessons de nous étonner des voies étranges de la biologie et de constater que l'axiome \"le plus long est le bon\" se vérifie dans ce cas.", "type": "PSYCHIATRIE", "options": {"1": "Streptococcus viridans.", "2": "Streptocoque alpha-hémolytique.", "3": "Streptocoque bêta-hémolytique du groupe A.", "4": "Entérocoque.", "5": null}, "correct_option": 3, "explanations": {"1": {"exist": false, "char_ranges": [], "word_ranges": [], "text": ""}, "2": {"exist": false, "char_ranges": [], "word_ranges": [], "text": ""}, "3": {"exist": false, "char_ranges": [], "word_ranges": [], "text": ""}, "4": {"exist": false, "char_ranges": [], "word_ranges": [], "text": ""}, "5": {"exist": false, "char_ranges": [], "word_ranges": [], "text": ""}}} +{"id": 619, "year": 2022, "question_id_specific": 121, "full_question": "Une femme de 79 ans est admise pour une fracture ostéoporotique de la hanche. En ce qui concerne la prévention secondaire des fractures de fragilité, cochez la MAUVAISE réponse :", "full_answer": "Ils doutent de l'efficacité du calcium ou de la vitamine D administrés en monothérapie pour le traitement de l'ostéoporose ; ils soulignent que \"administrés ensemble, ils semblent avoir un certain degré d'efficacité dans la prévention des fractures non vertébrales, ce qui est plus clair chez les personnes qui en sont déficientes, comme cela peut être le cas des personnes âgées vivant dans des maisons de retraite. Il n'y a pratiquement aucune preuve de leur efficacité dans la prévention des fractures vertébrales.", "type": "TRAUMATOLOGIE", "options": {"1": "Une mauvaise observance du traitement est associée à un risque accru de fracture.", "2": "Les marqueurs du remodelage osseux peuvent être utiles pour le suivi précoce de la réponse au traitement.", "3": "La monothérapie à la vitamine D est efficace pour réduire ces fractures chez les personnes âgées non institutionnalisées.", "4": "L'augmentation du calcium alimentaire ou la prise de suppléments de calcium ne protègent pas à elles seules contre les fractures.", "5": null}, "correct_option": 3, "explanations": {"1": {"exist": false, "char_ranges": [], "word_ranges": [], "text": ""}, "2": {"exist": false, "char_ranges": [], "word_ranges": [], "text": ""}, "3": {"exist": true, "char_ranges": [[0, 124]], "word_ranges": [[0, 20]], "text": "Ils doutent de l'efficacité du calcium ou de la vitamine D administrés en monothérapie pour le traitement de l'ostéoporose ;"}, "4": {"exist": false, "char_ranges": [], "word_ranges": [], "text": ""}, "5": {"exist": false, "char_ranges": [], "word_ranges": [], "text": ""}}} +{"id": 161, "year": 2013, "question_id_specific": 118, "full_question": "Une femme de 60 ans chez qui on a diagnostiqué une bronchopathie chronique, un diabète sucré, une hypertension artérielle et qui est traitée depuis 4 mois par oméprazole, metformine, salbutamol, bromure d'ipratropium et énalapril 20 mg + hydrochlorothiazide 25 mg se présente à votre cabinet en raison d'une fatigue, d'une diminution de l'appétit, d'une légère dyspnée et d'une toux occasionnelle, de selles variables, parfois molles, et sans symptômes urinaires. Les analyses montrent des leucocytes 10.000/mm³, Ht 35%, MCV 80, Glucose 150 mg/dl, Urée 80 mg/dl, Créatinine 1,6 mg/dl, Sodium 133 mEq/l et Potassium 2,9 mEq/l. Quelle est la cause la plus probable de l'hypokaliémie ?", "full_answer": "La réponse correcte est : 4. antihypertenseur. En particulier, l'hydrochlorothiazide, en tant que thiazide, est principalement responsable de l'hypokaliémie de ce patient.", "type": "NEPHROLOGIE", "options": {"1": "Insuffisance rénale.", "2": "Hyponatrémie.", "3": "Carence en potassium.", "4": "Antihypertenseur.", "5": "Metformine."}, "correct_option": 4, "explanations": {"1": {"exist": false, "char_ranges": [], "word_ranges": [], "text": ""}, "2": {"exist": false, "char_ranges": [], "word_ranges": [], "text": ""}, "3": {"exist": false, "char_ranges": [], "word_ranges": [], "text": ""}, "4": {"exist": true, "char_ranges": [[29, 171]], "word_ranges": [[6, 22]], "text": "antihypertenseur. En particulier, l'hydrochlorothiazide, en tant que thiazide, est principalement responsable de l'hypokaliémie de ce patient."}, "5": {"exist": false, "char_ranges": [], "word_ranges": [], "text": ""}}} +{"id": 269, "year": 2014, "question_id_specific": 141, "full_question": "Un patient asthmatique de 55 ans se présente aux urgences pour une exacerbation. Une heure après l'administration d'oxygène supplémentaire et de deux nébulisations de salbutamol, l'état du patient ne s'améliore pas. À l'examen, il respire à 42 tours/minute avec un tirage sus-claviculaire v et des sifflements inspiratoires et expiratoires dispersés. Le débit de pointe a diminué de 310 à 220 L/min. Un gaz du sang prélevé avec 28 % d'oxygène montre une p02 de 54 mmHg et une pC02 de 35 mm Hg. Laquelle des mesures suivantes vous semble la MOINS indiquée ?", "full_answer": "En cas de crise d'asthme grave, toutes les options sont indiquées ou peuvent être indiquées avant des options plus agressives. En cas d'issue défavorable (acidose, etc.), le patient serait soumis à une médication invasive.", "type": "PNEUMOLOGIE", "options": {"1": "Augmenter le flux d'oxygène.", "2": "Administrer 80 mg de méthylprednisolone iv.", "3": "Nébuliser le bromure d'ipratropium avec le salbutamol toutes les 20 minutes.", "4": "Administrer du magnésium par voie intraveineuse.", "5": " Mettre en place une ventilation mécanique non invasive."}, "correct_option": 5, "explanations": {"1": {"exist": false, "char_ranges": [], "word_ranges": [], "text": ""}, "2": {"exist": false, "char_ranges": [], "word_ranges": [], "text": ""}, "3": {"exist": false, "char_ranges": [], "word_ranges": [], "text": ""}, "4": {"exist": false, "char_ranges": [], "word_ranges": [], "text": ""}, "5": {"exist": true, "char_ranges": [[127, 222]], "word_ranges": [[20, 34]], "text": "En cas d'issue défavorable (acidose, etc.), le patient serait soumis à une médication invasive."}}} +{"id": 18, "year": 2011, "question_id_specific": 138, "full_question": "Si un patient signale qu'il a une langue hypertrophiée depuis des mois, sans autre altération de la langue provoquant une morsure constante, à laquelle des maladies suivantes penseriez-vous ?", "full_answer": "Question partagée avec l'ORL. L'amylose est un dépôt localisé ou systémique de protéines fibrillaires. Elle peut être primaire ou secondaire. Les organes les plus fréquemment touchés sont les reins et le cœur. Parfois, des dépôts amyloïdes apparaissent sur la langue, ce qui peut entraîner une macroglossie.", "type": "DERMATOLOGIE", "options": {"1": "Paralysie de l'hypoglosse.", "2": "Hyperthyroïdie.", "3": "Mastocytose.", "4": "Amyloïdose.", "5": "Granulome facial."}, "correct_option": 4, "explanations": {"1": {"exist": false, "char_ranges": [], "word_ranges": [], "text": ""}, "2": {"exist": false, "char_ranges": [], "word_ranges": [], "text": ""}, "3": {"exist": false, "char_ranges": [], "word_ranges": [], "text": ""}, "4": {"exist": true, "char_ranges": [[30, 307]], "word_ranges": [[4, 46]], "text": "L'amylose est un dépôt localisé ou systémique de protéines fibrillaires. Elle peut être primaire ou secondaire. Les organes les plus fréquemment touchés sont les reins et le cœur. Parfois, des dépôts amyloïdes apparaissent sur la langue, ce qui peut entraîner une macroglossie."}, "5": {"exist": false, "char_ranges": [], "word_ranges": [], "text": ""}}} +{"id": 250, "year": 2014, "question_id_specific": 78, "full_question": "Un homme de 55 ans, père d'un fils atteint de la maladie cœliaque, présentant une anémie ferriprive et une augmentation récente du rythme des selles, a été testé négatif pour les allèles HLA-DQ2 et HLA-DQ8. Quel est le test diagnostique le plus approprié dans ce cas ?", "full_answer": "Homme d'âge moyen souffrant d'une anémie ferriprive et d'une augmentation du nombre de selles, coloscopie pour écarter la possibilité d'un cancer colorectal ! Si le test HLA est négatif, il est très peu probable qu'il s'agisse d'une maladie cœliaque (valeur prédictive négative élevée).", "type": "SYSTÈME DIGESTIF", "options": {"1": "Détermination des anticorps IgA anti-transglutaminase.", "2": "Endoscopie gastro-intestinale supérieure avec biopsies duodénales.", "3": "Test au D-xylose.", "4": "Évaluer la réponse à un régime sans gluten.", "5": "Coloscopie."}, "correct_option": 5, "explanations": {"1": {"exist": false, "char_ranges": [], "word_ranges": [], "text": ""}, "2": {"exist": false, "char_ranges": [], "word_ranges": [], "text": ""}, "3": {"exist": false, "char_ranges": [], "word_ranges": [], "text": ""}, "4": {"exist": true, "char_ranges": [[159, 286]], "word_ranges": [[23, 43]], "text": "Si le test HLA est négatif, il est très peu probable qu'il s'agisse d'une maladie cœliaque (valeur prédictive négative élevée)."}, "5": {"exist": true, "char_ranges": [[0, 159]], "word_ranges": [[0, 23]], "text": "Homme d'âge moyen souffrant d'une anémie ferriprive et d'une augmentation du nombre de selles, coloscopie pour écarter la possibilité d'un cancer colorectal !"}}} +{"id": 5, "year": 2011, "question_id_specific": 44, "full_question": "Un homme de 66 ans signale des brûlures d'estomac quotidiennes depuis sa jeunesse, qu'il traite avec des alcalins. Une endoscopie du tractus gastro-intestinal supérieur a révélé une œsophagite érosive légère et une jonction pavimenteuse cylindrique déplacée d'environ 7 cm. Des biopsies de l'œsophage et de l'œsophage ont été réalisées. Les biopsies de l'œsophage distal révèlent que l'épithélium pavimenteux normal a été remplacé par un épithélium cylindrique de type intestinal présentant une dysplasie de bas grade. Quelle est l'option de prise en charge la plus appropriée pour ce patient ?", "full_answer": "Ce patient a un œsophage de Barrett avec une dysplasie de bas grade. Je n'aime pas cette question, car la dysplasie peut être conditionnée par l'inflammation et ne pas être réelle, puisqu'il n'y a pas d'inhibition de la production d'acide. Dans tous les cas, l'œsophage de Barrett doit être surveillé par endoscopie et traité avec des IPP à vie ou une chirurgie anti-reflux pour prévenir la progression. La bonne réponse est 4.", "type": "DIGESTIF", "options": {"1": "Étant donné que l'œsophagite est légère et que la dysplasie est de faible degré, il est conseillé de poursuivre le traitement avec des alcalins.", "2": "Il a été démontré qu'un traitement indéfini par des IPP à forte dose prévient la progression de l'œsophage de Barrett vers l'adénocarcinome, ce qui rend la surveillance endoscopique inutile.", "3": "Endoscopies de surveillance régulières et traitement par IPP pendant moins de 12 semaines, car un traitement prolongé est associé à un risque élevé de développement d'un gastrinome.", "4": "Endoscopies périodiques de surveillance et traitement indéfini par IPP.", "5": "Esophagectomie."}, "correct_option": 4, "explanations": {"1": {"exist": false, "char_ranges": [], "word_ranges": [], "text": ""}, "2": {"exist": false, "char_ranges": [], "word_ranges": [], "text": ""}, "3": {"exist": false, "char_ranges": [], "word_ranges": [], "text": ""}, "4": {"exist": true, "char_ranges": [[240, 427]], "word_ranges": [[40, 72]], "text": "Dans tous les cas, l'œsophage de Barrett doit être surveillé par endoscopie et traité avec des IPP à vie ou une chirurgie anti-reflux pour prévenir la progression. La bonne réponse est 4."}, "5": {"exist": false, "char_ranges": [], "word_ranges": [], "text": ""}}} +{"id": 359, "year": 2016, "question_id_specific": 91, "full_question": "La détermination chez une femme de 70 ans d'un hématocrite de 32 %, d'une Hb de 9,6 g/dL, d'un MCV de 85 fL (70-99), d'un Fe de 25 ug/dL (37-145) et d'une ferritine de 350 ng/mL (15-150) est très évocatrice :", "full_answer": "La polymyalgie rhumatismale. Exemple classique, je suppose, que l'on rencontre partout en clinique. Femme âgée présentant une anémie normocytaire et un schéma ferrocinétique clair de trouble chronique (hyposidérémie mais hyperferritinémie). Le seul qui corresponde, et très bien, est le numéro 3 ; tout ce dont ils avaient besoin, c'était de nous donner un ESR élevé. 1 : s'il s'agit d'une perte récente, le MCV pourrait rester normal, mais la ferritine devrait être affectée, et certainement pas élevée. 2 : idem. S'il s'agit d'une malabsorption du fer, la ferritine devrait être basse. Et s'il s'agit d'une malabsorption de la B12, le MCV sera clairement plus élevé. 4 : tachycardie à 70 et MCV normal. Celui-ci est trop élevé.", "type": "HÉMATOLOGIE", "options": {"1": "Perte récente de sang par le tube digestif.", "2": "Malabsorption intestinale.", "3": "Polymyalgie rhumatismale.", "4": "Thalassémie mineure.", "5": null}, "correct_option": 3, "explanations": {"1": {"exist": true, "char_ranges": [[368, 504]], "word_ranges": [[55, 77]], "text": "1 : s'il s'agit d'une perte récente, le MCV pourrait rester normal, mais la ferritine devrait être affectée, et certainement pas élevée."}, "2": {"exist": true, "char_ranges": [[505, 668]], "word_ranges": [[77, 105]], "text": "2 : idem. S'il s'agit d'une malabsorption du fer, la ferritine devrait être basse. Et s'il s'agit d'une malabsorption de la B12, le MCV sera clairement plus élevé."}, "3": {"exist": true, "char_ranges": [[100, 297]], "word_ranges": [[13, 42]], "text": "Femme âgée présentant une anémie normocytaire et un schéma ferrocinétique clair de trouble chronique (hyposidérémie mais hyperferritinémie). Le seul qui corresponde, et très bien, est le numéro 3 ;"}, "4": {"exist": true, "char_ranges": [[669, 729]], "word_ranges": [[105, 117]], "text": "4 : tachycardie à 70 et MCV normal. Celui-ci est trop élevé."}, "5": {"exist": false, "char_ranges": [], "word_ranges": [], "text": ""}}} +{"id": 108, "year": 2012, "question_id_specific": 219, "full_question": "Un homme présente une délétion partielle du gène de la dystrophine (chromosome Xp21) qui est à l'origine de la sémiologie de la dystrophie musculaire de Becker. Il se rend à un conseil génétique avec sa femme pour évaluer le risque de transmission de la maladie. Quelle information correcte sera fournie au cours du conseil génétique ?", "full_answer": "C'est l'homme qui est atteint de la maladie, dont on nous dit qu'elle est liée au chromosome X. Il ne reste donc que deux options : 2 et 3. 2 n'est pas correct parce que les hommes recevront le chromosome Y du père et le X de la mère, qui est en bonne santé, de sorte que les filles peuvent être porteuses de la maladie (option correcte 3).", "type": "GÉNÉTIQUE ET IMMUNOLOGIE", "options": {"1": "Selon le mode de transmission autosomique dominant, la moitié de leur progéniture héritera de la maladie, sans distinction de sexe.", "2": "Leurs filles n'hériteront pas de la maladie, mais tous leurs futurs fils seront porteurs et pourront transmettre la mutation à 50 %.", "3": "Leurs fils n'hériteront pas de la maladie, mais toutes leurs futures filles seront porteuses et pourront transmettre la mutation à 50 %.", "4": "Il n'y a pas de risque : l'hérédité de la maladie est de type mitochondrial, jamais transmise par les hommes.", "5": "Selon le mode de transmission autosomique récessif, 25 % de leurs enfants manifesteront la maladie dans l'enfance, quel que soit leur sexe."}, "correct_option": 3, "explanations": {"1": {"exist": false, "char_ranges": [], "word_ranges": [], "text": ""}, "2": {"exist": true, "char_ranges": [[0, 340]], "word_ranges": [[0, 68]], "text": "C'est l'homme qui est atteint de la maladie, dont on nous dit qu'elle est liée au chromosome X. Il ne reste donc que deux options : 2 et 3. 2 n'est pas correct parce que les hommes recevront le chromosome Y du père et le X de la mère, qui est en bonne santé, de sorte que les filles peuvent être porteuses de la maladie (option correcte 3)."}, "3": {"exist": true, "char_ranges": [[0, 340]], "word_ranges": [[0, 68]], "text": "C'est l'homme qui est atteint de la maladie, dont on nous dit qu'elle est liée au chromosome X. Il ne reste donc que deux options : 2 et 3. 2 n'est pas correct parce que les hommes recevront le chromosome Y du père et le X de la mère, qui est en bonne santé, de sorte que les filles peuvent être porteuses de la maladie (option correcte 3)."}, "4": {"exist": false, "char_ranges": [], "word_ranges": [], "text": ""}, "5": {"exist": false, "char_ranges": [], "word_ranges": [], "text": ""}}} +{"id": 255, "year": 2014, "question_id_specific": 94, "full_question": "Une femme de 65 ans, ayant des antécédents de douleurs articulaires et suivant un traitement aux anti-inflammatoires, a été adressée pour anémie. Dans les études complémentaires, elle présentait des globules rouges 3 164 000, Ht. 32%, Hb 11g/dl, MCV 69 fl, Leucocytes 7800, Plaquettes 370,000, CRP 0.29 mg/dl, Fe 20ng/ml, Ferritine 18 ng/ml, Glucose 105 mg/dl, GOT, GPT, GGT, F alcaline, Bilirubine totale, Cholestérol, Créatinine, Calcium et Phosphore normaux. Anticorps anti-transglutaminase et anti-gliadine négatifs. Gastroscopie : hernie hiatale de 3 cm, le reste sans altération. Coloscopie : jusqu'au cæcum, diverticules isolés dans le sigma. Transit intestinal sans altération. Echographie de l'abdomen sans altération. Parmi les examens suivants, lequel vous semble le plus approprié pour compléter l'étude ?", "full_answer": "Il s'agit d'une étude de l'anémie ferriprive avec une gastroscopie et une coloscopie normales, ayant exclu une éventuelle maladie cœliaque. Nous n'avons pas encore vu le jéjunum et l'iléon, une capsule endoscopique devra donc être réalisée. En cas d'anémie ferriprive, il faut toujours exclure une cause digestive.", "type": "SYSTÈME DIGESTIF", "options": {"1": "Biopsie du jéjunum.", "2": "Artériographie.", "3": "Endoscopie par capsule.", "4": "Radioisotopes.", "5": "IRM pelvienne."}, "correct_option": 3, "explanations": {"1": {"exist": false, "char_ranges": [], "word_ranges": [], "text": ""}, "2": {"exist": false, "char_ranges": [], "word_ranges": [], "text": ""}, "3": {"exist": true, "char_ranges": [[0, 240]], "word_ranges": [[0, 36]], "text": "Il s'agit d'une étude de l'anémie ferriprive avec une gastroscopie et une coloscopie normales, ayant exclu une éventuelle maladie cœliaque. Nous n'avons pas encore vu le jéjunum et l'iléon, une capsule endoscopique devra donc être réalisée."}, "4": {"exist": false, "char_ranges": [], "word_ranges": [], "text": ""}, "5": {"exist": false, "char_ranges": [], "word_ranges": [], "text": ""}}} +{"id": 303, "year": 2016, "question_id_specific": 111, "full_question": "Chez un patient atteint de polyarthrite rhumatoïde sous méthotrexate, prednisone et indométhacine qui présente de manière aiguë des œdèmes et une augmentation de la créatinine plasmatique avec un sédiment urinaire peu expressif et une protéinurie inférieure à 100 mg/24 h, la cause la plus probable est la suivante :", "full_answer": "L'amylose se manifeste par une protéinurie, généralement de l'ordre de la néphrologie, et a une évolution plus chronique. La GMN associée à la polyarthrite rhumatoïde modifie de façon caractéristique le sédiment, et l'insuffisance rénale due aux AINS a une évolution plus chronique, avec une nécrose papillaire. Ainsi, compte tenu de l'évolution et des altérations décrites, une néphrite interstitielle due au méthotrexate est plus probable, qui est plus directement toxique qu'allergique, d'où l'absence de pyurie, d'éosinophilie, de fièvre ou d'autres signes d'hypersensibilité.", "type": "NEPHROLOGIE", "options": {"1": "Amyloïdose rénale.", "2": "Glomérulonéphrite secondaire à la polyarthrite rhumatoïde.", "3": "Insuffisance rénale due aux anti-inflammatoires non stéroïdiens.", "4": "Néphrite interstitielle due au méthotrexate.", "5": null}, "correct_option": 4, "explanations": {"1": {"exist": true, "char_ranges": [[0, 121]], "word_ranges": [[0, 18]], "text": "L'amylose se manifeste par une protéinurie, généralement de l'ordre de la néphrologie, et a une évolution plus chronique."}, "2": {"exist": true, "char_ranges": [[122, 212]], "word_ranges": [[18, 31]], "text": "La GMN associée à la polyarthrite rhumatoïde modifie de façon caractéristique le sédiment,"}, "3": {"exist": true, "char_ranges": [[216, 311]], "word_ranges": [[32, 46]], "text": "l'insuffisance rénale due aux AINS a une évolution plus chronique, avec une nécrose papillaire."}, "4": {"exist": true, "char_ranges": [[319, 580]], "word_ranges": [[47, 81]], "text": "compte tenu de l'évolution et des altérations décrites, une néphrite interstitielle due au méthotrexate est plus probable, qui est plus directement toxique qu'allergique, d'où l'absence de pyurie, d'éosinophilie, de fièvre ou d'autres signes d'hypersensibilité."}, "5": {"exist": false, "char_ranges": [], "word_ranges": [], "text": ""}}} +{"id": 539, "year": 2021, "question_id_specific": 3, "full_question": "Une patiente de 45 ans a présenté un premier épisode de lombalgie qui a duré trois semaines et qui l'a empêchée de mener ses activités normales. Elle ne signale aucun traumatisme ni aucune autre pathologie sous-jacente. L'examen physique ne révèle aucun déficit neurologique. Quel examen d'imagerie serait indiqué ?", "full_answer": "Un premier épisode de lombalgie aiguë (d'une durée inférieure à 6 semaines) ne nécessite pas d'examens d'imagerie en l'absence de signaux d'alarme pouvant indiquer un processus sous-jacent tel qu'une fracture, une infection ou une tumeur maligne.", "type": "TRAUMATOLOGIE", "options": {"1": "Comme il s'agissait du premier épisode de lombalgie, seule une simple radiographie de la colonne lombaire a été effectuée.", "2": "L'imagerie par résonance magnétique, qui fournit davantage d'informations sur les tissus mous et les hernies éventuelles.", "3": "Le scanner, pour mieux évaluer la structure osseuse et les éventuelles fractures.", "4": "Pas d'indication de test d'imagerie.", "5": null}, "correct_option": 4, "explanations": {"1": {"exist": false, "char_ranges": [], "word_ranges": [], "text": ""}, "2": {"exist": false, "char_ranges": [], "word_ranges": [], "text": ""}, "3": {"exist": false, "char_ranges": [], "word_ranges": [], "text": ""}, "4": {"exist": true, "char_ranges": [[0, 246]], "word_ranges": [[0, 36]], "text": "Un premier épisode de lombalgie aiguë (d'une durée inférieure à 6 semaines) ne nécessite pas d'examens d'imagerie en l'absence de signaux d'alarme pouvant indiquer un processus sous-jacent tel qu'une fracture, une infection ou une tumeur maligne."}, "5": {"exist": false, "char_ranges": [], "word_ranges": [], "text": ""}}} +{"id": 408, "year": 2018, "question_id_specific": 190, "full_question": "Femme de 45 ans atteinte d'un cancer du sein de stade IV avec de multiples métastases osseuses, sous traitement hormonal et bisphosphonates iv. Elle s'est présentée à son cabinet en signalant des douleurs osseuses intenses (EVA=7), en divers endroits, sans symptômes neurologiques associés, et des exarcerbations de la douleur (EVA=9) plusieurs fois par jour qui l'obligeaient à rester au repos. Dans le cadre d'un traitement par ibuprofène (600 mg toutes les 8 h) et tramadol (150 mg par jour), quel traitement vous paraîtrait le plus approprié pour cette patiente afin d'améliorer la douleur ?", "full_answer": "La bonne réponse est 4 car si la patiente a des métastases osseuses et risque de souffrir de fractures, nous pouvons contrôler la douleur inflammatoire avec de l'ibuprofène et cela nous aidera également à utiliser la synergie antalgique. Nous devrions changer le traitement pour un opioïde majeur et traiter vigoureusement la douleur disruptive car c'est presque la douleur maximale qu'elle peut supporter.", "type": "SOINS PRIMAIRES", "options": {"1": "Arrêter l'ibuprofène et le tramadol et commencer un opioïde majeur comme la morphine ou l'oxycodone.", "2": "Arrêter le tramadol, maintenir l'ibuprofène et commencer un opioïde majeur comme la morphine et l'oxycodone.", "3": "Arrêter l'ibuprofène et le tramadol et commencer un opioïde majeur tel que la morphine ou l'oxycodone tout en commençant le traitement de la douleur de rupture avec le fentanyl sublingual.", "4": "Arrêter le tramadol, maintenir l'ibuprofène et commencer un opioïde majeur tel que la morphine et l'oxycodone et en même temps traiter les percées de douleur avec du fentanyl sublingual.", "5": null}, "correct_option": 4, "explanations": {"1": {"exist": false, "char_ranges": [], "word_ranges": [], "text": ""}, "2": {"exist": false, "char_ranges": [], "word_ranges": [], "text": ""}, "3": {"exist": false, "char_ranges": [], "word_ranges": [], "text": ""}, "4": {"exist": true, "char_ranges": [[0, 406]], "word_ranges": [[0, 62]], "text": "La bonne réponse est 4 car si la patiente a des métastases osseuses et risque de souffrir de fractures, nous pouvons contrôler la douleur inflammatoire avec de l'ibuprofène et cela nous aidera également à utiliser la synergie antalgique. Nous devrions changer le traitement pour un opioïde majeur et traiter vigoureusement la douleur disruptive car c'est presque la douleur maximale qu'elle peut supporter."}, "5": {"exist": false, "char_ranges": [], "word_ranges": [], "text": ""}}} +{"id": 29, "year": 2011, "question_id_specific": 60, "full_question": "Un homme de 71 ans, fumeur de 2 paquets de cigarettes par jour depuis 50 ans, avec une bonne qualité de vie et aucun autre antécédent pathologique intéressant, a consulté pour une toux et une expectoration hémoptotique. L'examen physique n'a révélé aucune anomalie pathologique significative. La radiographie thoracique simple a montré une masse de 4 x 3,4 cm en bas à droite, ce qui a été confirmé par la tomodensitométrie thoraco-abdominale, qui a localisé la lésion dans le lobe inférieur et n'a pas trouvé d'adénopathies ni d'images de métastases. La bronchoscopie a révélé une lésion infiltrante dans la pyramide basale droite, biopsiée comme un carcinome épidermoïde bien différencié. En ce qui concerne les épreuves fonctionnelles respiratoires, la CVF est de 3 300 cc (84 %) et le VEMS 1 est de 2 240 cc (80 %). Quelle devrait être la prochaine étape à suivre chez ce patient ?", "full_answer": "1 et 5 : Ce sont des tests diagnostiques, ils n'ont pas de sens puisque le diagnostic a déjà été posé. 2 : L'âge ne devrait pas être un facteur limitant, surtout si l'on considère qu'il n'a que 71 ans, mais surtout parce que la \"qualité de vie\" du patient est bonne (et je dis \"qualité de vie\" parce que c'est un terme utilisé à tort, il est censé signifier qu'il maintient un bon niveau fonctionnel). 3 : Le patient a une très légère limitation respiratoire et n'a pas besoin de ce test. 4 : C'est la bonne réponse.", "type": "NEUROLOGIE ET CHIRURGIE THORACIQUE", "options": {"1": "Effectuer une médiastinoscopie.", "2": "Une intervention chirurgicale à visée curative devrait être contre-indiquée en raison de l'âge avancé du patient.", "3": "Un test de consommation d'oxygène doit être effectué, compte tenu de la limitation respiratoire.", "4": "Une résection chirurgicale (lobectomie du lobe inférieur du poumon droit) doit être directement indiquée.", "5": "Une ponction trantoracique doit être effectuée."}, "correct_option": 4, "explanations": {"1": {"exist": true, "char_ranges": [[9, 102]], "word_ranges": [[4, 21]], "text": "Ce sont des tests diagnostiques, ils n'ont pas de sens puisque le diagnostic a déjà été posé."}, "2": {"exist": true, "char_ranges": [[107, 401]], "word_ranges": [[23, 76]], "text": "L'âge ne devrait pas être un facteur limitant, surtout si l'on considère qu'il n'a que 71 ans, mais surtout parce que la \"qualité de vie\" du patient est bonne (et je dis \"qualité de vie\" parce que c'est un terme utilisé à tort, il est censé signifier qu'il maintient un bon niveau fonctionnel)."}, "3": {"exist": true, "char_ranges": [[406, 488]], "word_ranges": [[78, 93]], "text": "Le patient a une très légère limitation respiratoire et n'a pas besoin de ce test."}, "4": {"exist": false, "char_ranges": [], "word_ranges": [], "text": ""}, "5": {"exist": true, "char_ranges": [[9, 102]], "word_ranges": [[4, 21]], "text": "Ce sont des tests diagnostiques, ils n'ont pas de sens puisque le diagnostic a déjà été posé."}}} +{"id": 10, "year": 2011, "question_id_specific": 37, "full_question": "Une femme de 76 ans, sans antécédents d'hypertension, consulte pour un ictère indolore et un prurit avec anorexie. Les examens de laboratoire révèlent un taux de bilirubine de 12 mg/dl (avec 9,5 de bilirubine directe). L'échographie abdominale montre une dilatation des voies biliaires intrahépatiques et extrahépatiques, ainsi qu'un nodule unique de 2 cm situé en périphérie sur la face antérieure du lobe gauche. La tomodensitométrie confirme les résultats, démontrant également la présence d'une masse de 3,5 cm dans la tête du pancréas. L'aspiration à l'aiguille fine confirme le diagnostic d'adénocarcinome. Indiquez la meilleure option thérapeutique :", "full_answer": "Je pense que la bonne réponse est 4, bien que les deux lésions soient résécables, l'adénocarcinome pancréatique avec métastases hépatiques synchrones n'a pas d'indication chirurgicale parce qu'il ne prolonge pas la survie et a une morbidité élevée. Le pontage chirurgical entraîne plus de complications que la mise en place d'une prothèse métallique auto-expansible dont la durée médiane de perméabilité est de 6 mois, ce qui est similaire à la médiane de survie d'un patient atteint de métastases hépatiques d'adénocarcinome pancréatique.", "type": "CHIRURGIE", "options": {"1": "Chimio-/radiothérapie adjuvante, chirurgie radicale de conditionnement en cas de réponse initiale.", "2": "Drainage externe percutané à titre palliatif, avec conversion possible en drainage interne en cas d'intolérance ou de complications.", "3": "Pontage biliaire chirurgical, avec ou sans gastrojéjunostomie prophylactique en fonction des résultats peropératoires.", "4": "Prothèse biliaire métallique par cholangiographie rétrograde endoscopique, avec possibilité de chimiothérapie palliative.", "5": "Duodénopancréatectomie céphalique avec ablation par radiofréquence ou alcoolisation de la lésion hépatique."}, "correct_option": 4, "explanations": {"1": {"exist": false, "char_ranges": [], "word_ranges": [], "text": ""}, "2": {"exist": false, "char_ranges": [], "word_ranges": [], "text": ""}, "3": {"exist": false, "char_ranges": [], "word_ranges": [], "text": ""}, "4": {"exist": true, "char_ranges": [[0, 248]], "word_ranges": [[0, 37]], "text": "Je pense que la bonne réponse est 4, bien que les deux lésions soient résécables, l'adénocarcinome pancréatique avec métastases hépatiques synchrones n'a pas d'indication chirurgicale parce qu'il ne prolonge pas la survie et a une morbidité élevée."}, "5": {"exist": false, "char_ranges": [], "word_ranges": [], "text": ""}}} +{"id": 284, "year": 2016, "question_id_specific": 57, "full_question": "Une femme de 72 ans se présente à l'hôpital avec une douleur thoracique oppressante depuis 2 heures. La tension artérielle est de 68/32 mm Hg, la fréquence cardiaque de 124 battements/min, la fréquence respiratoire de 32 respirations/min, la saturation en oxygène de 91 % avec de l'oxygène à 50 %, des crépitants bilatéraux à l'auscultation pulmonaire. L'ECG montre une lésion sous-épicardique sur la face antérieure, avec une image miroir sur la face inférieure. La radiographie du thorax montre un schéma alvéolaire bilatéral. Quelle est votre décision thérapeutique ?", "full_answer": "Effectuer une coronarographie d'urgence.", "type": "CARDIOLOGIE ET CHIRURGIE VASCULAIRE", "options": {"1": "Commencer la perfusion de nitrate par voie intraveineuse.", "2": "Effectuer une fibrinolyse avec du tenecteplase (TNK).", "3": "Réaliser une angioplastie coronaire urgente.", "4": "Administrer 40 mg de furosémide par voie i.v.", "5": null}, "correct_option": 3, "explanations": {"1": {"exist": false, "char_ranges": [], "word_ranges": [], "text": ""}, "2": {"exist": false, "char_ranges": [], "word_ranges": [], "text": ""}, "3": {"exist": true, "char_ranges": [[0, 40]], "word_ranges": [[0, 4]], "text": "Effectuer une coronarographie d'urgence."}, "4": {"exist": false, "char_ranges": [], "word_ranges": [], "text": ""}, "5": {"exist": false, "char_ranges": [], "word_ranges": [], "text": ""}}} +{"id": 188, "year": 2013, "question_id_specific": 230, "full_question": "Un patient de 57 ans a subi un infarctus aigu du myocarde et son analyse de sang a révélé un taux de cholestérol de 312 mg/dl, un taux de LDL de 241 mg/dl, un taux de HDL de 29 et un taux de TG normal. Il fumait 30 cigarettes par jour, n'était pas obèse, sa tension artérielle était de 145/90 mm Hg et il n'était pas diabétique. Interrogé sur ses antécédents familiaux, il a mentionné qu'un oncle du côté de sa mère était décédé subitement à l'âge de 53 ans. Quelle est la réponse avec laquelle vous êtes le plus d'accord ?", "full_answer": "Question sur un sujet à la mode ces dernières années : la dyslipidémie ; j'y répondrais en l'écartant. Le traitement de l'hypercholestérolémie étant les statines, nous excluons les réponses 2 et 4. La statine chez un patient atteint d'une maladie cardiovasculaire (comme un IAM) est un traitement à vie (nous écartons la réponse 1) car elle n'est pas prise tant que le taux de LDL n'est pas normalisé. Entre 3 et 5, 5 est plus complet car il inclut le traitement et le contrôle des autres facteurs de risque. Il est facile d'exclure une dyslipidémie secondaire grâce aux antécédents médicaux du patient et à un test de la fonction thyroïdienne.", "type": "ENDOCRINOLOGIE", "options": {"1": "Traiter avec des statines jusqu'à ce que le cholestérol soit normalisé et recommander un mode de vie sain (régime alimentaire, exercice physique et interdiction de fumer).", "2": "Éliminer l'hypothyroïdie ou une autre maladie systémique, recommander un mode de vie sain et traiter avec des fibrates pour normaliser le cholestérol.", "3": "Il n'est pas nécessaire d'exclure une dyslipidémie secondaire car elle est rare. Un profil lipidique familial doit être réalisé.", "4": "Éliminer la dyslipidémie secondaire, traiter avec des résines (résincholestyramine) et des fibrates et s'engager à nouveau dans un mode de vie sain. Interdire l'alcool.", "5": "Éliminer les dyslipidémies secondaires, demander une étude familiale et traiter avec des statines pour maintenir un bon profil lipidique. Contrôle des autres facteurs de risque."}, "correct_option": 5, "explanations": {"1": {"exist": true, "char_ranges": [[198, 401]], "word_ranges": [[32, 68]], "text": "La statine chez un patient atteint d'une maladie cardiovasculaire (comme un IAM) est un traitement à vie (nous écartons la réponse 1) car elle n'est pas prise tant que le taux de LDL n'est pas normalisé."}, "2": {"exist": true, "char_ranges": [[103, 197]], "word_ranges": [[18, 32]], "text": "Le traitement de l'hypercholestérolémie étant les statines, nous excluons les réponses 2 et 4."}, "3": {"exist": true, "char_ranges": [[402, 508]], "word_ranges": [[68, 89]], "text": "Entre 3 et 5, 5 est plus complet car il inclut le traitement et le contrôle des autres facteurs de risque."}, "4": {"exist": true, "char_ranges": [[103, 197]], "word_ranges": [[18, 32]], "text": "Le traitement de l'hypercholestérolémie étant les statines, nous excluons les réponses 2 et 4."}, "5": {"exist": true, "char_ranges": [[402, 508]], "word_ranges": [[68, 89]], "text": "Entre 3 et 5, 5 est plus complet car il inclut le traitement et le contrôle des autres facteurs de risque."}}} +{"id": 349, "year": 2016, "question_id_specific": 160, "full_question": "Une femme de 45 ans, mère de trois enfants, se présente à une consultation de diagnostic précoce. La cytologie cervicovaginale est compatible avec une lésion intraépithéliale malpighienne de haut grade. Parmi les options suivantes, laquelle choisiriez-vous ?", "full_answer": "La bonne réponse est 2. Selon l'Oncoguide SEGO 2014 sur la prévention du cancer du col de l'utérus, si la cytologie révèle une lésion malpighienne intraépithéliale de haut grade, la mesure à prendre est une colposcopie, avec ou sans biopsie selon les résultats (indépendamment de l'âge et de la parité de la patiente).", "type": "GYNÉCOLOGIE ET OBSTÉTRIQUE", "options": {"1": "Répéter la cytologie dans un mois.", "2": "Colposcopie avec éventuellement une biopsie.", "3": "Hystérectomie avec salpingectomie bilatérale et conservation des ovaires.", "4": "Curetage fractionné.", "5": null}, "correct_option": 2, "explanations": {"1": {"exist": false, "char_ranges": [], "word_ranges": [], "text": ""}, "2": {"exist": true, "char_ranges": [[24, 318]], "word_ranges": [[5, 53]], "text": "Selon l'Oncoguide SEGO 2014 sur la prévention du cancer du col de l'utérus, si la cytologie révèle une lésion malpighienne intraépithéliale de haut grade, la mesure à prendre est une colposcopie, avec ou sans biopsie selon les résultats (indépendamment de l'âge et de la parité de la patiente)."}, "3": {"exist": false, "char_ranges": [], "word_ranges": [], "text": ""}, "4": {"exist": false, "char_ranges": [], "word_ranges": [], "text": ""}, "5": {"exist": false, "char_ranges": [], "word_ranges": [], "text": ""}}} +{"id": 290, "year": 2016, "question_id_specific": 50, "full_question": "Un homme de 58 ans a signalé des lésions cutanées progressives et légèrement douloureuses sur son bras gauche depuis trois semaines. Ces lésions avaient commencé par une lésion érythémateuse sur le pouce gauche. Il présentait des stries rougeâtres visibles entre les lésions. Le patient n'avait pas de fièvre ni d'autres symptômes généraux. Il avait travaillé dans son jardin mais ne se souvenait d'aucune blessure. Le diagnostic étiologique a été établi par la culture d'une biopsie cutanée. Quel est l'agent causal le plus probable de ce processus ?", "full_answer": "Les lésions sporotrichoïdes ou lymphocutanées décrites peuvent correspondre à la sporotrichose ou au granulome de l'aquarium, de sorte que les deux premières seraient exclues. Le fait d'avoir travaillé dans le jardin oriente davantage vers la sporotrichose, bien que le fait que la question demande \"l'agent causal\" et non le nom de la maladie soit un peu trompeur, de sorte que si l'on considère la sporotrichose, la réponse devrait en fait être Sporothrix schenckii.", "type": "DERMATOLOGIE, VÉNÉRÉOLOGIE ET CHIRURGIE PLASTIQUE", "options": {"1": "Dermatophytose due à Microsporum gypseum.", "2": "Infection cutanée à Staphylococcus aureus.", "3": "Sporotrichose.", "4": "Infection cutanée à Mycobacterium marinum.", "5": null}, "correct_option": 3, "explanations": {"1": {"exist": true, "char_ranges": [[0, 175]], "word_ranges": [[0, 24]], "text": "Les lésions sporotrichoïdes ou lymphocutanées décrites peuvent correspondre à la sporotrichose ou au granulome de l'aquarium, de sorte que les deux premières seraient exclues."}, "2": {"exist": true, "char_ranges": [[0, 175]], "word_ranges": [[0, 24]], "text": "Les lésions sporotrichoïdes ou lymphocutanées décrites peuvent correspondre à la sporotrichose ou au granulome de l'aquarium, de sorte que les deux premières seraient exclues."}, "3": {"exist": true, "char_ranges": [[176, 257]], "word_ranges": [[24, 36]], "text": "Le fait d'avoir travaillé dans le jardin oriente davantage vers la sporotrichose,"}, "4": {"exist": true, "char_ranges": [[176, 257]], "word_ranges": [[24, 36]], "text": "Le fait d'avoir travaillé dans le jardin oriente davantage vers la sporotrichose,"}, "5": {"exist": false, "char_ranges": [], "word_ranges": [], "text": ""}}} +{"id": 305, "year": 2016, "question_id_specific": 174, "full_question": "Homme de 87 ans ayant des antécédents d'hypertension et de gonarthrose. Situation de base avec une autonomie fonctionnelle et cognitive complète qui lui permet de continuer à vivre seul dans la communauté. Il est régulièrement traité au périndopril et aux diurétiques thiazidiques pour contrôler sa tension artérielle et prend régulièrement de l'ibuprofène 1800 mg/jour pour contrôler les symptômes de sa gonarthrose. Après un examen de routine, on a observé une tension artérielle persistante de 190 et un TAD de 80 mmHg. Quelle serait la modification thérapeutique la plus raisonnable pour parvenir à contrôler les chiffres de la tension artérielle ?", "full_answer": "L'un des mécanismes à l'origine de l'effet hypotenseur des inhibiteurs de l'ECA tels que le périndopril est l'action accrue des prostaglandines vasodilatatrices sur le rein, ce qui diminue la rétention d'hydrosaline et augmente donc la diurèse et abaisse la tension artérielle. Les AINS inhibent la synthèse des prostaglandines et diminuent la perfusion rénale, antagonisant ainsi l'effet hypotenseur de nombreux antihypertenseurs, y compris les inhibiteurs de l'ECA. Par conséquent, il est préférable chez ce patient, avant d'augmenter le médicament, de passer de l'ibuprofène au paracétamol, qui n'a pas cet effet, et de réévaluer la situation ultérieurement.", "type": "NEPHROLOGIE", "options": {"1": "J'ajouterais un bloqueur de calcium.", "2": "J'augmenterais la dose d'hydrochlorothiazide à 25 mg/jour.", "3": "Je remplacerais l'ibuprofène par le paracétamol pour éviter l'influence possible du paracétamol sur l'effet des antihypertenseurs.", "4": "J'ajouterais un alpha-bloquant en raison de la forte prévalence du syndrome de la prostate chez les hommes de cet âge.", "5": null}, "correct_option": 3, "explanations": {"1": {"exist": false, "char_ranges": [], "word_ranges": [], "text": ""}, "2": {"exist": false, "char_ranges": [], "word_ranges": [], "text": ""}, "3": {"exist": true, "char_ranges": [[278, 661]], "word_ranges": [[41, 95]], "text": "Les AINS inhibent la synthèse des prostaglandines et diminuent la perfusion rénale, antagonisant ainsi l'effet hypotenseur de nombreux antihypertenseurs, y compris les inhibiteurs de l'ECA. Par conséquent, il est préférable chez ce patient, avant d'augmenter le médicament, de passer de l'ibuprofène au paracétamol, qui n'a pas cet effet, et de réévaluer la situation ultérieurement."}, "4": {"exist": false, "char_ranges": [], "word_ranges": [], "text": ""}, "5": {"exist": false, "char_ranges": [], "word_ranges": [], "text": ""}}} +{"id": 104, "year": 2012, "question_id_specific": 91, "full_question": "Un homme de 34 ans a été adressé à la clinique pour des taux persistants de cholestérol LDL supérieurs à 250 mg/dL. Le patient était asymptomatique mais, à l'examen, il présentait des plaques indurées blanchâtres sur les coudes, les genoux et les paupières. Sa tension est de 135/85 mmHg. Son père est décédé d'un infarctus du myocarde à l'âge de 48 ans. Sa prise de sang à jeun montre un cholestérol total de 346 mg/dl, un cholestérol LDL de 278 mg/dl, un cholestérol HDL de 42 mg/dl, des triglycérides de 130 mg/dl et un glucose de 93 mg/dl. En ce qui concerne la maladie dont souffre ce patient, l'une des affirmations suivantes est FAUSSE. Veuillez préciser laquelle :", "full_answer": "Il s'agit d'une hypercholestérolémie familiale hétérozygote. Il faut savoir reconnaître la maladie et connaître les critères. Cela me semble être une grande difficulté.", "type": "ENDOCRINOLOGIE", "options": {"1": "Il s'agit d'une maladie héréditaire co-dominante autosomique, avec une forte pénétrance, de sorte qu'il est fréquent que certains des parents et des frères et sœurs de l'individu en souffrent également.", "2": "Cette maladie génétique affecte le gène codant pour le récepteur du cholestérol LDL et plus de 900 mutations ont été décrites.", "3": "Cette maladie génétique affecte également la synthèse des triglycérides, ce qui entraîne une augmentation des triglycérides dans les stades avancés de la maladie, généralement au-dessus de 500 mg/dl.", "4": "Les hommes non traités ont près de 50 % de chances de souffrir d'un accident coronarien avant l'âge de 60 ans, et les femmes atteintes de la maladie ont une prévalence plus élevée de cardiopathie ischémique que la population féminine générale.", "5": "Les statines sont efficaces dans le traitement de la forme hétérozygote de la maladie, mais doivent souvent être associées à des inhibiteurs de l'absorption du cholestérol pour atteindre les objectifs thérapeutiques et contrôler la maladie."}, "correct_option": 3, "explanations": {"1": {"exist": false, "char_ranges": [], "word_ranges": [], "text": ""}, "2": {"exist": false, "char_ranges": [], "word_ranges": [], "text": ""}, "3": {"exist": true, "char_ranges": [[0, 125]], "word_ranges": [[0, 16]], "text": "Il s'agit d'une hypercholestérolémie familiale hétérozygote. Il faut savoir reconnaître la maladie et connaître les critères."}, "4": {"exist": false, "char_ranges": [], "word_ranges": [], "text": ""}, "5": {"exist": false, "char_ranges": [], "word_ranges": [], "text": ""}}} +{"id": 38, "year": 2011, "question_id_specific": 232, "full_question": "Une femme de 52 ans consulte pour avoir remarqué une coloration jaunâtre des conjonctives au cours de la semaine précédente. Elle ne signale aucun comportement sexuel à risque ni aucun antécédent épidémiologique de risque d'hépatite virale. Elle ne consomme pas d'alcool ni de médicaments hépatotoxiques. Elle signale un an d'antécédents de prurit généralisé, d'asthénie, de sécheresse buccale et d'absence de larmoiement pour une cause sans rapport avec l'hépatite virale. Le reste de l'anamnèse n'a révélé aucun signe pathologique. L'examen physique a révélé des lésions de grattage, un ictère conjonctival et une hépatomégalie non douloureuse. Le patient a fourni des analyses sanguines effectuées dans son entreprise et dont les résultats pathologiques sont les suivants : bilirubine totale 3 mg/dl, phosphatase alcaline 400 UI, vitesse de sédimentation 40 mm à une heure. Indiquez quelle serait la recommandation la plus appropriée pour établir le diagnostic étiologique des symptômes du patient :", "full_answer": "Une femme d'âge moyen présentant des symptômes semblables à ceux de Sjögren (absence de larmoiement, bouche sèche), avec hyperbilirubinémie, ictère et hépatomégalie, suggère une cirrhose biliaire primitive. Les anticorps antimitochondriaux (AMA IgG) sont utilisés pour diagnostiquer cette maladie. Il s'agit donc de la réponse 1.", "type": "OPHTHALMOLOGIE", "options": {"1": "Anticorps anti-mitochondriaux.", "2": "Étude du métabolisme du fer.", "3": "Étude du métabolisme du cuivre.", "4": "Imagerie par résonance magnétique du foie.", "5": "Sérologie du virus B et du virus C."}, "correct_option": 1, "explanations": {"1": {"exist": true, "char_ranges": [[0, 297]], "word_ranges": [[0, 38]], "text": "Une femme d'âge moyen présentant des symptômes semblables à ceux de Sjögren (absence de larmoiement, bouche sèche), avec hyperbilirubinémie, ictère et hépatomégalie, suggère une cirrhose biliaire primitive. Les anticorps antimitochondriaux (AMA IgG) sont utilisés pour diagnostiquer cette maladie."}, "2": {"exist": false, "char_ranges": [], "word_ranges": [], "text": ""}, "3": {"exist": false, "char_ranges": [], "word_ranges": [], "text": ""}, "4": {"exist": false, "char_ranges": [], "word_ranges": [], "text": ""}, "5": {"exist": false, "char_ranges": [], "word_ranges": [], "text": ""}}} +{"id": 376, "year": 2016, "question_id_specific": 130, "full_question": "Un homme de 39 ans a présenté au cours des trois derniers mois deux épisodes de vastes mouvements involontaires de saut des membres gauches qui, en l'espace de deux minutes environ, ont entraîné une perte de conscience. L'étude neuroradiologique a mis en évidence une lésion expansive frontale droite évoquant un gliome cérébral. Comment classeriez-vous l'épilepsie de ce patient ?", "full_answer": "Ils font référence à une crise partielle suivie d'une perte de conscience. Il s'agit donc à proprement parler d'une crise partielle complexe symptomatique et non d'une crise partielle secondairement généralisée (qui se produit lorsque les mouvements tonico-cloniques s'étendent au reste des membres). Pour cette raison, l'option la plus correcte serait la deuxième, car elle ne précise pas qu'il s'agit d'une crise partielle symptomatique simple, mais laisse ouverte la possibilité qu'il s'agisse d'une crise partielle symptomatique simple ou complexe.", "type": "NEUROLOGIE", "options": {"1": "Symptomatique généralisé.", "2": "Symptomatique partiel.", "3": "Cryptogène partiel.", "4": "Symptomatique partiel, secondairement généralisé.", "5": null}, "correct_option": 2, "explanations": {"1": {"exist": false, "char_ranges": [], "word_ranges": [], "text": ""}, "2": {"exist": true, "char_ranges": [[320, 552]], "word_ranges": [[45, 78]], "text": "l'option la plus correcte serait la deuxième, car elle ne précise pas qu'il s'agit d'une crise partielle symptomatique simple, mais laisse ouverte la possibilité qu'il s'agisse d'une crise partielle symptomatique simple ou complexe."}, "3": {"exist": false, "char_ranges": [], "word_ranges": [], "text": ""}, "4": {"exist": true, "char_ranges": [[0, 300]], "word_ranges": [[0, 42]], "text": "Ils font référence à une crise partielle suivie d'une perte de conscience. Il s'agit donc à proprement parler d'une crise partielle complexe symptomatique et non d'une crise partielle secondairement généralisée (qui se produit lorsque les mouvements tonico-cloniques s'étendent au reste des membres)."}, "5": {"exist": false, "char_ranges": [], "word_ranges": [], "text": ""}}} +{"id": 163, "year": 2013, "question_id_specific": 122, "full_question": "Un homme de 66 ans ayant des antécédents de spondylarthrite ankylosante de longue date. Il a présenté une protéinurie de 6 grammes par jour, une hypoalbuminémie et des œdèmes. Aucune hématurie n'a été détectée dans le sédiment urinaire. Sa créatinine plasmatique était de 1,6 mg/dl et son taux de filtration glomérulaire de 45 ml/min. Sa glycémie est de 110 mg/dl. Sa grave déformation de la colonne vertébrale rend difficile la réalisation d'une biopsie rénale percutanée. Quelle est l'approche initiale correcte ?", "full_answer": "La bonne réponse est : 3. biopsie de la graisse sous-cutanée. Le lupus érythémateux, certaines vascularites, la maladie de Goodpasture et d'autres maladies systémiques (dont la spondylarthrite ankylosante) affectent le rein et conditionnent son pronostic. L'apparition d'anomalies urinaires ou d'une atteinte rénale d'origine parenchymateuse sont des indications de biopsie rénale, même avec des gammes de protéinurie inférieures à celles admises dans les néphropathies primaires. La détermination des anticorps anti-GBM et l'ANCAS sont utiles au diagnostic, mais ne remplacent pas la biopsie rénale, car ils n'ont pas de valeur pronostique et n'aident pas à planifier le traitement. Dans ce cas, où nous sommes amenés à penser que la biopsie rénale n'est pas possible, nous devons relier l'EA à l'insuffisance rénale. Dans la plupart des cas, cette relation est établie par la néphropathie à IgA, qui se manifeste par une hématurie, un symptôme qui n'existe pas dans ce cas. Un nombre non négligeable de patients atteints de MA avancée présente une amyloïdose, qui correspond également à la symptomatologie du patient. Par conséquent, et étant donné que l'amylose peut être diagnostiquée par une biopsie de la graisse sous-cutanée, c'est la réponse 3 qui doit être considérée comme valide.", "type": "NEPHROLOGIE", "options": {"1": "Introduire des corticostéroïdes en cas de suspicion de maladie glomérulaire à modification minime.", "2": "Traiter par cyclophosphamide en cas de suspicion de glomérulopathie membraneuse.", "3": "Biopsie de la graisse sous-cutanée.", "4": "Commencer la dialyse.", "5": "Effectuer un test de surcharge en glucose pour exclure une néphropathie diabétique."}, "correct_option": 3, "explanations": {"1": {"exist": false, "char_ranges": [], "word_ranges": [], "text": ""}, "2": {"exist": false, "char_ranges": [], "word_ranges": [], "text": ""}, "3": {"exist": true, "char_ranges": [[1136, 1290]], "word_ranges": [[170, 195]], "text": "et étant donné que l'amylose peut être diagnostiquée par une biopsie de la graisse sous-cutanée, c'est la réponse 3 qui doit être considérée comme valide."}, "4": {"exist": false, "char_ranges": [], "word_ranges": [], "text": ""}, "5": {"exist": false, "char_ranges": [], "word_ranges": [], "text": ""}}} +{"id": 148, "year": 2012, "question_id_specific": 158, "full_question": "Une femme de 23 ans s'est présentée au service des urgences parce qu'elle était très nerveuse après une dispute avec son partenaire. Ses antécédents médicaux font état de plusieurs plaintes similaires au cours de l'année précédente, dont deux à la suite d'un geste d'automutilation. Des conflits fréquents dans sa relation avec son partenaire, des changements au travail et des disputes familiales sont également notés. Elle dit se sentir incomprise par tout le monde, y compris par les psychiatres qui la voient. Le diagnostic est le suivant :", "full_answer": "Cette femme de 23 ans est en colère. Elle a besoin de soutien et de beaucoup d'éducation émotionnelle pour freiner son impulsivité... elle se sent incomprise... elle a peut-être raison... Le problème est qu'elle ne supporte pas le rejet et qu'elle a des gestes d'automutilation. Le diagnostic est 1. 3, 4 et 5 ne sont même pas proches, il n'y a pas de description clinique dans la question. Ensuite, dans la vie réelle, en explorant bien, il y aurait presque certainement des symptômes dissociatifs et de dépersonnalisation, ainsi que des symptômes dépressifs... mais symptôme n'est pas trouble. Pourquoi pas 2 ? Il pourrait aussi s'agir d'un filage très spécifique et très fin... c'est là que la discussion est servie. Je vais mettre les critères de chacun et nous en parlerons,... mais l'intention de l'examinateur était le 1 et je vois qu'il est bien défendu. Elle ne peut être remise en cause. - La limite : Un modèle général d'instabilité dans les relations interpersonnelles, l'image de soi et l'auto-efficacité, et une impulsivité notable, commençant au début de l'âge adulte et se produisant dans une variété de contextes, comme indiqué par cinq (ou plus) des éléments suivants : 1. Efforts frénétiques pour éviter l'abandon réel ou imaginé. Un modèle de relations interpersonnelles instables et intenses caractérisées par l'alternance d'extrêmes d'idéalisation et de dévalorisation. 3. troubles de l'identité : instabilité marquée et persistante de l'image ou du sentiment de soi. Impulsivité dans au moins deux domaines, susceptible de porter atteinte à l'intégrité de la personne (par exemple, dépenses, sexe, abus de substances, conduite dangereuse, frénésie alimentaire). Note : Ne pas inclure les comportements suicidaires ou d'automutilation énumérés dans le critère 5. 5. comportements, tentatives ou menaces suicidaires récurrents, ou comportements d'automutilation. 6. instabilité affective due à une réactivité marquée de l'humeur (par exemple, épisodes de dysphorie, d'irritabilité ou d'anxiété intenses, durant généralement quelques heures et rarement quelques jours) 7. un sentiment chronique de vide 8. colère inappropriée et intense ou difficultés à gérer la colère (par exemple, manifestations fréquentes de colère, colère constante, bagarres physiques récurrentes) 9. une idéation paranoïaque transitoire liée au stress ou des symptômes dissociatifs graves. - L'histrionique : un schéma général d'émotivité excessive et de recherche d'attention, débutant au début de l'âge adulte et se produisant dans divers contextes, comme indiqué par cinq (ou plus) des éléments suivants : 1. L'interaction avec les autres est souvent caractérisée par un comportement sexuellement séduisant ou provocateur. 3. 3. expression émotionnelle superficielle et changeant rapidement 4. utilise constamment son apparence physique pour attirer l'attention sur lui/elle. 5. a une façon de parler excessivement subjective et sans nuances 6. fait preuve d'autodramatisation, de théâtralité et d'expression émotionnelle exagérée 7. est influençable, c'est-à-dire qu'il est facilement influencé par les autres ou par les circonstances. 8. considère ses relations comme plus intimes qu'elles ne le sont en réalité.", "type": "PSYCHIATRIE", "options": {"1": "Trouble de la personnalité limite.", "2": "Trouble de la personnalité histrionique.", "3": "Dysthymie.", "4": "Trouble dissociatif.", "5": "Trouble de la dépersonnalisation."}, "correct_option": 1, "explanations": {"1": {"exist": true, "char_ranges": [[0, 299]], "word_ranges": [[0, 49]], "text": "Cette femme de 23 ans est en colère. Elle a besoin de soutien et de beaucoup d'éducation émotionnelle pour freiner son impulsivité... elle se sent incomprise... elle a peut-être raison... Le problème est qu'elle ne supporte pas le rejet et qu'elle a des gestes d'automutilation. Le diagnostic est 1."}, "2": {"exist": false, "char_ranges": [], "word_ranges": [], "text": ""}, "3": {"exist": true, "char_ranges": [[300, 390]], "word_ranges": [[49, 68]], "text": "3, 4 et 5 ne sont même pas proches, il n'y a pas de description clinique dans la question."}, "4": {"exist": true, "char_ranges": [[300, 390]], "word_ranges": [[49, 68]], "text": "3, 4 et 5 ne sont même pas proches, il n'y a pas de description clinique dans la question."}, "5": {"exist": true, "char_ranges": [[300, 390]], "word_ranges": [[49, 68]], "text": "3, 4 et 5 ne sont même pas proches, il n'y a pas de description clinique dans la question."}}} +{"id": 178, "year": 2013, "question_id_specific": 94, "full_question": "Un garçon de 18 mois, dont le calendrier de vaccination est complet à ce jour, consulte le service des urgences pour un gonflement du genou droit après avoir joué dans un parc, sans traumatisme évident. Dans l'anamnèse, la mère mentionne qu'un de ses oncles a eu des problèmes similaires. L'échographie est compatible avec une hémarthrose et le bilan sanguin réalisé ne met en évidence qu'un allongement du TCA de 52″ (normale 25-35″). Quelle est l'hypothèse diagnostique la plus probable ?", "full_answer": "Dans cette question aussi, ils ne cessent de lâcher des indices pour nous lancer tête baissée dans le bon diagnostic. Reprenons : un petit enfant sans traumatisme évident souffre d'une hémarthrose. Un de ses oncles ayant eu des problèmes similaires, nous soupçonnons une maladie héréditaire. Le TCA, qui évalue la voie de coagulation intrinsèque (impliquant les facteurs V, VIII, IX, X, XI et XII) est allongé. Problème de coagulation héréditaire. On écarte donc les réponses 1, 2 et 5, il reste la maladie de von Willebrand et l'hémophilie A. Je considère que la bonne réponse est la 4, l'hémophilie A, car le mode d'hérédité est lié à l'X et sa principale manifestation clinique est la contusion des tissus mous et l'hémarthrose spontanée, sans traumatisme apparent, alors que la maladie de von Willebrand se caractérise par des saignements après une intervention chirurgicale ou un traumatisme et qu'un temps de saignement prolongé avec une numération plaquettaire normale est caractéristique (donnée non mentionnée dans l'énoncé).", "type": "HÉMATOLOGIE", "options": {"1": "Le syndrome de Marfan.", "2": "La maladie de Von Willebrand.", "3": "La maladie d'Ehlers-Danlos.", "4": "Hémophilie A.", "5": "Maladie de Bemard-Soulier."}, "correct_option": 4, "explanations": {"1": {"exist": true, "char_ranges": [[292, 486]], "word_ranges": [[45, 79]], "text": "Le TCA, qui évalue la voie de coagulation intrinsèque (impliquant les facteurs V, VIII, IX, X, XI et XII) est allongé. Problème de coagulation héréditaire. On écarte donc les réponses 1, 2 et 5,"}, "2": {"exist": true, "char_ranges": [[292, 486]], "word_ranges": [[45, 79]], "text": "Le TCA, qui évalue la voie de coagulation intrinsèque (impliquant les facteurs V, VIII, IX, X, XI et XII) est allongé. Problème de coagulation héréditaire. On écarte donc les réponses 1, 2 et 5,"}, "3": {"exist": true, "char_ranges": [[769, 975]], "word_ranges": [[125, 155]], "text": "alors que la maladie de von Willebrand se caractérise par des saignements après une intervention chirurgicale ou un traumatisme et qu'un temps de saignement prolongé avec une numération plaquettaire normale"}, "4": {"exist": true, "char_ranges": [[544, 768]], "word_ranges": [[89, 125]], "text": "Je considère que la bonne réponse est la 4, l'hémophilie A, car le mode d'hérédité est lié à l'X et sa principale manifestation clinique est la contusion des tissus mous et l'hémarthrose spontanée, sans traumatisme apparent,"}, "5": {"exist": true, "char_ranges": [[292, 486]], "word_ranges": [[45, 79]], "text": "Le TCA, qui évalue la voie de coagulation intrinsèque (impliquant les facteurs V, VIII, IX, X, XI et XII) est allongé. Problème de coagulation héréditaire. On écarte donc les réponses 1, 2 et 5,"}}} +{"id": 100, "year": 2012, "question_id_specific": 85, "full_question": "Homme de 24 ans signalant une asthénie progressive depuis au moins 6 mois, une voix rauque, une élocution lente, une somnolence et un gonflement des mains, des pieds et du visage. Examen : pouls à 52 battements par minute, visage abotarada et peau sèche et pâle. NFS : anémie légère, cholestérol 385 mg/dL (normale <220), créatinine 1,3 mg/dL (normale 0,5-1,1), protéinurie négative, TSH 187μIU/mL (normale 0,35-5,5) et T4 libre 0,2 ng/dL (normale 0,85-1,86). Quelle stratégie vous semble la plus appropriée ?", "full_answer": "Il s'agit d'une hypothyroïdie primaire sévère. Le traitement consiste à administrer de la thyroxine. En l'absence de nodule, ni l'écho ni la FNA ne sont indiqués.", "type": "ENDOCRINOLOGIE", "options": {"1": "Effectuer une échographie de la thyroïde avant de commencer le traitement.", "2": "Déterminer la T3 libre et effectuer une IRM de l'hypophyse.", "3": "Traiter avec de la L-triiodothyronine et une statine.", "4": "Traiter avec de la L-Tyroxine et déterminer les anticorps anti-thyroïdiens.", "5": "Effectuer une cytologie thyroïdienne avant le traitement."}, "correct_option": 4, "explanations": {"1": {"exist": true, "char_ranges": [[101, 162]], "word_ranges": [[14, 26]], "text": "En l'absence de nodule, ni l'écho ni la FNA ne sont indiqués."}, "2": {"exist": false, "char_ranges": [], "word_ranges": [], "text": ""}, "3": {"exist": false, "char_ranges": [], "word_ranges": [], "text": ""}, "4": {"exist": true, "char_ranges": [[0, 100]], "word_ranges": [[0, 14]], "text": "Il s'agit d'une hypothyroïdie primaire sévère. Le traitement consiste à administrer de la thyroxine."}, "5": {"exist": true, "char_ranges": [[101, 162]], "word_ranges": [[14, 26]], "text": "En l'absence de nodule, ni l'écho ni la FNA ne sont indiqués."}}} +{"id": 398, "year": 2016, "question_id_specific": 140, "full_question": "Vous évaluez une patiente de 66 ans qui présente des douleurs à l'aine accentuées par une station debout prolongée quelques jours par mois. La radiographie simple des hanches montre un rétrécissement de l'espace articulaire fémoro-acétabulaire, une sclérose et des ostéophytes. Quelle est votre approche ?", "full_answer": "Il s'agit d'une patiente de 66 ans qui présente des douleurs certains jours, des douleurs occasionnelles. Bien que le diagnostic soit clairement celui de l'arthrose, il ne conduit pas directement à une indication chirurgicale. La première étape consiste à essayer un traitement conservateur (nous avons exclu la première option). L'arthrose est un processus dégénératif que les opioïdes n'arrêtent certainement pas, de sorte que la deuxième option est exclue. L'option quatre peut également être écartée rapidement : l'énoncé précise le diagnostic et le traitement à suivre : dans ce cas, l'IRM ne va pas nous fournir d'informations intéressantes.", "type": "TRAUMATOLOGIE ET ORTHOPÉDIE", "options": {"1": "Je pose le diagnostic de coxarthrose et envoie le traumatologue poser une prothèse de hanche.", "2": "Initiation d'un traitement avec des opioïdes faibles qui ont démontré leur capacité à stopper la progression de la maladie.", "3": "J'ai commencé le traitement avec du paracétamol, j'ai expliqué que l'évolution est très variable et que l'indication d'une intervention chirurgicale dépend de la fonctionnalité et du contrôle de la douleur.", "4": "En raison des caractéristiques radiologiques décrites, j'ai besoin d'une IRM de la hanche avant de prendre une décision thérapeutique.", "5": null}, "correct_option": 3, "explanations": {"1": {"exist": true, "char_ranges": [[227, 329]], "word_ranges": [[34, 49]], "text": "La première étape consiste à essayer un traitement conservateur (nous avons exclu la première option)."}, "2": {"exist": true, "char_ranges": [[330, 459]], "word_ranges": [[49, 68]], "text": "L'arthrose est un processus dégénératif que les opioïdes n'arrêtent certainement pas, de sorte que la deuxième option est exclue."}, "3": {"exist": true, "char_ranges": [[227, 329]], "word_ranges": [[34, 49]], "text": "La première étape consiste à essayer un traitement conservateur (nous avons exclu la première option)."}, "4": {"exist": true, "char_ranges": [[460, 647]], "word_ranges": [[68, 97]], "text": "L'option quatre peut également être écartée rapidement : l'énoncé précise le diagnostic et le traitement à suivre : dans ce cas, l'IRM ne va pas nous fournir d'informations intéressantes."}, "5": {"exist": false, "char_ranges": [], "word_ranges": [], "text": ""}}} +{"id": 28, "year": 2011, "question_id_specific": 58, "full_question": "Un âne de 30 ans subit une radiographie pulmonaire pour officialiser un contrat de travail avec une entreprise. La radiographie montre un tableau interstitiel bilatéral de type réticulaire, des adénopathies hilaires et médiastinales bilatérales. Une bronchofibroscopie avec lavage broncho-alvéolaire est indiquée et la numération cellulaire donne les résultats suivants : lymphocytes 50%, histiocytes 40%, éosinophiles 2%, polymorphonucléaires 8% et rapport CD4/Cd8 5 Quel est le diagnostic le plus probable ?", "full_answer": "Vous décrivez une sarcoïdose.", "type": "NEUROLOGIE ET CHIRURGIE THORACIQUE", "options": {"1": "Alvéolite allergique extrinsèque.", "2": "Pneumoconiose.", "3": "La sarcoïdose.", "4": "Lymphangite carcinomateuse.", "5": "Hémosidérose pulmonaire idiopathique."}, "correct_option": 3, "explanations": {"1": {"exist": false, "char_ranges": [], "word_ranges": [], "text": ""}, "2": {"exist": false, "char_ranges": [], "word_ranges": [], "text": ""}, "3": {"exist": true, "char_ranges": [[0, 29]], "word_ranges": [[0, 4]], "text": "Vous décrivez une sarcoïdose."}, "4": {"exist": false, "char_ranges": [], "word_ranges": [], "text": ""}, "5": {"exist": false, "char_ranges": [], "word_ranges": [], "text": ""}}} +{"id": 587, "year": 2022, "question_id_specific": 76, "full_question": "Une femme de 40 ans consulte parce qu'elle a remarqué une grosseur dans le quadrant supéro-externe du sein droit depuis un mois. Elle présente un rapport de mammographie décrivant une lésion de type BIRADS 3. Quelle est la meilleure conduite à tenir ?", "full_answer": "Toutes les autres réponses sont incorrectes. Avec un BIRADS 3, c'est l'attitude à adopter.", "type": "OBSTÉTRIQUE ET GYNÉCOLOGIE", "options": {"1": "Rassurez-le, car un examen d'imagerie a déjà été effectué et une tumeur maligne a été exclue.", "2": "Cette classification implique probablement une intervention chirurgicale étant donné que la probabilité de cancer est supérieure à 10 %. Il le lui explique et l'oriente préférentiellement vers l'unité du sein.", "3": "Il s'agit probablement d'une découverte bénigne, le risque de cancer étant inférieur à 2 %. Il explique que cela nécessite un suivi tous les 6 à 12 mois jusqu'à 24 mois ou une biopsie.", "4": "Les résultats sont peu suspects de cancer (2-10%), mais une biopsie est nécessaire.", "5": null}, "correct_option": 3, "explanations": {"1": {"exist": false, "char_ranges": [], "word_ranges": [], "text": ""}, "2": {"exist": false, "char_ranges": [], "word_ranges": [], "text": ""}, "3": {"exist": true, "char_ranges": [[45, 90]], "word_ranges": [[6, 14]], "text": "Avec un BIRADS 3, c'est l'attitude à adopter."}, "4": {"exist": false, "char_ranges": [], "word_ranges": [], "text": ""}, "5": {"exist": false, "char_ranges": [], "word_ranges": [], "text": ""}}} +{"id": 554, "year": 2022, "question_id_specific": 50, "full_question": "Un homme de 58 ans se présente pour un bilan de santé programmé pour un diabète de type 2 diagnostiqué il y a 6 ans. Il a des antécédents personnels d'hypertension, de dyslipidémie, de cardiopathie ischémique et de pancréatite d'origine biliaire. Son traitement comprend du lisinopril, du métoprolol, de la metformine, de l'AAS et de l'atorvastatine. L'examen physique révèle une tension artérielle de 151/93 mmHg, un IMC de 27,1 kg/m2. L'hémoglobine glyquée est de 8,3 %. Lequel des traitements suivants est le plus approprié pour ce patient ?", "full_answer": "Patient souffrant d'obésité, d'hypertension et d'une augmentation de l'HbA1C. Dans des études récentes, l'Empaglifozin, un inhibiteur hautement sélectif du SGLT210, réduit de manière significative la tension artérielle, l'HbA1C et l'IMC. Le glipizide fait partie du groupe des sulfonylurées. Son utilisation doit être évitée chez les patients souffrant de maladies cardiaques. L'acarbose peut interagir avec les bêta-bloquants, tels que ceux pris par notre patient. Il a été démontré que la sitagliptine diminue l'HbA1C mais pas l'HT ni l'IMC.", "type": "ENDOCRINOLOGIE", "options": {"1": "Empaglifozin.", "2": "Glipizide.", "3": "Acarbose.", "4": "Sitagliptine.", "5": null}, "correct_option": 1, "explanations": {"1": {"exist": true, "char_ranges": [[0, 237]], "word_ranges": [[0, 30]], "text": "Patient souffrant d'obésité, d'hypertension et d'une augmentation de l'HbA1C. Dans des études récentes, l'Empaglifozin, un inhibiteur hautement sélectif du SGLT210, réduit de manière significative la tension artérielle, l'HbA1C et l'IMC."}, "2": {"exist": true, "char_ranges": [[238, 376]], "word_ranges": [[30, 50]], "text": "Le glipizide fait partie du groupe des sulfonylurées. Son utilisation doit être évitée chez les patients souffrant de maladies cardiaques."}, "3": {"exist": true, "char_ranges": [[377, 465]], "word_ranges": [[50, 63]], "text": "L'acarbose peut interagir avec les bêta-bloquants, tels que ceux pris par notre patient."}, "4": {"exist": true, "char_ranges": [[466, 543]], "word_ranges": [[63, 77]], "text": "Il a été démontré que la sitagliptine diminue l'HbA1C mais pas l'HT ni l'IMC."}, "5": {"exist": false, "char_ranges": [], "word_ranges": [], "text": ""}}} +{"id": 251, "year": 2014, "question_id_specific": 81, "full_question": "Une femme de 50 ans s'est présentée aux urgences pour asthénie, anorexie, perte de poids, jaunisse, fièvre et douleurs abdominales. Elle n'a pas signalé d'antécédents récents de transfusion sanguine, de contacts sexuels à risque ou de dépendance à l'égard de drogues injectables. Consommation habituelle de 60 grammes d'alcool par jour au cours des cinq dernières années, qui a augmenté au cours du mois précédent en raison de problèmes familiaux. Température de 38,5ºC, tension artérielle de 100/60 mmHg. L'examen physique a révélé une désorientation temporelle et spatiale, une malnutrition, un astérixis, un ictère et une hépatomégalie douloureuse. Il n'y avait pas d'ascite ni de signe d'irritation péritonéale. Les examens de laboratoire montrent des leucocytes à 15 000/microlitre dont 90 % de polymorphonucléaires, des globules rouges à 3 millions/mm3, une hémoglobine à 10g/dl, un volume corpusculaire moyen à 115 fl, une bilirubine à 15 mg/dl à prédominance directe, une AST (GOT) à 300 UI/L, une ALT (GPT) à 120 UI/L, une GGT à 635 UI/L, un allongement du temps de prothrombine supérieur à 50 %. Quel est le diagnostic le plus probable de l'état de la patiente ?", "full_answer": "Un patient présentant une consommation d'alcool importante (60g et une femme), une fièvre élevée, une encéphalopathie hépatique (désorientation, astérixis), une bilirubine directe très élevée, un ictère, une hépatomégalie douloureuse, une coagulopathie (temps de prothrombine élevé) et une légère élévation des transaminases (moins de 5 fois la limite supérieure de la normale) suggèrent une hépatite alcoolique aiguë.", "type": "SYSTÈME DIGESTIF", "options": {"1": "Abcès du foie.", "2": "Cholécystite aiguë.", "3": "Cholangite aiguë.", "4": "Hépatite alcoolique.", "5": "Pancréatite aiguë."}, "correct_option": 4, "explanations": {"1": {"exist": false, "char_ranges": [], "word_ranges": [], "text": ""}, "2": {"exist": false, "char_ranges": [], "word_ranges": [], "text": ""}, "3": {"exist": false, "char_ranges": [], "word_ranges": [], "text": ""}, "4": {"exist": true, "char_ranges": [[0, 418]], "word_ranges": [[0, 56]], "text": "Un patient présentant une consommation d'alcool importante (60g et une femme), une fièvre élevée, une encéphalopathie hépatique (désorientation, astérixis), une bilirubine directe très élevée, un ictère, une hépatomégalie douloureuse, une coagulopathie (temps de prothrombine élevé) et une légère élévation des transaminases (moins de 5 fois la limite supérieure de la normale) suggèrent une h��patite alcoolique aiguë."}, "5": {"exist": false, "char_ranges": [], "word_ranges": [], "text": ""}}} +{"id": 128, "year": 2012, "question_id_specific": 79, "full_question": "Une femme mariée de 42 ans, mère de deux enfants en âge scolaire, consulte son médecin de famille pour des céphalées sévères qui se développent depuis environ 7 jours. Bien qu'elle ait connu des épisodes similaires pendant plus d'un an, ceux-ci se sont considérablement aggravés au cours des deux derniers mois. La douleur, semblable à un poids, commence dans la région occipitale, s'étend aux deux régions temporales et n'est guère soulagée par la prise de comprimés de 650 mg de paracétamol, si bien qu'elle demande un scanner (elle est convaincue que \"quelque chose ne va pas dans sa tête\"). Laquelle des causes suivantes est la plus plausible pour expliquer son mal de tête ?", "full_answer": "Tout comme l'année dernière, ils ont voulu dépeindre une infirmière comme mauvaise pour nous avoir réveillés pour un patient atteint de myasthénie, cette année ils nous divertissent en présentant un patient qui demande un scanner. Je les interprète comme montrant une patiente stressée par ses enfants et souffrant de maux de tête récurrents de localisation occipitale qui étaient soulagés par une faible dose de paracétamol et qui ne le sont plus aujourd'hui. La localisation initialement occipitale, la durée d'évolution de 2 ans et la sensation de pesanteur orientent toutes vers la réponse 4, une céphalée de tension. La migraine et les troubles vasculaires, accompagnés d'autres symptômes neurologiques ou de nausées, de photophobie, etc., sont rapidement écartés, de même que la rare artérite temporale avant l'âge de 50 ans. Les céphalées tumorales sont généralement \"en crescendo\", et non récurrentes comme le suggère le cas, et s'accompagnent d'autres symptômes neurologiques ou d'une hypertension intracrânienne.", "type": "NEUROLOGIE ET NEUROCHIRURGIE", "options": {"1": "La migraine.", "2": "Troubles vasculaires.", "3": "Artérite de l'artère temporale.", "4": "Le tensionnel.", "5": "Oncologie."}, "correct_option": 4, "explanations": {"1": {"exist": true, "char_ranges": [[622, 831]], "word_ranges": [[97, 128]], "text": "La migraine et les troubles vasculaires, accompagnés d'autres symptômes neurologiques ou de nausées, de photophobie, etc., sont rapidement écartés, de même que la rare artérite temporale avant l'âge de 50 ans."}, "2": {"exist": true, "char_ranges": [[622, 831]], "word_ranges": [[97, 128]], "text": "La migraine et les troubles vasculaires, accompagnés d'autres symptômes neurologiques ou de nausées, de photophobie, etc., sont rapidement écartés, de même que la rare artérite temporale avant l'âge de 50 ans."}, "3": {"exist": true, "char_ranges": [[622, 831]], "word_ranges": [[97, 128]], "text": "La migraine et les troubles vasculaires, accompagnés d'autres symptômes neurologiques ou de nausées, de photophobie, etc., sont rapidement écartés, de même que la rare artérite temporale avant l'âge de 50 ans."}, "4": {"exist": true, "char_ranges": [[461, 621]], "word_ranges": [[72, 97]], "text": "La localisation initialement occipitale, la durée d'évolution de 2 ans et la sensation de pesanteur orientent toutes vers la réponse 4, une céphalée de tension."}, "5": {"exist": true, "char_ranges": [[832, 1022]], "word_ranges": [[128, 152]], "text": "Les céphalées tumorales sont généralement \"en crescendo\", et non récurrentes comme le suggère le cas, et s'accompagnent d'autres symptômes neurologiques ou d'une hypertension intracrânienne."}}} +{"id": 0, "year": 2011, "question_id_specific": 34, "full_question": "Une femme de 27 ans a été examinée dans le service de dermatologie pour des épisodes récurrents d'aphtes buccaux. Elle est adressée au service gastro-intestinal pour une analyse sanguine montrant une Hb de 11,5 g/dl Fe 38 AST 52 ALT 64 Anti-trasglutaminase IgA 177. La patiente ne signale qu'une gêne abdominale diffuse occasionnelle. Elle ne signale aucun signe d'hémorragie ou de jaunisse. L'examen physique est normal. Lequel des tests suivants vous semble le plus approprié pour établir le diagnostic ?", "full_answer": "Dans cette question, ils donnent un élément d'information important lorsqu'il s'agit de suspecter un diagnostic, à savoir les anticorps anti-transglutaminase qui devraient vous amener à suspecter une maladie cœliaque. Les autres symptômes correspondent parfaitement au tableau et, bien que plusieurs des tests mentionnés soient probablement effectués, la réponse qui nous permettra de poser le diagnostic est la réponse 4 (biopsie intestinale).", "type": "DIGESTIF", "options": {"1": "Endoscopie par capsule.", "2": "Coloscopie.", "3": "Sérologie des virus de l'hépatite b et c.", "4": "Biopsie intestinale.", "5": "Échographie abdominale."}, "correct_option": 4, "explanations": {"1": {"exist": false, "char_ranges": [], "word_ranges": [], "text": ""}, "2": {"exist": false, "char_ranges": [], "word_ranges": [], "text": ""}, "3": {"exist": false, "char_ranges": [], "word_ranges": [], "text": ""}, "4": {"exist": true, "char_ranges": [[0, 444]], "word_ranges": [[0, 61]], "text": "Dans cette question, ils donnent un élément d'information important lorsqu'il s'agit de suspecter un diagnostic, à savoir les anticorps anti-transglutaminase qui devraient vous amener à suspecter une maladie cœliaque. Les autres symptômes correspondent parfaitement au tableau et, bien que plusieurs des tests mentionnés soient probablement effectués, la réponse qui nous permettra de poser le diagnostic est la réponse 4 (biopsie intestinale)."}, "5": {"exist": false, "char_ranges": [], "word_ranges": [], "text": ""}}} +{"id": 256, "year": 2014, "question_id_specific": 162, "full_question": "Une femme de 80 ans consulte pour une augmentation progressive de la circonférence abdominale au cours du dernier mois. Au cours des 4 à 5 mois précédents, elle avait noté une asthénie, une anorexie et une perte de poids non précisée. L'examen physique a révélé une ascite de tension et un œdème malléolaire, sans autre signe pertinent. L'échographie et la tomodensitométrie de l'abdomen ont montré la présence d'une ascite de densité homogène, sans implants péritonéaux ni masses abdominales ou pelviennes. Le foie, le pancréas, les glandes surrénales, la rate et les reins n'ont rien révélé de significatif. Une paracentèse a été réalisée à l'aide d'une aiguille épaisse, obtenant difficilement un liquide jaunâtre, dense et gélatineux. Quelle est l'étiologie la plus probable ?", "full_answer": "Les caractéristiques du liquide décrit sont laiteuses, c'est-à-dire un liquide ascitique chyleux. Compte tenu de l'âge du patient et du syndrome constitutionnel décrit, il est très probable qu'il s'agisse d'un lymphome non hodgkinien et non de la tuberculose, qui présente également un liquide ascitique chyleux, étant donné qu'il n'est pas fait mention de symptômes infectieux. Les autres entités décrites ne présentent pas de liquide ascitique chyleux.", "type": "SYSTÈME DIGESTIF", "options": {"1": "Décompensation hydropique secondaire à une cirrhose du foie.", "2": "Ascite chyleuse due à un lymphome non hodgkinien.", "3": "Tuberculose péritonéale.", "4": "Métastases péritonéales d'un adénocarcinome.", "5": "Ascite secondaire à une péricardite constrictive."}, "correct_option": 2, "explanations": {"1": {"exist": true, "char_ranges": [[379, 454]], "word_ranges": [[55, 66]], "text": "Les autres entités décrites ne présentent pas de liquide ascitique chyleux."}, "2": {"exist": true, "char_ranges": [[98, 378]], "word_ranges": [[12, 55]], "text": "Compte tenu de l'âge du patient et du syndrome constitutionnel décrit, il est très probable qu'il s'agisse d'un lymphome non hodgkinien et non de la tuberculose, qui présente également un liquide ascitique chyleux, étant donné qu'il n'est pas fait mention de symptômes infectieux."}, "3": {"exist": true, "char_ranges": [[98, 378]], "word_ranges": [[12, 55]], "text": "Compte tenu de l'âge du patient et du syndrome constitutionnel décrit, il est très probable qu'il s'agisse d'un lymphome non hodgkinien et non de la tuberculose, qui présente également un liquide ascitique chyleux, étant donné qu'il n'est pas fait mention de symptômes infectieux."}, "4": {"exist": true, "char_ranges": [[379, 454]], "word_ranges": [[55, 66]], "text": "Les autres entités décrites ne présentent pas de liquide ascitique chyleux."}, "5": {"exist": true, "char_ranges": [[379, 454]], "word_ranges": [[55, 66]], "text": "Les autres entités décrites ne présentent pas de liquide ascitique chyleux."}}} +{"id": 604, "year": 2022, "question_id_specific": 113, "full_question": "Un homme de 35 ans, facteur, sans antécédents, s'est présenté aux urgences pour des douleurs cervicales aiguës depuis 24 heures, sans antécédents traumatiques, irradiant dans le bras gauche jusqu'à la main et accompagnées d'une paresthésie du bord radial de l'avant-bras. Il n'y a pas de perte objective de force, le patient conserve la mobilité du cou bien qu'il soit douloureux et il y a une contracture des muscles paravertébraux. La première approche est la suivante :", "full_answer": "Douleur cervicale sans signe avant-coureur. Traitement conservateur avec un taux de réussite de 75%-90%.", "type": "TRAUMATOLOGIE", "options": {"1": "Traitement conservateur à base d'anti-inflammatoires non stéroïdiens, de chaleur locale et de repos relatif.", "2": "Appel urgent au neurochirurgien pour une évaluation chirurgicale.", "3": "Demande préférentielle pour l'IRM et l'électromyogramme.", "4": "Orientation préférentielle vers les cliniques ambulatoires spécialisées dans les traumatismes.", "5": null}, "correct_option": 1, "explanations": {"1": {"exist": true, "char_ranges": [[0, 104]], "word_ranges": [[0, 14]], "text": "Douleur cervicale sans signe avant-coureur. Traitement conservateur avec un taux de réussite de 75%-90%."}, "2": {"exist": false, "char_ranges": [], "word_ranges": [], "text": ""}, "3": {"exist": false, "char_ranges": [], "word_ranges": [], "text": ""}, "4": {"exist": false, "char_ranges": [], "word_ranges": [], "text": ""}, "5": {"exist": false, "char_ranges": [], "word_ranges": [], "text": ""}}} +{"id": 362, "year": 2016, "question_id_specific": 96, "full_question": "Une femme de 25 ans s'est présentée au service des urgences avec des ecchymoses et des pétéchies sur les membres inférieurs et des gingivorragies depuis quelques jours. Son hémogramme montre les données suivantes : Hb 13 g/dL, leucocytes 8 500/microL avec formule leucocytaire normale et plaquettes 9 000/mm3. Le temps de coagulation est normal. Lequel des tests diagnostiques suivants ne doit PAS être effectué ?", "full_answer": "La première chose à faire est de connaître le diagnostic du cas clinique qui nous est présenté, il s'agit d'une thrombocytopénie immunitaire primaire (ITP), seule la série plaquettaire est altérée avec une coagulation normale. Le premier test à effectuer est un frottis de sang périphérique pour vérifier si ces plaquettes sont réelles et s'il n'y a pas de pseudothrombopénie due à des agrégats plaquettaires. Dans l'étude d'extension, les anticorps antinucléaires (ANA) sont demandés pour exclure les pathologies auto-immunes qui sont la cause de la thrombopénie et un autre test qui serait indiqué bien que dans ce cas le patient soit jeune et ne présente pas de signes de gravité (leucopénie, anémie, pas de réponse au traitement de première ligne...) est une aspiration de la moelle osseuse, pour voir s'il y a des mégacaryocytes formant des thrombocytes et confirmer ainsi que la destruction des plaquettes est d'origine périphérique. Mais ce qui n'est pas indiqué, c'est la détermination de la mutation JAK-2, qui est utilisée dans le diagnostic des syndromes myéloprolifératifs chroniques, ce qui n'est pas le cas ici.", "type": "HÉMATOLOGIE", "options": {"1": "Mutation JAK-2.", "2": "Aspiration de la moelle osseuse.", "3": "Anticorps antinucléaires.", "4": "Frottis sanguin périphérique.", "5": null}, "correct_option": 1, "explanations": {"1": {"exist": true, "char_ranges": [[945, 1125]], "word_ranges": [[148, 177]], "text": "ce qui n'est pas indiqué, c'est la détermination de la mutation JAK-2, qui est utilisée dans le diagnostic des syndromes myéloprolifératifs chroniques, ce qui n'est pas le cas ici."}, "2": {"exist": true, "char_ranges": [[567, 939]], "word_ranges": [[86, 147]], "text": "un autre test qui serait indiqué bien que dans ce cas le patient soit jeune et ne présente pas de signes de gravité (leucopénie, anémie, pas de réponse au traitement de première ligne...) est une aspiration de la moelle osseuse, pour voir s'il y a des mégacaryocytes formant des thrombocytes et confirmer ainsi que la destruction des plaquettes est d'origine périphérique."}, "3": {"exist": true, "char_ranges": [[410, 563]], "word_ranges": [[64, 85]], "text": "Dans l'étude d'extension, les anticorps antinucléaires (ANA) sont demandés pour exclure les pathologies auto-immunes qui sont la cause de la thrombopénie"}, "4": {"exist": true, "char_ranges": [[227, 409]], "word_ranges": [[34, 64]], "text": "Le premier test à effectuer est un frottis de sang périphérique pour vérifier si ces plaquettes sont réelles et s'il n'y a pas de pseudothrombopénie due à des agrégats plaquettaires."}, "5": {"exist": false, "char_ranges": [], "word_ranges": [], "text": ""}}} +{"id": 410, "year": 2018, "question_id_specific": 195, "full_question": "Un homme de 75 ans est admis à l'hôpital parce qu'il refuse de s'alimenter et a perdu 30 % de son poids. Il a des antécédents de sclérose en plaques qui se sont aggravés au cours des derniers mois. Sa seule parente, sa sœur, est décédée il y a trois mois et depuis, son état s'est détérioré. Il est devenu incontinent, a cessé de manger et de participer aux événements sociaux. Il souffre de dépression psychotique. Physiquement, il est capable de manger. Des médicaments psychotropes sont mis en place et interrompus en raison d'effets indésirables. Si son état nutritionnel s'améliore, il est possible de reprendre ses médicaments. Il refuse les fluides intraveineux, puis les accepte, mais la perfusion est commencée quelques heures plus tard. Une alternative consiste à pratiquer une gastrostomie endoscopique percutanée (GEP) et à le renvoyer à son domicile. Laquelle des mesures suivantes est la plus appropriée pour ce patient ?", "full_answer": "La bonne réponse est 1, car le patient est dans une situation de dépression psychotique et n'est pas en mesure de décider de sa vie et il ne semble pas que le patient ait rempli ses dernières volontés, la chose la plus appropriée à faire est de présenter le cas au comité d'éthique du centre afin qu'il prenne une décision sur la question conformément à la loi.", "type": "SOINS PRIMAIRES", "options": {"1": "Demandez au comité d'éthique de l'hôpital de vous aider à déterminer la décision qui est dans le meilleur intérêt du patient.", "2": "Évaluer si la résidence accueille le patient si le PEG n'est pas effectué.", "3": "Demander une ordonnance du tribunal pour effectuer une PEG", "4": "Inscription du patient dans un programme de soins palliatifs en phase terminale", "5": null}, "correct_option": 1, "explanations": {"1": {"exist": true, "char_ranges": [[28, 361]], "word_ranges": [[6, 67]], "text": "le patient est dans une situation de dépression psychotique et n'est pas en mesure de décider de sa vie et il ne semble pas que le patient ait rempli ses dernières volontés, la chose la plus appropriée à faire est de présenter le cas au comité d'éthique du centre afin qu'il prenne une décision sur la question conformément à la loi."}, "2": {"exist": false, "char_ranges": [], "word_ranges": [], "text": ""}, "3": {"exist": false, "char_ranges": [], "word_ranges": [], "text": ""}, "4": {"exist": false, "char_ranges": [], "word_ranges": [], "text": ""}, "5": {"exist": false, "char_ranges": [], "word_ranges": [], "text": ""}}} +{"id": 361, "year": 2016, "question_id_specific": 94, "full_question": "Un homme de 51 ans a été admis aux urgences pour une anémie macrocytaire (Hb 6,3 g/dL, MCV 120 fL). Les études ont exclu une origine carentielle. Les réticulocytes sont au nombre de 24000/microL. L'étude de la moelle osseuse est compatible avec un syndrome myélodysplasique (SMD). La cytogénétique montre une délétion 5q. Quelle est l'affirmation correcte concernant ce patient ?", "full_answer": "Il s'agit d'un jeune patient sans comorbidité, qui est donc candidat à un traitement à visée curative, de sorte que l'option 4 est exclue. L'option 3 n'est pas correcte non plus car nous ne pouvons pas connaître l'IPSS du patient puisque nous ne connaissons pas le nombre de blastes et le nombre de leucocytes et de plaquettes pour calculer le risque. L'option 2 est également incorrecte car l'indication d'une greffe de moelle osseuse dans le cas d'un SMD concerne les patients présentant une SSPI élevée ou intermédiaire-2, et dans ce cas, nous ne connaissons pas le risque, la seule chose que nous savons à propos du risque est la cytogénétique, qui est de bon pronostic, ce qui nous donnerait 0 point. La bonne réponse est donc 1, car il est vrai que la cytogénétique est favorable (5q-) et qu'il existe un traitement spécifique, le lénalidomide, également utilisé dans d'autres pathologies telles que le myélome multiple.", "type": "HÉMATOLOGIE", "options": {"1": "Cette délétion (5q-) est une maladie de bon pronostic qui bénéficie d'un traitement spécifique (lénalidomide).", "2": "Chez ce patient, il serait souhaitable d'effectuer un typage HLA afin d'organiser une transplantation allogénique.", "3": "Il s'agit d'un patient dont l'indice pronostique international (IPSS) est élevé.", "4": "Dans ce cas, le traitement consisterait uniquement en une transfusion.", "5": null}, "correct_option": 1, "explanations": {"1": {"exist": true, "char_ranges": [[706, 926]], "word_ranges": [[121, 155]], "text": "La bonne réponse est donc 1, car il est vrai que la cytogénétique est favorable (5q-) et qu'il existe un traitement spécifique, le lénalidomide, également utilisé dans d'autres pathologies telles que le myélome multiple."}, "2": {"exist": true, "char_ranges": [[352, 705]], "word_ranges": [[61, 121]], "text": "L'option 2 est également incorrecte car l'indication d'une greffe de moelle osseuse dans le cas d'un SMD concerne les patients présentant une SSPI élevée ou intermédiaire-2, et dans ce cas, nous ne connaissons pas le risque, la seule chose que nous savons à propos du risque est la cytogénétique, qui est de bon pronostic, ce qui nous donnerait 0 point."}, "3": {"exist": true, "char_ranges": [[139, 351]], "word_ranges": [[24, 61]], "text": "L'option 3 n'est pas correcte non plus car nous ne pouvons pas connaître l'IPSS du patient puisque nous ne connaissons pas le nombre de blastes et le nombre de leucocytes et de plaquettes pour calculer le risque."}, "4": {"exist": true, "char_ranges": [[0, 138]], "word_ranges": [[0, 24]], "text": "Il s'agit d'un jeune patient sans comorbidité, qui est donc candidat à un traitement à visée curative, de sorte que l'option 4 est exclue."}, "5": {"exist": false, "char_ranges": [], "word_ranges": [], "text": ""}}} +{"id": 172, "year": 2013, "question_id_specific": 50, "full_question": "Un patient souffrant d'une BPCO sévère se présente au service des urgences avec un gaz du sang artériel mesurant 31 % de Fi02 au niveau de la mer, une Pa02 de 86 mm Hg, une PaC02 de 65 mm Hg, un pH de 7,13 et un bicarbonate de 27 mmol/litre. Laquelle des affirmations suivantes est FAUSSE ?", "full_answer": "L'analyse des gaz du sang montre une acidose respiratoire aiguë, due à une augmentation de la PCO2 avec un temps d'évolution court car le rein n'a pas encore eu le temps de retenir les bicarbonates (taux de bicarbonate à la limite supérieure de la normalité). Le patient n'est pas en hyperventilation mais en hypoventilation car la PCO2 est élevée. Le gradient alvéolo-artériel d'oxygène est altéré car le patient a une PaO2 normale, mais parce qu'il a une FiO2 élevée. S'il était privé d'oxygène, il serait hypoxémique. Une acidose respiratoire inférieure à 7,20 est une raison d'envisager une ventilation mécanique non invasive de manière aiguë et en principe temporaire chez un patient atteint d'une BPCO sévère.", "type": "PNEUMOLOGIE", "options": {"1": "Le patient est en hyperventilation.", "2": "Le gradient d'oxygène artériel alvéolaire est élevé.", "3": "Le patient est en acidose respiratoire.", "4": "Le taux de bicarbonate est normal.", "5": "La mise en place d'une ventilation mécanique doit être envisagée."}, "correct_option": 1, "explanations": {"1": {"exist": true, "char_ranges": [[260, 348]], "word_ranges": [[45, 59]], "text": "Le patient n'est pas en hyperventilation mais en hypoventilation car la PCO2 est élevée."}, "2": {"exist": true, "char_ranges": [[349, 469]], "word_ranges": [[59, 79]], "text": "Le gradient alvéolo-artériel d'oxygène est altéré car le patient a une PaO2 normale, mais parce qu'il a une FiO2 élevée."}, "3": {"exist": true, "char_ranges": [[0, 259]], "word_ranges": [[0, 45]], "text": "L'analyse des gaz du sang montre une acidose respiratoire aiguë, due à une augmentation de la PCO2 avec un temps d'évolution court car le rein n'a pas encore eu le temps de retenir les bicarbonates (taux de bicarbonate à la limite supérieure de la normalité)."}, "4": {"exist": true, "char_ranges": [[135, 259]], "word_ranges": [[23, 45]], "text": "le rein n'a pas encore eu le temps de retenir les bicarbonates (taux de bicarbonate à la limite supérieure de la normalité)."}, "5": {"exist": true, "char_ranges": [[521, 715]], "word_ranges": [[86, 115]], "text": "Une acidose respiratoire inférieure à 7,20 est une raison d'envisager une ventilation mécanique non invasive de manière aiguë et en principe temporaire chez un patient atteint d'une BPCO sévère."}}} +{"id": 83, "year": 2012, "question_id_specific": 48, "full_question": "Quelle intervention thérapeutique envisageriez-vous chez un patient de 67 ans présentant une cardiomyopathie dilatée idiopathique, un bloc de branche gauche, une fraction d'éjection ventriculaire gauche de 26 %, une régurgitation mitrale de grade II-III/IV, sous traitement par furosémide, spironolactone, énalapril et bisoprolol et qui reste en classe fonctionnelle III de la NYHA ?", "full_answer": "Il s'agit d'un patient souffrant d'insuffisance cardiaque avec dysfonctionnement ventriculaire gauche sévère (FE < 35 %), LBBB (QRS large), qui suit un traitement médical optimal et qui reste malgré tout dans une classe fonctionnelle avancée. La revascularisation coronaire ne semble pas apporter beaucoup d'avantages à ce patient à l'heure actuelle. Le remplacement de la valve mitrale non plus, d'abord parce que la régurgitation mitrale est très probablement due à une dilatation de l'anneau mitral, et ensuite parce que nous aurions besoin d'une évaluation échocardiographique de la valve mitrale pour déterminer s'il y a une altération. L'implantation d'un ballon intra-aortique n'est pas indiquée chez ce patient (il est utilisé dans les situations de choc cardiogénique, dans les complications mécaniques post-AMI, en tant que pont vers le Tx cardiaque...). Le dispositif d'assistance ventriculaire ne serait pas non plus indiqué chez ce patient. Ce qui lui serait vraiment bénéfique parmi toutes les réponses est le numéro 5, le dispositif de resynchronisation. En stimulant simultanément les deux ventricules, on obtient une synergie mécanique qui améliore la fonction cardiaque. Les indications pour l'application de la CRT (thérapie de resynchronisation cardiaque) sont précisément celles mentionnées au début : dysfonctionnement ventriculaire sévère, asynchronisme démontré par la présence d'un QRS large à l'ECG (généralement dû à un LBBB) et classe fonctionnelle avancée (III-IV) en dépit d'un traitement médical optimal.", "type": "CARDIOLOGIE ET CHIRURGIE VASCULAIRE", "options": {"1": "Revascularisation coronaire chirurgicale.", "2": "Remplacement de la valve mitrale.", "3": "Implantation d'une pompe à ballonnet aortique.", "4": "Implantation d'un dispositif d'assistance ventriculaire.", "5": "Implantation d'un système de resynchronisation cardiaque."}, "correct_option": 5, "explanations": {"1": {"exist": true, "char_ranges": [[243, 350]], "word_ranges": [[35, 50]], "text": "La revascularisation coronaire ne semble pas apporter beaucoup d'avantages à ce patient à l'heure actuelle."}, "2": {"exist": true, "char_ranges": [[351, 641]], "word_ranges": [[50, 95]], "text": "Le remplacement de la valve mitrale non plus, d'abord parce que la régurgitation mitrale est très probablement due à une dilatation de l'anneau mitral, et ensuite parce que nous aurions besoin d'une évaluation échocardiographique de la valve mitrale pour déterminer s'il y a une altération."}, "3": {"exist": true, "char_ranges": [[642, 864]], "word_ranges": [[95, 127]], "text": "L'implantation d'un ballon intra-aortique n'est pas indiquée chez ce patient (il est utilisé dans les situations de choc cardiogénique, dans les complications mécaniques post-AMI, en tant que pont vers le Tx cardiaque...)."}, "4": {"exist": true, "char_ranges": [[865, 953]], "word_ranges": [[127, 140]], "text": "Le dispositif d'assistance ventriculaire ne serait pas non plus indiqué chez ce patient."}, "5": {"exist": true, "char_ranges": [[1070, 1188]], "word_ranges": [[158, 174]], "text": "En stimulant simultanément les deux ventricules, on obtient une synergie mécanique qui améliore la fonction cardiaque."}}} +{"id": 141, "year": 2012, "question_id_specific": 135, "full_question": "Garçon de 2 ans, ses antécédents personnels comprennent : 3 épisodes d'otite moyenne aiguë, 1 méningite à méningocoques et 2 pneumonies (l'une dans le lobe moyen et l'autre dans le lobe supérieur gauche). Elle a été admise trois fois pour un purpura thrombocytopénique (à trois reprises, les anticorps antiplaquettaires étaient négatifs et la moelle osseuse montrait des mégacaryocytes normaux). Plusieurs garçons de la famille maternelle sont décédés en bas âge à la suite de processus infectieux. L'examen a révélé des lésions typiques de la dermatite atopique. L'étude immunologique a montré une légère diminution des sous-populations de lymphocytes T ; une élévation des IgA et des IgE ; une diminution des IgM et des IgG à la limite inférieure de la normale. Quel est le diagnostic le plus probable ?", "full_answer": "La bonne réponse est 1. Le syndrome de Wiskott-Aldrich associe un déficit immunitaire, une thrombopénie et une dermatite atopique à l'étude immunologique décrite dans l'énoncé.", "type": "PÉDIATRIE", "options": {"1": "Syndrome de Wiskott-Aldrich.", "2": "Syndrome d'hyperIgE.", "3": "Hypogammaglobulinémie transitoire de l'enfance.", "4": "Déficit immunitaire combiné sévère associé au chromosome X.", "5": "Déficit immunitaire commun variable."}, "correct_option": 1, "explanations": {"1": {"exist": true, "char_ranges": [[24, 176]], "word_ranges": [[5, 25]], "text": "Le syndrome de Wiskott-Aldrich associe un déficit immunitaire, une thrombopénie et une dermatite atopique à l'étude immunologique décrite dans l'énoncé."}, "2": {"exist": false, "char_ranges": [], "word_ranges": [], "text": ""}, "3": {"exist": false, "char_ranges": [], "word_ranges": [], "text": ""}, "4": {"exist": false, "char_ranges": [], "word_ranges": [], "text": ""}, "5": {"exist": false, "char_ranges": [], "word_ranges": [], "text": ""}}} +{"id": 317, "year": 2016, "question_id_specific": 139, "full_question": "Une femme de 70 ans ayant des antécédents d'anorexie, de perte de poids, de gêne dans la musculature et les articulations proximales et de douleur dans la région temporo-mandibulaire se présente au service des urgences pour une perte soudaine et indolore de la vision unilatérale (mouvement des mains) (anomalie pupillaire afférente).", "full_answer": "Il s'agit probablement d'une artérite temporale, le tableau clinique et un test sanguin avec augmentation des réactifs suffisent à confirmer la suspicion. Il faut ensuite procéder à un examen oculaire pour exclure une AIN, commencer un traitement urgent (en cas d'AIN, 3 bolus de méthylprednisolone 1 g, sinon, prednisone mg/kg) et procéder à une biopsie.", "type": "RHEUMATOLOGIE", "options": {"1": "Ponction lombaire.", "2": "Protéine C-Réactive.", "3": "Angiographie par résonance magnétique.", "4": "Échographie carotidienne.", "5": null}, "correct_option": 2, "explanations": {"1": {"exist": false, "char_ranges": [], "word_ranges": [], "text": ""}, "2": {"exist": true, "char_ranges": [[0, 154]], "word_ranges": [[0, 22]], "text": "Il s'agit probablement d'une artérite temporale, le tableau clinique et un test sanguin avec augmentation des réactifs suffisent à confirmer la suspicion."}, "3": {"exist": false, "char_ranges": [], "word_ranges": [], "text": ""}, "4": {"exist": false, "char_ranges": [], "word_ranges": [], "text": ""}, "5": {"exist": false, "char_ranges": [], "word_ranges": [], "text": ""}}} +{"id": 550, "year": 2022, "question_id_specific": 129, "full_question": "Un homme de 65 ans s'est présenté aux urgences en raison de l'apparition de lésions cyanosées tachetées sur les orteils des deux pieds. Antécédents personnels : tabagisme, hypertension et dyslipidémie. Radiographie du thorax : hypertrophie médiastinale. Angiographie par tomodensitométrie thoracique-abdominale-pelvienne : anévrisme de l'aorte thoracique descendante de 7 cm de diamètre, distal par rapport à l'artère sous-clavière gauche et avec un thrombus mural. Parmi les éléments suivants, cochez la bonne option :", "full_answer": "L'implantation d'un stent thoracique est indiquée.", "type": "CARDIOLOGIE", "options": {"1": "Un traitement par statines est indiqué pour stabiliser le thrombus.", "2": "Une aortographie diagnostique urgente est nécessaire.", "3": "Lors de l'intervention chirurgicale, le segment anévrismal est remplacé par une prothèse tubulaire avec réimplantation des troncs supra-aortiques.", "4": "L'implantation d'un stent thoracique est indiquée.", "5": null}, "correct_option": 4, "explanations": {"1": {"exist": false, "char_ranges": [], "word_ranges": [], "text": ""}, "2": {"exist": false, "char_ranges": [], "word_ranges": [], "text": ""}, "3": {"exist": false, "char_ranges": [], "word_ranges": [], "text": ""}, "4": {"exist": true, "char_ranges": [[0, 50]], "word_ranges": [[0, 6]], "text": "L'implantation d'un stent thoracique est indiquée."}, "5": {"exist": false, "char_ranges": [], "word_ranges": [], "text": ""}}} +{"id": 110, "year": 2012, "question_id_specific": 152, "full_question": "Patiente âgée de 76 ans, parité 3-0-3-1, ménopausée à l'âge de 52 ans. Elle signale que depuis au moins 4-5 ans, elle souffre de démangeaisons vulvaires d'intensité variable qui ont été traitées tantôt par automédication, tantôt sur les conseils de son médecin généraliste, avec des préparations topiques (crèmes et lavages). Le prurit s'est développé de façon intermittente, mais depuis 3-4 mois, elle a également remarqué une petite grosseur sur le grand labium gauche de la vulve, qui fuit de façon séro-hématique au frottement depuis quelques jours ; elle a donc consulté un gynécologue. À l'examen, elle signale une dysurie occasionnelle et son état général est bon. Quel est le diagnostic le plus probable chez cette patiente ?", "full_answer": "La bonne réponse est 5. Le carcinome vulvaire se caractérise par des démangeaisons vulvaires chroniques résistantes à de multiples traitements, l'apparition d'une tumeur ou d'une ulcération, une dysurie et un besoin urgent d'uriner, ainsi que des douleurs et des hémorragies tardives.", "type": "GYNÉCOLOGIE ET OBSTÉTRIQUE", "options": {"1": "Herpès génital.", "2": "Maladie de Paget de la vulve.", "3": "Caroncule urétral.", "4": "Granulome chronique de la vulve.", "5": "Carcinome épidermoïde de la vulve."}, "correct_option": 5, "explanations": {"1": {"exist": false, "char_ranges": [], "word_ranges": [], "text": ""}, "2": {"exist": false, "char_ranges": [], "word_ranges": [], "text": ""}, "3": {"exist": false, "char_ranges": [], "word_ranges": [], "text": ""}, "4": {"exist": false, "char_ranges": [], "word_ranges": [], "text": ""}, "5": {"exist": true, "char_ranges": [[24, 284]], "word_ranges": [[5, 41]], "text": "Le carcinome vulvaire se caractérise par des démangeaisons vulvaires chroniques résistantes à de multiples traitements, l'apparition d'une tumeur ou d'une ulcération, une dysurie et un besoin urgent d'uriner, ainsi que des douleurs et des hémorragies tardives."}}} +{"id": 419, "year": 2018, "question_id_specific": 77, "full_question": "Une femme de 80 ans s'est présentée aux urgences avec des douleurs abdominales débutant dans l'épigastre et irradiant ensuite vers la fosse iliaque gauche. Elle était accompagnée d'une fièvre de 37,5 ºC. Un scanner abdominal a été réalisé, montrant une inflammation des parois du sigma et un abcès mésentérique de 2 cm. Le traitement de choix est le suivant :", "full_answer": "Dans cette question, on nous présente un tableau clinique dont les manifestations sont compatibles avec une diverticulite aiguë, question qui a été posée à de nombreuses reprises dans le passé. Nous présentons un cas de diverticulite aiguë compliquée d'un petit abcès mésentérique (< 2-3 cm). Les abcès doivent être drainés par ponction percutanée guidée par scanner, car l'approche intra-abdominale peut répandre le contenu dans la cavité abdominale. Cependant, les petits abcès avec divertciulite de grade I de Hinchey, chez les patients sans détérioration majeure de l'état général comme dans le cas proposé, peuvent être gérés par un traitement conservateur avec une thérapie antiobiotique intraveineuse et un régime alimentaire absolu, par conséquent l'option correcte est 1. L'approche chirurgicale urgente doit être réservée aux cas présentant une péritonite (grade III, IV) par résection du segment atteint et anastomose primaire chez les patients stables, ou par la procédure de Hartamn comprenant une résection avec colostomie terminale et fermeture du moignon rectal distal avec anastomose dans un second temps chirurgical.", "type": "CHIRURGIE GÉNÉRALE", "options": {"1": "Admission dans le service avec régime alimentaire absolu et traitement antibiotique à large spectre.", "2": "Décharge de la colostomie.", "3": "Drainage par chirurgie laparoscopique.", "4": "Chirurgie urgente avec sigmoïdectomie et anastomose colorectale.", "5": null}, "correct_option": 1, "explanations": {"1": {"exist": true, "char_ranges": [[463, 780]], "word_ranges": [[68, 115]], "text": "les petits abcès avec divertciulite de grade I de Hinchey, chez les patients sans détérioration majeure de l'état général comme dans le cas proposé, peuvent être gérés par un traitement conservateur avec une thérapie antiobiotique intraveineuse et un régime alimentaire absolu, par conséquent l'option correcte est 1."}, "2": {"exist": false, "char_ranges": [], "word_ranges": [], "text": ""}, "3": {"exist": false, "char_ranges": [], "word_ranges": [], "text": ""}, "4": {"exist": true, "char_ranges": [[781, 881]], "word_ranges": [[115, 129]], "text": "L'approche chirurgicale urgente doit être réservée aux cas présentant une péritonite (grade III, IV)"}, "5": {"exist": false, "char_ranges": [], "word_ranges": [], "text": ""}}} +{"id": 37, "year": 2011, "question_id_specific": 227, "full_question": "Une femme de 78 ans consulte pour une perte aiguë de la vision de l'œil gauche. Au cours des trois dernières semaines, elle a présenté de la fièvre, des douleurs dans les épaules, le cou et les hanches, ainsi que des céphalées modérées. L'examen du fond d'œil révèle un disque optique pâle et œdémateux. La mobilisation des épaules et des hanches provoque des douleurs. La palpation des artères temporales ne révèle aucune altération. Données de laboratoire : hémoglobine 9,7 g/dL, ferritine 450 ng/mL, vitesse de sédimentation des érythrocytes 115 mm/h. Quelle est la mesure immédiate la plus appropriée pour ce patient ?", "full_answer": "La présentation clinique est fortement évocatrice d'une artérite à cellules géantes (ACG), avec une fièvre, des douleurs au niveau de la ceinture scapulaire et de la ceinture pelvienne, et des céphalées. L'âge et le sexe sont également des éléments qui plaident en faveur de cette possibilité. Une perte de vision unilatérale soudaine avec un œdème papillaire pâle est très évocatrice d'une neuropathie optique ischémique artéritique due à l'ACG. L'ESR est élevé, ce qui clarifie encore le diagnostic. Nous devons commencer rapidement un traitement aux corticostéroïdes. Nous pouvons d'ores et déjà exclure les points 1 et 5 car ils n'apportent rien. Le 4 pourrait nous faire douter, et il est vrai que nous devrons demander une biopsie de l'artère temporale, mais ce n'est pas immédiat : l'énoncé indique très clairement qu'il faut agir immédiatement. La tendance actuelle est d'utiliser des mégadoses de corticostéroïdes intraveineux, et bien que l'aspirine ait traditionnellement été proposée pour réduire les événements ischémiques, les preuves suggèrent aujourd'hui que les médicaments antiplaquettaires ne contribuent pas beaucoup. Une dose de corticostéroïdes par voie orale d'environ 1mg/kg serait également acceptable. Par conséquent, si nous excluons le point 3 parce que la dose est trop faible, nous devrions nous en tenir au point 2. Avec des réserves, car l'aspirine a un rôle plutôt douteux et la dose de corticostéroïde est ambiguë (nous ne savons pas combien pèse le patient). De toutes les questions d'ophtalmologie, c'est peut-être celle qui risque le plus d'être contestée.", "type": "OPHTHALMOLOGIE", "options": {"1": "Demander une échographie Doppler des artères temporales.", "2": "Commencez le traitement avec 60 mg de prednisone par jour et 100 mg d'aspirine par jour.", "3": "Commencer le traitement avec 10 mg de prednisone par jour.", "4": "Demander une biopsie de l'artère temporale.", "5": "Demander une imagerie par résonance magnétique du cerveau."}, "correct_option": 2, "explanations": {"1": {"exist": false, "char_ranges": [], "word_ranges": [], "text": ""}, "2": {"exist": true, "char_ranges": [[0, 446]], "word_ranges": [[0, 68]], "text": "La présentation clinique est fortement évocatrice d'une artérite à cellules géantes (ACG), avec une fièvre, des douleurs au niveau de la ceinture scapulaire et de la ceinture pelvienne, et des céphalées. L'âge et le sexe sont également des éléments qui plaident en faveur de cette possibilité. Une perte de vision unilatérale soudaine avec un œdème papillaire pâle est très évocatrice d'une neuropathie optique ischémique artéritique due à l'ACG."}, "3": {"exist": true, "char_ranges": [[1247, 1306]], "word_ranges": [[183, 195]], "text": "nous excluons le point 3 parce que la dose est trop faible,"}, "4": {"exist": true, "char_ranges": [[651, 1137]], "word_ranges": [[100, 168]], "text": "Le 4 pourrait nous faire douter, et il est vrai que nous devrons demander une biopsie de l'artère temporale, mais ce n'est pas immédiat : l'énoncé indique très clairement qu'il faut agir immédiatement. La tendance actuelle est d'utiliser des mégadoses de corticostéroïdes intraveineux, et bien que l'aspirine ait traditionnellement été proposée pour réduire les événements ischémiques, les preuves suggèrent aujourd'hui que les médicaments antiplaquettaires ne contribuent pas beaucoup."}, "5": {"exist": false, "char_ranges": [], "word_ranges": [], "text": ""}}} +{"id": 532, "year": 2021, "question_id_specific": 67, "full_question": "Une femme de 26 ans, ayant des antécédents d'une grossesse à l'âge de 24 ans qui s'est terminée par une fausse couche à la 12e semaine et n'a pas nécessité de curetage, se présente à la clinique avec des saignements vaginaux quotidiens d'une durée d'un mois, sans fièvre ni douleur. L'échographie montre une augmentation de la muqueuse endométriale, avec une image intracavitaire d'aspect polypoïde. La biopsie de l'endomètre montre une prolifération du trophoblaste intermédiaire. Quel est le bon diagnostic ?", "full_answer": "Augmente la gonadotrophine chorionique. Il s'agit d'un traitement curatif.", "type": "ONCOLOGIE", "options": {"1": "Tumeur du lit placentaire.", "2": "Hyperplasie atypique de l'endomètre.", "3": "Polype de l'endomètre.", "4": "Choriocarcinome.", "5": null}, "correct_option": 4, "explanations": {"1": {"exist": false, "char_ranges": [], "word_ranges": [], "text": ""}, "2": {"exist": false, "char_ranges": [], "word_ranges": [], "text": ""}, "3": {"exist": false, "char_ranges": [], "word_ranges": [], "text": ""}, "4": {"exist": true, "char_ranges": [[0, 39]], "word_ranges": [[0, 4]], "text": "Augmente la gonadotrophine chorionique."}, "5": {"exist": false, "char_ranges": [], "word_ranges": [], "text": ""}}} +{"id": 272, "year": 2016, "question_id_specific": 69, "full_question": "Un homme de 60 ans atteint d'une colite ulcéreuse étendue depuis 15 ans et en rémission clinique depuis 3 ans, vient dans notre clinique pour s'enquérir du risque de cancer colorectal et de la possibilité de participer à des programmes de prévention. Il convient de l'informer que :", "full_answer": "Les patients atteints de maladies inflammatoires de l'intestin (MII) présentent un risque accru de cancer colorectal, qu'ils soient fumeurs ou non. Il est recommandé de commencer le dépistage 8 à 10 ans après le diagnostic de la MII (ou plus tôt si une cholangite sclérosante primaire ou d'autres événements surviennent dans cet intervalle). Les techniques recommandées sont la coloscopie avec biopsies aléatoires (réponse 3) et la chromoendoscopie pancolonique avec coloration, qui est actuellement l'examen de référence car elle détecte mieux la dysplasie, bien qu'elle nécessite une plus grande expertise technique.", "type": "SYSTÈME DIGESTIF", "options": {"1": "La colite ulcéreuse n'est associée à un risque accru de cancer colorectal que chez les fumeurs.", "2": "Dans votre cas, étant donné que vous êtes en rémission de longue durée, le dépistage conseillé pour la population générale est considéré comme approprié.", "3": "Dans des cas comme le vôtre, il est considéré comme approprié de subir des coloscopies régulières avec des biopsies multiples effectuées à des intervalles échelonnés tout au long du côlon.", "4": "Dans des cas comme le vôtre, le dépistage par des tests périodiques de recherche de sang occulte dans les selles est considéré comme approprié, mais à une fréquence plus élevée que celle utilisée pour le dépistage dans la population générale.", "5": null}, "correct_option": 3, "explanations": {"1": {"exist": false, "char_ranges": [], "word_ranges": [], "text": ""}, "2": {"exist": false, "char_ranges": [], "word_ranges": [], "text": ""}, "3": {"exist": true, "char_ranges": [[148, 478]], "word_ranges": [[21, 70]], "text": "Il est recommandé de commencer le dépistage 8 à 10 ans après le diagnostic de la MII (ou plus tôt si une cholangite sclérosante primaire ou d'autres événements surviennent dans cet intervalle). Les techniques recommandées sont la coloscopie avec biopsies aléatoires (réponse 3) et la chromoendoscopie pancolonique avec coloration,"}, "4": {"exist": false, "char_ranges": [], "word_ranges": [], "text": ""}, "5": {"exist": false, "char_ranges": [], "word_ranges": [], "text": ""}}} +{"id": 428, "year": 2018, "question_id_specific": 61, "full_question": "Un homme de 62 ans se présente au service des urgences de l'hôpital avec une fièvre de 38,3″C et des frissons. Il est tachycarde et tachypnéique. Examens sanguins : leucocytose avec déplacement vers la gauche. La tension artérielle est de 90/60 mmHg. Aucun souffle n'a été entendu à l'auscultation cardiaque et la radiographie du thorax a exclu une pneumonie. Dans ses antécédents, il mentionne des infections urinaires répétées. Il y a une semaine, le médecin généraliste lui a prescrit du céfuroxime, qu'il prend actuellement. Deux hémocultures ont été prélevées et l'urine a été envoyée pour examen du sédiment et culture microbiologique. Le sédiment était pathologique, avec une pyurie intense, mais négatif pour les nitrites. Après 24 heures, le laboratoire de microbiologie a signalé que les cultures d'urine et de sang étaient positives et que les résultats montraient des cocci gram-positifs groupés en chaînes, en attente d'identification et d'antibiogramme. Dans l'attente du résultat définitif, que recommanderiez-vous ?", "full_answer": "Le test de nitrite urinaire permet de détecter la présence de nitrites dans l'urine. Ceux-ci apparaissent en raison de la présence dans l'urine d'une forte concentration de bactéries qui possèdent l'enzyme nitrate réductase, capable de réduire les nitrates présents dans l'urine en nitrites. Cette enzyme est active dans la plupart des bactéries qui causent le plus fréquemment des infections urinaires, comme de nombreux bacilles à Gram négatif. D'autres micro-organismes, causes relativement fréquentes d'infections urinaires, ne possèdent pas cette enzyme et ne réduisent pas les nitrates, comme les entérocoques, les staphylocoques et les levures. Comme la coloration de Gram indique que les bactéries sont groupées en chaînes, ce qui est typique des entérocoques, nous suggérons la nécessité de couvrir ce groupe de bactéries par un traitement antibiotique. Les staphylocoques sont typiquement observés dans la coloration de Gram comme des cocci Gram-positifs formant des grappes et les levures, bien que présentant la même coloration, ont une morphologie typique différente de celle des cocci Gram-positifs.", "type": "MALADIES INFECTIEUSES ET MICROBIOLOGIE", "options": {"1": "Poursuivre le céfuroxime dans l'attente du résultat définitif, car le patient a subi plusieurs traitements et nous ne devons pas commettre d'erreur dans son traitement actuel.", "2": "Passer à l'ertapénème, compte tenu de la possibilité de micro-organismes multirésistants.", "3": "Passer à un traitement antimicrobien à large spectre couvrant Enterococcus spp.", "4": "Passer à une thérapie antimicrobienne à large spectre contenant de l'imipénème, en considérant la possibilité de Staphylococcus aureus, car notre hôpital a une incidence élevée de S. aureus résistant à la méthicilline (MRSA).", "5": null}, "correct_option": 3, "explanations": {"1": {"exist": false, "char_ranges": [], "word_ranges": [], "text": ""}, "2": {"exist": false, "char_ranges": [], "word_ranges": [], "text": ""}, "3": {"exist": true, "char_ranges": [[652, 862]], "word_ranges": [[93, 126]], "text": "Comme la coloration de Gram indique que les bactéries sont groupées en chaînes, ce qui est typique des entérocoques, nous suggérons la nécessité de couvrir ce groupe de bactéries par un traitement antibiotique."}, "4": {"exist": true, "char_ranges": [[863, 1113]], "word_ranges": [[126, 162]], "text": "Les staphylocoques sont typiquement observés dans la coloration de Gram comme des cocci Gram-positifs formant des grappes et les levures, bien que présentant la même coloration, ont une morphologie typique différente de celle des cocci Gram-positifs."}, "5": {"exist": false, "char_ranges": [], "word_ranges": [], "text": ""}}} +{"id": 393, "year": 2016, "question_id_specific": 224, "full_question": "Une patiente se présente avec une longue histoire de symptômes physiques multiples : perte de mémoire, maux de tête, vertiges, vomissements, douleurs génitales, douleurs dans les membres, distension abdominale et irrégularités menstruelles. Les différents examens médicaux effectués ont permis d'exclure toute maladie. De laquelle des affections suivantes est-elle le plus susceptible de souffrir ?", "full_answer": "Selon la CIM-10, les critères de diagnostic du trouble de la somatisation (F45.0) sont les suivants : A. Antécédents de symptômes physiques multiples débutant avant l'âge de 30 ans, persistant pendant plusieurs années et nécessitant une prise en charge médicale ou entraînant un handicap significatif sur le plan social, professionnel ou dans d'autres domaines importants de l'activité de l'individu. B. Quatre symptômes douloureux : antécédents de douleurs liées à au moins quatre régions du corps ou quatre fonctions (par exemple, tête, abdomen, dos, articulations, extrémités, poitrine, rectum ; pendant les règles, les rapports sexuels ou la miction). 2) Deux symptômes gastro-intestinaux : antécédents d'au moins deux symptômes gastro-intestinaux autres que la douleur (par exemple, nausées, ballonnements, vomissements [pas pendant la grossesse], diarrhée ou intolérance à différents aliments). 3. un symptôme sexuel : antécédents d'au moins un symptôme sexuel ou reproductif autre que la douleur (par exemple, indifférence sexuelle, dysfonctionnement érectile ou éjaculatoire, menstruations irrégulières, saignements menstruels excessifs, vomissements pendant la grossesse). 4. Un symptôme pseudo-neurologique : antécédents d'au moins un symptôme ou déficit évoquant un trouble neurologique non limité à la douleur (symptômes de conversion du type trouble de la coordination psychomotrice ou de l'équilibre, paralysie ou faiblesse musculaire localisée, difficulté à avaler, sensation de boule dans la gorge, aphonie, rétention urinaire, hallucinations, perte des sensations tactiles et douloureuses, diplopie, cécité, surdité, crises d'épilepsie ; symptômes dissociatifs tels que l'amnésie ; ou perte de conscience autre que l'évanouissement). C. L'une ou l'autre des deux caractéristiques suivantes : 1. après un examen adéquat, aucun des symptômes du critère B ne peut être expliqué par la présence d'une maladie connue ou par les effets directs d'une substance (par exemple, des drogues ou des médicaments) ; 2. en cas de présence d'une maladie, les symptômes ne peuvent être expliqués que par la présence d'une substance. 2) Si une maladie est présente, les symptômes physiques ou la déficience sociale ou professionnelle sont excessifs par rapport à ce que l'on pourrait attendre des antécédents, de l'examen physique ou des résultats de laboratoire. D. Les symptômes ne sont pas produits intentionnellement et ne sont pas simulés (contrairement au trouble factice et à la simulation). Tout correspond à la description de l'énoncé. Les troubles conversifs ou dissociatifs sont exclus car ils sont généralement liés à des facteurs psychologiques, associés au symptôme ou au déficit (non intentionnel). L'apparition ou l'exacerbation de la maladie est précédée d'un conflit ou d'autres facteurs déclenchants, ce qui n'est pas mentionné dans l'énoncé. Il ne s'agit pas non plus d'un trouble hypocondriaque, car cela implique une préoccupation et une peur d'avoir, ou la conviction d'avoir, une maladie grave basée sur l'interprétation personnelle des symptômes somatiques. Mais cela n'implique pas l'existence de signes ou de symptômes physiques.", "type": "PSYCHIATRIE", "options": {"1": "Trouble de la conversion.", "2": "Trouble hypocondriaque.", "3": "Trouble de la somatisation.", "4": "Trouble dissociatif.", "5": null}, "correct_option": 3, "explanations": {"1": {"exist": true, "char_ranges": [[2544, 2860]], "word_ranges": [[365, 410]], "text": "Les troubles conversifs ou dissociatifs sont exclus car ils sont généralement liés à des facteurs psychologiques, associés au symptôme ou au déficit (non intentionnel). L'apparition ou l'exacerbation de la maladie est précédée d'un conflit ou d'autres facteurs déclenchants, ce qui n'est pas mentionné dans l'énoncé."}, "2": {"exist": true, "char_ranges": [[2861, 3155]], "word_ranges": [[410, 453]], "text": "Il ne s'agit pas non plus d'un trouble hypocondriaque, car cela implique une préoccupation et une peur d'avoir, ou la conviction d'avoir, une maladie grave basée sur l'interprétation personnelle des symptômes somatiques. Mais cela n'implique pas l'existence de signes ou de symptômes physiques."}, "3": {"exist": false, "char_ranges": [], "word_ranges": [], "text": ""}, "4": {"exist": true, "char_ranges": [[2544, 2860]], "word_ranges": [[365, 410]], "text": "Les troubles conversifs ou dissociatifs sont exclus car ils sont généralement liés à des facteurs psychologiques, associés au symptôme ou au déficit (non intentionnel). L'apparition ou l'exacerbation de la maladie est précédée d'un conflit ou d'autres facteurs déclenchants, ce qui n'est pas mentionné dans l'énoncé."}, "5": {"exist": false, "char_ranges": [], "word_ranges": [], "text": ""}}} +{"id": 124, "year": 2012, "question_id_specific": 46, "full_question": "Une femme de 58 ans vient en consultation de suivi pour un diabète sucré et une hypertension. Elle se sent bien mais déclare qu'elle a arrêté de prendre du vérapamil à cause de la constipation. Elle présente une intolérance aux inhibiteurs de l'ECA en raison de sa toux. À l'examen, la tension artérielle est de 156/92 mm Hg. Les examens de laboratoire montrent une créatinine de 1,6 mg/dl, une excrétion de protéines dans les urines de 24 heures de 1,5 g/jour et une clairance de la créatinine de 45 ml/min. Sur cette base, quel est le traitement le plus efficace pour ralentir la progression de la néphropathie diabétique de type 2 du patient ?", "full_answer": "Il s'agit d'une question typique et assez facile. Les médicaments antihypertenseurs dont il a été démontré qu'ils ralentissent la progression de la néphropathie diabétique sont les inhibiteurs de l'enzyme de conversion de l'angiotensine (IECA) et les antagonistes des récepteurs de l'angiotensine II. En particulier, les bloqueurs des récepteurs de l'angiotensine II se sont révélés efficaces dans ce domaine pour le diabète de type 2, ce qui, combiné aux effets secondaires des inhibiteurs de l'ECA sur le patient, fait d'un ARAI le médicament de choix. Cette question ne me semble pas contestable (sans entrer dans les questions pharmacoéconomiques, bien sûr...).", "type": "NEPHROLOGIE", "options": {"1": "Inhibiteur de l'enzyme de conversion de l'angiotensine.", "2": "Bloqueur des récepteurs de l'angiotensine.", "3": "Antagoniste du calcium.", "4": "Alpha-bloquant.", "5": "Bêta-bloquant."}, "correct_option": 2, "explanations": {"1": {"exist": true, "char_ranges": [[317, 554]], "word_ranges": [[44, 84]], "text": "les bloqueurs des récepteurs de l'angiotensine II se sont révélés efficaces dans ce domaine pour le diabète de type 2, ce qui, combiné aux effets secondaires des inhibiteurs de l'ECA sur le patient, fait d'un ARAI le médicament de choix."}, "2": {"exist": true, "char_ranges": [[317, 554]], "word_ranges": [[44, 84]], "text": "les bloqueurs des récepteurs de l'angiotensine II se sont révélés efficaces dans ce domaine pour le diabète de type 2, ce qui, combiné aux effets secondaires des inhibiteurs de l'ECA sur le patient, fait d'un ARAI le médicament de choix."}, "3": {"exist": false, "char_ranges": [], "word_ranges": [], "text": ""}, "4": {"exist": false, "char_ranges": [], "word_ranges": [], "text": ""}, "5": {"exist": false, "char_ranges": [], "word_ranges": [], "text": ""}}} +{"id": 405, "year": 2016, "question_id_specific": 31, "full_question": "Une femme de 20 ans présentant une tumeur ovarienne kystique solide de 15 cm détectée par échographie après avoir présenté des symptômes abdominaux non spécifiques. Lors de l'étude histopathologique de l'échantillon correspondant, des dents, des poils, des zones d'épithélium intestinal, des zones d'épithélium malpighien (15 %) et d'épithélium bronchique, ainsi que des éléments neuroectodermiques et embryonnaires ont été trouvés dans plusieurs des préparations histologiques. En ce qui concerne ce cas, veuillez indiquer le bon diagnostic :", "full_answer": "Le tératome est la tumeur germinale la plus fréquente. Il se différencie en éléments des trois couches embryonnaires : endoderme, mésoderme et ectoderme. Macroscopiquement, ils peuvent être kystiques ou solides, et histologiquement, les tissus composant la tumeur peuvent être matures (bien différenciés, comme les tissus adultes) ou immatures (comme les tissus embryonnaires). Les tératomes kystiques matures sont les plus fréquents : ils représentent en moyenne 10 % des tumeurs ovariennes (5-25 % selon les cas). Ils surviennent à tout âge. Les tissus ectodermiques, comme la peau, prédominent, tapissant une cavité à contenu kératinique. Un éperon peut être reconnu dans la cavité, d'où naissent souvent des cheveux ou des structures dentaires. Les tissus communs comprennent : le tissu nerveux, généralement l'épithélium glial et épendymaire, les épithéliums respiratoires et digestifs et diverses structures mésodermiques. Ils sont bilatéraux dans environ 10 % des cas. Le tératome kystique mature est bénin, mais dans 2 % des cas, une tumeur maligne peut se développer à partir de l'un des composants du tissu (carcinome spinocellulaire, carcinoïde, adénocarcinome, carcinome du tissu thyroïdien, sarcome). Dans ce cas, nous avons d'abord pensé qu'il s'agissait d'un tératome kystique mature, mais grâce à Ramón, lorsque nous l'avons lu plus attentivement, nous avons découvert qu'il présentait des éléments neuroectodermiques et embryonnaires dans plusieurs des préparations histologiques, de sorte que nous considérons que la réponse correcte est 2.", "type": "ONCOLOGIE (ECTOPIQUE)", "options": {"1": "Tératocarcinome.", "2": "Tératome immature.", "3": "Tératome kystique mature.", "4": "Dysgerminome.", "5": null}, "correct_option": 2, "explanations": {"1": {"exist": false, "char_ranges": [], "word_ranges": [], "text": ""}, "2": {"exist": true, "char_ranges": [[1364, 1558]], "word_ranges": [[199, 225]], "text": "nous avons découvert qu'il présentait des éléments neuroectodermiques et embryonnaires dans plusieurs des préparations histologiques, de sorte que nous considérons que la réponse correcte est 2."}, "3": {"exist": true, "char_ranges": [[1364, 1558]], "word_ranges": [[199, 225]], "text": "nous avons découvert qu'il présentait des éléments neuroectodermiques et embryonnaires dans plusieurs des préparations histologiques, de sorte que nous considérons que la réponse correcte est 2."}, "4": {"exist": false, "char_ranges": [], "word_ranges": [], "text": ""}, "5": {"exist": false, "char_ranges": [], "word_ranges": [], "text": ""}}} +{"id": 497, "year": 2020, "question_id_specific": 127, "full_question": "Un homme de 60 ans, fumeur de 40 paquets/an, consulte pour une douleur au niveau du bord ulnaire du bras gauche et une ptose palpébrale gauche depuis un mois. La radiographie du thorax montre une masse dans le lobe supérieur gauche et la tomodensitométrie confirme la lésion avec envahissement de la deuxième côte. Compte tenu du diagnostic suspecté, le traitement le plus approprié est le suivant :", "full_answer": "Pancoast typique du fumeur, probablement épidermoïde, avec sdr de Horner. La meilleure solution serait la chimioradio suivie d'un sauvetage chirurgical. Autrefois, il n'y avait que la RT et la chirurgie. L'immunothérapie de première ligne n'est peut-être pas loin. Il y a de plus en plus de nouveautés.", "type": "ONCOLOGIE MÉDICALE", "options": {"1": "Chimiothérapie néoadjuvante avec chimioradiothérapie combinée, suivie d'une intervention chirurgicale.", "2": "Chimiothérapie néoadjuvante sans chirurgie, suivie d'une radiothérapie.", "3": "Chirurgie suivie d'une chimioradiothérapie combinée.", "4": "Radiothérapie sans chimiothérapie, suivie d'une intervention chirurgicale.", "5": null}, "correct_option": 1, "explanations": {"1": {"exist": true, "char_ranges": [[0, 152]], "word_ranges": [[0, 20]], "text": "Pancoast typique du fumeur, probablement épidermoïde, avec sdr de Horner. La meilleure solution serait la chimioradio suivie d'un sauvetage chirurgical."}, "2": {"exist": false, "char_ranges": [], "word_ranges": [], "text": ""}, "3": {"exist": false, "char_ranges": [], "word_ranges": [], "text": ""}, "4": {"exist": true, "char_ranges": [[153, 203]], "word_ranges": [[20, 30]], "text": "Autrefois, il n'y avait que la RT et la chirurgie."}, "5": {"exist": false, "char_ranges": [], "word_ranges": [], "text": ""}}} +{"id": 307, "year": 2016, "question_id_specific": 207, "full_question": "Un patient de 34 ans jouant au tennis a reçu une balle de tennis dans l'orbite gauche. À l'examen, il présente un large hématome palpébral, une hyposphagmie, une diplopie de la vision supérieure avec limitation de la version supérieure du globe oculaire. Que suspecteriez-vous ?", "full_answer": "Cette question comporte une petite faute de frappe (il faudrait lire \"au tennis\"), mais personnellement je suis un peu plus gêné par l'erreur dans la description du scanner. En particulier lorsqu'il est dit \"diplopie à la vision supérieure avec limitation de la version supérieure du globe oculaire\". Les versions sont des mouvements conjugués bilatéraux des deux yeux. C'est-à-dire qu'une version est constituée de la somme de deux ductions, puisque la duction est le mouvement d'un seul œil. Lorsque vous explorez les versions, à savoir la version supérieure ou supraversion, les deux yeux se tournent vers le haut. À ce stade, vous pouvez rencontrer une limitation du mouvement d'un œil. Dans ce cas, lorsque vous scrutez la supraversion, vous soupçonnez ou détectez une limitation du mouvement de l'œil gauche. C'est ce que nous appelons une limitation de la supraversion de cet œil. Cela n'a pas de sens de parler d'une limitation de la version du globe oculaire, car la version est toujours celle des deux globes oculaires. C'est compréhensible, mais c'est une erreur importante qui ne devrait pas être commise par la personne qui écrit la question (qui est censée être un expert en la matière), mais je ne pense pas que ce soit une raison pour contester la question, alors continuons. La question porte sur un patient qui a reçu un coup. Les quatre réponses sont des fractures, nous avons donc une image assez claire. L'hématome palpébral et l'hyposphagma (hémorragie sous-conjonctivale) ne nous fournissent pas les données essentielles. Ce qui est important, c'est que l'œil qui a reçu le coup ne monte pas et qu'il y a donc une double vision dans le regard supérieur. Il pourrait être tentant de répondre 3, en pensant que le muscle droit supérieur est coincé, qu'il ne fonctionne pas et que l'œil ne peut donc pas remonter. Cependant, les muscles coincés dans une fracture ne perdent pas automatiquement leur action musculaire. Ce qui se passe, c'est qu'ils sont piégés, \"coincés\", et qu'ils ne peuvent pas être étirés. Le problème est que la paroi inférieure a été brisée, le droit inférieur a été piégé, et lorsque l'œil doit regarder vers le haut, ce droit inférieur ne peut pas s'étirer comme il le devrait. Il ne s'agit pas d'une paralysie, mais d'un problème de restriction. On peut également répondre à cette question en écartant les options. Les options 2 et 4 peuvent être ignorées parce qu'elles sont trop éloignées des muscles extraoculaires pour produire une diplopie. En ce qui concerne les parois de l'orbite, celles qui se rompent le plus souvent sont les parois inférieure et médiale. La paroi supérieure est très rarement rompue.", "type": "OPHTHALMOLOGIE", "options": {"1": "Fracture de la paroi inférieure du plancher de l'orbite avec coincement du muscle droit inférieur.", "2": "Fracture de l'arcade zygomatique.", "3": "Fracture de la paroi supérieure de l'orbite avec coincement du muscle droit supérieur.", "4": "Fracture dentoalvéolaire.", "5": null}, "correct_option": 1, "explanations": {"1": {"exist": true, "char_ranges": [[2491, 2610]], "word_ranges": [[410, 431]], "text": "En ce qui concerne les parois de l'orbite, celles qui se rompent le plus souvent sont les parois inférieure et médiale."}, "2": {"exist": true, "char_ranges": [[2360, 2490]], "word_ranges": [[390, 410]], "text": "Les options 2 et 4 peuvent être ignorées parce qu'elles sont trop éloignées des muscles extraoculaires pour produire une diplopie."}, "3": {"exist": true, "char_ranges": [[2611, 2656]], "word_ranges": [[431, 438]], "text": "La paroi supérieure est très rarement rompue."}, "4": {"exist": true, "char_ranges": [[2360, 2490]], "word_ranges": [[390, 410]], "text": "Les options 2 et 4 peuvent être ignorées parce qu'elles sont trop éloignées des muscles extraoculaires pour produire une diplopie."}, "5": {"exist": false, "char_ranges": [], "word_ranges": [], "text": ""}}} +{"id": 589, "year": 2022, "question_id_specific": 63, "full_question": "Un homme de 36 ans consulte pour une hyperémie conjonctivale et une sensation de corps étranger. A l'examen, il présente une atteinte du ganglion préauriculaire. Parmi les pathologies suivantes, laquelle ne correspond PAS à cet examen ?", "full_answer": "On nous présente un tableau d'inflammation conjonctivale ou de conjonctivite et on nous demande lequel de ces cas n'est pas associé à une lymphadénopathie pré-auriculaire. La conjonctivite à adénovirus peut être associée à une inflammation du ganglion lymphatique pré-auriculaire ou sub-mandibulaire ipsilatéral (option 1 écartée). La conjonctivite allergique est un état d'hypersensibilité qui peut survenir immédiatement après le contact avec le stimulus déclencheur ou après 24 à 72 heures, et qui s'accompagne de signes tels que l'œdème et l'érythème des paupières, des modifications de la pigmentation de la peau périoculaire et un symptôme clé : les démangeaisons. Elle n'est pas associée à une lymphadénopathie (option 2 correcte). Le syndrome oculoglandulaire de Parinaud est une affection qui, par définition, se compose d'une conjonctivite d'origine infectieuse associée à une lymphadénopathie pré-auriculaire ou latéro-cervicale ipsilatérale (une ou plusieurs), généralement causée par des bactéries telles que Bartonella henselae (maladie des griffes du chat) ou Francisella tularensis (tularémie ou fièvre du lapin) : l'option 3 n'a pas été retenue. La conjonctivite à chlamydia peut se présenter sous la forme de trois tableaux cliniques bien définis : le trachome, l'ophtalmie néonatale ou la conjonctivite à corps inclusif de l'adulte. Cette dernière est considérée comme une maladie sexuellement transmissible et est associée à des ganglions pré-auriculaires ipsilatéraux (option 4 écartée).", "type": "OPHTHALMOLOGIE", "options": {"1": "Conjonctivite adénovirale.", "2": "Conjonctivite allergique.", "3": "Syndrome oculoglandulaire de Parinaud.", "4": "Conjonctivite à Chlamydia.", "5": null}, "correct_option": 2, "explanations": {"1": {"exist": true, "char_ranges": [[172, 331]], "word_ranges": [[25, 45]], "text": "La conjonctivite à adénovirus peut être associée à une inflammation du ganglion lymphatique pré-auriculaire ou sub-mandibulaire ipsilatéral (option 1 écartée)."}, "2": {"exist": true, "char_ranges": [[332, 738]], "word_ranges": [[45, 107]], "text": "La conjonctivite allergique est un état d'hypersensibilité qui peut survenir immédiatement après le contact avec le stimulus déclencheur ou après 24 à 72 heures, et qui s'accompagne de signes tels que l'œdème et l'érythème des paupières, des modifications de la pigmentation de la peau périoculaire et un symptôme clé : les démangeaisons. Elle n'est pas associée à une lymphadénopathie (option 2 correcte)."}, "3": {"exist": true, "char_ranges": [[739, 1162]], "word_ranges": [[107, 164]], "text": "Le syndrome oculoglandulaire de Parinaud est une affection qui, par définition, se compose d'une conjonctivite d'origine infectieuse associée à une lymphadénopathie pré-auriculaire ou latéro-cervicale ipsilatérale (une ou plusieurs), généralement causée par des bactéries telles que Bartonella henselae (maladie des griffes du chat) ou Francisella tularensis (tularémie ou fièvre du lapin) : l'option 3 n'a pas été retenue."}, "4": {"exist": true, "char_ranges": [[1163, 1508]], "word_ranges": [[164, 213]], "text": "La conjonctivite à chlamydia peut se présenter sous la forme de trois tableaux cliniques bien définis : le trachome, l'ophtalmie néonatale ou la conjonctivite à corps inclusif de l'adulte. Cette dernière est considérée comme une maladie sexuellement transmissible et est associée à des ganglions pré-auriculaires ipsilatéraux (option 4 écartée)."}, "5": {"exist": false, "char_ranges": [], "word_ranges": [], "text": ""}}} +{"id": 350, "year": 2016, "question_id_specific": 160, "full_question": "Une femme de 45 ans, mère de trois enfants, se présente à une consultation de diagnostic précoce. La cytologie cervicovaginale est compatible avec une lésion intraépithéliale malpighienne de haut grade. Parmi les options suivantes, laquelle choisiriez-vous ?", "full_answer": "Colposcopie avec éventuellement une biopsie. H-SIL (High-grade squamous intraepithelial lesion)* : comprend les changements évocateurs de CIN2 et CIN3/CIS. Le diagnostic cytologique doit être confirmé par une biopsie, ce qui nécessite une biopsie dirigée par colposcopie.", "type": "GYNÉCOLOGIE ET OBSTÉTRIQUE", "options": {"1": "Répéter la cytologie dans un mois.", "2": "Colposcopie avec éventuellement une biopsie.", "3": "Hystérectomie avec salpingectomie bilatérale et conservation des ovaires.", "4": "Curetage fractionné.", "5": null}, "correct_option": 2, "explanations": {"1": {"exist": false, "char_ranges": [], "word_ranges": [], "text": ""}, "2": {"exist": true, "char_ranges": [[156, 271]], "word_ranges": [[19, 36]], "text": "Le diagnostic cytologique doit être confirmé par une biopsie, ce qui nécessite une biopsie dirigée par colposcopie."}, "3": {"exist": false, "char_ranges": [], "word_ranges": [], "text": ""}, "4": {"exist": false, "char_ranges": [], "word_ranges": [], "text": ""}, "5": {"exist": false, "char_ranges": [], "word_ranges": [], "text": ""}}} +{"id": 25, "year": 2011, "question_id_specific": 116, "full_question": "Une femme de 40 ans est admise aux urgences pour une fièvre de 38°C et des douleurs lombaires droites. L'analyse sanguine montre 25 000 leucocytes/mm3 avec une déviation à gauche et l'échographie abdominale révèle une lithiase coralliforme. Quel micro-organisme peut-on s'attendre à trouver ?", "full_answer": "Question d'épidémiologie partagée avec l'URO. La lithiase coroniforme est associée à des germes fendeurs d'urée : Proteus en premier lieu, Pseudomonas, Klebsiella et certains staphylocoques.", "type": "INFECTIEUX", "options": {"1": "Escherichia coli.", "2": "Enterococcus faecalis.", "3": "Salmonella typhi.", "4": "Candida albicans.", "5": "Proteus mirabilis."}, "correct_option": 5, "explanations": {"1": {"exist": false, "char_ranges": [], "word_ranges": [], "text": ""}, "2": {"exist": false, "char_ranges": [], "word_ranges": [], "text": ""}, "3": {"exist": false, "char_ranges": [], "word_ranges": [], "text": ""}, "4": {"exist": false, "char_ranges": [], "word_ranges": [], "text": ""}, "5": {"exist": true, "char_ranges": [[46, 190]], "word_ranges": [[5, 25]], "text": "La lithiase coroniforme est associée à des germes fendeurs d'urée : Proteus en premier lieu, Pseudomonas, Klebsiella et certains staphylocoques."}}} +{"id": 103, "year": 2012, "question_id_specific": 89, "full_question": "Patient de 55 ans dont l'hypertension a été diagnostiquée il y a deux mois lors d'un bilan de santé. Bilan sanguin : glycémie 129 mg/dl, cholestérol 202 mg/dl, LDLc 160 mg/dl, HDLc 38 mg/dl, triglycérides 171 mg/dl, créatinine 1,1 mg/dl. Consultation pour mauvais contrôle de la tension artérielle, œdème malléolaire et dyspnée, parfois nocturne. Examen physique : IMC 38 kg/m2, circonférence abdominale 110 cm, TA 157/93, FC 70 bpm, FC 14 rpm. Souffle systolique avec renforcement du deuxième bruit. Pouls périphériques avec discrète asymétrie au niveau du pédius droit et du tibialis postérieur, plus faibles que ceux de l'extrémité gauche. Radiographie du thorax et ECG sans altération Lequel des examens complémentaires suivants est le moins nécessaire à la détection des lésions des organes cibles ?", "full_answer": "L'HbA1c serait utile chez ce patient pour le diagnostic du diabète mais pas pour l'atteinte des organes cibles de l'HT.", "type": "ENDOCRINOLOGIE", "options": {"1": "Échocardiographie transthoracique.", "2": "Examen du fond d'œil.", "3": "Détermination de la microalbuminurie.", "4": "Indice brachial de la cheville.", "5": "Détermination de l'hémoglobine glycosylée."}, "correct_option": 5, "explanations": {"1": {"exist": false, "char_ranges": [], "word_ranges": [], "text": ""}, "2": {"exist": false, "char_ranges": [], "word_ranges": [], "text": ""}, "3": {"exist": false, "char_ranges": [], "word_ranges": [], "text": ""}, "4": {"exist": false, "char_ranges": [], "word_ranges": [], "text": ""}, "5": {"exist": true, "char_ranges": [[0, 119]], "word_ranges": [[0, 20]], "text": "L'HbA1c serait utile chez ce patient pour le diagnostic du diabète mais pas pour l'atteinte des organes cibles de l'HT."}}} +{"id": 22, "year": 2011, "question_id_specific": 112, "full_question": "Un patient se présente aux urgences avec une fièvre sans origine depuis 24 heures, une hypothyroïdie et une impression de gravité. Il a subi une splénectomie il y a deux ans pour stadification de la maladie de Hosgkin. Quel micro-organisme est le plus souvent impliqué ?", "full_answer": "Autre épidémiologie. Chez les patients splénectomisés, le risque de contracter des infections graves par des germes encapsulés est accru, le pneumocoque étant là encore le plus fréquent.", "type": "INFECTIEUX", "options": {"1": "Streptococcus pneumoniae.", "2": "Pseudomonas aeruginosa.", "3": "Salmonella non-typhi.", "4": "Pneumocystis jirovecii.", "5": "Streptococcus viridans."}, "correct_option": 1, "explanations": {"1": {"exist": true, "char_ranges": [[21, 186]], "word_ranges": [[2, 27]], "text": "Chez les patients splénectomisés, le risque de contracter des infections graves par des germes encapsulés est accru, le pneumocoque étant là encore le plus fréquent."}, "2": {"exist": false, "char_ranges": [], "word_ranges": [], "text": ""}, "3": {"exist": false, "char_ranges": [], "word_ranges": [], "text": ""}, "4": {"exist": false, "char_ranges": [], "word_ranges": [], "text": ""}, "5": {"exist": false, "char_ranges": [], "word_ranges": [], "text": ""}}} +{"id": 488, "year": 2020, "question_id_specific": 105, "full_question": "Patient de 67 ans qui, au cours des 6 derniers mois, a présenté une lymphocytose progressive lors de deux analyses de routine. Dans la dernière, hémoglobine 15,4 g/dl ; leucocytes 18,5 x103/μL avec 82% de lymphocytes matures qui, par cytométrie de flux, expriment les antigènes CD5/CD19/CD23 et plaquettes 240 x103/μL. Quelle est, selon vous, l'approche correcte ?", "full_answer": "Une question qui, à mon avis, pourrait être compliquée pour un candidat MIR. On nous parle d'une leucocytose aux dépens des lymphocytes avec un phénotype B-CLL. Il semble qu'il n'y ait pas d'autres données alarmantes. Aucune donnée clinique n'est donnée (nous supposons donc que le patient est asymptomatique). Pas de cytopénies... Conclusion : FOLLOW UP. Le principal doute concernant la réponse devrait porter sur l'option 1. Pourquoi est-ce que je la considère comme incorrecte ? Parce que la mutation TP53, bien qu'elle établisse le pronostic, n'est pas indiquée au moment du diagnostic, mais au moment où le traitement va commencer (première ligne et avant les lignes successives si nécessaire). Il serait possible d'aller trop loin dans cette question où le pourcentage donné ne se réfère pas directement aux lymphocytes clonaux, il pourrait même s'agir d'une lymphocytose B monoclonale. Mais c'est aller trop loin.", "type": "HÉMATOLOGIE", "options": {"1": "Étude des mutations TP53 pour établir un pronostic.", "2": "Aspiration/biopsie osseuse pour confirmer le diagnostic.", "3": "PET/CT pour établir l'approche thérapeutique.", "4": "Nouveau contrôle clinique et analytique en 6 mois.", "5": null}, "correct_option": 4, "explanations": {"1": {"exist": true, "char_ranges": [[493, 700]], "word_ranges": [[77, 109]], "text": "la mutation TP53, bien qu'elle établisse le pronostic, n'est pas indiquée au moment du diagnostic, mais au moment où le traitement va commencer (première ligne et avant les lignes successives si nécessaire)."}, "2": {"exist": false, "char_ranges": [], "word_ranges": [], "text": ""}, "3": {"exist": false, "char_ranges": [], "word_ranges": [], "text": ""}, "4": {"exist": true, "char_ranges": [[77, 355]], "word_ranges": [[13, 55]], "text": "On nous parle d'une leucocytose aux dépens des lymphocytes avec un phénotype B-CLL. Il semble qu'il n'y ait pas d'autres données alarmantes. Aucune donnée clinique n'est donnée (nous supposons donc que le patient est asymptomatique). Pas de cytopénies... Conclusion : FOLLOW UP."}, "5": {"exist": false, "char_ranges": [], "word_ranges": [], "text": ""}}} +{"id": 111, "year": 2012, "question_id_specific": 153, "full_question": "Chez une femme de 59 ans qui consulte pour un écoulement spontané, unilatéral et non antérieur du mamelon droit sans nodularité palpable, quel est le diagnostic le plus probable ?", "full_answer": "La bonne réponse est 2. En présence d'une télorrhée hémorragique unilatérale et non artificielle, la cause la plus fréquente est le papillome intraductal (50 %), suivi de l'ectasie canalaire et du carcinome. Lorsqu'ils sont associés à une masse palpable, 60 % correspondent à un carcinome. La maladie de Paget est de type eczémateux. Dans l'hyperprolactinémie, la télorrhée est bilatérale et pluriorificielle. La mammographie ne montre souvent pas les papillomes, l'échographie et la galactographie étant plus utiles.", "type": "GYNÉCOLOGIE ET OBSTÉTRIQUE", "options": {"1": "Compte tenu de la nature hématologique de l'écoulement, le diagnostic le plus probable est celui d'un carcinome canalaire infiltrant.", "2": "Papillome intraductal.", "3": "Maladie de Paget du mamelon.", "4": "Hyperprolactinémie tumorale.", "5": "La mammographie permet de poser le diagnostic."}, "correct_option": 2, "explanations": {"1": {"exist": true, "char_ranges": [[208, 289]], "word_ranges": [[32, 45]], "text": "Lorsqu'ils sont associés à une masse palpable, 60 % correspondent à un carcinome."}, "2": {"exist": true, "char_ranges": [[24, 161]], "word_ranges": [[5, 25]], "text": "En présence d'une télorrhée hémorragique unilatérale et non artificielle, la cause la plus fréquente est le papillome intraductal (50 %),"}, "3": {"exist": true, "char_ranges": [[290, 333]], "word_ranges": [[45, 53]], "text": "La maladie de Paget est de type eczémateux."}, "4": {"exist": true, "char_ranges": [[334, 409]], "word_ranges": [[53, 61]], "text": "Dans l'hyperprolactinémie, la télorrhée est bilatérale et pluriorificielle."}, "5": {"exist": true, "char_ranges": [[410, 517]], "word_ranges": [[61, 76]], "text": "La mammographie ne montre souvent pas les papillomes, l'échographie et la galactographie étant plus utiles."}}} +{"id": 119, "year": 2012, "question_id_specific": 119, "full_question": "Une jeune femme de 17 ans s'est présentée aux urgences avec une forte fièvre, des douleurs pharyngées et une lymphadénopathie cervicale. Une pharyngite aiguë avait déjà été diagnostiquée et traitée à l'amoxicilline. Elle a ensuite présenté une éruption cutanée maculaire généralisée. Les examens de laboratoire montrent une légère leucocytose avec lymphocytose et présence de lymphocytes activés, une légère thrombopénie et une légère augmentation des transaminases. Quel serait le diagnostic le plus probable de ce tableau clinique ?", "full_answer": "Il s'agit certainement d'un tableau typique (EBV, CMV, VH6, Toxoplasma), que vous ne verrez pas aussi clairement à la porte de la salle d'urgence lorsque vous êtes de garde. Attention au mauvais sang dans la réponse 3, car la toxoplasmose aiguë peut se comporter de la sorte et provoquer également un exanthème maculaire, bien que le cas de cette jeune fille soit lié à la prise d'amoxicilline. À propos, j'ajouterais une remarque : si, au lieu d'une jeune fille de 17 ans, il s'agissait d'un jeune homme ou d'une jeune fille ayant des relations sexuelles à risque, n'oubliez pas que la primo-infection par le VIH peut être à l'origine du syndrome mononucléosique.", "type": "INFECTOLOGIE", "options": {"1": "Il s'agit d'un tableau typique de la mononucléose infectieuse.", "2": "Infection par le virus de la varicelle et du zona.", "3": "Toxoplasmose aiguë.", "4": "Maladie de Lyme.", "5": "Infection par le virus de l'herpès 8."}, "correct_option": 1, "explanations": {"1": {"exist": false, "char_ranges": [], "word_ranges": [], "text": ""}, "2": {"exist": false, "char_ranges": [], "word_ranges": [], "text": ""}, "3": {"exist": true, "char_ranges": [[223, 394]], "word_ranges": [[38, 67]], "text": "la toxoplasmose aiguë peut se comporter de la sorte et provoquer également un exanthème maculaire, bien que le cas de cette jeune fille soit lié à la prise d'amoxicilline."}, "4": {"exist": false, "char_ranges": [], "word_ranges": [], "text": ""}, "5": {"exist": false, "char_ranges": [], "word_ranges": [], "text": ""}}} +{"id": 562, "year": 2022, "question_id_specific": 183, "full_question": "Une femme de 45 ans, sans antécédents particuliers, a consulté pour une dyspnée qui durait depuis environ 4 jours. L'examen a montré une tachycardie rythmique, sans souffle, et l'auscultation pulmonaire était normale. Les gaz du sang artériel montrent une pO2 de 70 mmHg et une pCO2 de 32 mmHg. L'hémogramme et les fonctions rénale et hépatique sont normaux. Le temps de prothrombine est de 90 %, le temps de céphaline activé (TCA) est de 2 par rapport au contrôle (N <1,2). Parmi les diagnostics suivants, lequel est le plus probable ?", "full_answer": "Dans cette question, on suppose qu'il s'agit d'une femme de 45 ans à qui l'on a diagnostiqué une thromboembolie pulmonaire (TEP). Avec ce diagnostic, les options 3 et 4 sont directement exclues (en outre, il s'agit d'une femme, donc nous n'aurions pas à penser à l'hémophilie). Par conséquent, ils veulent que vous sachiez qu'elle a un ac lupique positif et que vous fassiez le lien direct avec le syndrome des antiphospholipides (SAP). En tout état de cause, bien qu'elle ne fasse pas partie des réponses possibles, la positivité au lupus peut être liée à une myriade de situations cliniques, de syndromes et de pathologies. Supposer qu'un PTE + lupus est un PBS me semble quelque peu exagéré, mais c'est le plus probable avec les données fournies.", "type": "HÉMATOLOGIE", "options": {"1": "Syndrome des antiphospholipides.", "2": "Facteur V de Leiden.", "3": "Hémophilie.", "4": "Péricardite aiguë.", "5": null}, "correct_option": 1, "explanations": {"1": {"exist": true, "char_ranges": [[278, 436]], "word_ranges": [[46, 72]], "text": "Par conséquent, ils veulent que vous sachiez qu'elle a un ac lupique positif et que vous fassiez le lien direct avec le syndrome des antiphospholipides (SAP)."}, "2": {"exist": false, "char_ranges": [], "word_ranges": [], "text": ""}, "3": {"exist": true, "char_ranges": [[0, 277]], "word_ranges": [[0, 46]], "text": "Dans cette question, on suppose qu'il s'agit d'une femme de 45 ans à qui l'on a diagnostiqué une thromboembolie pulmonaire (TEP). Avec ce diagnostic, les options 3 et 4 sont directement exclues (en outre, il s'agit d'une femme, donc nous n'aurions pas à penser à l'hémophilie)."}, "4": {"exist": true, "char_ranges": [[0, 277]], "word_ranges": [[0, 46]], "text": "Dans cette question, on suppose qu'il s'agit d'une femme de 45 ans à qui l'on a diagnostiqué une thromboembolie pulmonaire (TEP). Avec ce diagnostic, les options 3 et 4 sont directement exclues (en outre, il s'agit d'une femme, donc nous n'aurions pas à penser à l'hémophilie)."}, "5": {"exist": false, "char_ranges": [], "word_ranges": [], "text": ""}}} +{"id": 514, "year": 2021, "question_id_specific": 97, "full_question": "Après un accident de la route, vous vous occupez d'un blessé. Le patient ouvre les yeux à votre appel, mais ne prononce que des mots inappropriés. A l'examen moteur, vous localisez la douleur aux extrémités droites, mais elle s'étend à la stimulation des extrémités gauches. Comment décririez-vous son état selon l'échelle de coma de Glasgow ?", "full_answer": "La première chose à savoir lorsqu'on applique l'échelle de coma de Glasgow à un patient est qu'il faut toujours prendre la meilleure valeur dans chaque domaine (réponse oculaire, réponse motrice et réponse verbale). Dans le cas présent, nous avons un patient qui ouvre les yeux pour appeler (réponse oculaire 3 sur 4), prononce des mots inappropriés (réponse verbale 3 sur 5) et localise la douleur avec les membres droits (réponse motrice 5 sur 6). Le score de l'échelle de coma de Glasgow serait de 11 (sur 15). La meilleure valeur de l'examen est retenue car dans de nombreux cas (évaluation d'un polytraumatisé, d'un accident vasculaire cérébral ischémique ou d'une hémorragie cérébrale aiguë spontanée), l'examen moteur peut être artéfactualisé par d'autres éléments. Par exemple : chez un polytraumatisé (comme le patient en question), la réponse présentée avec les extrémités gauches peut être due à un traumatisme localisé dans cette zone, et non à proprement parler à une lésion cérébrale traumatique sous-jacente. De même, la réponse verbale peut parfois être sous-estimée chez le polytraumatisé en raison de l'obstruction des voies respiratoires (par exemple, parce que la langue est ramenée dans l'oropharynx). En évaluant toujours la meilleure réponse, nous aurons une image plus fidèle de l'état du patient.", "type": "SOINS CRITIQUES ET D'URGENCE", "options": {"1": "E3V2M4.", "2": "E3V3M5.", "3": "E2V3M4.", "4": "E2V4M5.", "5": null}, "correct_option": 2, "explanations": {"1": {"exist": false, "char_ranges": [], "word_ranges": [], "text": ""}, "2": {"exist": false, "char_ranges": [], "word_ranges": [], "text": ""}, "3": {"exist": false, "char_ranges": [], "word_ranges": [], "text": ""}, "4": {"exist": false, "char_ranges": [], "word_ranges": [], "text": ""}, "5": {"exist": false, "char_ranges": [], "word_ranges": [], "text": ""}}} +{"id": 101, "year": 2012, "question_id_specific": 86, "full_question": "Une femme de 55 ans consulte pour fatigue et polyurie depuis deux ans. Les analyses révèlent une hypercalcémie, une augmentation de la PTH et des marqueurs de renouvellement osseux. L'étude radiographique montre une résorption sous-périostée et une ostéoporose. La scintigraphie au sestamibi a révélé un adénome parathyroïdien. Une parathyroïdectomie a été réalisée par chirurgie mini-invasive. En postopératoire, le patient a développé une hypocalcémie sévère et une tétanie, avec des taux de PTH inférieurs à 5 ng/l. Le patient répond bien au traitement initial par calcium intraveineux, puis par calcium oral et vitamine D. Quel est le diagnostic le plus probable ?", "full_answer": "Question très difficile, je ne m'inquiéterais pas d'échouer. Il est facile de se méprendre sur la réponse concernant l'hypoparathyroïdie. L'hypoparathyroïdie permanente post-chirurgicale après parathyroïdectomie d'un adénome par chirurgie mini-invasive est rare. J'écris un bref résumé du syndrome de l'os affamé que j'ai trouvé dans un rapport de cas dans les Annals of Internal Medicine [1] : \"La physiopathologie du syndrome de l'os affamé (HBS) est liée à un déséquilibre entre la formation et la résorption osseuse, qui est associé à une hypocalcémie, une hypophosphatémie et une hypomagnésémie (1). Ce syndrome peut être observé après une intervention chirurgicale chez des patients atteints d'hyperparathyroïdie primaire (HPT), ainsi que dans l'HPT tertiaire en cas d'insuffisance rénale chronique, et dans une moindre mesure après un traitement pour des processus impliquant un excès d'hormones thyroïdiennes circulantes (2). Dans l'HPT, il y a un excès d'hormone parathyroïdienne (PTH) qui stimule l'activité ostéoclastique, entraînant une déminéralisation de la matrice osseuse et libérant du calcium dans la circulation sanguine. Après une parathyroïdectomie, les taux sériques de PTH chutent de façon spectaculaire, la résorption osseuse induite par la PTH cesse donc, tandis que l'activité ostéoblastique se poursuit, entraînant une augmentation de l'absorption osseuse de calcium, de phosphate et de magnésium, d'où la SHH (3)\".", "type": "ENDOCRINOLOGIE", "options": {"1": "Hypoparathyroïdie chirurgicale permanente.", "2": "Transfusion de sang citraté.", "3": "Insuffisance en vitamine D.", "4": "Le syndrome de l'os affamé.", "5": "Ostéomalacie."}, "correct_option": 4, "explanations": {"1": {"exist": true, "char_ranges": [[138, 262]], "word_ranges": [[19, 31]], "text": "L'hypoparathyroïdie permanente post-chirurgicale après parathyroïdectomie d'un adénome par chirurgie mini-invasive est rare."}, "2": {"exist": false, "char_ranges": [], "word_ranges": [], "text": ""}, "3": {"exist": false, "char_ranges": [], "word_ranges": [], "text": ""}, "4": {"exist": true, "char_ranges": [[395, 599]], "word_ranges": [[56, 87]], "text": "\"La physiopathologie du syndrome de l'os affamé (HBS) est liée à un déséquilibre entre la formation et la résorption osseuse, qui est associé à une hypocalcémie, une hypophosphatémie et une hypomagnésémie"}, "5": {"exist": false, "char_ranges": [], "word_ranges": [], "text": ""}}} +{"id": 230, "year": 2014, "question_id_specific": 179, "full_question": "Un garçon de 4 ans s'est présenté au centre de santé 5 minutes plus tôt avec un angio-œdème du visage, une conjonctivite, une congestion nasale et un enrouement, coïncidant avec l'ingestion d'une cuillerée de yaourt qui lui avait été donnée par erreur à l'école. Ses antécédents médicaux comprenaient un diagnostic d'allergie aux protéines du lait de vache. L'examen révèle une légère hypotension, une fréquence cardiaque de 110 bpm, une saturation en O2 de 93%, un patient pâle et légèrement en sueur, avec des sifflements épars. Quel est le premier traitement de choix ?", "full_answer": "Il s'agit d'un cas d'anaphylaxie, la première mesure consiste donc à administrer de l'adrénaline 1/1000 par voie intramusculaire (0,01mg/kg). Ensuite, des corticostéroïdes systémiques tels que la méthylprednisolone seront administrés, qui mettront quelques heures à agir. Donc l'adrénaline d'abord, car elle agit plus vite, puis le reste (question très similaire à celle de l'année dernière, au lieu du yaourt, c'était l'omelette).", "type": "PÉDIATRIE", "options": {"1": "Faire vomir.", "2": "Adrénaline 1/1000 sous-cutanée.", "3": "Adrénaline 1/1000 intramusculaire.", "4": "Méthylprednisolone intramusculaire.", "5": "Salbutamol en nébulisation."}, "correct_option": 3, "explanations": {"1": {"exist": false, "char_ranges": [], "word_ranges": [], "text": ""}, "2": {"exist": false, "char_ranges": [], "word_ranges": [], "text": ""}, "3": {"exist": true, "char_ranges": [[277, 323]], "word_ranges": [[36, 43]], "text": "l'adrénaline d'abord, car elle agit plus vite,"}, "4": {"exist": true, "char_ranges": [[142, 271]], "word_ranges": [[19, 35]], "text": "Ensuite, des corticostéroïdes systémiques tels que la méthylprednisolone seront administrés, qui mettront quelques heures à agir."}, "5": {"exist": false, "char_ranges": [], "word_ranges": [], "text": ""}}} +{"id": 479, "year": 2020, "question_id_specific": 164, "full_question": "Homme de 72 ans présentant une fièvre de 38,7°C depuis 48 heures, une toux, une expectoration purulente et une dyspnée. Signes vitaux : TA 85/60 mmHg, fréquence cardiaque 100 bpm, fréquence respiratoire 35 rpm, SatO2 80%. On note la présence d'une confusion et de crépitants dans le champ pulmonaire inférieur gauche. La radiographie thoracique confirme l'existence d'une pneumonie de la lingula et du lobe inférieur gauche. Quel serait le lieu de soins approprié ?", "full_answer": "En appliquant l'échelle de gravité CURB-65 (l'une des plus utilisées aux urgences pour stratifier la gravité de la pneumonie, bien que dans ce cas, en appliquant simplement le bon sens, le patient devrait être admis en USI), nous aurions un score d'au moins 4 points (il nous manquerait les données relatives à l'urée sérique), ce qui nous donne des critères de gravité. Étant donné qu'aucun critère ne contre-indique l'admission à l'USI, l'admission dans cette unité serait indiquée.", "type": "SOINS INTENSIFS", "options": {"1": "Traitement ambulatoire à domicile.", "2": "Admission dans une unité d'observation d'urgence.", "3": "Admission à l'hôpital dans le service.", "4": "Admission à l'unité de soins intensifs.", "5": null}, "correct_option": 4, "explanations": {"1": {"exist": false, "char_ranges": [], "word_ranges": [], "text": ""}, "2": {"exist": false, "char_ranges": [], "word_ranges": [], "text": ""}, "3": {"exist": false, "char_ranges": [], "word_ranges": [], "text": ""}, "4": {"exist": true, "char_ranges": [[0, 224]], "word_ranges": [[0, 37]], "text": "En appliquant l'échelle de gravité CURB-65 (l'une des plus utilisées aux urgences pour stratifier la gravité de la pneumonie, bien que dans ce cas, en appliquant simplement le bon sens, le patient devrait être admis en USI),"}, "5": {"exist": false, "char_ranges": [], "word_ranges": [], "text": ""}}} +{"id": 515, "year": 2021, "question_id_specific": 101, "full_question": "Homme de 15 ans, admis après avoir été renversé dans la rue et éjecté. Il a été admis conscient et orienté, avec une douleur intense dans l'hémithorax gauche et une dyspnée. L'examen physique révèle une tension artérielle de 90/60 mmHg, une fréquence cardiaque de 130 bpm, une saturation basale en oxygène de 90 %, une fréquence respiratoire de 35 rpm. L'hémithorax gauche est complètement hypophonique et la percussion est sourde. La radiographie du thorax montre de multiples fractures des côtes gauches et un épanchement pleural massif ipsilatéral. La mise en place d'un drain pleural est indiquée, 1700 cc de liquide hématopoïétique s'en échappant. Quelle est la décision à prendre ?", "full_answer": "Il y a deux réponses que nous pouvons immédiatement écarter si nous nous en tenons à la prise en charge initiale des traumatismes thoraciques graves : l'option 1 (la ventilation non invasive n'est pas indiquée car elle ne sécurise pas les voies respiratoires) et l'option 3 (l'observation dans ces cas, en attendant une aggravation, ne conduit qu'à une augmentation de la morbidité et de la mortalité). Entre les options 2 et 4, il pourrait y avoir une controverse entre la littérature et la pratique clinique habituelle, dans laquelle nous devons généralement trouver un équilibre délicat entre la stabilité clinique du patient (et la mesure dans laquelle nous pouvons \"déguiser\" cette stabilité avec nos thérapies de maintien en vie) et les actions qui aboutissent à une meilleure prise en charge du patient pour le traitement définitif des lésions. Nous avons affaire à un patient polytraumatisé grave. Avec les données fournies par la question, un patient qui semble en état de choc (tachycardie, hypoxémie, tachypnée... bien qu'on ne nous donne pas de données, par exemple, sur la lactacidémie), et chez qui nous avons, à notre connaissance, un traumatisme thoracique gauche, ce qui implique non seulement une lésion osseuse, mais peut masquer d'autres lésions très graves qui ne sont pas diagnostiquées par la radiographie thoracique : contusion myocardique, épanchement péricardique, lésion splénique, lésion diaphragmatique... Le patient a été placé sur un drain pleural gauche et nous avons obtenu un débit de 1700cc de liquide hématogène. Si nous suivons les directives ATLS (ATLS-Advanced Trauma Life Support 10th edition) pour les soins aux patients polytraumatisés sévères, elles nous disent qu'un saignement immédiat de plus de 1500cc de sang est une indication de thoracotomie urgente, et que même chez les patients avec un saignement de moins de 1500cc, si le débit persiste à un taux de 200mL/h pendant 2 à 4h, cela serait également une indication. Cependant, dans la réalité, la conduite à tenir dépendra de la stabilité du patient une fois que nous aurons appliqué les traitements indiqués dans l'option 2 : analgésie adéquate, oxygénothérapie et initiation de la transfusion sanguine. Si, en optimisant le patient, nous parvenons à le stabiliser et à estimer qu'il est possible de le transférer en toute sécurité, la réalisation d'une tomodensitométrie corporelle permettra de définir de manière beaucoup plus détaillée les lésions dont il souffre et contribuera à garantir que la chirurgie qu'il subira probablement sera définitive et ne se limitera pas à un simple contrôle des dommages. Dans ce cas, l'option 4 susciterait des doutes, même si, comme je l'ai dit, tout dépendra de la stabilité clinique du patient et du temps dont nous disposons avant de parvenir à un contrôle définitif du foyer hémorragique (ce qui est, après tout, ce que nous visons dans cette situation).", "type": "SOINS CRITIQUES ET D'URGENCE", "options": {"1": "Instaurer une ventilation mécanique non invasive et demander une transfusion sanguine.", "2": "Analgésie, oxygénothérapie, transfusion sanguine et indication d'une intervention chirurgicale urgente.", "3": "Intubation orotrachéale, demande de transfusion sanguine et observation pour, en cas d'aggravation, poser l'indication d'une intervention chirurgicale.", "4": "Intubation orotrachéale et tomodensitométrie urgente pour une évaluation précise des lésions.", "5": null}, "correct_option": 2, "explanations": {"1": {"exist": true, "char_ranges": [[162, 259]], "word_ranges": [[28, 43]], "text": "(la ventilation non invasive n'est pas indiquée car elle ne sécurise pas les voies respiratoires)"}, "2": {"exist": true, "char_ranges": [[1966, 2204]], "word_ranges": [[310, 346]], "text": "Cependant, dans la réalité, la conduite à tenir dépendra de la stabilité du patient une fois que nous aurons appliqué les traitements indiqués dans l'option 2 : analgésie adéquate, oxygénothérapie et initiation de la transfusion sanguine."}, "3": {"exist": true, "char_ranges": [[275, 402]], "word_ranges": [[46, 66]], "text": "(l'observation dans ces cas, en attendant une aggravation, ne conduit qu'à une augmentation de la morbidité et de la mortalité)."}, "4": {"exist": true, "char_ranges": [[2205, 2609]], "word_ranges": [[346, 409]], "text": "Si, en optimisant le patient, nous parvenons à le stabiliser et à estimer qu'il est possible de le transférer en toute sécurité, la réalisation d'une tomodensitométrie corporelle permettra de définir de manière beaucoup plus détaillée les lésions dont il souffre et contribuera à garantir que la chirurgie qu'il subira probablement sera définitive et ne se limitera pas à un simple contrôle des dommages."}, "5": {"exist": false, "char_ranges": [], "word_ranges": [], "text": ""}}} +{"id": 234, "year": 2014, "question_id_specific": 105, "full_question": "Un homme de 72 ans s'est présenté aux urgences pour une fracture pathologique du fémur gauche. Après l'opération, une étude diagnostique a été réalisée pour déterminer la pathologie sous-jacente avec les résultats suivants : hémoglobine 9,5 g/dl, protéines totales 11 g/dl, (VN : 6-8 g/dl), albumine sérique 2 g/dl, (VN 3,5-5,0 g/dl), bêta 2 microglobuline 6 mg/l (VN 1,1-2,4 mg/l), créatinine sérique 1,8 mg/dl (VN : 0,1-1,4 mg/dl). Indiquez quels tests diagnostiques seraient nécessaires pour confirmer le diagnostic le plus probable :", "full_answer": "Vous allez avoir un myélome. Pour le diagnostic, un prélèvement (moelle osseuse) et une électrophorèse permettent de déterminer le type de pic monoclonal. Ensuite, pour l'extension, la valeur pronostique et pour décider des traitements, le reste, mais une fois diagnostiqué.", "type": "HÉMATOLOGIE", "options": {"1": "Série osseuse radiologique et aspiration de la moelle osseuse.", "2": "Électrophorèse du sérum et de l'urine et tests de la fonction rénale.", "3": "Aspiration de la moelle osseuse et concentration de calcium sérique.", "4": "Aspiration de la moelle osseuse et électrophorèse du sérum et de l'urine.", "5": "Biopsie de la fracture pathologique et série osseuse radiologique."}, "correct_option": 4, "explanations": {"1": {"exist": false, "char_ranges": [], "word_ranges": [], "text": ""}, "2": {"exist": false, "char_ranges": [], "word_ranges": [], "text": ""}, "3": {"exist": false, "char_ranges": [], "word_ranges": [], "text": ""}, "4": {"exist": true, "char_ranges": [[0, 154]], "word_ranges": [[0, 23]], "text": "Vous allez avoir un myélome. Pour le diagnostic, un prélèvement (moelle osseuse) et une électrophorèse permettent de déterminer le type de pic monoclonal."}, "5": {"exist": false, "char_ranges": [], "word_ranges": [], "text": ""}}} +{"id": 174, "year": 2013, "question_id_specific": 52, "full_question": "Un homme de 67 ans consulte pour une dyspnée d'effort modérée d'apparition progressive depuis quelques années. Il présente une expectoration blanchâtre quotidienne et parfois des bruits respiratoires, surtout en hiver lors d'infections respiratoires. Il fume environ 20 cigarettes par jour depuis 45 ans. À l'examen, il est eupnéique, de couleur normale, avec une saturation en oxygène de 94 %, et son murmure vésiculaire à l'auscultation thoracique est en diminution généralisée, ce qui constitue le seul élément intéressant. La radiographie du thorax a montré une silhouette cardiaque allongée, avec des signes d'hyperinflation ou de piégeage de l'air pulmonaire, sans autre altération. La spirométrie a été effectuée avec les résultats suivants : CVF 84%, VEMS 58%, VEMS/CVF 61%, inchangés après bronchodilatateur. Parmi les traitements indiqués, indiquez celui qui convient le mieux à ce patient :", "full_answer": "Chez un patient souffrant d'une BPCO peu symptomatique qui pourrait probablement être classé GOLD 2, le traitement initial peut être soit du tiotropium inhalé, soit un bêta-2 à longue durée d'action. La réponse correcte ne fait donc aucun doute.", "type": "PNEUMOLOGIE", "options": {"1": "Corticostéroïde inhalé quotidien.", "2": "Oxygénothérapie à domicile avec source d'oxygène portable pour la marche.", "3": "Antagoniste oral des leucotriènes.", "4": "Corticostéroïde oral pendant trois mois.", "5": "Tiotropium en inhalation."}, "correct_option": 5, "explanations": {"1": {"exist": false, "char_ranges": [], "word_ranges": [], "text": ""}, "2": {"exist": false, "char_ranges": [], "word_ranges": [], "text": ""}, "3": {"exist": false, "char_ranges": [], "word_ranges": [], "text": ""}, "4": {"exist": false, "char_ranges": [], "word_ranges": [], "text": ""}, "5": {"exist": true, "char_ranges": [[0, 199]], "word_ranges": [[0, 31]], "text": "Chez un patient souffrant d'une BPCO peu symptomatique qui pourrait probablement être classé GOLD 2, le traitement initial peut être soit du tiotropium inhalé, soit un bêta-2 à longue durée d'action."}}} +{"id": 143, "year": 2012, "question_id_specific": 141, "full_question": "Chez un enfant de 7 mois présentant de la fièvre et de l'irritabilité, une fontanelle bombée et une étude du liquide céphalo-rachidien avec 110 cellules/mm3 (75 % de lymphocytes), 120 mg/dl de protéines et 28 mg/dl de glucose (glycémie sérique de 89 mg/dl), quelle est la suspicion diagnostique la plus raisonnable ?", "full_answer": "La bonne réponse est 3. Elle définit un liquide céphalo-rachidien typique de la tuberculose. Dans la pratique clinique, les choses ne sont pas toujours aussi simples......", "type": "PÉDIATRIE", "options": {"1": "Méningite virale.", "2": "Méningite bactérienne.", "3": "Méningite tuberculeuse.", "4": "Syndrome de la mononucléose.", "5": "Syndrome de Guillain-Barré."}, "correct_option": 3, "explanations": {"1": {"exist": false, "char_ranges": [], "word_ranges": [], "text": ""}, "2": {"exist": false, "char_ranges": [], "word_ranges": [], "text": ""}, "3": {"exist": true, "char_ranges": [[24, 92]], "word_ranges": [[5, 14]], "text": "Elle définit un liquide céphalo-rachidien typique de la tuberculose."}, "4": {"exist": false, "char_ranges": [], "word_ranges": [], "text": ""}, "5": {"exist": false, "char_ranges": [], "word_ranges": [], "text": ""}}} +{"id": 9, "year": 2011, "question_id_specific": 36, "full_question": "Une cholécystite calcaire aiguë a été diagnostiquée chez un patient de 87 ans ayant des antécédents de bronchite chronique et d'insuffisance cardiaque. Après quatre jours de traitement par régime absolu, thérapie sérique et pipéracilline/tazobactam, le patient continue à avoir de la fièvre, des douleurs abdominales persistantes et une leucocytose. L'approche la plus appropriée à ce stade serait la suivante :", "full_answer": "Je pense que la bonne réponse est 2. Bien que le traitement de la cholécystite aiguë soit la cholécystectomie, il faut pour cela que le patient soit un candidat à la chirurgie, dans ce cas il s'agit d'un patient âgé, avec des maladies antérieures qui augmentent le risque chirurgical et qui serait un ASA IV pour une intervention urgente. Dans ces cas, la cholécystostomie peut guérir la cholécystite ; après la guérison, la chirurgie programmée peut être évaluée en fonction de la situation du patient.", "type": "CHIRURGIE", "options": {"1": "Traitement chirurgical (cholécystectomie d'urgence).", "2": "Drainage biliaire par cholécystostomie percutanée.", "3": "Remplacer pipéracilline/tazobactam par métronidazole+cefotaxime.", "4": "Remplacer pipéracilline/tazobactam par amikacine+clindamycine.", "5": "Ajouter la gentamicine."}, "correct_option": 2, "explanations": {"1": {"exist": false, "char_ranges": [], "word_ranges": [], "text": ""}, "2": {"exist": true, "char_ranges": [[0, 338]], "word_ranges": [[0, 59]], "text": "Je pense que la bonne réponse est 2. Bien que le traitement de la cholécystite aiguë soit la cholécystectomie, il faut pour cela que le patient soit un candidat à la chirurgie, dans ce cas il s'agit d'un patient âgé, avec des maladies antérieures qui augmentent le risque chirurgical et qui serait un ASA IV pour une intervention urgente."}, "3": {"exist": false, "char_ranges": [], "word_ranges": [], "text": ""}, "4": {"exist": false, "char_ranges": [], "word_ranges": [], "text": ""}, "5": {"exist": false, "char_ranges": [], "word_ranges": [], "text": ""}}} +{"id": 601, "year": 2022, "question_id_specific": 112, "full_question": "Une femme de 61 ans, administrative, aux antécédents de surpoids, d'hypertension, de dyslipidémie et de syndrome métabolique, consulte pour des douleurs dans les deux fesses, la région trochantérienne gauche, la face latérale de la cuisse gauche jusqu'au genou et la jambe gauche jusqu'au tiers moyen. La douleur apparaît lorsque le membre inférieur est soulevé avec le genou en extension, mais est soulagée lorsque le genou est fléchi. Quelle est la première suspicion clinique ?", "full_answer": "Signe de Lasegue positif, réapparition des symptômes lors de la manœuvre d'extension du membre atteint, compatible avec une atteinte des racines nerveuses au niveau de la colonne lombo-sacrée.", "type": "TRAUMATOLOGIE", "options": {"1": "Arthrite goutteuse de la hanche gauche.", "2": "Arthrose coxo-fémorale gauche.", "3": "Lombalgie irradiée / lombosciatique.", "4": "Claudication due à une sténose du canal.", "5": null}, "correct_option": 3, "explanations": {"1": {"exist": false, "char_ranges": [], "word_ranges": [], "text": ""}, "2": {"exist": false, "char_ranges": [], "word_ranges": [], "text": ""}, "3": {"exist": true, "char_ranges": [[0, 192]], "word_ranges": [[0, 28]], "text": "Signe de Lasegue positif, réapparition des symptômes lors de la manœuvre d'extension du membre atteint, compatible avec une atteinte des racines nerveuses au niveau de la colonne lombo-sacrée."}, "4": {"exist": false, "char_ranges": [], "word_ranges": [], "text": ""}, "5": {"exist": false, "char_ranges": [], "word_ranges": [], "text": ""}}} +{"id": 202, "year": 2013, "question_id_specific": 44, "full_question": "Patient atteint de pancréatite aiguë. La tomographie axiale informatisée (TAO) réalisée 72 heures après l'admission a montré une nécrose de 50 % du pancréas. À la troisième semaine d'admission, le patient a commencé à présenter une forte fièvre et une leucocytose. Une radiographie pulmonaire urgente et un sédiment urinaire sont demandés, tous deux normaux. Indiquez la prochaine étape à suivre :", "full_answer": "bien que les dernières recommandations sur la prise en charge de la pancréatite aiguë sévère déconseillent la FNA en raison du risque d'infection par nécrose stérile. La prise en charge actuelle est basée sur un traitement par étapes. Dans la situation décrite dans la question, le traitement actuel consiste à commencer par des antibiotiques et à évaluer la réponse.", "type": "CHIRURGIE GÉNÉRALE", "options": {"1": "Ponction à l'aiguille fine de la nécrose pancréatique sous guidage échographique ou tomodensitométrique.", "2": "Aspiration broncho-alvéolaire, culture d'urine et hémocultures.", "3": "Imagerie par résonance magnétique nucléaire de l'abdomen.", "4": "Cholécystectomie urgente.", "5": "Echoendoscopie avec ponction de la nécrose."}, "correct_option": 1, "explanations": {"1": {"exist": false, "char_ranges": [], "word_ranges": [], "text": ""}, "2": {"exist": false, "char_ranges": [], "word_ranges": [], "text": ""}, "3": {"exist": false, "char_ranges": [], "word_ranges": [], "text": ""}, "4": {"exist": false, "char_ranges": [], "word_ranges": [], "text": ""}, "5": {"exist": false, "char_ranges": [], "word_ranges": [], "text": ""}}} +{"id": 259, "year": 2014, "question_id_specific": 97, "full_question": "Un homme de 54 ans s'est rendu à son entreprise pour un bilan de santé. Son indice de masse corporelle est de 32,8 kg/m2 et sa glycémie à jeun est de 138 mg/l. Un mois plus tard, sa glycémie est de 13 mg/dl. Un mois plus tard, sa glycémie était de 13 mg/dl. Quelle recommandation de traitement feriez-vous en premier ?", "full_answer": "Changements de comportement : régime alimentaire et exercice physique. C'est la première chose que nous faisons lorsqu'on nous diagnostique un diabète de type 2. C'est une bonne question pour nous rappeler qu'avant tout médicament, il faut insister sur les changements d'habitudes. Ensuite, nous prescrivons de la Metformine.", "type": "ENDOCRINOLOGIE", "options": {"1": "Administrer la metformine.", "2": "Prescrire une sulfonylurée.", "3": "Changements de comportement. Régime alimentaire et exercice physique.", "4": "Insuline avant chaque repas.", "5": "Prendre l'acarbose le soir, avant de se coucher."}, "correct_option": 3, "explanations": {"1": {"exist": false, "char_ranges": [], "word_ranges": [], "text": ""}, "2": {"exist": false, "char_ranges": [], "word_ranges": [], "text": ""}, "3": {"exist": true, "char_ranges": [[71, 161]], "word_ranges": [[9, 24]], "text": "C'est la première chose que nous faisons lorsqu'on nous diagnostique un diabète de type 2."}, "4": {"exist": false, "char_ranges": [], "word_ranges": [], "text": ""}, "5": {"exist": false, "char_ranges": [], "word_ranges": [], "text": ""}}} +{"id": 204, "year": 2014, "question_id_specific": 220, "full_question": "Un homme de 43 ans consulte pour un syndrome diarrhéique et évoque trois pneumonies survenues à l'âge adulte. Parmi les examens immunologiques suivants, lequel doit-on demander ?", "full_answer": "Plus de deux pneumonies lobaires imposent d'écarter la possibilité d'un déficit immunitaire.", "type": "GÉNÉTIQUE ET IMMUNOLOGIE", "options": {"1": "Numération des immunoglobulines sériques et test de la capacité de production d'anticorps.", "2": "Test de phagocytose des neutrophiles et de métabolisme oxydatif.", "3": "Test d'apoptose (mort cellulaire programmée) sur les lymphocytes circulants du patient.", "4": "Étude du répertoire et de la clonalité des lymphocytes T (alpha/beta).", "5": "Dans le cas de ce patient, il ne serait pas approprié de demander une étude immunologique."}, "correct_option": 1, "explanations": {"1": {"exist": true, "char_ranges": [[0, 92]], "word_ranges": [[0, 12]], "text": "Plus de deux pneumonies lobaires imposent d'écarter la possibilité d'un déficit immunitaire."}, "2": {"exist": false, "char_ranges": [], "word_ranges": [], "text": ""}, "3": {"exist": false, "char_ranges": [], "word_ranges": [], "text": ""}, "4": {"exist": false, "char_ranges": [], "word_ranges": [], "text": ""}, "5": {"exist": false, "char_ranges": [], "word_ranges": [], "text": ""}}} +{"id": 4, "year": 2011, "question_id_specific": 40, "full_question": "Un homme présente soudainement une asthénie et un ictère avec des transaminases supérieures à 2000 UI/L. Les marqueurs sérologiques présentent le profil suivant : IgM anti-VHA négatifs, HBsAg négatifs, IgM anti-HBc positifs, anti-VHC négatifs. Quel est le diagnostic ?", "full_answer": "Il s'agit d'un cas intéressant qui doit être étudié en général. D'après sa présentation, il s'agit clairement d'une hépatite aiguë, ce qui exclut la possibilité d'une hépatite 1. Des IgM négatifs pour le VHA excluent le 5 et un Ag HBs négatif exclut le 4 (pour avoir le virus delta, il faut l'antigène de surface B). Lorsque vous avez des IgM anti-HBc, il s'agit d'une hépatite B aiguë (3), car il y a une période de négativation de l'antigène.", "type": "DIGESTIF", "options": {"1": "Hépatite B chronique.", "2": "Hépatite aiguë non virale.", "3": "Hépatite B aiguë.", "4": "Surinfection du virus D (delta).", "5": "Hépatite aiguë A et B."}, "correct_option": 3, "explanations": {"1": {"exist": true, "char_ranges": [[64, 178]], "word_ranges": [[11, 28]], "text": "D'après sa présentation, il s'agit clairement d'une hépatite aiguë, ce qui exclut la possibilité d'une hépatite 1."}, "2": {"exist": false, "char_ranges": [], "word_ranges": [], "text": ""}, "3": {"exist": true, "char_ranges": [[317, 444]], "word_ranges": [[56, 79]], "text": "Lorsque vous avez des IgM anti-HBc, il s'agit d'une hépatite B aiguë (3), car il y a une période de négativation de l'antigène."}, "4": {"exist": true, "char_ranges": [[222, 316]], "word_ranges": [[37, 56]], "text": "et un Ag HBs négatif exclut le 4 (pour avoir le virus delta, il faut l'antigène de surface B)."}, "5": {"exist": true, "char_ranges": [[179, 221]], "word_ranges": [[28, 37]], "text": "Des IgM négatifs pour le VHA excluent le 5"}}} +{"id": 372, "year": 2016, "question_id_specific": 127, "full_question": "Un homme de 55 ans consulte pour une dysphonie. Dans l'anamnèse, il signale un mois d'asthénie et une perte de poids non quantifiée. La radiographie thoracique montre une augmentation de la densité du lobe supérieur gauche et une occupation de la fenêtre aorto-pulmonaire. La bronchoscopie montre une paralysie de la corde vocale gauche, sans image endoscopique évocatrice de néoplasie. Quel est le diagnostic le plus probable ?", "full_answer": "La cause la plus fréquente de paralysie du nerf récurrent est le carcinome broncho-pulmonaire. Ce n'est pas parce qu'il n'est pas accessible à la vision endoscopique qu'il n'existe pas.", "type": "PNEUMOLOGIE ET CHIRURGIE THORACIQUE", "options": {"1": "Néoplasie pulmonaire.", "2": "La sarcoïdose.", "3": "La silicose.", "4": "La tuberculose.", "5": null}, "correct_option": 1, "explanations": {"1": {"exist": true, "char_ranges": [[0, 94]], "word_ranges": [[0, 14]], "text": "La cause la plus fréquente de paralysie du nerf récurrent est le carcinome broncho-pulmonaire."}, "2": {"exist": false, "char_ranges": [], "word_ranges": [], "text": ""}, "3": {"exist": false, "char_ranges": [], "word_ranges": [], "text": ""}, "4": {"exist": false, "char_ranges": [], "word_ranges": [], "text": ""}, "5": {"exist": false, "char_ranges": [], "word_ranges": [], "text": ""}}} +{"id": 377, "year": 2016, "question_id_specific": 131, "full_question": "Une patiente de 65 ans a présenté une apraxie de la parole deux ans auparavant et a développé un parkinsonisme rigide-akinétique prédominant dans l'hémicorps droit avec des myoclonies superposées et un phénomène de main étrangère ou de membre étranger. Quel est le diagnostic que vous considérez comme le plus probable ?", "full_answer": "Comme elle est décrite comme un phénomène apraxique suivi d'un parkinsonisme rigide-acinétique asymétrique et de myoclonies supplémentaires, elle est typique de la dégénérescence corticobasale. La maladie de Parkinson ne peut pas l'être car elle ne présente pas d'apraxie comme caractéristique initiale, la maladie d'Alzheimer ne présente pas de parkinsonisme asymétrique et la maladie de Huntington typique ne se trouve pas chez les personnes âgées et lorsqu'elle fait son apparition à un âge avancé, elle se comporte comme une chorée.", "type": "NEUROLOGIE", "options": {"1": "La maladie de Parkinson.", "2": "Dégénérescence corticobasale.", "3": "La maladie d'Alzheimer.", "4": "La maladie de Huntington.", "5": null}, "correct_option": 2, "explanations": {"1": {"exist": true, "char_ranges": [[194, 303]], "word_ranges": [[24, 41]], "text": "La maladie de Parkinson ne peut pas l'être car elle ne présente pas d'apraxie comme caractéristique initiale,"}, "2": {"exist": true, "char_ranges": [[0, 193]], "word_ranges": [[0, 24]], "text": "Comme elle est décrite comme un phénomène apraxique suivi d'un parkinsonisme rigide-acinétique asymétrique et de myoclonies supplémentaires, elle est typique de la dégénérescence corticobasale."}, "3": {"exist": true, "char_ranges": [[305, 371]], "word_ranges": [[41, 50]], "text": "la maladie d'Alzheimer ne présente pas de parkinsonisme asymétrique"}, "4": {"exist": true, "char_ranges": [[375, 536]], "word_ranges": [[51, 79]], "text": "la maladie de Huntington typique ne se trouve pas chez les personnes âgées et lorsqu'elle fait son apparition à un âge avancé, elle se comporte comme une chorée."}, "5": {"exist": false, "char_ranges": [], "word_ranges": [], "text": ""}}} +{"id": 145, "year": 2012, "question_id_specific": 143, "full_question": "Un enfant d'un mois, exclusivement nourri au sein, consulte parce qu'il va à la selle tous les 5 ou 6 jours avec des efforts mais avec une consistance molle, quelle doit être notre attitude ?", "full_answer": "La bonne réponse est 4. La plage normale de production de selles chez les jeunes nourrissons est très large. Pour suspecter un mégacôlon aganglionnaire, on observe généralement un retard dans l'émission du méconium et souvent d'autres symptômes associés tels qu'une distension abdominale, une difficulté à prendre du poids, etc.", "type": "PÉDIATRIE", "options": {"1": "Mettre en place un traitement laxatif.", "2": "Compléter l'allaitement par une formule anti-constipation.", "3": "Initier une stimulation rectale quotidienne.", "4": "Il est considéré comme un rythme intestinal normal.", "5": "Adressé au service de gastro-entérologie pédiatrique pour exclure un mégacôlon aganglionnaire."}, "correct_option": 4, "explanations": {"1": {"exist": false, "char_ranges": [], "word_ranges": [], "text": ""}, "2": {"exist": false, "char_ranges": [], "word_ranges": [], "text": ""}, "3": {"exist": false, "char_ranges": [], "word_ranges": [], "text": ""}, "4": {"exist": true, "char_ranges": [[24, 108]], "word_ranges": [[5, 19]], "text": "La plage normale de production de selles chez les jeunes nourrissons est très large."}, "5": {"exist": true, "char_ranges": [[109, 328]], "word_ranges": [[19, 49]], "text": "Pour suspecter un mégacôlon aganglionnaire, on observe généralement un retard dans l'émission du méconium et souvent d'autres symptômes associés tels qu'une distension abdominale, une difficulté à prendre du poids, etc."}}} +{"id": 567, "year": 2022, "question_id_specific": 150, "full_question": "Une femme de 47 ans a consulté pour un syndrome mictionnel. On lui a diagnostiqué une infection urinaire non compliquée et elle a été traitée à la ciprofloxacine pendant 5 jours. Une semaine plus tard, elle consulte à nouveau en raison d'un mauvais état général, d'arthralgies et de l'apparition d'une éruption cutanée. Les examens de laboratoire montrent un taux de filtration glomérulaire de 45 ml/min (CKD-EPI), contre 100 ml/min six mois plus tôt. Dans le sédiment, des leucocytes sont identifiés, avec un résultat négatif pour les nitrites. Parmi les éléments suivants, quelle est la cause la plus probable de son insuffisance rénale ?", "full_answer": "Un malaise général, des arthralgies, une éruption cutanée, une insuffisance rénale aiguë et un sédiment actif avec une leucocyturie stérile à la suite de la prise d'un médicament sont très évocateurs d'une NIA (option 4 correcte). Parmi les médicaments, les antibiotiques sont la cause la plus fréquente, la ciprofloxacine étant l'une des plus courantes. La néphrotoxicité directe est très rare et se caractérise par une cristallisation dans les tubules rénaux (option 2 incorrecte). La glomérulonéphrite post-infectieuse est plus fréquente chez les enfants et les adultes de plus de 60 ans et se manifeste généralement par une hématurie après des infections à streptocoques (option 1 incorrecte). La pyélonéphrite aiguë se manifeste par une forte fièvre et une douleur dans la fosse rénale (option 3 incorrecte).", "type": "NEPHROLOGIE", "options": {"1": "Glomérulonéphrite post-infectieuse.", "2": "Néphrotoxicité de la ciprofloxacine.", "3": "Pyélonéphrite aiguë.", "4": "Néphrite interstitielle aiguë.", "5": null}, "correct_option": 4, "explanations": {"1": {"exist": true, "char_ranges": [[484, 697]], "word_ranges": [[73, 105]], "text": "La glomérulonéphrite post-infectieuse est plus fréquente chez les enfants et les adultes de plus de 60 ans et se manifeste généralement par une hématurie après des infections à streptocoques (option 1 incorrecte)."}, "2": {"exist": true, "char_ranges": [[355, 483]], "word_ranges": [[54, 73]], "text": "La néphrotoxicité directe est très rare et se caractérise par une cristallisation dans les tubules rénaux (option 2 incorrecte)."}, "3": {"exist": true, "char_ranges": [[698, 813]], "word_ranges": [[105, 124]], "text": "La pyélonéphrite aiguë se manifeste par une forte fièvre et une douleur dans la fosse rénale (option 3 incorrecte)."}, "4": {"exist": true, "char_ranges": [[0, 230]], "word_ranges": [[0, 36]], "text": "Un malaise général, des arthralgies, une éruption cutanée, une insuffisance rénale aiguë et un sédiment actif avec une leucocyturie stérile à la suite de la prise d'un médicament sont très évocateurs d'une NIA (option 4 correcte)."}, "5": {"exist": false, "char_ranges": [], "word_ranges": [], "text": ""}}} +{"id": 177, "year": 2013, "question_id_specific": 93, "full_question": "Une femme de 30 ans, asymptomatique, est trouvée anémique lors d'un examen de routine. L'examen physique révèle un ictère conjonctival et une splénomégalie. La patiente a signalé des antécédents familiaux de lithiase biliaire à un âge précoce. Tous ces éléments suggèrent le diagnostic le plus probable de :", "full_answer": "Vos yeux se sont probablement arrêtés sur la réponse 5, la sphérocytose héréditaire, associée à une lithiase biliaire à un âge précoce. Exactement, c'est la bonne réponse. Dans la déclaration, on ne cesse de donner des indices : jaunisse conjonctivale, splénomégalie et antécédents familiaux de lithiase biliaire à un âge précoce. Il a atteint l'âge de 30 ans sans subir de crise hémolytique, sans souffrir d'aucun symptôme. Il n'y a aucune mention de microcytose ou de macrocytose pour suggérer une thalassémie mineure ou une carence en B12 et/ou en acide folique. Ni de crise hémolytique associée à la consommation de médicaments, d'infections ou à l'ingestion de haricots ou de pois. Ils n'ont donné aucun indice permettant de s'arrêter et de réfléchir à d'autres possibilités.", "type": "HÉMATOLOGIE", "options": {"1": "Déficit en glucose 6-phosphate déshydrogénase.", "2": "Thalassémie mineure.", "3": "Carence en vitamine Bl2 et/ou en acide folique.", "4": "Déficit familial en pyruvate kinase.", "5": "Sphérocytose héréditaire."}, "correct_option": 5, "explanations": {"1": {"exist": false, "char_ranges": [], "word_ranges": [], "text": ""}, "2": {"exist": false, "char_ranges": [], "word_ranges": [], "text": ""}, "3": {"exist": false, "char_ranges": [], "word_ranges": [], "text": ""}, "4": {"exist": false, "char_ranges": [], "word_ranges": [], "text": ""}, "5": {"exist": true, "char_ranges": [[229, 686]], "word_ranges": [[38, 110]], "text": "jaunisse conjonctivale, splénomégalie et antécédents familiaux de lithiase biliaire à un âge précoce. Il a atteint l'âge de 30 ans sans subir de crise hémolytique, sans souffrir d'aucun symptôme. Il n'y a aucune mention de microcytose ou de macrocytose pour suggérer une thalassémie mineure ou une carence en B12 et/ou en acide folique. Ni de crise hémolytique associée à la consommation de médicaments, d'infections ou à l'ingestion de haricots ou de pois."}}} +{"id": 125, "year": 2012, "question_id_specific": 105, "full_question": "Femme de 19 ans, poids 60 kg, présentant une déshydratation aiguë due à une longue exposition au soleil. Tension artérielle en position couchée 100/60 mmHg. Debout 70/50 mmHg avec vertiges. Sodium sérique 155 mmol/L. Quel est le traitement le plus correct, dans les premières 24 heures, compte tenu des données dont nous disposons ?", "full_answer": "Cette question n'est pas facile. La littérature disponible est quelque peu controversée (certaines directives conseillent l'utilisation de sérum glucosé, d'autres l'interdisent absolument...), j'ai donc décidé de m'appuyer sur le traitement recommandé par Harrison, qui est le manuel sur lequel les examinateurs du MIR s'appuient généralement. Cette fille présente une hypernatrémie due à une perte d'eau libre extrarénale. L'objectif est de remplacer l'eau perdue et de réduire la natrémie à un taux ne dépassant pas 12 mmol/jour. Pour calculer la thérapie liquidienne, il faut d'abord calculer le déficit hydrique. Déficit hydrique Eau = (Natrémie-140/140)xEau corporelle totale. Eau corporelle totale = poids en kg x 0,4 pour les femmes/0,5 pour les hommes. Dans ce cas, le déficit hydrique est de : ([[tel:155-140/140|155-140/140]])x60x0,4 = 2,57 litres. Seules deux réponses s'approchent de ce chiffre, mais la réponse 2 semble plus correcte, car c'est la seule qui couvre complètement le déficit hydrique et qui utilise une solution hyposodée.", "type": "NEPHROLOGIE", "options": {"1": "Solution saline hypertonique (3 %), 500 ml + 500 ml de glucose à 5 %.", "2": "Solution saline hyposaline (0,45%), 3000 ml.", "3": "Sérum de glucose 5% 1000 ml.", "4": "Hydratation orale avec 1 litre d'eau.", "5": "Solution saline isotonique (0,9 %), 2000 ml."}, "correct_option": 2, "explanations": {"1": {"exist": false, "char_ranges": [], "word_ranges": [], "text": ""}, "2": {"exist": true, "char_ranges": [[761, 1049]], "word_ranges": [[110, 153]], "text": "Dans ce cas, le déficit hydrique est de : ([[tel:155-140/140|155-140/140]])x60x0,4 = 2,57 litres. Seules deux réponses s'approchent de ce chiffre, mais la réponse 2 semble plus correcte, car c'est la seule qui couvre complètement le déficit hydrique et qui utilise une solution hyposodée."}, "3": {"exist": false, "char_ranges": [], "word_ranges": [], "text": ""}, "4": {"exist": false, "char_ranges": [], "word_ranges": [], "text": ""}, "5": {"exist": true, "char_ranges": [[761, 1049]], "word_ranges": [[110, 153]], "text": "Dans ce cas, le déficit hydrique est de : ([[tel:155-140/140|155-140/140]])x60x0,4 = 2,57 litres. Seules deux réponses s'approchent de ce chiffre, mais la réponse 2 semble plus correcte, car c'est la seule qui couvre complètement le déficit hydrique et qui utilise une solution hyposodée."}}} +{"id": 510, "year": 2021, "question_id_specific": 151, "full_question": "Un homme de 43 ans, sans antécédents particuliers, a consulté pour une jaunisse de la peau et des muqueuses, une cholurie et une acholie qui duraient depuis dix jours. Il ne signale pas de douleurs abdominales, de perte de poids ou d'autres symptômes, à l'exception d'un prurit généralisé. Elle nie avoir consommé de l'alcool. Il ne prend pas de médicaments, à l'exception d'ibuprofène occasionnel pour des douleurs musculaires après une activité sportive. Les analyses sanguines montrent une augmentation de la bilirubine au détriment de la bilirubine directe. Une échographie urgente montre un foie normal, une cholestérose vésiculaire et l'absence de dilatation des voies biliaires, sans autre altération intéressante. Quel est le diagnostic le plus probable ?", "full_answer": "Nous pensons que cette question peut être remise en question. L'hépatite à l'ibuprofène est un effet indésirable rare dont la prévalence est d'environ 3,7/100 000 utilisateurs et 1,1/100 000 prescriptions. Il semble qu'elle puisse être liée à la dose et à la durée du traitement, mais il y a peu de preuves à ce sujet. Parmi les AINS présentant le risque le plus élevé, on trouve le diclofénac et le sulindac. En termes de fréquence, la prévalence du cholangiocarcinome est de 2/100 000 patients, une fréquence très comparable. L'absence de dilatation des voies biliaires dans les cholangiocarcinomes peut être observée dans les cas de cholangiocarcinomes intrahépatiques ou dans ceux dont l'histologie est mixte avec un hépatocarcinome. Il est vrai que l'évolution aiguë et l'absence de syndrome constitutionnel peuvent nous faire renoncer à cette option et faire pencher la balance vers l'hépatite à l'ibuprofène. En ce qui concerne les autres options : la cholangite aiguë est caractérisée par la triade ictère, douleurs abdominales et fièvre ; le syndrome de Gilbert est une augmentation de la bilirubine aux dépens de l'indirect.", "type": "DIGESTIF", "options": {"1": "Cholangite aiguë due à une boue biliaire.", "2": "Hépatite toxique à l'ibuprofène.", "3": "Cholangiocarcinome.", "4": "Syndrome de Gilbert.", "5": null}, "correct_option": 2, "explanations": {"1": {"exist": true, "char_ranges": [[956, 1047]], "word_ranges": [[151, 165]], "text": "la cholangite aiguë est caractérisée par la triade ictère, douleurs abdominales et fièvre ;"}, "2": {"exist": true, "char_ranges": [[738, 915]], "word_ranges": [[116, 143]], "text": "Il est vrai que l'évolution aiguë et l'absence de syndrome constitutionnel peuvent nous faire renoncer à cette option et faire pencher la balance vers l'hépatite à l'ibuprofène."}, "3": {"exist": true, "char_ranges": [[738, 915]], "word_ranges": [[116, 143]], "text": "Il est vrai que l'évolution aiguë et l'absence de syndrome constitutionnel peuvent nous faire renoncer à cette option et faire pencher la balance vers l'hépatite à l'ibuprofène."}, "4": {"exist": true, "char_ranges": [[1048, 1134]], "word_ranges": [[165, 179]], "text": "le syndrome de Gilbert est une augmentation de la bilirubine aux dépens de l'indirect."}, "5": {"exist": false, "char_ranges": [], "word_ranges": [], "text": ""}}} +{"id": 268, "year": 2014, "question_id_specific": 140, "full_question": "Patient de 50 ans présentant un épanchement pleural avec les caractéristiques suivantes : aspect pailleux, pH 7,3, rapport protéines pleurales/sérum 0,8, rapport LDH pleural/sérum 0,9, Gram et Ziehl négatifs, lipides totaux, cholestérol et triglycérides normaux, cellules mésothéliales <5%, lymphocytose intense sans atypie, ADA 64 U/l. Quel diagnostic suggérez-vous ?", "full_answer": "Il s'agit d'un exsudat lymphocytaire dont les diagnostics principaux sont la tumeur et la pleurite tuberculeuse. Ils ne donnent pas la valeur du glucose qui devrait être abaissée, mais la cytologie montre l'absence de cellules malignes, on en reste donc à la pleurite tuberculeuse. D'autres diagnostics possibles, tels que le lymphome, ne sont pas mentionnés dans les réponses.", "type": "PNEUMOLOGIE", "options": {"1": "Empyème pleural.", "2": "Épanchement pleural dû à une insuffisance cardiaque (transsudat).", "3": "Mésothéliome pleural.", "4": "Épanchement pleural tuberculeux.", "5": "Accident vasculaire cérébral secondaire à un infarctus pulmonaire."}, "correct_option": 4, "explanations": {"1": {"exist": false, "char_ranges": [], "word_ranges": [], "text": ""}, "2": {"exist": false, "char_ranges": [], "word_ranges": [], "text": ""}, "3": {"exist": false, "char_ranges": [], "word_ranges": [], "text": ""}, "4": {"exist": true, "char_ranges": [[0, 281]], "word_ranges": [[0, 44]], "text": "Il s'agit d'un exsudat lymphocytaire dont les diagnostics principaux sont la tumeur et la pleurite tuberculeuse. Ils ne donnent pas la valeur du glucose qui devrait être abaissée, mais la cytologie montre l'absence de cellules malignes, on en reste donc à la pleurite tuberculeuse."}, "5": {"exist": false, "char_ranges": [], "word_ranges": [], "text": ""}}} +{"id": 465, "year": 2020, "question_id_specific": 116, "full_question": "Chez un sujet de plus de 65 ans, un test à la tuberculine a montré une induration de 3 mm. Lors d'un second test, effectué 10 jours plus tard, l'induration est de 13 mm. Cochez la bonne réponse :", "full_answer": "Voyons voir, si ce sujet ne faisait partie d'aucun groupe à risque, le positif est dans les 15 mm, donc les deux tests sont négatifs, et comme il n'y a qu'une seule réponse pour les négatifs, c'est celle-là. Mais il pourrait s'agir dans les deux cas de faux négatifs.", "type": "BIOSTATISTIQUE", "options": {"1": "La première réaction est un faux positif.", "2": "La deuxième réaction est vraiment positive.", "3": "La première réaction est vraiment négative.", "4": "La deuxième réaction est un faux positif.", "5": null}, "correct_option": 3, "explanations": {"1": {"exist": false, "char_ranges": [], "word_ranges": [], "text": ""}, "2": {"exist": false, "char_ranges": [], "word_ranges": [], "text": ""}, "3": {"exist": true, "char_ranges": [[0, 207]], "word_ranges": [[0, 38]], "text": "Voyons voir, si ce sujet ne faisait partie d'aucun groupe à risque, le positif est dans les 15 mm, donc les deux tests sont négatifs, et comme il n'y a qu'une seule réponse pour les négatifs, c'est celle-là."}, "4": {"exist": false, "char_ranges": [], "word_ranges": [], "text": ""}, "5": {"exist": false, "char_ranges": [], "word_ranges": [], "text": ""}}} +{"id": 340, "year": 2016, "question_id_specific": 33, "full_question": "Une femme de 67 ans a été diagnostiquée avec un carcinome canalaire infiltrant du sein sans antécédents familiaux de néoplasie. Quelles sont les études complémentaires à réaliser sur la tumeur pour ses implications cliniques et thérapeutiques ?", "full_answer": "La réponse est 2, étude des récepteurs hormonaux et HER2. Cela s'explique par le fait qu'il n'y a pas lieu d'étudier les parents au premier degré, car il ne semble pas s'agir d'un cancer du sein héréditaire, puisqu'il n'y a pas d'antécédents familiaux de néoplasie. D'autre part, il n'y a pas d'indication pour l'étude de BRCA 1-2.", "type": "GYNÉCOLOGIE ET OBSTÉTRIQUE", "options": {"1": "Étude phénotypique complète par cytométrie de flux.", "2": "Étude des récepteurs hormonaux et de HER2.", "3": "Étude des récepteurs hormonaux, de l'e-cadhérine et étude des parents au premier degré.", "4": "Dépistage BRCA 1-2 et dépistage des parents au premier degré.", "5": null}, "correct_option": 2, "explanations": {"1": {"exist": false, "char_ranges": [], "word_ranges": [], "text": ""}, "2": {"exist": false, "char_ranges": [], "word_ranges": [], "text": ""}, "3": {"exist": true, "char_ranges": [[86, 265]], "word_ranges": [[15, 45]], "text": "qu'il n'y a pas lieu d'étudier les parents au premier degré, car il ne semble pas s'agir d'un cancer du sein héréditaire, puisqu'il n'y a pas d'antécédents familiaux de néoplasie."}, "4": {"exist": true, "char_ranges": [[86, 265]], "word_ranges": [[15, 45]], "text": "qu'il n'y a pas lieu d'étudier les parents au premier degré, car il ne semble pas s'agir d'un cancer du sein héréditaire, puisqu'il n'y a pas d'antécédents familiaux de néoplasie."}, "5": {"exist": false, "char_ranges": [], "word_ranges": [], "text": ""}}} +{"id": 561, "year": 2022, "question_id_specific": 180, "full_question": "Une femme de 26 ans, chez qui on a diagnostiqué un lupus érythémateux disséminé et qui est traitée à l'hydroxychloroquine, consulte pour une sensation de faiblesse généralisée qui s'est progressivement développée au cours des 15 derniers jours. L'examen physique a révélé une pâleur cutanée et l'analyse sanguine a révélé une Hb de 7,4 g/dL, une Hct de 31 % et une MCV de 108. Lequel des tests suivants serait le plus utile pour décider de la marche à suivre ?", "full_answer": "Il nous parle d'un patient diagnostiqué avec un lupus et une anémie macrocytaire. En fin de compte, toutes les données sont généralement fournies pour une raison. L'apparition des symptômes semble relativement rapide et elle est également associée à une maladie auto-immune, de sorte qu'il semble que l'on veuille l'associer à une anémie hémolytique auto-immune. Le faible taux d'haptoglobine nous indiquerait qu'il s'agit d'une anémie hémolytique, le test de Coombs nous indiquerait qu'il s'agit d'une anémie auto-immune. Vitamine B12, le MCV serait probablement supérieur à 108 et l'apparition du tableau clinique serait plus lente. L'ANA ne nous donnerait rien en ce qui concerne l'anémie, puisqu'elle a déjà été diagnostiquée. La réponse 2 est donc correcte. Pour mémoire, les anémies hémolytiques auto-immunes sont généralement légèrement macrocytaires en raison d'une réticulocytose. N'oubliez pas de demander des réticulocytes.", "type": "HÉMATOLOGIE", "options": {"1": "Haptoglobine.", "2": "Test de Coombs.", "3": "Vitamine B12.", "4": "Anticorps antinucléaires.", "5": null}, "correct_option": 2, "explanations": {"1": {"exist": false, "char_ranges": [], "word_ranges": [], "text": ""}, "2": {"exist": true, "char_ranges": [[163, 522]], "word_ranges": [[26, 76]], "text": "L'apparition des symptômes semble relativement rapide et elle est également associée à une maladie auto-immune, de sorte qu'il semble que l'on veuille l'associer à une anémie hémolytique auto-immune. Le faible taux d'haptoglobine nous indiquerait qu'il s'agit d'une anémie hémolytique, le test de Coombs nous indiquerait qu'il s'agit d'une anémie auto-immune."}, "3": {"exist": true, "char_ranges": [[523, 634]], "word_ranges": [[76, 93]], "text": "Vitamine B12, le MCV serait probablement supérieur à 108 et l'apparition du tableau clinique serait plus lente."}, "4": {"exist": true, "char_ranges": [[635, 731]], "word_ranges": [[93, 108]], "text": "L'ANA ne nous donnerait rien en ce qui concerne l'anémie, puisqu'elle a déjà été diagnostiquée."}, "5": {"exist": false, "char_ranges": [], "word_ranges": [], "text": ""}}} +{"id": 451, "year": 2018, "question_id_specific": 175, "full_question": "Un homme de 36 ans n'ayant pas subi d'examen toxicologique se présente aux urgences pour une douleur intense d'apparition soudaine à la face postérieure du membre inférieur droit, qui descend jusqu'au pied. À l'examen, il a présenté Lasège droit positif à 10º, diminution de la force de flexion plantaire du pied droit, hypoesthésie sur le bord externe du pied droit et absence de réflexe ischio-jambier droit. La radiographie simple de la colonne lombaire ne montre pas d'altérations significatives. Lequel des diagnostics suivants est le plus probable ?", "full_answer": "Sciatique aiguë probablement due à une extrusion discale. La distribution sensorielle, la parésie pour la flexion plantaire (pointe du pied) et l'absence de réflexe d'Achille sont typiques d'une atteinte de la racine S1, qui est typiquement affectée par une hernie discale L5/S1 (postéro-latérale). Une hernie L1/L2 (postéro-latérale) affecterait la racine L2, une hernie L4/L5 la racine L5 et la cauda equina impliquerait plus de symptomatologie qu'une simple atteinte de S1.", "type": "NEUROSURGÉRIE", "options": {"1": "Hernie discale L1/L2.", "2": "Syndrome de la cauda équine.", "3": "Hernie discale L4/L5.", "4": "Hernie discale L5/S1.", "5": null}, "correct_option": 4, "explanations": {"1": {"exist": true, "char_ranges": [[299, 360]], "word_ranges": [[43, 51]], "text": "Une hernie L1/L2 (postéro-latérale) affecterait la racine L2,"}, "2": {"exist": true, "char_ranges": [[394, 476]], "word_ranges": [[58, 70]], "text": "la cauda equina impliquerait plus de symptomatologie qu'une simple atteinte de S1."}, "3": {"exist": true, "char_ranges": [[361, 390]], "word_ranges": [[51, 57]], "text": "une hernie L4/L5 la racine L5"}, "4": {"exist": true, "char_ranges": [[0, 298]], "word_ranges": [[0, 43]], "text": "Sciatique aiguë probablement due à une extrusion discale. La distribution sensorielle, la parésie pour la flexion plantaire (pointe du pied) et l'absence de réflexe d'Achille sont typiques d'une atteinte de la racine S1, qui est typiquement affectée par une hernie discale L5/S1 (postéro-latérale)."}, "5": {"exist": false, "char_ranges": [], "word_ranges": [], "text": ""}}} +{"id": 399, "year": 2016, "question_id_specific": 141, "full_question": "Garçon de 20 ans, qui a consulté pour des douleurs lombo-sacrées avec un rythme inflammatoire de 4 mois d'évolution. Il souffre également de thalgies bilatérales et d'une raideur matinale d'une heure. Au cours des deux derniers mois, il a développé une diarrhée avec une perte de poids de 4 kg. Quelle est l'approche diagnostique la plus correcte ?", "full_answer": "Le patient présente les caractéristiques cliniques d'une spondylarthropathie, qui est souvent associée à une maladie inflammatoire de l'intestin, comme le suggère ce cas.", "type": "TRAUMATOLOGIE ET ORTHOPÉDIE", "options": {"1": "Compte tenu de l'âge du patient, il est très probable qu'il souffre de lombalgies non spécifiques et de tendinites aux pieds. Si la diarrhée persiste, un bilan digestif doit être effectué.", "2": "J'effectuerais une étude digestive pour exclure une pathologie tumorale. La douleur lombaire peut être due à une pathologie viscérale.", "3": "Le tableau clinique est très évocateur d'une spondylarthrite. Une maladie inflammatoire de l'intestin doit être exclue.", "4": "Je demanderais une IRM lombaire pour exclure une hernie discale et, si la diarrhée persiste, une étude digestive.", "5": null}, "correct_option": 3, "explanations": {"1": {"exist": false, "char_ranges": [], "word_ranges": [], "text": ""}, "2": {"exist": false, "char_ranges": [], "word_ranges": [], "text": ""}, "3": {"exist": true, "char_ranges": [[0, 170]], "word_ranges": [[0, 23]], "text": "Le patient présente les caractéristiques cliniques d'une spondylarthropathie, qui est souvent associée à une maladie inflammatoire de l'intestin, comme le suggère ce cas."}, "4": {"exist": false, "char_ranges": [], "word_ranges": [], "text": ""}, "5": {"exist": false, "char_ranges": [], "word_ranges": [], "text": ""}}} +{"id": 12, "year": 2011, "question_id_specific": 92, "full_question": "Un homme de 30 ans est tombé d'une hauteur de 2 mètres sur ses pieds. À son arrivée au service des urgences sur un brancard, il est conscient et orienté, avec un cathétérisme vésical et des urines claires. Il présente un gonflement du talon gauche et de légers picotements dans la partie antérieure de la cuisse gauche et du scrotum, ainsi qu'une dysesthésie des deux pieds. Le patient dit au revoir au brancardier par une poignée de main et une tape simultanée sur l'épaule. La rotation des membres inférieurs sur le brancard par les chevilles est indolore et la compression du bassin est asymptomatique. Suivant :", "full_answer": "Je pense que la bonne réponse est 3. Le patient, en raison du mécanisme de la blessure (chute du pied), de la fracture du calcanéum et des symptômes neurologiques, suggère une lésion de la moelle épinière basse due à une fracture de la colonne vertébrale lombaire. La blessure n'est pas haute car il bouge les bras. Il ne semble pas non plus s'agir d'une lésion pelvienne en raison de l'absence de douleur à la mobilisation ou à la palpation. En cas de suspicion de lésion pelvienne lors de l'évaluation primaire de toutes les victimes de polytraumatismes (ATLS), un examen de la colonne vertébrale doit être effectué en effectuant une rotation en bloc de la colonne vertébrale afin d'éviter de provoquer d'autres lésions. La palpation abdominale fait également partie de l'évaluation initiale. Une échographie abdominale peut être indiquée chez les patients hémodynamiquement instables si des lésions abdominales sont suspectées, mais dans tous les cas, elle est effectuée après l'évaluation initiale.", "type": "CHIRURGIE", "options": {"1": "Vous serez assis sur le brancard afin que la colonne cervicale puisse être explorée.", "2": "Adressez-vous à la radiologie pour l'étude des extrémités inférieures (pieds, fémurs, bassin).", "3": "Palpez l'abdomen et roulez en bloc pour palper la colonne thoraco-lombaire.", "4": "Vous serez placé dans un collier cervical et envoyé en radiologie pour une imagerie des membres inférieurs et de la colonne cervicale.", "5": "Nous demanderons une échographie abdominale."}, "correct_option": 3, "explanations": {"1": {"exist": false, "char_ranges": [], "word_ranges": [], "text": ""}, "2": {"exist": false, "char_ranges": [], "word_ranges": [], "text": ""}, "3": {"exist": true, "char_ranges": [[0, 264]], "word_ranges": [[0, 46]], "text": "Je pense que la bonne réponse est 3. Le patient, en raison du mécanisme de la blessure (chute du pied), de la fracture du calcanéum et des symptômes neurologiques, suggère une lésion de la moelle épinière basse due à une fracture de la colonne vertébrale lombaire."}, "4": {"exist": false, "char_ranges": [], "word_ranges": [], "text": ""}, "5": {"exist": false, "char_ranges": [], "word_ranges": [], "text": ""}}} +{"id": 605, "year": 2022, "question_id_specific": 113, "full_question": "Un homme de 35 ans, facteur, sans antécédents, s'est présenté aux urgences pour des douleurs cervicales aiguës depuis 24 heures, sans antécédents traumatiques, irradiant dans le bras gauche jusqu'à la main et accompagnées d'une paresthésie du bord radial de l'avant-bras. Il n'y a pas de perte objective de force, le patient conserve la mobilité du cou bien qu'il soit douloureux et il y a une contracture des muscles paravertébraux. La première approche est la suivante :", "full_answer": "Ce cas décrit une cervicobrachialgie ne présentant aucun signe d'alerte à l'heure actuelle, le traitement le plus indiqué à ce stade serait donc conservateur, basé sur les AINS, la chaleur locale et le repos. Il ne s'agit pas d'une pathologie urgente, la réponse 2 n'est donc pas valable. Une IRM et un électromyogramme sont des options valables pour l'étude étiologique de l'affection, mais il convient de noter qu'il faut d'abord traiter correctement la douleur, de sorte que, étant donné que nous sommes interrogés sur la première approche, l'option 1 serait plus valable.", "type": "TRAUMATOLOGIE", "options": {"1": "Traitement conservateur à base d'anti-inflammatoires non stéroïdiens, de chaleur locale et de repos relatif.", "2": "Appel urgent au neurochirurgien pour une évaluation chirurgicale.", "3": "Demande préférentielle pour l'IRM et l'électromyogramme.", "4": "Orientation préférentielle vers les cliniques ambulatoires spécialisées dans les traumatismes.", "5": null}, "correct_option": 1, "explanations": {"1": {"exist": true, "char_ranges": [[0, 208]], "word_ranges": [[0, 34]], "text": "Ce cas décrit une cervicobrachialgie ne présentant aucun signe d'alerte à l'heure actuelle, le traitement le plus indiqué à ce stade serait donc conservateur, basé sur les AINS, la chaleur locale et le repos."}, "2": {"exist": true, "char_ranges": [[209, 288]], "word_ranges": [[34, 48]], "text": "Il ne s'agit pas d'une pathologie urgente, la réponse 2 n'est donc pas valable."}, "3": {"exist": true, "char_ranges": [[289, 575]], "word_ranges": [[48, 92]], "text": "Une IRM et un électromyogramme sont des options valables pour l'étude étiologique de l'affection, mais il convient de noter qu'il faut d'abord traiter correctement la douleur, de sorte que, étant donné que nous sommes interrogés sur la première approche, l'option 1 serait plus valable."}, "4": {"exist": false, "char_ranges": [], "word_ranges": [], "text": ""}, "5": {"exist": false, "char_ranges": [], "word_ranges": [], "text": ""}}} +{"id": 443, "year": 2018, "question_id_specific": 102, "full_question": "Femme de 64 ans, sans comorbidités, opérée d'un cancer du sein droit par tumorectomie élargie et biopsie sélective du ganglion lymphatique sentinelle, avec le résultat anatomopathologique suivant : carcinome canalaire infiltrant peu différencié (grade 3), taille 2,2 cm (pT2), récepteurs aux œstrogènes et à la progestérone positifs, Ki67 25 %, HER2 négatif. Il y a une atteinte macroscopique du ganglion lymphatique sentinelle, bien qu'il n'y ait pas d'atteinte du reste des ganglions axillaires isolés (pN1). Quel traitement systémique adjuvant considérez-vous comme le plus approprié pour cette patiente ?", "full_answer": "Il s'agit d'une tumeur luminale B. Selon le SEGO, cette patiente est candidate à un traitement par chimiothérapie en raison de son grade histologique élevé et de la forte prolifération de la tumeur.", "type": "GYNÉCOLOGIE ET OBSTÉTRIQUE", "options": {"1": "Thérapie hormonale adjuvante comprenant des inhibiteurs de l'aromatase.", "2": "Chimiothérapie adjuvante, de préférence avec des anthracyclines et des taxanes, suivie d'une thérapie hormonale.", "3": "Chimiothérapie adjuvante en association avec l'anticorps monoclonal trastuzumab, suivie d'une hormonothérapie.", "4": "Chimiothérapie adjuvante, de préférence avec des anthracyclines et des taxanes.", "5": null}, "correct_option": 2, "explanations": {"1": {"exist": false, "char_ranges": [], "word_ranges": [], "text": ""}, "2": {"exist": true, "char_ranges": [[0, 198]], "word_ranges": [[0, 33]], "text": "Il s'agit d'une tumeur luminale B. Selon le SEGO, cette patiente est candidate à un traitement par chimiothérapie en raison de son grade histologique élevé et de la forte prolifération de la tumeur."}, "3": {"exist": false, "char_ranges": [], "word_ranges": [], "text": ""}, "4": {"exist": false, "char_ranges": [], "word_ranges": [], "text": ""}, "5": {"exist": false, "char_ranges": [], "word_ranges": [], "text": ""}}} +{"id": 197, "year": 2013, "question_id_specific": 71, "full_question": "Garçon de 10 ans présentant de brefs épisodes de distraction (< 1 minute) au cours desquels il ne répond pas aux appels et cligne des yeux. Un EEG montre des décharges de pointes-ondes à 3 cycles par seconde. Le traitement électif de première intention serait l'utilisation de :", "full_answer": "Les absences se présentent comme de brefs épisodes de perte de conscience brutale sans altération du contrôle postural ; elles durent typiquement quelques secondes et peuvent être répétées plusieurs fois par jour, sont généralement accompagnées de petits signes moteurs bilatéraux (clignement des yeux, mastication) et la conscience est récupérée tout aussi brutalement, sans confusion ultérieure ni souvenir de l'épisode (MIR 03-04, 248 ; MIR 97-98, 51). L'âge d'apparition se situe généralement entre 4 ans et le début de l'adolescence, cette maladie étant la cause la plus fréquente de crises dans cette tranche d'âge. Les résultats de l'EEG sont typiquement des décharges généralisées et symétriques de pointes-ondes à 3 Hz (MIR 99-00F, 67) coïncidant avec les crises, bien qu'il y ait plus de périodes d'activité anormale dans l'EEG interictal qu'il n'y en a de cliniquement visibles. Les médicaments les plus appropriés pour le traitement des absences sont le Valproate et l'Ethosuximide (Manuel CTO).", "type": "NEUROLOGIE", "options": {"1": "Valproate.", "2": "Carbamazépine.", "3": "Phénytoïne.", "4": "Gabapentin.", "5": "Clonazepam."}, "correct_option": 1, "explanations": {"1": {"exist": true, "char_ranges": [[890, 1007]], "word_ranges": [[135, 152]], "text": "Les médicaments les plus appropriés pour le traitement des absences sont le Valproate et l'Ethosuximide (Manuel CTO)."}, "2": {"exist": false, "char_ranges": [], "word_ranges": [], "text": ""}, "3": {"exist": false, "char_ranges": [], "word_ranges": [], "text": ""}, "4": {"exist": false, "char_ranges": [], "word_ranges": [], "text": ""}, "5": {"exist": false, "char_ranges": [], "word_ranges": [], "text": ""}}} +{"id": 478, "year": 2020, "question_id_specific": 134, "full_question": "Un homme de 35 ans a été amené au service des urgences après un accident de la route. À l'admission, il présente un score de Glasgow de 15, une tension artérielle de 140/90 mmHg, une fréquence respiratoire de 35 rpm et une fréquence cardiaque de 110 bpm, avec une saturation en oxygène de base de 91 %. A l'examen, il présente une hypophonie thoracique droite et un tympanisme à la percussion. Quel est le diagnostic présumé ? :", "full_answer": "Question facile et classique que vous pouvez également appliquer à la pratique clinique quotidienne. Hypophonie dans un hémithorax + tympanisme = pneumothorax. S'il y a une matité à la percussion : hémothorax. Dans les deux cas, en cas d'hypophonie d'un hémithorax chez un patient gravement traumatisé et instable : drainage thoracique (si possible de gros calibre s'il est placé en milieu hospitalier, car si la lésion prédominante est le pneumothorax, il peut y avoir une composante hémothorax qui, si un drain fin est placé, pourrait obstruer le drainage).", "type": "SOINS INTENSIFS", "options": {"1": "Tamponnade cardiaque traumatique.", "2": "Pneumothorax sous tension à droite.", "3": "Hémothorax massif droit.", "4": "Contusion pulmonaire.", "5": null}, "correct_option": 2, "explanations": {"1": {"exist": false, "char_ranges": [], "word_ranges": [], "text": ""}, "2": {"exist": true, "char_ranges": [[101, 159]], "word_ranges": [[14, 22]], "text": "Hypophonie dans un hémithorax + tympanisme = pneumothorax."}, "3": {"exist": true, "char_ranges": [[160, 209]], "word_ranges": [[22, 32]], "text": "S'il y a une matité à la percussion : hémothorax."}, "4": {"exist": false, "char_ranges": [], "word_ranges": [], "text": ""}, "5": {"exist": false, "char_ranges": [], "word_ranges": [], "text": ""}}} +{"id": 97, "year": 2012, "question_id_specific": 171, "full_question": "Le patient présente depuis 2 semaines une éruption prurigineuse constituée de multiples papules polygonales érythémato-violacées à surface aplatie, localisées principalement sur la face antérieure des poignets, la zone prétibiale et la zone lombaire. Il présente également des lésions réticulées blanchâtres sur la muqueuse jugale. Quel est le diagnostic le plus probable ?", "full_answer": "Ils décrivent sans aucun doute les lésions typiques (sur la peau et la muqueuse buccale) d'un lichen plan (5).", "type": "DERMATOLOGIE, VÉNÉRÉOLOGIE ET CHIRURGIE PLASTIQUE", "options": {"1": "Pityriasis versicolor.", "2": "Mycosis fungoides.", "3": "Psoriasis en gouttes.", "4": "Pityriasis rosea de Gibert.", "5": "Lichen plan."}, "correct_option": 5, "explanations": {"1": {"exist": false, "char_ranges": [], "word_ranges": [], "text": ""}, "2": {"exist": false, "char_ranges": [], "word_ranges": [], "text": ""}, "3": {"exist": false, "char_ranges": [], "word_ranges": [], "text": ""}, "4": {"exist": false, "char_ranges": [], "word_ranges": [], "text": ""}, "5": {"exist": true, "char_ranges": [[0, 105]], "word_ranges": [[0, 18]], "text": "Ils décrivent sans aucun doute les lésions typiques (sur la peau et la muqueuse buccale) d'un lichen plan"}}} +{"id": 426, "year": 2018, "question_id_specific": 94, "full_question": "Un patient de 73 ans... présente une dysphagie neurologique complète, secondaire à un accident vasculaire cérébral cardioembolique dans le territoire de l'artère cérébrale moyenne gauche. aura besoin d'un soutien nutritionnel à long terme. Parmi les modalités de traitement suivantes, laquelle vous semble la plus appropriée dans ce cas ?", "full_answer": "La nutrition entérale est une technique de soutien nutritionnel qui consiste à administrer des nutriments directement dans le tractus gastro-intestinal par l'intermédiaire d'une sonde. Le choix de la voie d'administration de la formule dépend de l'état nutritionnel, de l'âge du patient et de la maladie sous-jacente. Si la nutrition entérale est considérée comme étant de courte durée, la voie de choix est la sonde nasogastrique ou nasoduodénale-nasojéjunale... Si une nutrition prolongée est prévue, la voie de choix est la gastrostomie (option 3), la jéjunostomie étant réservée aux cas où la voie gastrique ne peut pas être utilisée.", "type": "ENDOCRINOLOGIE", "options": {"1": "Nutrition entérale par sonde nasogastrique.", "2": "Nutrition entérale par sonde nasoduodénale.", "3": "Nutrition entérale par gastrostomie.", "4": "Nutrition entérale par jéjunostomie.", "5": null}, "correct_option": 3, "explanations": {"1": {"exist": true, "char_ranges": [[318, 463]], "word_ranges": [[46, 67]], "text": "Si la nutrition entérale est considérée comme étant de courte durée, la voie de choix est la sonde nasogastrique ou nasoduodénale-nasojéjunale..."}, "2": {"exist": true, "char_ranges": [[318, 463]], "word_ranges": [[46, 67]], "text": "Si la nutrition entérale est considérée comme étant de courte durée, la voie de choix est la sonde nasogastrique ou nasoduodénale-nasojéjunale..."}, "3": {"exist": true, "char_ranges": [[464, 551]], "word_ranges": [[67, 82]], "text": "Si une nutrition prolongée est prévue, la voie de choix est la gastrostomie (option 3),"}, "4": {"exist": true, "char_ranges": [[552, 638]], "word_ranges": [[82, 97]], "text": "la jéjunostomie étant réservée aux cas où la voie gastrique ne peut pas être utilisée."}, "5": {"exist": false, "char_ranges": [], "word_ranges": [], "text": ""}}} +{"id": 262, "year": 2014, "question_id_specific": 58, "full_question": "Une femme de 70 ans, ayant des antécédents d'hypertension et d'insuffisance cardiaque modérée, se présente à la clinique avec une toux sèche persistante qui commence par une sensation de grattement dans la gorge. Quel est le médicament suspecté d'être à l'origine des symptômes et de l'altération des résultats de laboratoire de la patiente ?", "full_answer": "Il s'agit d'effets indésirables typiques des inhibiteurs de l'ECA ; dans le cas de la toux, elle est généralement sèche et irritative et survient chez jusqu'à 3 % des patients. Une hyperkaliémie peut également survenir chez 1 % des patients.", "type": "PHARMACOLOGIE", "options": {"1": "Hydrochlorothiazide.", "2": "Bisoprolol.", "3": "Furosémide.", "4": "Enalapril.", "5": "Hydralazine."}, "correct_option": 4, "explanations": {"1": {"exist": false, "char_ranges": [], "word_ranges": [], "text": ""}, "2": {"exist": false, "char_ranges": [], "word_ranges": [], "text": ""}, "3": {"exist": false, "char_ranges": [], "word_ranges": [], "text": ""}, "4": {"exist": true, "char_ranges": [[0, 241]], "word_ranges": [[0, 40]], "text": "Il s'agit d'effets indésirables typiques des inhibiteurs de l'ECA ; dans le cas de la toux, elle est généralement sèche et irritative et survient chez jusqu'à 3 % des patients. Une hyperkaliémie peut également survenir chez 1 % des patients."}, "5": {"exist": false, "char_ranges": [], "word_ranges": [], "text": ""}}} +{"id": 346, "year": 2016, "question_id_specific": 159, "full_question": "Une femme de 69 ans se présente à la clinique en se plaignant de saignements génitaux depuis plusieurs mois. Elle refuse le traitement hormonal substitutif et l'anticoagulation. Elle a fourni une cytologie cervicovaginale normale. L'examen physique général et génital n'a rien révélé d'intéressant. IMC de 38 kg/m2. Indiquez l'approche la plus appropriée :", "full_answer": "La bonne réponse est 2. Il s'agit d'un cas de métrorragie post-ménopausique, dont la cause la plus fréquente est l'atrophie génitale en premier lieu et le cancer de l'endomètre en second lieu. Les données fournies sur l'obésité devraient faire suspecter un cancer de l'endomètre, puisque les graisses périphériques sont transformées en œstrogènes, et que ces œstrogènes provoquent la prolifération de l'endomètre. D'autre part, elle présente une cytologie normale, ce qui exclut l'option 3. L'évaluation hormonale n'est pas appropriée, car nous comprenons que la patiente est ménopausée. Nous ne devrions pas prescrire de traitement sans avoir d'abord écarté les causes organiques.", "type": "GYNÉCOLOGIE ET OBSTÉTRIQUE", "options": {"1": "Prescrire de la progestérone cyclique.", "2": "Biopsie de l'endomètre.", "3": "Biopsies cervicales aléatoires.", "4": "Évaluation hormonale avec FSH, LH et œstradiol.", "5": null}, "correct_option": 2, "explanations": {"1": {"exist": true, "char_ranges": [[588, 681]], "word_ranges": [[86, 100]], "text": "Nous ne devrions pas prescrire de traitement sans avoir d'abord écarté les causes organiques."}, "2": {"exist": true, "char_ranges": [[24, 192]], "word_ranges": [[5, 32]], "text": "Il s'agit d'un cas de métrorragie post-ménopausique, dont la cause la plus fréquente est l'atrophie génitale en premier lieu et le cancer de l'endomètre en second lieu."}, "3": {"exist": true, "char_ranges": [[428, 490]], "word_ranges": [[63, 73]], "text": "elle présente une cytologie normale, ce qui exclut l'option 3."}, "4": {"exist": true, "char_ranges": [[491, 587]], "word_ranges": [[73, 86]], "text": "L'évaluation hormonale n'est pas appropriée, car nous comprenons que la patiente est ménopausée."}, "5": {"exist": false, "char_ranges": [], "word_ranges": [], "text": ""}}} +{"id": 170, "year": 2013, "question_id_specific": 89, "full_question": "Un patient de 62 ans avec une bioprothèse péricardique en position mitrale depuis deux ans. Il a été admis en urgence pour un syndrome coronarien aigu. Deux jours plus tard, après une évolution favorable, il a soudainement développé une insuffisance cardiaque avec œdème pulmonaire aigu. Sélectionnez l'option diagnostique dans ce cas parmi les possibilités suivantes :", "full_answer": "La valve est biologique et les feuillets ne sont généralement pas cousus aux muscles papillaires, de sorte que nous ne nous soucions pas de ce qui leur arrive. Un anévrisme ventriculaire déprime la FEVG, mais n'entraîne pas soudainement une MAP. En revanche, une TAV augmente fortement le flux vers le lit pulmonaire, provoquant un œdème pulmonaire, et constitue également l'une des complications les plus courantes de l'IAM. La TSA ischémique... Je ne nie pas qu'elle existe, mais elle ne figure même pas dans les lignes directrices. Et l'endocardite ne nécessiterait pas autant de sacoches.", "type": "CARDIOLOGIE ET CHIRURGIE CARDIOVASCULAIRE", "options": {"1": "Régurgitation mitrale aiguë post-infarctus.", "2": "Anévrisme du ventricule gauche.", "3": "Déficience septale ventriculaire aiguë.", "4": "Défaut septal auriculaire post-infarctus.", "5": "Dysfonctionnement de la bioprothèse dû à une probable endocardite aiguë."}, "correct_option": 3, "explanations": {"1": {"exist": true, "char_ranges": [[426, 534]], "word_ranges": [[67, 86]], "text": "La TSA ischémique... Je ne nie pas qu'elle existe, mais elle ne figure même pas dans les lignes directrices."}, "2": {"exist": true, "char_ranges": [[160, 245]], "word_ranges": [[28, 40]], "text": "Un anévrisme ventriculaire déprime la FEVG, mais n'entraîne pas soudainement une MAP."}, "3": {"exist": true, "char_ranges": [[259, 425]], "word_ranges": [[42, 67]], "text": "une TAV augmente fortement le flux vers le lit pulmonaire, provoquant un œdème pulmonaire, et constitue également l'une des complications les plus courantes de l'IAM."}, "4": {"exist": true, "char_ranges": [[426, 534]], "word_ranges": [[67, 86]], "text": "La TSA ischémique... Je ne nie pas qu'elle existe, mais elle ne figure même pas dans les lignes directrices."}, "5": {"exist": true, "char_ranges": [[538, 592]], "word_ranges": [[87, 94]], "text": "l'endocardite ne nécessiterait pas autant de sacoches."}}} +{"id": 40, "year": 2011, "question_id_specific": 136, "full_question": "Un fumeur et grand buveur de 64 ans a signalé une lymphadénopathie jugulodigastrique droite indolore qui s'est développée progressivement au cours des deux derniers mois et dont la ponction à l'aiguille fine a révélé la présence d'un carcinome épidermoïde. Laquelle des localisations suivantes est la plus probable pour la tumeur primaire ?", "full_answer": "Vous êtes au milieu du bloc ORL. Vous avez déjà passé les blocs Pneumologie et Digestif. La dermatologie doit encore sortir, mais un dermatologue (réponse 1) ne poserait pas cette question. Il nous reste donc la parotide et le larynx ; mais le carcinome épidermoïde parotidien est rare ; je mettrais 5 et je serais rassuré. Peut-être parce que toutes les adénopathies à ce niveau que j'ai vues ont fini par correspondre à des carcinomes des voies aérodigestives supérieures.", "type": "OTORHINOLARYNGOLOGIE ET CHIRURGIE MAXILLO-FACIALE", "options": {"1": "Cuir chevelu.", "2": "Arrêtée.", "3": "Poumon.", "4": "L'œsophage.", "5": "Larynx."}, "correct_option": 5, "explanations": {"1": {"exist": false, "char_ranges": [], "word_ranges": [], "text": ""}, "2": {"exist": false, "char_ranges": [], "word_ranges": [], "text": ""}, "3": {"exist": false, "char_ranges": [], "word_ranges": [], "text": ""}, "4": {"exist": false, "char_ranges": [], "word_ranges": [], "text": ""}, "5": {"exist": true, "char_ranges": [[344, 474]], "word_ranges": [[59, 79]], "text": "toutes les adénopathies à ce niveau que j'ai vues ont fini par correspondre à des carcinomes des voies aérodigestives supérieures."}}} +{"id": 126, "year": 2012, "question_id_specific": 59, "full_question": "Un homme de 65 ans, employé de bureau à la retraite et fumeur d'un paquet de cigarettes par jour, présente une toux persistante, généralement sèche, et une dyspnée progressive depuis 2 ans, qui est actuellement de grade 2. Le patient nie avoir d'autres symptômes. L'examen physique ne révèle aucune donnée pertinente. L'approche à suivre serait la suivante :", "full_answer": "La bonne réponse est 2 (effectuer une radiographie pulmonaire et une spirométrie avec test au bronchodilatateur). Il s'agit probablement d'une BPCO, mais cela doit être confirmé et la gravité de l'obstruction doit être évaluée par spirométrie avant de commencer le traitement. Il s'agit d'un patient présentant un risque de cancer du poumon, il est donc nécessaire d'effectuer une radiographie du thorax afin d'exclure cette pathologie ou d'autres pathologies qui justifieraient les symptômes, bien qu'il soit plus probable qu'ils soient dus à l'apparition de sa BPCO.", "type": "PNEUMOLOGIE ET CHIRURGIE THORACIQUE", "options": {"1": "Commencer le traitement avec des bronchodilatateurs inhalés.", "2": "Effectuer une radiographie pulmonaire et une spirométrie avec test au bronchodilatateur.", "3": "Prescrire des corticostéroïdes oraux.", "4": "Réaliser un scanner thoracique.", "5": "Effectuer les gaz du sang artériel de référence."}, "correct_option": 2, "explanations": {"1": {"exist": false, "char_ranges": [], "word_ranges": [], "text": ""}, "2": {"exist": true, "char_ranges": [[277, 568]], "word_ranges": [[41, 85]], "text": "Il s'agit d'un patient présentant un risque de cancer du poumon, il est donc nécessaire d'effectuer une radiographie du thorax afin d'exclure cette pathologie ou d'autres pathologies qui justifieraient les symptômes, bien qu'il soit plus probable qu'ils soient dus à l'apparition de sa BPCO."}, "3": {"exist": false, "char_ranges": [], "word_ranges": [], "text": ""}, "4": {"exist": false, "char_ranges": [], "word_ranges": [], "text": ""}, "5": {"exist": false, "char_ranges": [], "word_ranges": [], "text": ""}}} +{"id": 568, "year": 2022, "question_id_specific": 151, "full_question": "Un homme de 22 ans s'est présenté au service des urgences avec des urines \"presque noires\" depuis 12 heures. Il n'avait aucun antécédent et ne prenait aucun médicament. Il a signalé un catarrhe des voies respiratoires supérieures depuis 4-5 jours pour lequel il avait pris du paracétamol. Examen : température 37,3 ºC, TA 150/95, FC 85 bpm, léger érythème pharyngé, le reste normal. Les examens de laboratoire étaient normaux : Hb 12,8 g/dl, Hcto 39%, leucocytes 10 500/mm3 avec formule normale, plaquettes 250 000/mm3, coagulation normale, urée 25 mg/dl, créatinine 0,8 mg/dl, ions, hépatique, lipides, albumine et protéines totales normales. Etude d'auto-immunité normale et sérologie virale négative. Urines de 24 h avec protéinurie de 0,75 g/24 h, sédiment avec 10 érythrocytes par champ (90% dysmorphiques), pas de leucocyturie. L'échographie rénale est normale. Quel est le diagnostic le plus probable ?", "full_answer": "Un tableau d'hématurie macroscopique et de protéinurie légère avec des taux de créatinine normaux, accompagné de symptômes catarrhaux des voies supérieures, est très évocateur d'une néphropathie à IgA (option 3 correcte).", "type": "NEPHROLOGIE", "options": {"1": "Néphropathie avec changements minimes.", "2": "Glomérulonéphrite post-infectieuse.", "3": "Néphropathie à IgA.", "4": "Néphrite interstitielle aiguë.", "5": null}, "correct_option": 3, "explanations": {"1": {"exist": false, "char_ranges": [], "word_ranges": [], "text": ""}, "2": {"exist": false, "char_ranges": [], "word_ranges": [], "text": ""}, "3": {"exist": true, "char_ranges": [[0, 221]], "word_ranges": [[0, 31]], "text": "Un tableau d'hématurie macroscopique et de protéinurie légère avec des taux de créatinine normaux, accompagné de symptômes catarrhaux des voies supérieures, est très évocateur d'une néphropathie à IgA (option 3 correcte)."}, "4": {"exist": false, "char_ranges": [], "word_ranges": [], "text": ""}, "5": {"exist": false, "char_ranges": [], "word_ranges": [], "text": ""}}} +{"id": 286, "year": 2016, "question_id_specific": 60, "full_question": "Une insuffisance cardiaque a récemment été diagnostiquée chez un homme de 60 ans ayant des antécédents d'hypertension et de dyslipidémie. Après avoir effectué les examens nécessaires, il a été constaté que sa fraction d'éjection ventriculaire gauche était préservée (>50%). En ce qui concerne le traitement, cochez la bonne réponse :", "full_answer": "Le traitement par diurétiques doit être instauré à fortes doses.", "type": "CARDIOLOGIE ET CHIRURGIE VASCULAIRE", "options": {"1": "Le traitement initial doit viser le processus pathologique sous-jacent.", "2": "Le traitement par diurétiques doit être instauré à fortes doses.", "3": "Si un traitement aux nitrates est nécessaire, il doit être commencé à faible dose.", "4": "La dyspnée peut être traitée en réduisant l'activation neurohormonale à l'aide d'inhibiteurs de l'enzyme de conversion de l'angiotensine ou d'antagonistes des récepteurs de l'angiotensine.", "5": null}, "correct_option": 2, "explanations": {"1": {"exist": false, "char_ranges": [], "word_ranges": [], "text": ""}, "2": {"exist": true, "char_ranges": [[0, 64]], "word_ranges": [[0, 10]], "text": "Le traitement par diurétiques doit être instauré à fortes doses."}, "3": {"exist": false, "char_ranges": [], "word_ranges": [], "text": ""}, "4": {"exist": false, "char_ranges": [], "word_ranges": [], "text": ""}, "5": {"exist": false, "char_ranges": [], "word_ranges": [], "text": ""}}} +{"id": 228, "year": 2014, "question_id_specific": 75, "full_question": "Chez un patient souffrant d'insuffisance cardiaque et présentant une ascite disproportionnée par rapport à l'œdème périphérique, l'étiologie la plus probable est la suivante :", "full_answer": "On nous parle d'une insuffisance cardiaque droite, avec œdème et ascite. Nous avons donc exclu une insuffisance cardiaque gauche : EAo, dilatée avec LVSD et une MHO. Sur les deux qui restent, dans la péricardite constrictive, nous avons une augmentation des pressions de remplissage des cavités droites, ce qui provoque une augmentation de la pression rétrograde avec congestion hépatique, ascite, œdèmes, etc. En revanche, dans l'hypertension pulmonaire, la valve tricuspide est chargée (a priori) de \"contenir\" cette surpression.", "type": "CARDIOLOGIE", "options": {"1": "Sténose aortique sévère.", "2": "Cardiomyopathie dilatée avec dysfonctionnement ventriculaire gauche important.", "3": "Hypertension pulmonaire primaire.", "4": "Cardiomyopathie obstructive hypertrophique.", "5": "Péricardite constrictive."}, "correct_option": 5, "explanations": {"1": {"exist": true, "char_ranges": [[0, 165]], "word_ranges": [[0, 27]], "text": "On nous parle d'une insuffisance cardiaque droite, avec œdème et ascite. Nous avons donc exclu une insuffisance cardiaque gauche : EAo, dilatée avec LVSD et une MHO."}, "2": {"exist": true, "char_ranges": [[0, 165]], "word_ranges": [[0, 27]], "text": "On nous parle d'une insuffisance cardiaque droite, avec œdème et ascite. Nous avons donc exclu une insuffisance cardiaque gauche : EAo, dilatée avec LVSD et une MHO."}, "3": {"exist": true, "char_ranges": [[424, 531]], "word_ranges": [[64, 78]], "text": "dans l'hypertension pulmonaire, la valve tricuspide est chargée (a priori) de \"contenir\" cette surpression."}, "4": {"exist": true, "char_ranges": [[0, 165]], "word_ranges": [[0, 27]], "text": "On nous parle d'une insuffisance cardiaque droite, avec œdème et ascite. Nous avons donc exclu une insuffisance cardiaque gauche : EAo, dilatée avec LVSD et une MHO."}, "5": {"exist": true, "char_ranges": [[193, 410]], "word_ranges": [[32, 62]], "text": "dans la péricardite constrictive, nous avons une augmentation des pressions de remplissage des cavités droites, ce qui provoque une augmentation de la pression rétrograde avec congestion hépatique, ascite, œdèmes, etc."}}} +{"id": 217, "year": 2014, "question_id_specific": 122, "full_question": "On a diagnostiqué chez un homme de 54 ans une tumeur rénale gauche évoquant un carcinome à cellules rénales. L'examen analytique préopératoire révèle des taux élevés de GPT, de phosphatase alcaline et d'alpha-2-globuline, ainsi qu'un allongement du temps de prothrombine. Le foie est élargi de façon diffuse, mais ne présente pas de signes d'infiltration hépatique. La justification la plus probable de ces résultats est due à :", "full_answer": "Une question très facile dans laquelle on nous présente un patient atteint du syndrome de Stauffer, un syndrome paranéoplasique consistant en un dysfonctionnement hépatique secondaire à des produits toxiques sécrétés par une série de tumeurs (la plus fréquente étant le carcinome à cellules rénales, comme dans ce cas). Comme dans ce cas, la cholestase (phosphatase alcaline élevée), la mobilisation des enzymes hépatiques et l'allongement du temps de prothrombine indiquent un dysfonctionnement hépatique. Les métastases sont exclues (il n'y a pas de défauts d'infiltration hépatique), la thrombose tumorale intrahépatique (elle donnerait d'autres symptômes et ce serait plus aigu), et l'hépatite aiguë et l'hémochromatose (elles ne sont pas dans le contexte et ne sont pas ce que l'auteur de la question veut nous faire penser).", "type": "NEPHROLOGIE", "options": {"1": "Métastases hépatiques.", "2": "Thrombose tumorale intrahépatique.", "3": "Hépatite aiguë.", "4": "Présence de substances hépatotoxiques produites par la tumeur.", "5": "Hémochromatose."}, "correct_option": 4, "explanations": {"1": {"exist": true, "char_ranges": [[507, 586]], "word_ranges": [[72, 84]], "text": "Les métastases sont exclues (il n'y a pas de défauts d'infiltration hépatique),"}, "2": {"exist": true, "char_ranges": [[587, 683]], "word_ranges": [[84, 97]], "text": "la thrombose tumorale intrahépatique (elle donnerait d'autres symptômes et ce serait plus aigu),"}, "3": {"exist": true, "char_ranges": [[687, 830]], "word_ranges": [[98, 123]], "text": "l'hépatite aiguë et l'hémochromatose (elles ne sont pas dans le contexte et ne sont pas ce que l'auteur de la question veut nous faire penser)."}, "4": {"exist": true, "char_ranges": [[320, 506]], "word_ranges": [[48, 72]], "text": "Comme dans ce cas, la cholestase (phosphatase alcaline élevée), la mobilisation des enzymes hépatiques et l'allongement du temps de prothrombine indiquent un dysfonctionnement hépatique."}, "5": {"exist": true, "char_ranges": [[687, 830]], "word_ranges": [[98, 123]], "text": "l'hépatite aiguë et l'hémochromatose (elles ne sont pas dans le contexte et ne sont pas ce que l'auteur de la question veut nous faire penser)."}}} +{"id": 142, "year": 2012, "question_id_specific": 138, "full_question": "Fillette de 12 ans, fille unique de parents en bonne santé et non consanguins. Elle n'a pas d'antécédents personnels ou familiaux intéressants. Lors de l'examen clinique pour commencer une activité sportive, vous trouvez une discrète hépatomégalie de consistance normale, vous demandez donc une analyse de sang, qui met en évidence un ASAT de 80 U/l, un ALAT de 105 U/l, un protéinogramme électrophorétique avec toutes les fractions protéiques dans la plage normale et une sérologie négative pour les virus hépatotropes. Que devriez-vous exclure et quel test indiquerez-vous dans ce cas ?", "full_answer": "En fait, je ne testerais pas ce patient pour l'instant et je vérifierais si le taux de transaminases s'est normalisé ultérieurement, car la cause la plus fréquente, du moins chez les enfants, est une élévation transitoire dans le cadre de maladies virales (le plus souvent le cytomégalovirus et le virus d'Epstein-Barr). L'élévation minime des enzymes hépatiques et le fait que le patient présente un profil protéique normal vont à l'encontre de la chronicité... Si je devais choisir, j'opterais pour la réponse 5, qui me semble la moins agressive, et si je devais faire un test sanguin, j'ajouterais le dosage de l'alpha-1-antitrypsine (sans CT) et les marqueurs de la maladie cœliaque, car la maladie cœliaque peut également être associée à une certaine hypertransaminémie.", "type": "PÉDIATRIE", "options": {"1": "Déficit en alpha 1 antitrypsine. Tomodensitométrie thoracique pour confirmer l'emphysème.", "2": "Mucoviscidose. Détermination du chlore dans la sueur.", "3": "Hépatite auto-immune. Biopsie du foie.", "4": "Glycogénose de type VI (déficit en phosphorylase). Biopsie musculaire.", "5": "Maladie de Wilson. Déterminer la céruloplasmine et le cuivre dans le sang et l'urine."}, "correct_option": 5, "explanations": {"1": {"exist": false, "char_ranges": [], "word_ranges": [], "text": ""}, "2": {"exist": false, "char_ranges": [], "word_ranges": [], "text": ""}, "3": {"exist": false, "char_ranges": [], "word_ranges": [], "text": ""}, "4": {"exist": false, "char_ranges": [], "word_ranges": [], "text": ""}, "5": {"exist": false, "char_ranges": [], "word_ranges": [], "text": ""}}} +{"id": 267, "year": 2014, "question_id_specific": 139, "full_question": "Chez un patient atteint d'une cirrhose du foie et d'ascite, qui développe un épanchement pleural avec les données suivantes dans le liquide pleural : LDH 45 U/L (sérum 220 U/L), rapport protéines pleurales/sérum 0,3 et rapport LDH plèvre/sérum 0,2. Quelle serait l'approche appropriée ?", "full_answer": "Ils décrivent des paramètres de liquide pleural de transsudat, donc si le patient n'est pas en insuffisance cardiaque, il s'agit d'une décompensation hydropique. Aucune autre étude n'est nécessaire.", "type": "PNEUMOLOGIE", "options": {"1": "L'étude du liquide avec numération cellulaire, glucose, pH, ADA, cholestérol et culture doit être prolongée. ADA, cholestérol et culture.", "2": "Biopsie pleurale à l'aveugle.", "3": "Vidéothoracoscopie diagnostique.", "4": "Antibiothérapie empirique en cas de suspicion d'épanchement parapneumonique.", "5": "Poursuivre le traitement de votre maladie hépatique."}, "correct_option": 5, "explanations": {"1": {"exist": false, "char_ranges": [], "word_ranges": [], "text": ""}, "2": {"exist": false, "char_ranges": [], "word_ranges": [], "text": ""}, "3": {"exist": false, "char_ranges": [], "word_ranges": [], "text": ""}, "4": {"exist": false, "char_ranges": [], "word_ranges": [], "text": ""}, "5": {"exist": true, "char_ranges": [[0, 198]], "word_ranges": [[0, 28]], "text": "Ils décrivent des paramètres de liquide pleural de transsudat, donc si le patient n'est pas en insuffisance cardiaque, il s'agit d'une décompensation hydropique. Aucune autre étude n'est nécessaire."}}} +{"id": 99, "year": 2012, "question_id_specific": 84, "full_question": "Un patient diabétique de 60 ans consulte pour la première fois au sujet du traitement de sa maladie. Parmi les objectifs suivants, lequel recommanderiez-vous d'atteindre en premier ?", "full_answer": "Les autres ne sont pas des indications. La tension cible est < 130/85 mmHg.", "type": "ENDOCRINOLOGIE", "options": {"1": "Maintenir une pression artérielle inférieure à 110/70 mmHg.", "2": "Arrêt du tabac.", "3": "Maintenir un indice de masse corporelle (IMC) inférieur à 21.", "4": "Effectuer une surveillance quotidienne de la glycémie capillaire basale.", "5": "Éviter les graisses animales dans l'alimentation."}, "correct_option": 2, "explanations": {"1": {"exist": true, "char_ranges": [[0, 75]], "word_ranges": [[0, 14]], "text": "Les autres ne sont pas des indications. La tension cible est < 130/85 mmHg."}, "2": {"exist": true, "char_ranges": [[0, 39]], "word_ranges": [[0, 7]], "text": "Les autres ne sont pas des indications."}, "3": {"exist": true, "char_ranges": [[0, 39]], "word_ranges": [[0, 7]], "text": "Les autres ne sont pas des indications."}, "4": {"exist": true, "char_ranges": [[0, 39]], "word_ranges": [[0, 7]], "text": "Les autres ne sont pas des indications."}, "5": {"exist": true, "char_ranges": [[0, 39]], "word_ranges": [[0, 7]], "text": "Les autres ne sont pas des indications."}}} +{"id": 380, "year": 2016, "question_id_specific": 135, "full_question": "Au cours d'un match de tennis, un patient de 25 ans ressent une douleur intense au niveau du cou et de l'œil gauche. Le lendemain matin, il se réveille avec une sensation d'instabilité de la démarche et présente une ptose palpébrale de l'œil gauche et une anisocorie, la pupille gauche étant plus petite que la pupille droite. Le patient conserve une bonne acuité visuelle. Où situeriez-vous le plus probablement la lésion ?", "full_answer": "Le tableau qui nous est présenté est celui d'un syndrome de Horner (myosis avec ptosis) donc les possibilités sont réduites aux options 3 ou 4, mais on précise que le patient présente une instabilité de la marche, ce qui n'est pas justifié par un syndrome de Horner dû à une dissection carotidienne touchant le ganglion cervical supérieur, ce que l'on veut nous faire croire en parlant de douleur au cou et à l'œil. Si le patient souffre réellement d'ataxie, la seule localisation possible serait le bulbe rachidien.", "type": "NEUROLOGIE", "options": {"1": "III nerf crânien.", "2": "Chiasme optique.", "3": "Ganglion cervical supérieur.", "4": "Moelle épinière.", "5": null}, "correct_option": 4, "explanations": {"1": {"exist": false, "char_ranges": [], "word_ranges": [], "text": ""}, "2": {"exist": false, "char_ranges": [], "word_ranges": [], "text": ""}, "3": {"exist": true, "char_ranges": [[152, 339]], "word_ranges": [[27, 57]], "text": "précise que le patient présente une instabilité de la marche, ce qui n'est pas justifié par un syndrome de Horner dû à une dissection carotidienne touchant le ganglion cervical supérieur,"}, "4": {"exist": true, "char_ranges": [[416, 516]], "word_ranges": [[73, 87]], "text": "Si le patient souffre réellement d'ataxie, la seule localisation possible serait le bulbe rachidien."}, "5": {"exist": false, "char_ranges": [], "word_ranges": [], "text": ""}}} +{"id": 341, "year": 2016, "question_id_specific": 33, "full_question": "Une femme de 67 ans a été diagnostiquée avec un carcinome canalaire infiltrant du sein sans antécédents familiaux de néoplasie. Quelles sont les études complémentaires à réaliser sur la tumeur pour ses implications cliniques et thérapeutiques ?", "full_answer": "Étude des récepteurs hormonaux et de HER2. Tant pour le type de traitement que pour l'étude du facteur pronostique et du traitement adjuvant, il est nécessaire de connaître les différents récepteurs hormonaux (œstrogènes et progestérone) ainsi que HER2-neu.", "type": "GYNÉCOLOGIE ET OBSTÉTRIQUE", "options": {"1": "Étude phénotypique complète par cytométrie de flux.", "2": "Étude des récepteurs hormonaux et de HER2.", "3": "Étude des récepteurs hormonaux, de l'e-cadhérine et étude des parents au premier degré.", "4": "Dépistage BRCA 1-2 et dépistage des parents au premier degré.", "5": null}, "correct_option": 2, "explanations": {"1": {"exist": false, "char_ranges": [], "word_ranges": [], "text": ""}, "2": {"exist": true, "char_ranges": [[43, 257]], "word_ranges": [[7, 38]], "text": "Tant pour le type de traitement que pour l'étude du facteur pronostique et du traitement adjuvant, il est nécessaire de connaître les différents récepteurs hormonaux (œstrogènes et progestérone) ainsi que HER2-neu."}, "3": {"exist": false, "char_ranges": [], "word_ranges": [], "text": ""}, "4": {"exist": false, "char_ranges": [], "word_ranges": [], "text": ""}, "5": {"exist": false, "char_ranges": [], "word_ranges": [], "text": ""}}} +{"id": 308, "year": 2016, "question_id_specific": 215, "full_question": "Un homme de 47 ans atteint de myopie magna, opéré de la cataracte il y a deux ans, se présente aux urgences en signalant une perte de vision profonde et indolore de l'œil droit. Parmi les diagnostics suivants, lequel peut être à l'origine de cette symptomatologie ?", "full_answer": "Il s'agit d'un patient qui a subi une opération de la cataracte il y a deux ans et qui a subi une perte profonde et indolore de l'acuité visuelle. L'endophtalmie post-chirurgicale est plus souvent associée aux patients dans la période postopératoire immédiate, elle est également très douloureuse et s'accompagne d'une baisse rapide de l'acuité visuelle, et n'est donc pas correcte. Le décollement de la rétine correspond au cas clinique car il s'agit également d'un patient myope qui tend à avoir une prédisposition particulière au décollement. L'ARMD survient généralement chez des patients plus âgés et la baisse de la vision est progressive. Dans le cas de la PVD, il peut y avoir ou non une légère baisse de l'acuité visuelle et le patient signale généralement une gêne ou une douleur à l'apparition du tableau clinique.", "type": "OPHTHALMOLOGIE", "options": {"1": "Endophtalmie post-chirurgicale.", "2": "Décollement de la rétine.", "3": "Dégénérescence maculaire liée à l'âge, forme humide.", "4": "Décollement du vitré postérieur.", "5": null}, "correct_option": 2, "explanations": {"1": {"exist": true, "char_ranges": [[147, 382]], "word_ranges": [[29, 60]], "text": "L'endophtalmie post-chirurgicale est plus souvent associée aux patients dans la période postopératoire immédiate, elle est également très douloureuse et s'accompagne d'une baisse rapide de l'acuité visuelle, et n'est donc pas correcte."}, "2": {"exist": true, "char_ranges": [[383, 545]], "word_ranges": [[60, 85]], "text": "Le décollement de la rétine correspond au cas clinique car il s'agit également d'un patient myope qui tend à avoir une prédisposition particulière au décollement."}, "3": {"exist": true, "char_ranges": [[546, 645]], "word_ranges": [[85, 101]], "text": "L'ARMD survient généralement chez des patients plus âgés et la baisse de la vision est progressive."}, "4": {"exist": true, "char_ranges": [[646, 825]], "word_ranges": [[101, 134]], "text": "Dans le cas de la PVD, il peut y avoir ou non une légère baisse de l'acuité visuelle et le patient signale généralement une gêne ou une douleur à l'apparition du tableau clinique."}, "5": {"exist": false, "char_ranges": [], "word_ranges": [], "text": ""}}} +{"id": 519, "year": 2021, "question_id_specific": 176, "full_question": "Une femme de 25 ans se présente au service des urgences accompagnée d'une amie, déclarant qu'elle a été victime d'une agression sexuelle il y a quatre heures. La patiente est très angoissée, répond à ses questions de manière incohérente et lente, et est incapable de se souvenir de la manière dont l'agression s'est déroulée. Ses signes vitaux (tension artérielle, fréquence cardiaque et température) sont normaux. Lequel des énoncés suivants serait FAUX ?", "full_answer": "Toutes les actions détaillées sont correctes à l'exception de celle détaillée dans l'option 2. Le prélèvement d'échantillons d'intérêt légal ne sera effectué qu'en présence du médecin légiste et, sans la présence du médecin légiste, la femme doit donner son consentement écrit. En outre, le prélèvement gynécologique doit être adapté à l'état psychologique de la victime, de sorte que dans la plupart des cas, il est contre-productif de le faire à ce moment-là (a fortiori lorsque l'état de la patiente est décrit comme \"très affectée, répondant à ses questions de manière incongrue, lentement et sans pouvoir se souvenir de la manière dont l'agression s'est déroulée\").", "type": "SOINS CRITIQUES ET D'URGENCE", "options": {"1": "Contactez le tribunal de permanence.", "2": "Effectuer immédiatement un examen gynécologique avec cytologie cervicale.", "3": "Administrer 500 mg de ceftriaxone par voie intramusculaire, 1 g d'azithromycine par voie orale et 2 g de tinidazole par voie orale en doses uniques.", "4": "Administrer 1500 mg de lévonorgestrel par voie orale en une seule dose.", "5": null}, "correct_option": 2, "explanations": {"1": {"exist": false, "char_ranges": [], "word_ranges": [], "text": ""}, "2": {"exist": true, "char_ranges": [[95, 277]], "word_ranges": [[14, 41]], "text": "Le prélèvement d'échantillons d'intérêt légal ne sera effectué qu'en présence du médecin légiste et, sans la présence du médecin légiste, la femme doit donner son consentement écrit."}, "3": {"exist": false, "char_ranges": [], "word_ranges": [], "text": ""}, "4": {"exist": false, "char_ranges": [], "word_ranges": [], "text": ""}, "5": {"exist": false, "char_ranges": [], "word_ranges": [], "text": ""}}} +{"id": 63, "year": 2011, "question_id_specific": 200, "full_question": "Un maçon de 31 ans se présente aux urgences pour une ténosynovite consécutive à un traumatisme pénétrant. L'examen révèle une infection de la bourse cubitale suivie d'une nécrose tendineuse. Quel tendon sera touché ?", "full_answer": "Le flexor carpi radialis et l'abductor pollicis longus sont anatomiquement éloignés de l'ulnaire. Le long palmaire est médial, tout comme le flexor digitorum profundus. Le tendon qui longe le nerf cubital et s'insère dans le pisiforme est le flexor carpi ulnaris, d'où la réponse 1.", "type": "ANATOMIE", "options": {"1": "Tendon du fléchisseur ulnaire du carpe.", "2": "Tendon de l'abducteur pollicis longus.", "3": "Tendon du flexor digitorum profundus.", "4": "Tendon du flexor carpi radialis.", "5": "Tendon du grand palmier."}, "correct_option": 1, "explanations": {"1": {"exist": true, "char_ranges": [[169, 282]], "word_ranges": [[24, 45]], "text": "Le tendon qui longe le nerf cubital et s'insère dans le pisiforme est le flexor carpi ulnaris, d'où la réponse 1."}, "2": {"exist": true, "char_ranges": [[0, 97]], "word_ranges": [[0, 13]], "text": "Le flexor carpi radialis et l'abductor pollicis longus sont anatomiquement éloignés de l'ulnaire."}, "3": {"exist": true, "char_ranges": [[98, 168]], "word_ranges": [[13, 24]], "text": "Le long palmaire est médial, tout comme le flexor digitorum profundus."}, "4": {"exist": true, "char_ranges": [[0, 97]], "word_ranges": [[0, 13]], "text": "Le flexor carpi radialis et l'abductor pollicis longus sont anatomiquement éloignés de l'ulnaire."}, "5": {"exist": true, "char_ranges": [[98, 168]], "word_ranges": [[13, 24]], "text": "Le long palmaire est médial, tout comme le flexor digitorum profundus."}}} +{"id": 30, "year": 2011, "question_id_specific": 61, "full_question": "Un patient précédemment diagnostiqué avec une BPCO s'est présenté aux urgences depuis 3 jours avec une augmentation de sa dyspnée habituelle jusqu'au repos, une toux avec expectoration blanchâtre et une somnolence marquée. Après un traitement à base d'oxygène à faible concentration, de bronchodilatateurs et de coticoïdes, il subit une seconde analyse des gaz du sang artériel avec une fraction d'oxygène inspiratoire de 28% qui montre un pH de 7,32, une pO2 de 61 mmHg, une pCO2 de 58 mmHg, une HCO3- de 29 mmol/l. Quelle interprétation feriez-vous de l'analyse des gaz du sang artériel et quel traitement mettriez-vous en oeuvre ?", "full_answer": "C'est une très mauvaise question, je parierai sur une réponse, mais en fonction des données que l'on ne nous donne pas, trois réponses pourraient être valables. Le piège est qu'ils nous donnent le tableau clinique initial SANS GASOMÉTRIE et qu'ils nous donnent ensuite les gaz du sang SANS CLINIQUE, de sorte qu'il n'y a aucun moyen de savoir quelle a été l'évolution du patient avec notre traitement, et il est donc impossible de savoir ce qu'il faut faire. Si le patient est arrivé avec un gaz du sang dont le pH était de 7,05 et qu'il a maintenant celui que l'on nous donne, nous faisons bien (bien que l'on ne nous dise pas l'intervalle de temps entre une chose et l'autre) et, dans ce cas, il peut être intéressant de poursuivre le même traitement. Si rien n'a évolué, que les manifestations cliniques sont les mêmes et que les gaz du sang au début étaient similaires, il faut faire quelque chose (cela dépend aussi de l'état de base du patient, que l'on ne nous dit pas non plus). Si nous décidons de faire quelque chose, à l'heure actuelle, nous pourrions décider d'utiliser la ventilation plutôt que la théophylline, bien qu'avec une \" si petite \" altération du pH (et encore plus si nous pensons qu'il s'agit d'un patient chronique qui peut avoir un gaz sanguin de base similaire à celui qui nous a été présenté), cela peut sembler \" exagéré \". Nous n'avons rien \"contre\" la théophylline (aucune contre-indication ne nous est signalée), mais il est rare aujourd'hui d'y avoir recours. 1 : Faux, c'est un patient \"rétentif\" et l'augmentation du débit d'O2 risque fort d'augmenter l'hypercapnie et donc l'acidose. 2 : Nous garderons celle-ci comme correcte. 3 : Il ne s'agit pas d'une acidose métabolique, le bicarbonate n'est pas diminué, au contraire, il est légèrement augmenté pour tenter de compenser l'acidose respiratoire. 4 : Il ne s'agit pas d'une insuffisance respiratoire par définition (pO2 60), mais d'une hypercapnie et d'une légère acidose. Si nous connaissions la lecture du premier gaz du sang et que son état était bien pire (c'est-à-dire qu'elle progressait très bien), continuer avec le même médicament serait une option, mais cette donnée est manquante !!!, en tout cas il semble que ce soit une réponse piège. 5 : À l'heure actuelle, et - comme nous l'avons déjà mentionné - en tenant compte du fait qu'il n'y a pas de contre-indication formelle, ce médicament n'est généralement pas utilisé.", "type": "NEUROLOGIE ET CHIRURGIE THORACIQUE", "options": {"1": "Cela augmenterait le débit d'oxygène car il souffre d'acidose respiratoire aiguë et d'hypoxémie.", "2": "Je mettrais en place une ventilation mécanique non invasive car il souffre d'une insuffisance respiratoire hypercapnique aiguë avec une acidose respiratoire modérée.", "3": "J'ajouterais du bicarbonate de sodium pour corriger l'acidose métabolique aiguë.", "4": "Le patient ne souffre pas d'insuffisance respiratoire et continuerait donc à suivre le même régime pharmacologique.", "5": "J'ajouterais l'aminophylline intraveineuse comme stimulant respiratoire, car je constate une hypoxémie et une hypercapnie."}, "correct_option": 4, "explanations": {"1": {"exist": true, "char_ranges": [[1498, 1620]], "word_ranges": [[262, 279]], "text": "Faux, c'est un patient \"rétentif\" et l'augmentation du débit d'O2 risque fort d'augmenter l'hypercapnie et donc l'acidose."}, "2": {"exist": true, "char_ranges": [[987, 1353]], "word_ranges": [[177, 240]], "text": "Si nous décidons de faire quelque chose, à l'heure actuelle, nous pourrions décider d'utiliser la ventilation plutôt que la théophylline, bien qu'avec une \" si petite \" altération du pH (et encore plus si nous pensons qu'il s'agit d'un patient chronique qui peut avoir un gaz sanguin de base similaire à celui qui nous a été présenté), cela peut sembler \" exagéré \"."}, "3": {"exist": true, "char_ranges": [[1669, 1836]], "word_ranges": [[288, 312]], "text": "Il ne s'agit pas d'une acidose métabolique, le bicarbonate n'est pas diminué, au contraire, il est légèrement augmenté pour tenter de compenser l'acidose respiratoire."}, "4": {"exist": true, "char_ranges": [[1841, 1962]], "word_ranges": [[314, 332]], "text": "Il ne s'agit pas d'une insuffisance respiratoire par définition (pO2 60), mais d'une hypercapnie et d'une légère acidose."}, "5": {"exist": true, "char_ranges": [[2243, 2421]], "word_ranges": [[381, 410]], "text": "À l'heure actuelle, et - comme nous l'avons déjà mentionné - en tenant compte du fait qu'il n'y a pas de contre-indication formelle, ce médicament n'est généralement pas utilisé."}}} +{"id": 226, "year": 2014, "question_id_specific": 71, "full_question": "Une femme de 82 ans, hypertendue, sous aténolol, hydrochlorothiazide et digoxine. Elle s'est présentée aux urgences pour une fibrillation auriculaire et on lui a administré du vérapamil IV. L'ECG montre un bloc auriculo-ventriculaire complet. Quelle est la cause la plus probable de cette situation clinique ?", "full_answer": "Il prend déjà deux inhibiteurs du nœud AV et vous en ajoutez un troisième, et vous vous étonnez qu'il obtienne un bloc AV complet après avoir utilisé des médicaments de familles différentes mais ayant le même effet (pharmacodynamique) ? Les trois. Et rappelez-vous que les médicaments sont comme les boissons dans les bars : il ne faut pas les mélanger !", "type": "CARDIOLOGIE", "options": {"1": "Intoxication digitalique due à une interaction pharmacocinétique avec le vérapamil.", "2": "Hypokaliémie due à l'administration de thiazide et de digoxine.", "3": "Interaction pharmacodynamique entre les bêta-bloquants, la digoxine et le vérapamil.", "4": "Effet hypotenseur des diurétiques thiazidiques.", "5": "Arythmie cardiaque due au vérapamil."}, "correct_option": 3, "explanations": {"1": {"exist": false, "char_ranges": [], "word_ranges": [], "text": ""}, "2": {"exist": false, "char_ranges": [], "word_ranges": [], "text": ""}, "3": {"exist": true, "char_ranges": [[0, 236]], "word_ranges": [[0, 39]], "text": "Il prend déjà deux inhibiteurs du nœud AV et vous en ajoutez un troisième, et vous vous étonnez qu'il obtienne un bloc AV complet après avoir utilisé des médicaments de familles différentes mais ayant le même effet (pharmacodynamique) ?"}, "4": {"exist": false, "char_ranges": [], "word_ranges": [], "text": ""}, "5": {"exist": false, "char_ranges": [], "word_ranges": [], "text": ""}}} +{"id": 565, "year": 2022, "question_id_specific": 148, "full_question": "Un patient de 48 ans a été adressé au service de néphrologie pour un débit de filtration glomérulaire estimé (DFGe) de 32 ml/min/1,72 m². Parmi les données suivantes, lesquelles ne suggèrent PAS une maladie rénale chronique ?", "full_answer": "Une différenciation corticomédullaire adéquate suggère l'absence d'atteinte parenchymateuse chronique et est une observation courante dans l'insuffisance rénale aiguë (option 4 correcte). Des antécédents familiaux de néphropathie peuvent indiquer l'existence d'une maladie héréditaire (option 2 incorrecte). Des reins de petite taille et une hyperphosphatémie sont des caractéristiques d'une maladie rénale chronique (options 1 et 3 incorrectes).", "type": "NEPHROLOGIE", "options": {"1": "Petits reins.", "2": "Antécédents familiaux de maladie rénale.", "3": "Des niveaux élevés de phosphore dans le sang.", "4": "Bonne différenciation échographique du cortex et de la médulla rénaux.", "5": null}, "correct_option": 4, "explanations": {"1": {"exist": true, "char_ranges": [[308, 446]], "word_ranges": [[35, 55]], "text": "Des reins de petite taille et une hyperphosphatémie sont des caractéristiques d'une maladie rénale chronique (options 1 et 3 incorrectes)."}, "2": {"exist": true, "char_ranges": [[188, 307]], "word_ranges": [[21, 35]], "text": "Des antécédents familiaux de néphropathie peuvent indiquer l'existence d'une maladie héréditaire (option 2 incorrecte)."}, "3": {"exist": true, "char_ranges": [[308, 446]], "word_ranges": [[35, 55]], "text": "Des reins de petite taille et une hyperphosphatémie sont des caractéristiques d'une maladie rénale chronique (options 1 et 3 incorrectes)."}, "4": {"exist": true, "char_ranges": [[0, 187]], "word_ranges": [[0, 21]], "text": "Une différenciation corticomédullaire adéquate suggère l'absence d'atteinte parenchymateuse chronique et est une observation courante dans l'insuffisance rénale aiguë (option 4 correcte)."}, "5": {"exist": false, "char_ranges": [], "word_ranges": [], "text": ""}}} +{"id": 135, "year": 2012, "question_id_specific": 38, "full_question": "Un homme de 64 ans atteint d'une cirrhose hépatique de type Child A sans antécédent de décompensation. Après avoir détecté une lésion hépatique focale lors de l'échographie de dépistage, l'étude a été complétée par une tomodensitométrie thoraco-abdominale. Cet examen révèle la présence de 4 lésions hépatiques (dont une mesurant jusqu'à 6 cm) avec un profil de captation typique d'un hépatocarcinome, une invasion vasculaire tumorale et une métastase dans la glande surrénale droite. Aucune ascite n'est observée. Le patient ne signale qu'une asthénie, mais pas de syndrome général. Le traitement de ce patient serait le suivant :", "full_answer": "Commentaire : le patient est en bon état général avec un hépatocarcinome récemment diagnostiqué, et il ne serait pas correct de lui donner un autre type de traitement que des soins symptomatiques et de soutien à ce stade. La TACE n'est pas indiquée en raison de l'atteinte extra-hépatique. La transplantation n'est pas non plus indiquée en cas de maladie extrahépatique. Il nous reste les réponses 3 et 4, qui prévoient toutes deux un traitement systémique (indiqué parce qu'il s'agit d'une maladie métastatique) et la bonne réponse est la réponse 4, pour toutes les raisons invoquées.", "type": "ONCOLOGIE", "options": {"1": "Traitement symptomatique et de soutien, le patient étant atteint d'un hépatocarcinome avancé avec invasion vasculaire et métastases.", "2": "La chimio-embolisation transartérielle (TACE), car ce traitement augmente la survie des patients atteints d'un hépatocarcinome avancé.", "3": "Chimiothérapie systémique avec doxorubicine iv car il a une maladie disséminée mais est en bon état général.", "4": "Traitement par sorafenib oral, car il s'agit d'un patient en bon état général, enfant A et atteint d'un hépatocarcinome BCLC-C de stade avancé.", "5": "Le traitement approprié est la transplantation du foie, car c'est la seule procédure qui permet d'éliminer complètement la tumeur primaire et d'éviter les complications futures de la cirrhose."}, "correct_option": 4, "explanations": {"1": {"exist": true, "char_ranges": [[14, 221]], "word_ranges": [[2, 38]], "text": "le patient est en bon état général avec un hépatocarcinome récemment diagnostiqué, et il ne serait pas correct de lui donner un autre type de traitement que des soins symptomatiques et de soutien à ce stade."}, "2": {"exist": true, "char_ranges": [[222, 289]], "word_ranges": [[38, 48]], "text": "La TACE n'est pas indiquée en raison de l'atteinte extra-hépatique."}, "3": {"exist": true, "char_ranges": [[406, 585]], "word_ranges": [[68, 95]], "text": "qui prévoient toutes deux un traitement systémique (indiqué parce qu'il s'agit d'une maladie métastatique) et la bonne réponse est la réponse 4, pour toutes les raisons invoquées."}, "4": {"exist": true, "char_ranges": [[406, 585]], "word_ranges": [[68, 95]], "text": "qui prévoient toutes deux un traitement systémique (indiqué parce qu'il s'agit d'une maladie métastatique) et la bonne réponse est la réponse 4, pour toutes les raisons invoquées."}, "5": {"exist": true, "char_ranges": [[290, 370]], "word_ranges": [[48, 60]], "text": "La transplantation n'est pas non plus indiquée en cas de maladie extrahépatique."}}} +{"id": 556, "year": 2022, "question_id_specific": 55, "full_question": "Un patient diabétique de 63 ans dont le médecin de famille a demandé une prise de sang pour déterminer son taux de vitamine B12. Quel est le médicament antidiabétique qu'il prend et qui justifie cette demande ?", "full_answer": "La metformine est un antidiabétique oral largement utilisé dans le traitement du diabète de type 2. Les effets secondaires les plus connus sont les effets gastro-intestinaux et l'acidose lactique, mais la malabsorption de la vitamine B12 est moins bien connue. Une diminution des taux de vitamine B 12 est observée chez les patients traités à la metformine. Le mécanisme par lequel ce déficit se produit n'est pas clair et on sait qu'il est réversible à l'arrêt du traitement.", "type": "ENDOCRINOLOGIE", "options": {"1": "Gliclazide.", "2": "Metformine.", "3": "Repaglinide.", "4": "Pioglitazone.", "5": null}, "correct_option": 2, "explanations": {"1": {"exist": false, "char_ranges": [], "word_ranges": [], "text": ""}, "2": {"exist": true, "char_ranges": [[0, 260]], "word_ranges": [[0, 40]], "text": "La metformine est un antidiabétique oral largement utilisé dans le traitement du diabète de type 2. Les effets secondaires les plus connus sont les effets gastro-intestinaux et l'acidose lactique, mais la malabsorption de la vitamine B12 est moins bien connue."}, "3": {"exist": false, "char_ranges": [], "word_ranges": [], "text": ""}, "4": {"exist": false, "char_ranges": [], "word_ranges": [], "text": ""}, "5": {"exist": false, "char_ranges": [], "word_ranges": [], "text": ""}}} +{"id": 448, "year": 2018, "question_id_specific": 148, "full_question": "Une femme de 59 ans ayant récemment eu des crises d'épilepsie s'est présentée au service des urgences. L'examen a révélé des signes pyramidaux gauches et un œdème papillaire. L'IRM cérébrale montre une masse hémisphérique droite avec œdème, déviation de la ligne médiane et signes d'hernie tentorielle. Elle signale qu'elle souffre depuis une semaine de céphalées dont l'intensité augmente. Parmi les caractéristiques suivantes associées aux céphalées, laquelle vous paraît la plus probable chez cette patiente ?", "full_answer": "Un diagnostic récent de LOE cérébrale avec des signes d'hypertension intracrânienne (œdème, déviation de la ligne médiane, œdème papillaire) est décrit. Les céphalées associées sont typiquement des céphalées matinales, avec une augmentation des valeurs de la PIC pendant la nuit.", "type": "NEUROSURGÉRIE", "options": {"1": "Prédominance matinale.", "2": "Elle ne change pas avec l'effort.", "3": "Photophobie.", "4": "Sonophobie.", "5": null}, "correct_option": 1, "explanations": {"1": {"exist": true, "char_ranges": [[0, 279]], "word_ranges": [[0, 40]], "text": "Un diagnostic récent de LOE cérébrale avec des signes d'hypertension intracrânienne (œdème, déviation de la ligne médiane, œdème papillaire) est décrit. Les céphalées associées sont typiquement des céphalées matinales, avec une augmentation des valeurs de la PIC pendant la nuit."}, "2": {"exist": false, "char_ranges": [], "word_ranges": [], "text": ""}, "3": {"exist": false, "char_ranges": [], "word_ranges": [], "text": ""}, "4": {"exist": false, "char_ranges": [], "word_ranges": [], "text": ""}, "5": {"exist": false, "char_ranges": [], "word_ranges": [], "text": ""}}} +{"id": 591, "year": 2022, "question_id_specific": 64, "full_question": "Une femme de 68 ans porte des lunettes de +4,00 dioptries pour la vision de loin dans les deux yeux. Elle consulte parce que depuis quelques semaines, elle ressent des douleurs oculaires accompagnées de maux de tête et d'une vision floue le soir lorsqu'elle regarde la télévision dans une lumière faible. Parmi les pathologies suivantes, laquelle est la plus probable ?", "full_answer": "Nous sommes en présence d'une femme de 68 ans qui présente une hypermétropie modérée et qui n'a pas d'antécédents de chirurgie de la cataracte. Ce seul fait n'est pas une coïncidence : il est important que la patiente soit hypermétrope, car les globes oculaires des hypermétropes sont plus courts, avec un certain conflit d'espace dans le segment antérieur lorsque le cristallin subit un processus d'intumescence secondaire à l'âge (ce que les patients appellent la cataracte naissante). En outre, dans le cas clinique, la douleur oculaire et la céphalée surviennent dans des situations scotopiques, c'est-à-dire dans des situations de faible éclairage ambiant, ce qui produit une mydriase moyenne pour optimiser l'afflux de lumière vers le globe oculaire. Il est donc clair que le patient souffre de processus primaires de fermeture angulaire qui se résorbent éventuellement d'eux-mêmes lorsque la situation déclenchante cesse, plutôt que d'une véritable crise aiguë de glaucome. Toutefois, ces fermetures angulaires intermittentes sont incluses dans le spectre du glaucome par fermeture angulaire primaire (option 1 correcte). La migraine, n'étant pas une question neurologique, peut parfois être un facteur de confusion dans le tableau clinique habituel ; cependant, les patients sont généralement plus jeunes, ou ont au moins des antécédents clairs de migraine, et, surtout, ont une aggravation symptomatologique avec des stimuli sensoriels intenses (photophobie et phonophobie), mais les symptômes ne s'aggravent pas précisément en cas d'éclairage faible (option 2 incorrecte). La kératite actinique survient précisément chez les patients exposés à des rayons ultraviolets qui ne sont pas correctement filtrés avant d'atteindre la cornée ; elle est typique des soudeurs (appelée kératite du soudeur), des skieurs et d'autres personnes exposées à une lumière très intense et sans protection oculaire adéquate (option 3 incorrecte). Le syndrome de l'œil sec ne provoque généralement pas de maux de tête et la gêne oculaire ne va pas au-delà d'une gêne persistante que les patients pourraient définir comme une douleur en raison de sa composante neuropathique récemment plus étudiée. Certaines formes de sécheresse oculaire peuvent s'aggraver le soir, d'autres le matin, et cela dépend principalement de la fréquence du clignement des yeux. En outre, le syndrome de l'œil sec est un terme dépassé ; de nos jours, il est préférable de parler de maladie de l'œil sec (option 4 incorrecte).", "type": "OPHTHALMOLOGIE", "options": {"1": "Glaucome angulaire primaire.", "2": "La migraine.", "3": "Kératite actinique.", "4": "Syndrome de l'œil sec.", "5": null}, "correct_option": 1, "explanations": {"1": {"exist": true, "char_ranges": [[757, 1128]], "word_ranges": [[116, 167]], "text": "Il est donc clair que le patient souffre de processus primaires de fermeture angulaire qui se résorbent éventuellement d'eux-mêmes lorsque la situation déclenchante cesse, plutôt que d'une véritable crise aiguë de glaucome. Toutefois, ces fermetures angulaires intermittentes sont incluses dans le spectre du glaucome par fermeture angulaire primaire (option 1 correcte)."}, "2": {"exist": true, "char_ranges": [[1129, 1582]], "word_ranges": [[167, 231]], "text": "La migraine, n'étant pas une question neurologique, peut parfois être un facteur de confusion dans le tableau clinique habituel ; cependant, les patients sont généralement plus jeunes, ou ont au moins des antécédents clairs de migraine, et, surtout, ont une aggravation symptomatologique avec des stimuli sensoriels intenses (photophobie et phonophobie), mais les symptômes ne s'aggravent pas précisément en cas d'éclairage faible (option 2 incorrecte)."}, "3": {"exist": true, "char_ranges": [[1583, 1935]], "word_ranges": [[231, 283]], "text": "La kératite actinique survient précisément chez les patients exposés à des rayons ultraviolets qui ne sont pas correctement filtrés avant d'atteindre la cornée ; elle est typique des soudeurs (appelée kératite du soudeur), des skieurs et d'autres personnes exposées à une lumière très intense et sans protection oculaire adéquate (option 3 incorrecte)."}, "4": {"exist": true, "char_ranges": [[1936, 2489]], "word_ranges": [[283, 375]], "text": "Le syndrome de l'œil sec ne provoque généralement pas de maux de tête et la gêne oculaire ne va pas au-delà d'une gêne persistante que les patients pourraient définir comme une douleur en raison de sa composante neuropathique récemment plus étudiée. Certaines formes de sécheresse oculaire peuvent s'aggraver le soir, d'autres le matin, et cela dépend principalement de la fréquence du clignement des yeux. En outre, le syndrome de l'œil sec est un terme dépassé ; de nos jours, il est préférable de parler de maladie de l'œil sec (option 4 incorrecte)."}, "5": {"exist": false, "char_ranges": [], "word_ranges": [], "text": ""}}} +{"id": 291, "year": 2016, "question_id_specific": 157, "full_question": "Un patient de 8 ans présentant des lésions pustuleuses superficielles, des érosions et des croûtes jaunâtres autour de la bouche depuis 3 jours doit être pris en considération :", "full_answer": "C'est un tableau clinique typique et fréquent chez l'enfant. Vous décrivez un impétigo contagieux. Il n'y a pas beaucoup de discussion. L'agent causal le plus fréquent dans la forme bulleuse est S. aureus phage II.", "type": "DERMATOLOGIE, VÉNÉRÉOLOGIE ET CHIRURGIE PLASTIQUE", "options": {"1": "Erythème exsudatif multiforme.", "2": "Impétigo contagieux.", "3": "Acné de l'enfant.", "4": "Psoriasis pustuleux.", "5": null}, "correct_option": 2, "explanations": {"1": {"exist": false, "char_ranges": [], "word_ranges": [], "text": ""}, "2": {"exist": true, "char_ranges": [[0, 98]], "word_ranges": [[0, 14]], "text": "C'est un tableau clinique typique et fréquent chez l'enfant. Vous décrivez un impétigo contagieux."}, "3": {"exist": false, "char_ranges": [], "word_ranges": [], "text": ""}, "4": {"exist": false, "char_ranges": [], "word_ranges": [], "text": ""}, "5": {"exist": false, "char_ranges": [], "word_ranges": [], "text": ""}}} +{"id": 213, "year": 2014, "question_id_specific": 160, "full_question": "Femme de 75 ans, ménopausée à 52 ans, sans antécédents familiaux ou personnels de fracture, diagnostiquée avec une artérite temporale, qui va commencer un traitement avec une forte dose de prednisone et qui s'attend à un traitement d'au moins un an. Le délai de densitométrie (DXA) dans son centre est de 4 à 5 mois. Un traitement préventif de l'ostéoporose est envisagé. Laquelle des approches suivantes est la plus appropriée ?", "full_answer": "Dans les données fournies, il n'y a pas de facteurs de risque cliniques d'ostéoporose avant la maladie actuelle. Étant donné qu'un régime de prednisone prolongé et à forte dose sera mis en place, il est conseillé, en plus des mesures préventives non pharmacologiques, d'initier dans tous les cas un traitement visant à minimiser la perte secondaire de masse osseuse qui se produit avec l'administration de glucocorticoïdes (et, surtout, au cours des premiers mois). Par conséquent, et pour cette seule raison, les options 1, 2 et 5 seraient exclues (toutes les trois ayant un autre facteur de conditionnement). Dans ce contexte, et dans le cas d'une personne âgée de plus de 65 ans, une attitude appropriée serait d'administrer des suppléments de calcium (les corticoïdes sont ostéopéniants car ils produisent, entre autres mécanismes, un bilan négatif de cet élément) et de vitamine D. L'option d'associer un bisphosphonate pourrait également être envisagée, mais dans la proposition 3, le supplément de calcium est absent.", "type": "RHEUMATOLOGIE", "options": {"1": "Demander une DXA et attendre le résultat.", "2": "Évaluer le risque absolu de fracture à l'aide du questionnaire FRAX sans DMO et ne traiter que s'il est élevé.", "3": "Commencer un traitement par bisphosphonates et vitamine D (800 UI/jour).", "4": "Supplément de calcium (1 g) et de vitamine D (800 UI/jour).", "5": "Évaluer l'ostéopénie sur les radiographies et la traiter si elle est présente."}, "correct_option": 4, "explanations": {"1": {"exist": true, "char_ranges": [[113, 610]], "word_ranges": [[18, 97]], "text": "Étant donné qu'un régime de prednisone prolongé et à forte dose sera mis en place, il est conseillé, en plus des mesures préventives non pharmacologiques, d'initier dans tous les cas un traitement visant à minimiser la perte secondaire de masse osseuse qui se produit avec l'administration de glucocorticoïdes (et, surtout, au cours des premiers mois). Par conséquent, et pour cette seule raison, les options 1, 2 et 5 seraient exclues (toutes les trois ayant un autre facteur de conditionnement)."}, "2": {"exist": true, "char_ranges": [[113, 610]], "word_ranges": [[18, 97]], "text": "Étant donné qu'un régime de prednisone prolongé et à forte dose sera mis en place, il est conseillé, en plus des mesures préventives non pharmacologiques, d'initier dans tous les cas un traitement visant à minimiser la perte secondaire de masse osseuse qui se produit avec l'administration de glucocorticoïdes (et, surtout, au cours des premiers mois). Par conséquent, et pour cette seule raison, les options 1, 2 et 5 seraient exclues (toutes les trois ayant un autre facteur de conditionnement)."}, "3": {"exist": true, "char_ranges": [[887, 1024]], "word_ranges": [[141, 160]], "text": "L'option d'associer un bisphosphonate pourrait également être envisagée, mais dans la proposition 3, le supplément de calcium est absent."}, "4": {"exist": true, "char_ranges": [[611, 886]], "word_ranges": [[97, 141]], "text": "Dans ce contexte, et dans le cas d'une personne âgée de plus de 65 ans, une attitude appropriée serait d'administrer des suppléments de calcium (les corticoïdes sont ostéopéniants car ils produisent, entre autres mécanismes, un bilan négatif de cet élément) et de vitamine D."}, "5": {"exist": true, "char_ranges": [[113, 610]], "word_ranges": [[18, 97]], "text": "Étant donné qu'un régime de prednisone prolongé et à forte dose sera mis en place, il est conseillé, en plus des mesures préventives non pharmacologiques, d'initier dans tous les cas un traitement visant à minimiser la perte secondaire de masse osseuse qui se produit avec l'administration de glucocorticoïdes (et, surtout, au cours des premiers mois). Par conséquent, et pour cette seule raison, les options 1, 2 et 5 seraient exclues (toutes les trois ayant un autre facteur de conditionnement)."}}} +{"id": 442, "year": 2018, "question_id_specific": 100, "full_question": "Une femme de 34 ans a consulté pour des coitorragies répétées. Elle a présenté une cytologie avec une lésion intra-épithéliale de haut grade (U-SIL). Par la suite, une colposcopie et une biopsie ont été réalisées sur une zone en mosaïque et l'histologie a révélé un foyer de carcinome malpighien invasif de 2 mm de long. Quelle est l'option thérapeutique la plus appropriée pour cette patiente ?", "full_answer": "Selon la SEGO : \"Exciser la totalité de la lésion pour qu'elle puisse être évaluée histologiquement. Il s'agit de l'excision de toute la zone de transformation. D'une manière générale, l'exérèse doit être adaptée à la taille et aux caractéristiques de la lésion. On distingue trois types d'exérèse en fonction de la présence de la composante endocervicale de la lésion. L'excision de type 1 (applicable en cas de zone de transformation de type 1, où l'anse diathermique ne doit pas inclure le canal endocervical ni dépasser 8 mm de profondeur), l'excision de type 2 (indiquée en cas de zone de transformation de type 2, implique la résection d'une petite partie du canal endocervical visible par colposcopie) et l'excision de type 3 (indiquée en cas de zone de transformation de type 3, inclut une partie de l'épithélium endocervical)\". Quand on parle d'excision, c'est synonyme de conisation ; comme la lésion a moins de 8 mm de profondeur, c'est le traitement indiqué.", "type": "GYNÉCOLOGIE ET OBSTÉTRIQUE", "options": {"1": "Conisation.", "2": "Hystérectomie totale sans annexectomie.", "3": "Radiothérapie pelvienne à visée curative.", "4": "Répéter une biopsie plus importante.", "5": null}, "correct_option": 1, "explanations": {"1": {"exist": true, "char_ranges": [[895, 970]], "word_ranges": [[145, 159]], "text": "comme la lésion a moins de 8 mm de profondeur, c'est le traitement indiqué."}, "2": {"exist": false, "char_ranges": [], "word_ranges": [], "text": ""}, "3": {"exist": false, "char_ranges": [], "word_ranges": [], "text": ""}, "4": {"exist": false, "char_ranges": [], "word_ranges": [], "text": ""}, "5": {"exist": false, "char_ranges": [], "word_ranges": [], "text": ""}}} +{"id": 491, "year": 2020, "question_id_specific": 106, "full_question": "Une femme de 65 ans a été adressée au service des urgences pour de la fièvre et des altérations de ses analyses sanguines : hémoglobine 11,4 g/dL, leucocytes 0,86 x103/μL (neutrophiles 41,9 %, lymphocytes 55,8 %), plaquettes 48,0 x103/μL, fibrinogène 118 mg/dL, D-dimères 20,2 μg/mL. Un examen de la moelle osseuse a été effectué et le patient a été diagnostiqué avec une leucémie aiguë avec t(15;17) dans 60 % des cellules. Laquelle des réponses suivantes est correcte ?", "full_answer": "Je pense que c'est une très bonne question. Il n'y a aucun doute sur le diagnostic. Car si vous ne connaissez pas la translocation, ce n'est pas grave... les réponses vous donnent le diagnostic. Il est maintenant temps de savoir comment la leucémie aiguë promyélocytaire est actuellement traitée (ATRA mis à part... que vous connaissez sûrement). Depuis 2017, le protocole PETHEMA pour la LPA chez les patients à risque faible ou intermédiaire (et/ou âgés de plus de 70 ans) est ATO+ATRA. Pour les patients à haut risque, il s'agit d'ATRA+chimiothérapie. Je ne pense pas qu'ils veuillent que vous sachiez comment le risque est établi dans la promyélite, mais il est basé sur la numération des leucocytes et des plaquettes. Dans ce cas, vous n'avez pas de score du tout parce que vous avez <10 000 leucocytes et >40 000 plaquettes. En d'autres termes, le risque est faible. Réponse correcte 2. L'option 3 a peut-être dérouté certaines personnes. La coagulopathie est l'une des principales causes de mortalité lors de l'induction de la promyélocytose, mais l'option 3 est totalement incorrecte. Nous ne pourrions qu'aggraver la situation en ajoutant de l'héparine !!!! Je ne sais pas ce qu'ils expliquent en ce moment dans les académies, mais il est clair qu'ils voulaient que vous sachiez que beaucoup de promyélocytes, depuis 3 ans, ne sont pas traités par chimiothérapie.", "type": "HÉMATOLOGIE", "options": {"1": "Si elle est asymptomatique, l'acide transrétinoïque (ATRA) sera mis en place et une surveillance en hôpital de jour sera recommandée.", "2": "Un traitement au trioxyde d'arsenic, à l'ATRA et une thérapie de soutien seront mis en place.", "3": "Il s'agit d'une leucémie myéloblastique de type M3. Une chimiothérapie et de l'héparine seront donc mises en place pour contrôler la coagulation intravasculaire disséminée.", "4": "Un traitement antibiotique doit être mis en place. Une fois la fièvre disparue, le traitement de la leucémie doit être mis en place.", "5": null}, "correct_option": 2, "explanations": {"1": {"exist": false, "char_ranges": [], "word_ranges": [], "text": ""}, "2": {"exist": true, "char_ranges": [[347, 488]], "word_ranges": [[56, 81]], "text": "Depuis 2017, le protocole PETHEMA pour la LPA chez les patients à risque faible ou intermédiaire (et/ou âgés de plus de 70 ans) est ATO+ATRA."}, "3": {"exist": true, "char_ranges": [[945, 1166]], "word_ranges": [[157, 189]], "text": "La coagulopathie est l'une des principales causes de mortalité lors de l'induction de la promyélocytose, mais l'option 3 est totalement incorrecte. Nous ne pourrions qu'aggraver la situation en ajoutant de l'héparine !!!!"}, "4": {"exist": false, "char_ranges": [], "word_ranges": [], "text": ""}, "5": {"exist": false, "char_ranges": [], "word_ranges": [], "text": ""}}} +{"id": 277, "year": 2016, "question_id_specific": 226, "full_question": "Dans le cadre d'un programme de dépistage du cancer du côlon, un patient de 52 ans subit une coloscopie. L'ensemble du côlon semble normal, à l'exception d'un polype pédiculé de 2 cm dans le sigma, qui est retiré à l'aide d'une anse diathermique. Les résultats anatomopathologiques indiquent un carcinome in situ limité à la tête du polype. La tomodensitométrie thoraco-abdominale est normale. Indiquez la conduite à tenir.", "full_answer": "Le carcinome in situ est délimité par la membrane basale, c'est-à-dire qu'il n'est pas encore invasif, et le traitement consiste donc en une polypectomie, ce qui est déjà fait. Par la suite, un suivi endoscopique (avec biopsies) doit être effectué à un intervalle de 3 à 6 mois en fonction des antécédents du patient, de son âge et des pathologies concomitantes.", "type": "SYSTÈME DIGESTIF", "options": {"1": "Résection segmentaire du côlon atteint.", "2": "Surveillance endoscopique périodique.", "3": "Résection locale de la base du polype.", "4": "Sigmoïdectomie et lymphadénectomie.", "5": null}, "correct_option": 2, "explanations": {"1": {"exist": false, "char_ranges": [], "word_ranges": [], "text": ""}, "2": {"exist": true, "char_ranges": [[191, 362]], "word_ranges": [[32, 61]], "text": "un suivi endoscopique (avec biopsies) doit être effectué à un intervalle de 3 à 6 mois en fonction des antécédents du patient, de son âge et des pathologies concomitantes."}, "3": {"exist": false, "char_ranges": [], "word_ranges": [], "text": ""}, "4": {"exist": false, "char_ranges": [], "word_ranges": [], "text": ""}, "5": {"exist": false, "char_ranges": [], "word_ranges": [], "text": ""}}} +{"id": 155, "year": 2012, "question_id_specific": 83, "full_question": "Homme de 62 ans ayant des antécédents d'hypertension artérielle traitée au captopril, d'ulcère duodénal et d'urolithiase à l'acide urique. Il a consulté pour une crise de podagre typique, semblable aux autres crises présentées au cours des deux dernières années. L'examen a révélé la présence de tophi dans les deux pavillons de l'oreille. Les analyses ont révélé un taux d'acide urique de 10,1 mg/dl et un taux de créatine de 1,5 mg/dl. Veuillez indiquer laquelle des réponses suivantes est FAUSSE en ce qui concerne votre traitement éventuel par l'allopurinol.", "full_answer": "L'allopurinol est toujours le traitement de choix, surtout en cas de goutte tophacée. De plus, dans ce cas, les uricosuriques pourraient aggraver l'urolithiase en augmentant l'uricosurie (désolé pour la redondance...).", "type": "RHEUMATOLOGIE", "options": {"1": "Il est utilisé comme second choix lorsque les uricosuriques ont échoué.", "2": "Leur utilisation à long terme est pratiquement obligatoire en raison de la présence de tophi.", "3": "Son introduction doit être retardée jusqu'à ce que la crise actuelle ait été résolue par des médicaments anti-inflammatoires ou de la colchine.", "4": "Il doit être introduit progressivement jusqu'à une dose permettant d'obtenir une uricémie inférieure à 6 mg/dL.", "5": "Au cours des premiers mois de traitement, il est conseillé d'ajouter de faibles doses de colchicine pour prévenir d'autres crises."}, "correct_option": 1, "explanations": {"1": {"exist": true, "char_ranges": [[0, 85]], "word_ranges": [[0, 13]], "text": "L'allopurinol est toujours le traitement de choix, surtout en cas de goutte tophacée."}, "2": {"exist": false, "char_ranges": [], "word_ranges": [], "text": ""}, "3": {"exist": false, "char_ranges": [], "word_ranges": [], "text": ""}, "4": {"exist": false, "char_ranges": [], "word_ranges": [], "text": ""}, "5": {"exist": false, "char_ranges": [], "word_ranges": [], "text": ""}}} +{"id": 373, "year": 2016, "question_id_specific": 86, "full_question": "L'ophtalmologie nous a adressé un homme de 31 ans qui nous a consultés pour une perte de vision et des céphalées progressives. L'examen physique a révélé une hémianopsie temporale et l'IRM hypophysaire a montré une tournure de 35 x 30 x 20 mm comprimant le chiasma et le sinus caverneux. Les résultats de l'étude fonctionnelle sont : cortisol 14 microg/dL, TSH 1,4 microU/ml, T4L 1,2 ng/dL, prolactine 480 microg/L (vn < 15), testostérone 160 ng/dL (vn 300-1200), FSH 1,2 U/I., (vn 5-15) et LH 2 U/L (vn 3-15). Quelle est l'approche thérapeutique initiale ?", "full_answer": "D'après ce qui est décrit, il s'agit d'un macroprolactinome, c'est-à-dire d'une tumeur hypophysaire de plus d'un centimètre qui sécrète de la prolactine et qui présente une insuffisance des autres hormones hypophysaires. Le traitement de première intention est l'administration d'agonistes dopaminergiques, mais lorsque la tumeur affecte le champ visuel, la chirurgie peut être l'option de choix. Dans ce cas, comme le sinus caverneux est soumis à la pression de la masse, le risque chirurgical augmente. Par conséquent, dans ce cas particulier, l'option chirurgicale semble plus risquée. En gardant cela à l'esprit et en tenant compte du fait qu'il s'agit d'une tumeur qui produit des niveaux élevés de prolactine, ce qui indique qu'elle répond probablement à un blocage pharmacologique, l'option la plus appropriée serait l'option 1, le traitement avec des agonistes dopaminergiques.", "type": "NEUROLOGIE", "options": {"1": "Agonistes dopaminergiques.", "2": "Chirurgie transsphénoïdale.", "3": "Radiothérapie externe ou radiochirurgie.", "4": "Traitement de l'hypogonadisme hypogonadotrope par la testostérone.", "5": null}, "correct_option": 1, "explanations": {"1": {"exist": true, "char_ranges": [[0, 306]], "word_ranges": [[0, 40]], "text": "D'après ce qui est décrit, il s'agit d'un macroprolactinome, c'est-à-dire d'une tumeur hypophysaire de plus d'un centimètre qui sécrète de la prolactine et qui présente une insuffisance des autres hormones hypophysaires. Le traitement de première intention est l'administration d'agonistes dopaminergiques,"}, "2": {"exist": true, "char_ranges": [[397, 588]], "word_ranges": [[55, 85]], "text": "Dans ce cas, comme le sinus caverneux est soumis à la pression de la masse, le risque chirurgical augmente. Par conséquent, dans ce cas particulier, l'option chirurgicale semble plus risquée."}, "3": {"exist": false, "char_ranges": [], "word_ranges": [], "text": ""}, "4": {"exist": false, "char_ranges": [], "word_ranges": [], "text": ""}, "5": {"exist": false, "char_ranges": [], "word_ranges": [], "text": ""}}} +{"id": 523, "year": 2021, "question_id_specific": 124, "full_question": "Une infection par le VIH a été diagnostiquée chez un patient de 45 ans il y a trois mois, avec des CD4 de 45 lymphocytes/µL et une charge virale VIH-1 de 500 000 copies/ml à l'époque. Il avait alors un test de Mantoux négatif. Il a commencé un traitement avec un inhibiteur de l'intégrase et deux inhibiteurs de la transcriptase inverse, et au bout d'un mois, il avait 25 000 copies et son taux de CD4 était remonté à 80/µL. Il a consulté pour une lymphadénopathie cervicale et de la fièvre depuis deux semaines. La ponction d'un des ganglions lymphatiques a révélé la présence de bacilles acido-alcoolo-résistants isolés et de granulomes épithélioïdes :", "full_answer": "Il s'agit d'un patient infecté par le VIH en situation d'immunodépression sévère (<50 CD4+) et avec une charge virale très élevée, qui a suivi l'évolution attendue du point de vue immunovirologique après le début du traitement (après 12 semaines, la charge virale a ostensiblement diminué - la réponse 2 ne serait pas correcte, rappelons qu'il est souhaitable qu'elle devienne négative 24 semaines après le début du traitement, bien que chez des patients avec des CV plus élevés comme les nôtres, cela puisse prendre plus de temps - et le nombre de CD4+ a commencé à augmenter). Un test de Mantoux négatif n'exclut pas la tuberculose (mauvaise réponse 4), car chez les patients souffrant d'immunosuppression sévère, il peut donner de faux négatifs. Le diagnostic de tuberculose ganglionnaire est immédiat dès la détection de \"bacilles acido-alcoolo-résistants isolés et de granulomes épithélioïdes\" dans la ponction d'un des ganglions lymphatiques. Cependant, compte tenu du profil du patient, ce processus aura été cliniquement démasqué au cours de la reconstitution immunologique et l'infection n'est pas la conséquence d'une récupération immunologique incomplète (réponse correcte 3).", "type": "LES MALADIES INFECTIEUSES", "options": {"1": "Il s'agit d'une tuberculose des ganglions lymphatiques liée à une récupération immunitaire incomplète.", "2": "Elle se manifeste par un échec thérapeutique et constitue une infection opportuniste.", "3": "Il s'agit d'une infection opportuniste démasquée dans le contexte de la reconstitution immunitaire.", "4": "Un test de Mantoux négatif exclut la tuberculose.", "5": null}, "correct_option": 3, "explanations": {"1": {"exist": true, "char_ranges": [[749, 1187]], "word_ranges": [[121, 178]], "text": "Le diagnostic de tuberculose ganglionnaire est immédiat dès la détection de \"bacilles acido-alcoolo-résistants isolés et de granulomes épithélioïdes\" dans la ponction d'un des ganglions lymphatiques. Cependant, compte tenu du profil du patient, ce processus aura été cliniquement démasqué au cours de la reconstitution immunologique et l'infection n'est pas la conséquence d'une récupération immunologique incomplète (réponse correcte 3)."}, "2": {"exist": true, "char_ranges": [[291, 578]], "word_ranges": [[46, 96]], "text": "la réponse 2 ne serait pas correcte, rappelons qu'il est souhaitable qu'elle devienne négative 24 semaines après le début du traitement, bien que chez des patients avec des CV plus élevés comme les nôtres, cela puisse prendre plus de temps - et le nombre de CD4+ a commencé à augmenter)."}, "3": {"exist": true, "char_ranges": [[749, 1187]], "word_ranges": [[121, 178]], "text": "Le diagnostic de tuberculose ganglionnaire est immédiat dès la détection de \"bacilles acido-alcoolo-résistants isolés et de granulomes épithélioïdes\" dans la ponction d'un des ganglions lymphatiques. Cependant, compte tenu du profil du patient, ce processus aura été cliniquement démasqué au cours de la reconstitution immunologique et l'infection n'est pas la conséquence d'une récupération immunologique incomplète (réponse correcte 3)."}, "4": {"exist": true, "char_ranges": [[579, 748]], "word_ranges": [[96, 121]], "text": "Un test de Mantoux négatif n'exclut pas la tuberculose (mauvaise réponse 4), car chez les patients souffrant d'immunosuppression sévère, il peut donner de faux négatifs."}, "5": {"exist": false, "char_ranges": [], "word_ranges": [], "text": ""}}} +{"id": 417, "year": 2018, "question_id_specific": 76, "full_question": "Une femme de 78 ans atteinte de démence et institutionnalisée a été amenée par ses soignants pour de fortes douleurs abdominales accompagnées d'une détérioration de l'état général et d'une distension abdominale. Les examens de laboratoire révèlent une leucocytose, un hématocrite élevé, une insuffisance rénale et une acidose métabolique. L'ECG révèle une fibrillation auriculaire. La tomodensitométrie abdominale montre des boucles œdémateuses de l'intestin grêle, avec une accumulation intestinale et portale. Le diagnostic le plus probable est le suivant :", "full_answer": "Il s'agit d'un cas typique d'ischémie mésentérique aiguë, qui doit être suspectée chez les patients âgés présentant des douleurs abdominales et des ballonnements d'apparition soudaine, des diarrhées ou des vomissements et qui présentent des facteurs de risque cardiovasculaire, en particulier une cardiopathie emboligène. Dans ce cas, les données biochimiques caractéristiques (leucocytose, acidose métabolique due à l'acide lactique, hémoconcentration due à la perte de liquide dans le troisième espace, etc.) et les examens d'imagerie (boucles dilatées, pneumatose, gaz au niveau portal, etc.) sont évocateurs d'une ischémie mésentérique.", "type": "CHIRURGIE GÉNÉRALE", "options": {"1": "Perforation d'un ulcère gastrique ou duodénal.", "2": "canal biliaire.", "3": "Néoplasie sigmoïdienne obstructive avec perforation.", "4": "Ischémie mésentérique.", "5": null}, "correct_option": 4, "explanations": {"1": {"exist": false, "char_ranges": [], "word_ranges": [], "text": ""}, "2": {"exist": false, "char_ranges": [], "word_ranges": [], "text": ""}, "3": {"exist": false, "char_ranges": [], "word_ranges": [], "text": ""}, "4": {"exist": true, "char_ranges": [[0, 321]], "word_ranges": [[0, 43]], "text": "Il s'agit d'un cas typique d'ischémie mésentérique aiguë, qui doit être suspectée chez les patients âgés présentant des douleurs abdominales et des ballonnements d'apparition soudaine, des diarrhées ou des vomissements et qui présentent des facteurs de risque cardiovasculaire, en particulier une cardiopathie emboligène."}, "5": {"exist": false, "char_ranges": [], "word_ranges": [], "text": ""}}} +{"id": 403, "year": 2016, "question_id_specific": 135, "full_question": "Au cours d'un match de tennis, un patient de 25 ans ressent une douleur intense au niveau du cou et de l'œil gauche. Le lendemain matin, il se réveille avec une sensation d'instabilité de la démarche et présente une ptose palpébrale de l'œil gauche et une anisocorie, la pupille gauche étant plus petite que la pupille droite. Le patient conserve une bonne acuité visuelle. Où situeriez-vous le plus probablement la lésion ?", "full_answer": "Le syndrome de Claude-Bernard-Horner est une vieille connaissance de l'examen MIR. Nous ne le voyons pas tous les jours en pratique clinique, mais il apparaît dans de nombreux examens. Il se caractérise par une ptose de la paupière supérieure et un myosis de l'œil. Ce phénomène est dû à un déficit de l'innervation sympathique du territoire orbital. Le système nerveux sympathique innerve le muscle de Müller, qui est un muscle accessoire du releveur de la paupière supérieure. Si le muscle de Müller cesse de fonctionner, la paupière s'abaisse légèrement. Il innerve également le muscle dilatateur de l'iris. Ainsi, le muscle sphincter, son antagoniste, ne s'oppose pas et l'équilibre de la pupille se déplace vers le myosis. Cela explique le ptosis et le myosis de l'œil gauche, et nous soupçonnons donc une maladie de Horner dans cet œil. Ils décrivent également ce qui a pu se produire : après un éventuel mouvement brusque lors d'une partie de tennis, elle a une douleur dans le cou et un syndrome de Horner. Il faut suspecter un anévrisme carotidien disséquant, qui est l'une des causes les plus connues (et les plus graves) du syndrome de Horner acquis. Le ganglion cervical supérieur est un ganglion du système nerveux sympathique situé dans la gaine de l'artère carotide, et c'est à ce niveau qu'il a été touché. Nous pouvons également essayer de répondre correctement à cette question en l'excluant. Une altération du troisième nerf crânien peut produire un ptosis, mais la pupille, si elle est touchée, doit être en mydriase. Une atteinte du troisième nerf crânien devrait également produire une forme d'ophtalmoplégie, avec diplopie. Une atteinte du chiasma optique produirait une perte de vision bilatérale (probablement une hémianopsie bitemporale). La déclaration nous apprend qu'il conserve une bonne acuité visuelle. D'autre part, il n'y aurait pas non plus de ptosis ni d'anisocorie. Enfin, une altération du bulbe rachidien produirait des symptômes neurologiques différents qui ne sont pas décrits, et il ne devrait pas y avoir de ptosis ni d'altérations pupillaires, car la troisième paire est située dans le mésencéphale et les noyaux sympathiques dans la moelle épinière.", "type": "OPHTALMOLOGIE (ECTOPIQUE)", "options": {"1": "III nerf crânien.", "2": "Chiasme optique.", "3": "Ganglion cervical supérieur.", "4": "Moelle épinière.", "5": null}, "correct_option": 3, "explanations": {"1": {"exist": true, "char_ranges": [[1411, 1537]], "word_ranges": [[232, 253]], "text": "Une altération du troisième nerf crânien peut produire un ptosis, mais la pupille, si elle est touchée, doit être en mydriase."}, "2": {"exist": true, "char_ranges": [[1647, 1902]], "word_ranges": [[267, 304]], "text": "Une atteinte du chiasma optique produirait une perte de vision bilatérale (probablement une hémianopsie bitemporale). La déclaration nous apprend qu'il conserve une bonne acuité visuelle. D'autre part, il n'y aurait pas non plus de ptosis ni d'anisocorie."}, "3": {"exist": true, "char_ranges": [[1015, 1322]], "word_ranges": [[169, 220]], "text": "Il faut suspecter un anévrisme carotidien disséquant, qui est l'une des causes les plus connues (et les plus graves) du syndrome de Horner acquis. Le ganglion cervical supérieur est un ganglion du système nerveux sympathique situé dans la gaine de l'artère carotide, et c'est à ce niveau qu'il a été touché."}, "4": {"exist": true, "char_ranges": [[1910, 2194]], "word_ranges": [[305, 349]], "text": "une altération du bulbe rachidien produirait des symptômes neurologiques différents qui ne sont pas décrits, et il ne devrait pas y avoir de ptosis ni d'altérations pupillaires, car la troisième paire est située dans le mésencéphale et les noyaux sympathiques dans la moelle épinière."}, "5": {"exist": false, "char_ranges": [], "word_ranges": [], "text": ""}}} +{"id": 42, "year": 2011, "question_id_specific": 150, "full_question": "Un garçon d'un an et demi s'est présenté au service des urgences avec des douleurs abdominales et une jaunisse. À l'examen, une masse abdominale a été palpée. Une échographie a révélé la présence d'un kyste de la voie biliaire commune. Quelle approche thérapeutique devons-nous adopter ?", "full_answer": "La réponse correcte est 4. Le kyste du canal cholédoque est une pathologie congénitale qui se manifeste cliniquement au-delà de la période néonatale. Une image de qualité qui délimite la lésion est suffisante pour décider de l'approche chirurgicale.", "type": "PÉDIATRIE", "options": {"1": "Une ponction percutanée pour lavage péritonéal sera effectuée pour vérifier que le kyste contient de la bile afin de laisser un drainage pour améliorer la douleur abdominale.", "2": "Une laparotomie exploratoire est indiquée et l'ensemble du canal biliaire est réséqué radicalement et remplacé par une anse intestinale.", "3": "Une ex-laparotomie est indiquée pour drainer le kyste et lorsque la dilatation se résorbe, le drain est retiré.", "4": "Une scintigraphie par cholangio-résonance sera effectuée pour délimiter le kyste et une laparotomie sera indiquée pour la résection du kyste et l'anastomose des voies biliaires.", "5": "Une étude scintigraphique HIDA est nécessaire pour délimiter le kyste et pouvoir effectuer un drainage percutané en toute sécurité."}, "correct_option": 4, "explanations": {"1": {"exist": false, "char_ranges": [], "word_ranges": [], "text": ""}, "2": {"exist": false, "char_ranges": [], "word_ranges": [], "text": ""}, "3": {"exist": false, "char_ranges": [], "word_ranges": [], "text": ""}, "4": {"exist": true, "char_ranges": [[27, 249]], "word_ranges": [[5, 38]], "text": "Le kyste du canal cholédoque est une pathologie congénitale qui se manifeste cliniquement au-delà de la période néonatale. Une image de qualité qui délimite la lésion est suffisante pour décider de l'approche chirurgicale."}, "5": {"exist": false, "char_ranges": [], "word_ranges": [], "text": ""}}} +{"id": 445, "year": 2018, "question_id_specific": 233, "full_question": "Femme de 45 ans. Deux grossesses antérieures avec des accouchements normaux (G2PN2). Saignements menstruels abondants depuis environ un an. L'échographie montre un myome sous-séreux de 2 cm qui est resté stable pendant plusieurs années. Une biopsie de l'endomètre a été effectuée et s'est révélée normale. La patiente a un taux d'hémoglobine de 10 g/dl malgré une ferrothérapie orale et le volume du flux menstruel n'a pas diminué malgré un traitement à l'acide tranexamique et à l'acide méfénamique. Lequel des traitements suivants est le traitement de premier choix chez cette patiente ?", "full_answer": "Selon la SEGO, chez les femmes périménopausées ayant des saignements menstruels abondants et/ou prolongés qui ont besoin d'une contraception, le DIU au lévonorgestrel (également appelé DIU MIRENA) est le premier choix.", "type": "GYNÉCOLOGIE ET OBSTÉTRIQUE", "options": {"1": "Hystérectomie totale conservant les appendices.", "2": "Contraceptifs oraux combinés.", "3": "Ablation de l'endomètre.", "4": "DIU au lévonorgestrel.", "5": null}, "correct_option": 4, "explanations": {"1": {"exist": false, "char_ranges": [], "word_ranges": [], "text": ""}, "2": {"exist": false, "char_ranges": [], "word_ranges": [], "text": ""}, "3": {"exist": false, "char_ranges": [], "word_ranges": [], "text": ""}, "4": {"exist": true, "char_ranges": [[0, 218]], "word_ranges": [[0, 31]], "text": "Selon la SEGO, chez les femmes périménopausées ayant des saignements menstruels abondants et/ou prolongés qui ont besoin d'une contraception, le DIU au lévonorgestrel (également appelé DIU MIRENA) est le premier choix."}, "5": {"exist": false, "char_ranges": [], "word_ranges": [], "text": ""}}} +{"id": 511, "year": 2021, "question_id_specific": 152, "full_question": "Une femme de 79 ans, hypertendue et traitée à l'olmésartan, est venue consulter pour une diarrhée aqueuse avec 4 à 6 selles par jour au cours des deux derniers mois. Il y a trois mois, elle a été traitée par des anti-inflammatoires non stéroïdiens pendant trois semaines pour une lombosciatique. Une gastroscopie et une coloscopie ont été effectuées, toutes deux sans altération macroscopique. Les biopsies duodénales étaient normales, tandis que les biopsies coliques montraient un infiltrat inflammatoire chronique de la lamina propria avec une bande irrégulière de collagène immédiatement sous l'épithélium de surface de la muqueuse d'une épaisseur supérieure à 10 mm et un nombre de lymphocytes intra-épithéliaux >20 pour 100 cellules épithéliales. Quel est le diagnostic le plus probable ?", "full_answer": "Les critères diagnostiques de la colite microscopique sont les suivants : a) diarrhée aqueuse non sanglante chronique ou intermittente b) muqueuse colique évaluée par coloscopie comme étant macroscopiquement normale ou proche de la normale c) résultats histopathologiques caractéristiques. Dans notre cas, le patient présentait des signes compatibles avec une colite collagénique (bande de collagène sous-jacente à l'épithélium supérieure à 10 microns) et une colite lymphocytaire (plus de 20 lymphocytes intra-épithéliaux). Le traitement de cette pathologie consiste en des corticostéroïdes topiques à faible biodisponibilité orale (principalement le budésonide). L'entéropathie à l'olmésartan présente un tableau similaire à celui de la maladie cœliaque et se caractérise par une biopsie duodénale avec atrophie villositaire, une inflammation de la muqueuse avec augmentation des lymphocytes intraépithéliaux et une cryptite. Dans ce cas, une biopsie duodénale normale est décrite, mais il serait intéressant de conserver ce concept pour les examens futurs.", "type": "DIGESTIF", "options": {"1": "Colite microscopique.", "2": "Entéropathie aux AINS.", "3": "Entéropathie associée à l'olmésartan.", "4": "Syndrome du côlon irritable.", "5": null}, "correct_option": 1, "explanations": {"1": {"exist": true, "char_ranges": [[306, 452]], "word_ranges": [[41, 61]], "text": "le patient présentait des signes compatibles avec une colite collagénique (bande de collagène sous-jacente à l'épithélium supérieure à 10 microns)"}, "2": {"exist": false, "char_ranges": [], "word_ranges": [], "text": ""}, "3": {"exist": true, "char_ranges": [[665, 927]], "word_ranges": [[87, 123]], "text": "L'entéropathie à l'olmésartan présente un tableau similaire à celui de la maladie cœliaque et se caractérise par une biopsie duodénale avec atrophie villositaire, une inflammation de la muqueuse avec augmentation des lymphocytes intraépithéliaux et une cryptite."}, "4": {"exist": false, "char_ranges": [], "word_ranges": [], "text": ""}, "5": {"exist": false, "char_ranges": [], "word_ranges": [], "text": ""}}} +{"id": 33, "year": 2011, "question_id_specific": 65, "full_question": "Un homme de 64 ans, chez qui on avait diagnostiqué une myasthénie grave il y a un an, sous traitement de stéroïdes à faible dose (3 mg/jour de deflazacort) et d'anticholinestérasiques, asymptomatique depuis 6 mois, a consulté pour de légères difficultés de déglutition et une diplopie nocturne depuis quelques jours. Il a été admis à l'hôpital pour suspicion de crise myasthénique et un traitement a été mis en place. Le lendemain, le patient appelle à 3 heures du matin car il fait un bruit étrange à l'inspiration, comme un ronflement léger, il dort profondément et est très en sueur, mais ne semble pas fatigué. Quelle est l'approche la plus correcte ?", "full_answer": "Il s'agit d'un patient présentant une myasthénie décompensée avec des symptômes bulbaires (dysphagie) et donc un risque d'atteinte des muscles respiratoires. Les symptômes respiratoires correspondent à un épuisement des muscles, d'abord inspiratoires puis accessoires (le patient n'apparaît pas fatigué ou tachypnéique car les muscles n'arrivent plus à faire face), évoluant vers un coma carbonique.", "type": "NEUROLOGIE ET NEUROCHIRURGIE", "options": {"1": "Rassurez l'infirmière et la famille en leur disant que le patient est un ronfleur habituel et qu'il dort paisiblement. Le patient doit être placé en décubitus latéral.", "2": "Prévenir les soins intensifs en cas de suspicion d'insuffisance respiratoire aiguë, afin d'évaluer la nécessité d'une intubation orotrachéale et d'une ventilation assistée.", "3": "Effectuer une étude polysomnographique pour exclure l'apnée du sommeil.", "4": "Diminuer la dose de stéroïdes ; si vous souffrez de myopathie stéroïdienne, elle s'améliorera.", "5": "Demander un scanner thoracique pour exclure un thymome compressif sur la trachée associé à une myasthénie."}, "correct_option": 2, "explanations": {"1": {"exist": false, "char_ranges": [], "word_ranges": [], "text": ""}, "2": {"exist": true, "char_ranges": [[0, 157]], "word_ranges": [[0, 21]], "text": "Il s'agit d'un patient présentant une myasthénie décompensée avec des symptômes bulbaires (dysphagie) et donc un risque d'atteinte des muscles respiratoires."}, "3": {"exist": false, "char_ranges": [], "word_ranges": [], "text": ""}, "4": {"exist": false, "char_ranges": [], "word_ranges": [], "text": ""}, "5": {"exist": false, "char_ranges": [], "word_ranges": [], "text": ""}}} +{"id": 129, "year": 2012, "question_id_specific": 81, "full_question": "Un homme de 78 ans consulte pour un an de troubles cognitifs progressifs avec des problèmes de mémoire et d'orientation. Sa famille signale des hallucinations visuelles récurrentes, des chutes occasionnelles et un ralentissement moteur marqué. Quel est le diagnostic le plus probable ?", "full_answer": "La démence avec hallucinations visuelles au début de l'évolution (dans les deux premières années) est très suggestive des corps de Lewy et constitue le symptôme directeur dans ces questions. Si l'on ajoute à cela les chutes et le ralentissement moteur évoquant le parkinsonisme, la question devient plus facile. Par conséquent, le numéro 3 est correct.", "type": "NEUROLOGIE ET NEUROCHIRURGIE", "options": {"1": "Démence vasculaire à infarctus multiples.", "2": "Démence de type Alzheimer.", "3": "Démence à corps de Lewy diffus.", "4": "Démence fronto-temporale.", "5": "Hydrocéphalie à pression normale."}, "correct_option": 3, "explanations": {"1": {"exist": false, "char_ranges": [], "word_ranges": [], "text": ""}, "2": {"exist": false, "char_ranges": [], "word_ranges": [], "text": ""}, "3": {"exist": true, "char_ranges": [[0, 352]], "word_ranges": [[0, 55]], "text": "La démence avec hallucinations visuelles au début de l'évolution (dans les deux premières années) est très suggestive des corps de Lewy et constitue le symptôme directeur dans ces questions. Si l'on ajoute à cela les chutes et le ralentissement moteur évoquant le parkinsonisme, la question devient plus facile. Par conséquent, le numéro 3 est correct."}, "4": {"exist": false, "char_ranges": [], "word_ranges": [], "text": ""}, "5": {"exist": false, "char_ranges": [], "word_ranges": [], "text": ""}}} +{"id": 503, "year": 2020, "question_id_specific": 80, "full_question": "Un garçon de 8 ans consulte pour l'apparition de poils pubiens à la base du pénis depuis 12 mois. Le testicule n'a pas augmenté de volume et le pénis n'a pas augmenté de taille. Il n'y a pas eu d'augmentation de la vitesse de croissance au cours de l'année écoulée. L'âge osseux est supérieur d'un an à l'âge chronologique. Quel serait le diagnostic le plus probable ?", "full_answer": "Chez les garçons, le début de la puberté est considéré comme normal à partir de l'âge de 9 ans, il s'agit donc dans ce cas d'une puberté précoce. Le début de la puberté est défini comme une augmentation du volume des testicules (supérieure ou égale à 4 cm3) ; en précisant que la taille des testicules n'a pas augmenté, nous pouvons exclure les options 1 et 2. La sécrétion excessive d'androgènes produite par une tumeur surrénalienne entraînerait une augmentation de la pilosité et de la taille du pénis, ce qui n'est pas le cas de notre patient. Par conséquent, l'option correcte est l'adrénarche précoce.", "type": "PÉDIATRIE", "options": {"1": "Puberté précoce centrale.", "2": "Puberté précoce périphérique.", "3": "Adrénarche précoce.", "4": "Tumeur surrénalienne.", "5": null}, "correct_option": 3, "explanations": {"1": {"exist": true, "char_ranges": [[361, 607]], "word_ranges": [[67, 104]], "text": "La sécrétion excessive d'androgènes produite par une tumeur surrénalienne entraînerait une augmentation de la pilosité et de la taille du pénis, ce qui n'est pas le cas de notre patient. Par conséquent, l'option correcte est l'adrénarche précoce."}, "2": {"exist": true, "char_ranges": [[361, 607]], "word_ranges": [[67, 104]], "text": "La sécrétion excessive d'androgènes produite par une tumeur surrénalienne entraînerait une augmentation de la pilosité et de la taille du pénis, ce qui n'est pas le cas de notre patient. Par conséquent, l'option correcte est l'adrénarche précoce."}, "3": {"exist": true, "char_ranges": [[361, 607]], "word_ranges": [[67, 104]], "text": "La sécrétion excessive d'androgènes produite par une tumeur surrénalienne entraînerait une augmentation de la pilosité et de la taille du pénis, ce qui n'est pas le cas de notre patient. Par conséquent, l'option correcte est l'adrénarche précoce."}, "4": {"exist": false, "char_ranges": [], "word_ranges": [], "text": ""}, "5": {"exist": false, "char_ranges": [], "word_ranges": [], "text": ""}}} +{"id": 300, "year": 2016, "question_id_specific": 106, "full_question": "Un homme de 32 ans, originaire du Cameroun, a consulté pour fièvre, toux et douleur dans l'hémithorax gauche depuis un mois. Il avait pris de l'amoxicilline-acide clavulanique pendant une semaine sans amélioration de ses symptômes. Une analyse de sang a révélé un taux de globules blancs de 8000/microL et une hémoglobine de 12,8 g/dL. La radiographie du thorax montre un épanchement pleural gauche localisé occupant un tiers de l'hémithorax. Une thoracentèse montre un liquide jaunâtre avec les caractéristiques suivantes : globules rouges 2000/uL, leucocytes 2500/uL, avec 90% de lymphocytes, protéines 4,9 g/dL, lactate déshydrogénase 550 U/L, glucose 67 mg/dL et pas de cellules malignes à la cytologie. Lequel des tests suivants serait le plus utile pour diagnostiquer la cause de l'épanchement pleural ?", "full_answer": "La détermination de l'ADA, dans le cas d'une pneumonie de longue durée qui ne s'améliore pas avec un traitement antibiotique, avec un épanchement avec des leucocytes abondants et une prédominance lymphocytaire, dans lequel la présence d'une néoplasie est exclue, avec des valeurs supérieures à 70 UI/L, doit nous amener à avoir un haut degré de suspicion d'étiologie tuberculeuse et à envoyer le liquide pleural en microbiologie pour une étude de la tuberculose.", "type": "LES MALADIES INFECTIEUSES", "options": {"1": "Une tomodensitométrie thoracique (CT scan).", "2": "Mesure du pH du liquide pleural.", "3": "Mesure de l'adénosine désaminase du liquide pleural.", "4": "Test de tuberculine.", "5": null}, "correct_option": 3, "explanations": {"1": {"exist": false, "char_ranges": [], "word_ranges": [], "text": ""}, "2": {"exist": false, "char_ranges": [], "word_ranges": [], "text": ""}, "3": {"exist": true, "char_ranges": [[0, 462]], "word_ranges": [[0, 72]], "text": "La détermination de l'ADA, dans le cas d'une pneumonie de longue durée qui ne s'améliore pas avec un traitement antibiotique, avec un épanchement avec des leucocytes abondants et une prédominance lymphocytaire, dans lequel la présence d'une néoplasie est exclue, avec des valeurs supérieures à 70 UI/L, doit nous amener à avoir un haut degré de suspicion d'étiologie tuberculeuse et à envoyer le liquide pleural en microbiologie pour une étude de la tuberculose."}, "4": {"exist": false, "char_ranges": [], "word_ranges": [], "text": ""}, "5": {"exist": false, "char_ranges": [], "word_ranges": [], "text": ""}}} +{"id": 434, "year": 2018, "question_id_specific": 119, "full_question": "Vétérinaire âgée de 38 ans, elle était chargée de surveiller les animaux sauvages et d'aider à l'accouchement du bétail domestique. Elle a commencé par présenter une forte fièvre accompagnée de frissons, de céphalées, de myalgies et d'une toux non productive, qu'elle a interprétée comme un syndrome grippal. Elle a présenté des douleurs thoraciques. La radiographie du thorax a montré des infiltrats pulmonaires bilatéraux dans les champs inférieurs. Un test sérologique a été effectué et a révélé des titres élevés d'anticorps dirigés contre les antigènes de phase I. Laquelle des affirmations suivantes est VRAIE ?", "full_answer": "Infection aiguë à Coxiella burnetii, se manifestant dans ce cas par une pneumonie. L'infection à Coxiella n'est pas transmise par les tiques. Le contact est le plus probable au moment de la naissance d'un animal d'élevage infecté, où la bactérie se propage dans les aérosols. Le traitement de choix est la doxycycline dans la phase aiguë (en cas de méningite, les quinolones sont préférables en raison de leur pénétration de la barrière hémato-encéphalique). L'association de doxycycline et d'hicroxychloroquine peut être utilisée, mais pour la fièvre Q chronique (où l'on observe des anticorps élevés contre les antigènes de phase I et où le risque de mortalité est plus élevé).", "type": "MALADIES INFECTIEUSES ET MICROBIOLOGIE", "options": {"1": "Elle est transmise par les tiques.", "2": "La doxycycline et l'hydroxychloroquine sont efficaces pour traiter les formes aiguës de la maladie.", "3": "Dans la forme aiguë, on observe également une élévation des anticorps dirigés contre les antigènes de la phase l.", "4": "La mortalité dans les formes aiguës est presque inexistante.", "5": null}, "correct_option": 4, "explanations": {"1": {"exist": true, "char_ranges": [[83, 141]], "word_ranges": [[13, 22]], "text": "L'infection à Coxiella n'est pas transmise par les tiques."}, "2": {"exist": true, "char_ranges": [[459, 679]], "word_ranges": [[73, 108]], "text": "L'association de doxycycline et d'hicroxychloroquine peut être utilisée, mais pour la fièvre Q chronique (où l'on observe des anticorps élevés contre les antigènes de phase I et où le risque de mortalité est plus élevé)."}, "3": {"exist": false, "char_ranges": [], "word_ranges": [], "text": ""}, "4": {"exist": false, "char_ranges": [], "word_ranges": [], "text": ""}, "5": {"exist": false, "char_ranges": [], "word_ranges": [], "text": ""}}} +{"id": 563, "year": 2022, "question_id_specific": 175, "full_question": "Un patient de 62 ans consulte pour avoir été en contact étroit ces dernières semaines avec une personne atteinte de tuberculose active. Le test de Mantoux est positif (12 mm). Ses antécédents médicaux comprennent un traitement par anticoagulants oraux (acénocoumarol) pour une thrombose veineuse profonde survenue il y a quelques mois. Il ne signale ni fièvre ni toux et est asymptomatique. La radiographie du thorax est normale. Lequel des traitements suivants est le plus conseillé ?", "full_answer": "En 2018, les CDC recommandent quatre schémas posologiques pour le traitement de l'infection tuberculeuse latente : l'isoniazide (INH), la rifapentine (RPT) ou la rifampicine (RIF). Tous les traitements sont efficaces. Les prestataires de soins de santé devraient prescrire les traitements les plus courts, qui sont plus pratiques, chaque fois que cela est possible. La combinaison la plus courte serait Isoniazide + Rifapentine pendant trois mois ou Rifampicine pendant quatre mois. Cependant, le fait qu'il prenne de l'acénocoumarol signifie que l'indication pour ce patient est l'isoniazide pendant 9 mois, car il y a une interaction entre les dérivés de la rifamycine (rifapentine et rifampicine).", "type": "MÉDECINE PRÉVENTIVE", "options": {"1": "Isoniazide, pendant neuf mois.", "2": "Rifampicine, pendant quatre mois.", "3": "Isoniazide et rifampicine pendant trois mois.", "4": "Isoniazide et rifapentine pendant trois mois.", "5": null}, "correct_option": 4, "explanations": {"1": {"exist": false, "char_ranges": [], "word_ranges": [], "text": ""}, "2": {"exist": false, "char_ranges": [], "word_ranges": [], "text": ""}, "3": {"exist": false, "char_ranges": [], "word_ranges": [], "text": ""}, "4": {"exist": false, "char_ranges": [], "word_ranges": [], "text": ""}, "5": {"exist": false, "char_ranges": [], "word_ranges": [], "text": ""}}} +{"id": 89, "year": 2012, "question_id_specific": 57, "full_question": "Un homme de 64 ans consulte pour une douleur au mollet droit lorsqu'il monte des collines et des escaliers et lorsqu'il marche sur un sol plat pendant une période prolongée. Il signale que la douleur devient si intense à l'effort qu'il est obligé de se lever, ce qui fait disparaître la douleur en quelques minutes. En ce qui concerne les antécédents personnels, le patient est un fumeur de 20 cigarettes par jour, diabétique sous metformine et hypertendu. À l'examen, on observe une bonne perfusion périphérique du pied droit, mais le patient présente une diminution du pouls pédieux. Parmi les examens complémentaires suivants, lequel devrait être demandé initialement pour diagnostiquer et évaluer la gravité de la maladie du patient ?", "full_answer": "Bien que le diagnostic de la maladie artérielle périphérique soit essentiellement clinique (dans le cas de ce patient, le tableau est très typique), il est possible d'évaluer objectivement la gravité de la maladie à l'aide de techniques non sanguines. La plus importante est l'indice cheville-brachiale. La réponse 2 est donc correcte. Les techniques d'imagerie (artériographie, angioCT ou angioMRI) sont utilisées lorsqu'une réparation chirurgicale de l'occlusion artérielle est envisagée.", "type": "CARDIOLOGIE ET CHIRURGIE VASCULAIRE", "options": {"1": "Artériographie des membres inférieurs.", "2": "Indice brachial de la cheville.", "3": "Tomographie axiale informatisée avec reconstruction vasculaire des membres inférieurs.", "4": "Épreuve d'effort sur tapis roulant.", "5": "Imagerie par résonance magnétique."}, "correct_option": 2, "explanations": {"1": {"exist": true, "char_ranges": [[336, 490]], "word_ranges": [[51, 68]], "text": "Les techniques d'imagerie (artériographie, angioCT ou angioMRI) sont utilisées lorsqu'une réparation chirurgicale de l'occlusion artérielle est envisagée."}, "2": {"exist": true, "char_ranges": [[149, 303]], "word_ranges": [[23, 45]], "text": "il est possible d'évaluer objectivement la gravité de la maladie à l'aide de techniques non sanguines. La plus importante est l'indice cheville-brachiale."}, "3": {"exist": false, "char_ranges": [], "word_ranges": [], "text": ""}, "4": {"exist": false, "char_ranges": [], "word_ranges": [], "text": ""}, "5": {"exist": false, "char_ranges": [], "word_ranges": [], "text": ""}}} +{"id": 586, "year": 2022, "question_id_specific": 75, "full_question": "Une femme de 30 ans demande une méthode contraceptive. Elle a subi une césarienne d'urgence il y a sept mois, donnant naissance à une petite fille de 3 550 g. Elle allaite exclusivement. Elle allaite exclusivement. Cochez la bonne réponse :", "full_answer": "Compte tenu de sa situation et du fait qu'elle est exclusivement allaitée, la meilleure chose à faire serait de lui prescrire uniquement des gestagènes.", "type": "OBSTÉTRIQUE ET GYNÉCOLOGIE", "options": {"1": "Elle explique que si elle est aménorrhéique, elle n'a pas besoin d'une autre méthode de contraception car l'allaitement suffit comme méthode de contraception.", "2": "Il l'informe que le stérilet au lévonogestrel est contre-indiqué parce qu'elle a subi une césarienne il y a moins d'un an.", "3": "Elle explique qu'elle pourrait utiliser une méthode hormonale à base de gestagène uniquement.", "4": "Il offre une contraception hormonale combinée.", "5": null}, "correct_option": 3, "explanations": {"1": {"exist": false, "char_ranges": [], "word_ranges": [], "text": ""}, "2": {"exist": false, "char_ranges": [], "word_ranges": [], "text": ""}, "3": {"exist": true, "char_ranges": [[0, 152]], "word_ranges": [[0, 24]], "text": "Compte tenu de sa situation et du fait qu'elle est exclusivement allaitée, la meilleure chose à faire serait de lui prescrire uniquement des gestagènes."}, "4": {"exist": false, "char_ranges": [], "word_ranges": [], "text": ""}, "5": {"exist": false, "char_ranges": [], "word_ranges": [], "text": ""}}} +{"id": 461, "year": 2018, "question_id_specific": 148, "full_question": "Une femme de 59 ans ayant récemment eu des crises d'épilepsie s'est présentée au service des urgences. L'examen a révélé des signes pyramidaux gauches et un œdème papillaire. L'IRM cérébrale montre une masse hémisphérique droite avec œdème, une déviation de la ligne médiane et des signes d'hernie tentorielle. Elle dit souffrir de céphalées depuis une semaine, dont l'intensité augmente progressivement. Parmi les caractéristiques suivantes associées aux céphalées, laquelle semble la plus probable pour cette patiente ?", "full_answer": "Il s'agit d'un mal de tête dans le contexte d'un œdème papillaire. Dans ce cas, l'œdème papillaire est dû à une augmentation de la pression intracrânienne causée par une tumeur. La céphalée due à l'augmentation de la pression intracrânienne est positionnelle (elle augmente avec les manœuvres de Valsalva et le fait de se pencher), de sorte que l'option 2 n'est pas correcte. En outre, il s'agit principalement d'un mal de tête matinal (l'option 1 est correcte) car la pression intracrânienne augmente naturellement le matin. Les migraines peuvent être associées à une photophobie ou une sonophobie (réponses 3 et 4). Mais cette céphalée n'est pas une migraine.", "type": "OPHTALMOLOGIE (ECTOPIQUE)", "options": {"1": "Prédominance matinale.", "2": "Elle ne change pas avec l'effort.", "3": "Photophobie.", "4": "Sonophobie.", "5": null}, "correct_option": 1, "explanations": {"1": {"exist": true, "char_ranges": [[376, 525]], "word_ranges": [[62, 84]], "text": "En outre, il s'agit principalement d'un mal de tête matinal (l'option 1 est correcte) car la pression intracrânienne augmente naturellement le matin."}, "2": {"exist": true, "char_ranges": [[178, 375]], "word_ranges": [[30, 62]], "text": "La céphalée due à l'augmentation de la pression intracrânienne est positionnelle (elle augmente avec les manœuvres de Valsalva et le fait de se pencher), de sorte que l'option 2 n'est pas correcte."}, "3": {"exist": true, "char_ranges": [[526, 661]], "word_ranges": [[84, 106]], "text": "Les migraines peuvent être associées à une photophobie ou une sonophobie (réponses 3 et 4). Mais cette céphalée n'est pas une migraine."}, "4": {"exist": true, "char_ranges": [[526, 661]], "word_ranges": [[84, 106]], "text": "Les migraines peuvent être associées à une photophobie ou une sonophobie (réponses 3 et 4). Mais cette céphalée n'est pas une migraine."}, "5": {"exist": false, "char_ranges": [], "word_ranges": [], "text": ""}}} +{"id": 547, "year": 2022, "question_id_specific": 125, "full_question": "Un homme de 23 ans, diagnostiqué avec le syndrome de Wolff-Parkinson-White, s'est présenté aux urgences avec des palpitations. L'ECG montre une tachycardie irrégulière à QRS larges avec une fréquence cardiaque de 205 bpm, compatible avec une fibrillation auriculaire préexcitée. Parmi les médicaments suivants, lequel vous semble approprié pour arrêter la tachycardie ?", "full_answer": "Procaïnamide. Le traitement de choix du syndrome de Wolff-Parkinson-White est la cardioversion à courant continu. Les médicaments ralentissant la fréquence cardiaque, couramment utilisés dans la fibrillation auriculaire, sont inefficaces, et la digoxine et les inhibiteurs calciques non dihydropyridiniques (par exemple, le vérapamil, le diltiazem) sont contre-indiqués car ils peuvent augmenter la fréquence ventriculaire et provoquer une fibrillation ventriculaire. Si la cardioversion n'est pas possible, il convient d'utiliser des médicaments qui prolongent la période réfractaire de la connexion accessoire. La procaïnamide ou l'amiodarone IV sont préférables, mais tout médicament antiarythmique de classe Ia, Ic ou III peut être utilisé.", "type": "CARDIOLOGIE", "options": {"1": "Verapamil.", "2": "Digoxine.", "3": "Adénosine.", "4": "Procaïnamide.", "5": null}, "correct_option": 4, "explanations": {"1": {"exist": true, "char_ranges": [[242, 467]], "word_ranges": [[30, 58]], "text": "la digoxine et les inhibiteurs calciques non dihydropyridiniques (par exemple, le vérapamil, le diltiazem) sont contre-indiqués car ils peuvent augmenter la fréquence ventriculaire et provoquer une fibrillation ventriculaire."}, "2": {"exist": true, "char_ranges": [[242, 467]], "word_ranges": [[30, 58]], "text": "la digoxine et les inhibiteurs calciques non dihydropyridiniques (par exemple, le vérapamil, le diltiazem) sont contre-indiqués car ils peuvent augmenter la fréquence ventriculaire et provoquer une fibrillation ventriculaire."}, "3": {"exist": true, "char_ranges": [[114, 238]], "word_ranges": [[15, 29]], "text": "Les médicaments ralentissant la fréquence cardiaque, couramment utilisés dans la fibrillation auriculaire, sont inefficaces,"}, "4": {"exist": true, "char_ranges": [[468, 665]], "word_ranges": [[58, 85]], "text": "Si la cardioversion n'est pas possible, il convient d'utiliser des médicaments qui prolongent la période réfractaire de la connexion accessoire. La procaïnamide ou l'amiodarone IV sont préférables,"}, "5": {"exist": false, "char_ranges": [], "word_ranges": [], "text": ""}}} +{"id": 193, "year": 2013, "question_id_specific": 161, "full_question": "Chez un nourrisson de 3 mois, toutes les manifestations cliniques suivantes permettent d'exclure la mucoviscidose SAUF :", "full_answer": "La déshydratation peut être due à une diarrhée d'autres étiologies. J'ai des doutes sur la réponse 3 car elle ne précise pas quelles sont les sécrétions impliquées. Il faut comprendre que la croissance de Pseudomona aeruginosa se fait dans la culture des sécrétions bronchiques. Car il est clair que l'indication d'effectuer un dosage du chlore dans la sueur (pour exclure la PCP) est la déshydratation HIPOnatrémique et non la déshydratation hypernatrémique comme l'indique la réponse.", "type": "PÉDIATRIE", "options": {"1": "Retard dans l'évacuation du méconium.", "2": "Goût salé de la peau.", "3": "Isolement des sécrétions de Pseudomonas aeruginosa.", "4": "Détérioration/rétention de la courbe de poids.", "5": "Déshydratation hypernatrémique."}, "correct_option": 5, "explanations": {"1": {"exist": false, "char_ranges": [], "word_ranges": [], "text": ""}, "2": {"exist": false, "char_ranges": [], "word_ranges": [], "text": ""}, "3": {"exist": false, "char_ranges": [], "word_ranges": [], "text": ""}, "4": {"exist": false, "char_ranges": [], "word_ranges": [], "text": ""}, "5": {"exist": true, "char_ranges": [[290, 486]], "word_ranges": [[47, 75]], "text": "clair que l'indication d'effectuer un dosage du chlore dans la sueur (pour exclure la PCP) est la déshydratation HIPOnatrémique et non la déshydratation hypernatrémique comme l'indique la réponse."}}} +{"id": 431, "year": 2018, "question_id_specific": 116, "full_question": "Un homme de 38 ans s'est présenté au service des urgences avec une fièvre supérieure à 39 ºC et une détérioration de l'état de conscience après avoir voyagé en Guinée équatoriale sans prophylaxie antipaludique. Les examens de laboratoire révèlent une créatinine de 3,4 mg/dl, une AST de 764 Ull et une ALT de 678 Ull. Le laboratoire signale la présence de Plasmodium falciparum dans le sang avec un taux de parasitémie de 6 %. Quel traitement mettriez-vous en place à ce stade ?", "full_answer": "Dans les cas graves d'infection à Plasmodium, le plus grand risque de mortalité survient dans les 24 heures suivant la présentation clinique, de sorte que le traitement doit être instauré immédiatement. Le traitement par artésunate par voie parentérale ou intramusculaire est plus indiqué dans les cas graves d'infection à Plasmodium chez les adultes, les enfants et les femmes enceintes.", "type": "MALADIES INFECTIEUSES ET MICROBIOLOGIE", "options": {"1": "Méfloquine par voie orale.", "2": "Artésunate par voie intraveineuse.", "3": "Doxycycline par voie orale.", "4": "Sulfate de quinine par voie orale.", "5": null}, "correct_option": 2, "explanations": {"1": {"exist": false, "char_ranges": [], "word_ranges": [], "text": ""}, "2": {"exist": true, "char_ranges": [[0, 388]], "word_ranges": [[0, 59]], "text": "Dans les cas graves d'infection à Plasmodium, le plus grand risque de mortalité survient dans les 24 heures suivant la présentation clinique, de sorte que le traitement doit être instauré immédiatement. Le traitement par artésunate par voie parentérale ou intramusculaire est plus indiqué dans les cas graves d'infection à Plasmodium chez les adultes, les enfants et les femmes enceintes."}, "3": {"exist": false, "char_ranges": [], "word_ranges": [], "text": ""}, "4": {"exist": false, "char_ranges": [], "word_ranges": [], "text": ""}, "5": {"exist": false, "char_ranges": [], "word_ranges": [], "text": ""}}} +{"id": 569, "year": 2022, "question_id_specific": 152, "full_question": "Un homme de 24 ans présente une microhématurie dysmorphique, une protéinurie de 3 g/24 h, un DFGe (CKD-EPI) de 85 ml/min et une perte auditive neurosensorielle. Il signale que sa grand-mère maternelle a dû être dialysée à l'âge de 70 ans et que sa mère et sa sœur cadette présentent toutes deux une microhématurie isolée. Parmi les maladies suivantes, laquelle est la plus probable ?", "full_answer": "Cas d'école, peu de choses à ajouter. Microhématurie, protéinurie et perte auditive neurosensorielle chez un jeune homme, avec des antécédents familiaux de femmes souffrant d'hématurie = syndrome d'Alport.", "type": "NEPHROLOGIE", "options": {"1": "Syndrome d'Alport.", "2": "Néphropathie à IgA.", "3": "La maladie de Fabry.", "4": "Polykystose rénale autosomique dominante.", "5": null}, "correct_option": 1, "explanations": {"1": {"exist": true, "char_ranges": [[38, 205]], "word_ranges": [[7, 28]], "text": "Microhématurie, protéinurie et perte auditive neurosensorielle chez un jeune homme, avec des antécédents familiaux de femmes souffrant d'hématurie = syndrome d'Alport."}, "2": {"exist": false, "char_ranges": [], "word_ranges": [], "text": ""}, "3": {"exist": false, "char_ranges": [], "word_ranges": [], "text": ""}, "4": {"exist": false, "char_ranges": [], "word_ranges": [], "text": ""}, "5": {"exist": false, "char_ranges": [], "word_ranges": [], "text": ""}}} +{"id": 501, "year": 2020, "question_id_specific": 76, "full_question": "Un garçon de 5 ans a été diagnostiqué avec une insuffisance rénale chronique. Il s'est présenté au service des urgences en raison de vomissements, d'un malaise général et de palpitations. Un électrocardiogramme a été réalisé avec une élévation de l'onde T et les tests sanguins ont montré un taux de potassium de 5,9 mEq/L. L'administration immédiate de gluconate de calcium a été décidée. Quel est l'objectif de ce traitement ?", "full_answer": "Le gluconate de calcium n'a aucun effet sur les niveaux de potassium dans le sang (les options 1, 2 et 3 sont donc exclues). Sa fonction est de diminuer l'excitabilité des myocytes cardiaques afin de réduire la probabilité de développer des troubles du rythme cardiaque.", "type": "PÉDIATRIE", "options": {"1": "Favorisent le mouvement du potassium de l'espace plasmatique vers l'espace intracellulaire.", "2": "Chélater le potassium circulant pour favoriser son élimination hépatique.", "3": "Chélater le potassium circulant pour favoriser son élimination rénale.", "4": "Antagoniser l'action du potassium sur la membrane cellulaire du myocarde.", "5": null}, "correct_option": 4, "explanations": {"1": {"exist": true, "char_ranges": [[0, 124]], "word_ranges": [[0, 24]], "text": "Le gluconate de calcium n'a aucun effet sur les niveaux de potassium dans le sang (les options 1, 2 et 3 sont donc exclues)."}, "2": {"exist": true, "char_ranges": [[0, 124]], "word_ranges": [[0, 24]], "text": "Le gluconate de calcium n'a aucun effet sur les niveaux de potassium dans le sang (les options 1, 2 et 3 sont donc exclues)."}, "3": {"exist": true, "char_ranges": [[0, 124]], "word_ranges": [[0, 24]], "text": "Le gluconate de calcium n'a aucun effet sur les niveaux de potassium dans le sang (les options 1, 2 et 3 sont donc exclues)."}, "4": {"exist": true, "char_ranges": [[125, 270]], "word_ranges": [[24, 45]], "text": "Sa fonction est de diminuer l'excitabilité des myocytes cardiaques afin de réduire la probabilité de développer des troubles du rythme cardiaque."}, "5": {"exist": false, "char_ranges": [], "word_ranges": [], "text": ""}}} +{"id": 242, "year": 2014, "question_id_specific": 111, "full_question": "Un homme de 50 ans souffrant de bronchite chronique a été admis pour une pneumonie avec une hémoculture positive pour Streptococcus pneumoniae, avec une CMI de 0,0125 mg/l pour la pénicilline. Un traitement a été instauré avec 2 millions de pénicilline toutes les 4 heures. Le cinquième jour, il avait toujours une fièvre de 38ºC. Laquelle des décisions suivantes vous semble correcte ?", "full_answer": "Le patient était un homme souffrant d'une pneumonie avec un germe isolé dans l'hémoculture et l'antibiogramme. D'après les données fournies, il s'agissait d'un pneumocoque sensible à la pénicilline (CMI<0,125). Un traitement a donc été mis en place avec de la pénicilline à fortes doses toutes les 4 heures, malgré la persistance de la fièvre. Si l'on analyse attentivement le cas, le traitement devrait être efficace puisque le germe est totalement sensible au médicament et que le dosage est adéquat (il faut donc maintenir en permanence des concentrations élevées de médicament, car l'efficacité des bêta-lactamines dépend du temps et les concentrations d'antibiotiques dans le sang doivent être supérieures à la CMI en permanence pour obtenir une efficacité et éviter la production de résistance). Dans un tel cas, que se passe-t-il ? Il est probable que, bien que l'antibiotique soit le bon, il n'atteint pas le foyer infectieux de manière adéquate parce qu'un empyème s'est probablement formé. Il convient d'exclure l'existence d'un empyème et de le drainer en même temps que l'on poursuit le traitement antibiotique. Si le dosage n'avait pas été adéquat et qu'une résistance aux médicaments était possible, une option aurait pu être de traiter avec de la ceftriaxone à fortes doses, étant donné que le streptocoque ne produit pas de bêta-lactamase et que la résistance aux bêta-lactamines est produite par un autre mécanisme (l'amoxicilline-clavulanate ne serait d'aucune utilité). Avec les quinolones, nous n'augmentons pas le spectre et il est clair que la mauvaise évolution du patient n'est pas seulement un problème de temps.", "type": "LES MALADIES INFECTIEUSES", "options": {"1": "Je changerais le traitement pour la ceftriaxone en raison de sa plus grande efficacité.", "2": "J'ajouterais une quinolone au traitement.", "3": "Je passerais à l'amoxicilline/clavulanate.", "4": "J'exclurais la présence d'un empyème pleural.", "5": "Je poursuivrais le même traitement, en supposant qu'il s'agit simplement d'un problème de temps."}, "correct_option": 4, "explanations": {"1": {"exist": true, "char_ranges": [[1124, 1354]], "word_ranges": [[174, 212]], "text": "Si le dosage n'avait pas été adéquat et qu'une résistance aux médicaments était possible, une option aurait pu être de traiter avec de la ceftriaxone à fortes doses, étant donné que le streptocoque ne produit pas de bêta-lactamase"}, "2": {"exist": true, "char_ranges": [[1489, 1637]], "word_ranges": [[229, 254]], "text": "Avec les quinolones, nous n'augmentons pas le spectre et il est clair que la mauvaise évolution du patient n'est pas seulement un problème de temps."}, "3": {"exist": true, "char_ranges": [[1362, 1488]], "word_ranges": [[214, 229]], "text": "la résistance aux bêta-lactamines est produite par un autre mécanisme (l'amoxicilline-clavulanate ne serait d'aucune utilité)."}, "4": {"exist": true, "char_ranges": [[860, 1123]], "word_ranges": [[134, 174]], "text": "bien que l'antibiotique soit le bon, il n'atteint pas le foyer infectieux de manière adéquate parce qu'un empyème s'est probablement formé. Il convient d'exclure l'existence d'un empyème et de le drainer en même temps que l'on poursuit le traitement antibiotique."}, "5": {"exist": false, "char_ranges": [], "word_ranges": [], "text": ""}}} +{"id": 487, "year": 2020, "question_id_specific": 68, "full_question": "Quels conseils génétiques et reproductifs donneriez-vous à une femme de 30 ans qui consulte parce qu'elle est porteuse d'une prémutation du gène FMR1, responsable du syndrome de l'X fragile, et qu'elle souhaite avoir des enfants ? Cochez la bonne réponse :", "full_answer": "Un autre classique du MIR, posé en 2019 et 2017, le syndrome de l'X fragile. Toujours sur le même concept que la question 44, le phénomène d'anticipation génétique. Les maladies à expansion de triplet, comme celle-ci, peuvent présenter le phénomène d'anticipation selon lequel les femmes avec des prémutations (entre 55-200 répétitions) peuvent avoir des enfants avec des mutations complètes (plus de 200 répétitions du triplet) et des manifestations cliniques plus évidentes, plus sévères et/ou plus précoces, tant chez l'homme que chez la femme (syndrome de l'X fragile, insuffisance ovarienne précoce, tremblements/ataxie associés à l'X fragile).", "type": "GENETIQUE", "options": {"1": "Tous leurs fils seront porteurs et manifesteront donc la maladie.", "2": "Il existe un phénomène d'anticipation génétique, qui fait que leur progéniture présentera des symptômes plus précoces et plus graves.", "3": "Le diagnostic génétique préimplantatoire pour la sélection du sexe des embryons par hybridation fluorescente in situ (FISH) est la meilleure option pour obtenir une progéniture saine.", "4": "50 % de leurs filles seront porteuses, mais ne présenteront pas de manifestations cliniques de la maladie.", "5": null}, "correct_option": 2, "explanations": {"1": {"exist": false, "char_ranges": [], "word_ranges": [], "text": ""}, "2": {"exist": true, "char_ranges": [[165, 649]], "word_ranges": [[28, 95]], "text": "Les maladies à expansion de triplet, comme celle-ci, peuvent présenter le phénomène d'anticipation selon lequel les femmes avec des prémutations (entre 55-200 répétitions) peuvent avoir des enfants avec des mutations complètes (plus de 200 répétitions du triplet) et des manifestations cliniques plus évidentes, plus sévères et/ou plus précoces, tant chez l'homme que chez la femme (syndrome de l'X fragile, insuffisance ovarienne précoce, tremblements/ataxie associés à l'X fragile)."}, "3": {"exist": false, "char_ranges": [], "word_ranges": [], "text": ""}, "4": {"exist": false, "char_ranges": [], "word_ranges": [], "text": ""}, "5": {"exist": false, "char_ranges": [], "word_ranges": [], "text": ""}}} +{"id": 364, "year": 2016, "question_id_specific": 176, "full_question": "Femme de 78 ans ayant des antécédents de diabète de type 2 traité par antidiabétiques oraux, d'HTN traitée par bêta-bloquants et inhibiteurs de l'ECA, et d'insuffisance cardiaque congestive de grade 1 selon la NYHA avec une FEVG de 48 %, et actuellement asymptomatique. Statut fonctionnel : ECOG 0. Antécédents de petites masses dans le cou depuis 2 ans. Biopsie d'une adénopathie cervicale : lymphome folliculaire de grade 2. Étude d'extension : Hb 12 g/dL, leucocytes 6 900/microL (neutrophiles 60%, lymphocytes 27%, monocytes 6%, éosinophiles 4%, basophiles 4%), plaquettes 220 000/microL. MO : infiltré par un lymphome folliculaire. Créatinine 1,5 mg/dL, LDH 235 U/L, Beta2 microglobuline 2,1 microg/mL. CT : adénopathies de moins de 3 cm dans les territoires cervical, axillaire, rétropéritonéal, iliaque et inguinal ; foie et rate normaux. Lequel des traitements suivants est le plus approprié ?", "full_answer": "La question de savoir s'il faut commencer le Rituximab avec un autre médicament cytostatique ou en monothérapie peut faire l'objet d'un débat, mais étant donné l'absence de bénéfice en termes de survie en cas d'initiation précoce du traitement, les lignes directrices du NCCN recommandent d'attendre et de voir, à moins que les critères du GELF ne soient remplis pour initier le traitement : Critères du GELF (Groupe d'étude sur le lymphome folliculaire) : - Atteinte de ≥3 aires ganglionnaires, chacune d'un diamètre ≥ 3 cm. - Toute masse ganglionnaire ou extranodale d'un diamètre ≥ 7 cm. - Symptômes B. - Splénomégalie. - Épanchement pleural ou ascite. - Cytopénie (leucocytes < 1,0 x 109 / L et/ou plaquettes < 100 x 109 / L). - Leucémie (> 5,0 x 109 /L de cellules malignes).", "type": "HÉMATOLOGIE", "options": {"1": "Rituximab-CHOP (cyclophosphamide, adriamycine, vincristine, prednisone).", "2": "Rituximab-CVP (cyclophosphamide, vincristine, prednisone).", "3": "Ne pas traiter et surveiller (attendre et voir).", "4": "Rituxirnab-Bendamustine.", "5": null}, "correct_option": 3, "explanations": {"1": {"exist": true, "char_ranges": [[0, 311]], "word_ranges": [[0, 48]], "text": "La question de savoir s'il faut commencer le Rituximab avec un autre médicament cytostatique ou en monothérapie peut faire l'objet d'un débat, mais étant donné l'absence de bénéfice en termes de survie en cas d'initiation précoce du traitement, les lignes directrices du NCCN recommandent d'attendre et de voir,"}, "2": {"exist": true, "char_ranges": [[0, 311]], "word_ranges": [[0, 48]], "text": "La question de savoir s'il faut commencer le Rituximab avec un autre médicament cytostatique ou en monothérapie peut faire l'objet d'un débat, mais étant donné l'absence de bénéfice en termes de survie en cas d'initiation précoce du traitement, les lignes directrices du NCCN recommandent d'attendre et de voir,"}, "3": {"exist": true, "char_ranges": [[0, 311]], "word_ranges": [[0, 48]], "text": "La question de savoir s'il faut commencer le Rituximab avec un autre médicament cytostatique ou en monothérapie peut faire l'objet d'un débat, mais étant donné l'absence de bénéfice en termes de survie en cas d'initiation précoce du traitement, les lignes directrices du NCCN recommandent d'attendre et de voir,"}, "4": {"exist": true, "char_ranges": [[0, 311]], "word_ranges": [[0, 48]], "text": "La question de savoir s'il faut commencer le Rituximab avec un autre médicament cytostatique ou en monothérapie peut faire l'objet d'un débat, mais étant donné l'absence de bénéfice en termes de survie en cas d'initiation précoce du traitement, les lignes directrices du NCCN recommandent d'attendre et de voir,"}, "5": {"exist": false, "char_ranges": [], "word_ranges": [], "text": ""}}} +{"id": 90, "year": 2012, "question_id_specific": 32, "full_question": "On diagnostique chez une femme de 35 ans un cancer du côlon localisé dans l'angle hépatique. Elle a des antécédents familiaux de cancer du côlon chez sa mère, une tante âgée de 45 ans et un grand-père. L'intervention chirurgicale la plus acceptée est :", "full_answer": "L'énoncé des antécédents familiaux indique qu'il s'agit d'un cas de cancer colorectal héréditaire sans polypose (syndrome de Lynch). Les données relatives à l'âge et à la localisation droite nous conduisent à la réponse 4 : colectomie totale avec anastomose iléorectale. En cas de chirurgie conventionnelle de type hémicolectomie droite, les risques de récidive sont supérieurs à 30 %. L'ablation du rectum n'est pas nécessaire à titre prophylactique.", "type": "CHIRURGIE GÉNÉRALE", "options": {"1": "Hémicolectomie droite.", "2": "Hémicolectomie droite élargie.", "3": "Colectomie subtotale.", "4": "Colectomie totale avec anatomie iléorectale.", "5": "Colectomie totale avec anatomie iléoanale."}, "correct_option": 4, "explanations": {"1": {"exist": false, "char_ranges": [], "word_ranges": [], "text": ""}, "2": {"exist": false, "char_ranges": [], "word_ranges": [], "text": ""}, "3": {"exist": false, "char_ranges": [], "word_ranges": [], "text": ""}, "4": {"exist": true, "char_ranges": [[133, 270]], "word_ranges": [[18, 40]], "text": "Les données relatives à l'âge et à la localisation droite nous conduisent à la réponse 4 : colectomie totale avec anastomose iléorectale."}, "5": {"exist": false, "char_ranges": [], "word_ranges": [], "text": ""}}} +{"id": 528, "year": 2021, "question_id_specific": 134, "full_question": "Femme de 53 ans ayant des antécédents personnels d'obésité et de migraine. Sa tension artérielle a été prise à plusieurs reprises chez le médecin et l'infirmière, avec des chiffres inférieurs à 140/90 mmHg. Cependant, elle a acheté un appareil de mesure de la pression artérielle homologué et a appris à l'utiliser correctement. Le patient se présente avec des mesures de tension artérielle effectuées à domicile pendant plusieurs semaines et dont les valeurs sont supérieures à 140/90 mmHg. Cochez la bonne réponse :", "full_answer": "Le patient décrit correspond à la définition de l'HT masquée, avec des chiffres normaux en consultation et une élévation de l'AMPA ou du MAPA (option 2 correcte). Il ne s'agit pas d'une HT secondaire car aucune étude n'a été réalisée pour la confirmer (option 1 incorrecte). Il ne s'agit pas d'une HT isolée car les valeurs élevées sont maintenues pendant plusieurs semaines (option 3 incorrecte). Il ne s'agit pas d'une HT réfractaire car elle n'a pas encore été traitée (option 4 incorrecte).", "type": "NEPHROLOGIE", "options": {"1": "Hypertension artérielle secondaire.", "2": "Il souffre d'une hypertension masquée.", "3": "Hypertension clinique isolée.", "4": "Il souffre d'hypertension artérielle réfractaire.", "5": null}, "correct_option": 2, "explanations": {"1": {"exist": true, "char_ranges": [[163, 494]], "word_ranges": [[27, 82]], "text": "Il ne s'agit pas d'une HT secondaire car aucune étude n'a été réalisée pour la confirmer (option 1 incorrecte). Il ne s'agit pas d'une HT isolée car les valeurs élevées sont maintenues pendant plusieurs semaines (option 3 incorrecte). Il ne s'agit pas d'une HT réfractaire car elle n'a pas encore été traitée (option 4 incorrecte)."}, "2": {"exist": true, "char_ranges": [[0, 162]], "word_ranges": [[0, 27]], "text": "Le patient décrit correspond à la définition de l'HT masquée, avec des chiffres normaux en consultation et une élévation de l'AMPA ou du MAPA (option 2 correcte)."}, "3": {"exist": true, "char_ranges": [[275, 397]], "word_ranges": [[46, 65]], "text": "Il ne s'agit pas d'une HT isolée car les valeurs élevées sont maintenues pendant plusieurs semaines (option 3 incorrecte)."}, "4": {"exist": true, "char_ranges": [[398, 494]], "word_ranges": [[65, 82]], "text": "Il ne s'agit pas d'une HT réfractaire car elle n'a pas encore été traitée (option 4 incorrecte)."}, "5": {"exist": false, "char_ranges": [], "word_ranges": [], "text": ""}}} +{"id": 583, "year": 2022, "question_id_specific": 70, "full_question": "Une femme de 43 ans porteuse d'un dispositif intra-utérin au lévonorgestrel a consulté pour manque de repos nocturne avec apparition de chaleur et de palpitations. Les examens de laboratoire montrent une T4 libre de 10,5 pmol/L (9-19), une thyrotropine de 2,1 mUI/L (0,30-5,00), une FSH de 95,6 UI/L (1,38-16,7), une LH de 21 UI/L (2,4-9,3), un œstradiol < 0,07 nmol/L (0,07-1,14). Quel est le traitement le plus approprié ?", "full_answer": "Le profil hormonal rapporté se situe dans la plage de la ménopause (FSH > 20 ; œstradiol < 5) et la patiente présente des symptômes vasomoteurs systémiques. Par conséquent, l'administration systémique d'œstrogènes serait plus appropriée. Pour compenser, elle dispose déjà d'un progestatif utérin local (DIU au lévonorgestrel).", "type": "OBSTÉTRIQUE ET GYNÉCOLOGIE", "options": {"1": "Benzodiazépines.", "2": "Œstrogènes vaginaux.", "3": "Œstrogènes et progestatifs oraux en régime continu.", "4": "Œstrogènes transdermiques en régime continu.", "5": null}, "correct_option": 4, "explanations": {"1": {"exist": false, "char_ranges": [], "word_ranges": [], "text": ""}, "2": {"exist": false, "char_ranges": [], "word_ranges": [], "text": ""}, "3": {"exist": false, "char_ranges": [], "word_ranges": [], "text": ""}, "4": {"exist": true, "char_ranges": [[0, 156]], "word_ranges": [[0, 27]], "text": "Le profil hormonal rapporté se situe dans la plage de la ménopause (FSH > 20 ; œstradiol < 5) et la patiente présente des symptômes vasomoteurs systémiques."}, "5": {"exist": false, "char_ranges": [], "word_ranges": [], "text": ""}}} +{"id": 195, "year": 2013, "question_id_specific": 163, "full_question": "Un garçon de 15 mois a été amené en urgence au centre de santé car, après avoir mangé une bouchée d'omelette pendant le dîner, il a soudainement présenté : des rougeurs faciales, principalement péribuccales, des lésions hémorragiques sur le tronc et les extrémités et une toux. A son arrivée au centre, il est conscient et, en plus des éléments précédents, on observe : une oppression suprasternale, une rhinorrhée aqueuse abondante, une hypoventilation bilatérale sans respiration sifflante et un remplissage capillaire inférieur à 2 secondes. Parmi les affirmations suivantes, cochez la réponse qui convient :", "full_answer": "Vous décrivez une anaphylaxie à l'œuf, une affection potentiellement mortelle. Le traitement de choix est l'adrénaline intramusculaire, et de manière efficace, sans perdre trop de temps. La première chose à faire est de donner de l'adrénaline INTRAMUSCULAIRE MAINTENANT ! La voie veineuse, les corticoïdes, les antihistaminiques, etc. viendront plus tard.", "type": "PÉDIATRIE", "options": {"1": "La priorité est de poser une voie veineuse.", "2": "La méthylprednisolone intramusculaire est le traitement de choix.", "3": "Il s'agit d'une urticaire associée à l'asthme qui doit être traitée par des antihistaminiques et des bronchodilatateurs inhalés.", "4": "Il faut conseiller aux parents d'envoyer l'enfant au service des urgences d'un hôpital.", "5": "L'adrénaline intramusculaire doit être administrée sans délai."}, "correct_option": 5, "explanations": {"1": {"exist": true, "char_ranges": [[187, 355]], "word_ranges": [[26, 50]], "text": "La première chose à faire est de donner de l'adrénaline INTRAMUSCULAIRE MAINTENANT ! La voie veineuse, les corticoïdes, les antihistaminiques, etc. viendront plus tard."}, "2": {"exist": true, "char_ranges": [[187, 355]], "word_ranges": [[26, 50]], "text": "La première chose à faire est de donner de l'adrénaline INTRAMUSCULAIRE MAINTENANT ! La voie veineuse, les corticoïdes, les antihistaminiques, etc. viendront plus tard."}, "3": {"exist": true, "char_ranges": [[187, 355]], "word_ranges": [[26, 50]], "text": "La première chose à faire est de donner de l'adrénaline INTRAMUSCULAIRE MAINTENANT ! La voie veineuse, les corticoïdes, les antihistaminiques, etc. viendront plus tard."}, "4": {"exist": true, "char_ranges": [[187, 355]], "word_ranges": [[26, 50]], "text": "La première chose à faire est de donner de l'adrénaline INTRAMUSCULAIRE MAINTENANT ! La voie veineuse, les corticoïdes, les antihistaminiques, etc. viendront plus tard."}, "5": {"exist": true, "char_ranges": [[0, 186]], "word_ranges": [[0, 26]], "text": "Vous décrivez une anaphylaxie à l'œuf, une affection potentiellement mortelle. Le traitement de choix est l'adrénaline intramusculaire, et de manière efficace, sans perdre trop de temps."}}} +{"id": 84, "year": 2012, "question_id_specific": 49, "full_question": "Un homme de 75 ans, hypertendu et dyslipidémique, avec des antécédents d'insuffisance cardiaque due à un dysfonctionnement systolique du ventricule gauche (FE < 30 %). Il s'est présenté à notre clinique en raison de l'aggravation de sa dyspnée habituelle au cours des dernières semaines. Sa tension artérielle systolique était de 160/95 mmHg et sa fréquence cardiaque de 65 bpm. L'examen physique ne révèle pas de crépitants et la pression veineuse jugulaire est normale. Il présente un dosage de la créatinine de 3,7 mg/dl avec des ions dans la normale. Parmi les médicaments suivants, lequel est le plus approprié pour améliorer le pronostic de son insuffisance cardiaque ?", "full_answer": "Chez un patient souffrant de CVRF multiples avec insuffisance cardiaque et rénale concomitante (probablement d'étiologie multiple : hypoperfusion rénale, artériosclérose...), nous devons être très prudents lors de l'utilisation de diurétiques et d'inhibiteurs de l'ECA/ARA-II, car nous pouvons aggraver la fonction rénale et/ou augmenter les taux de potassium (parmi les diurétiques, la spironolactone et l'éplérénone sont \"épargnant le potassium\"). Par conséquent, dans ce cas, nous utiliserons le bisoprolol.", "type": "CARDIOLOGIE ET CHIRURGIE VASCULAIRE", "options": {"1": "Enalapril.", "2": "Losartan.", "3": "Spironolactone.", "4": "Bisoprolol.", "5": "Eplérénone."}, "correct_option": 4, "explanations": {"1": {"exist": true, "char_ranges": [[0, 449]], "word_ranges": [[0, 58]], "text": "Chez un patient souffrant de CVRF multiples avec insuffisance cardiaque et rénale concomitante (probablement d'étiologie multiple : hypoperfusion rénale, artériosclérose...), nous devons être très prudents lors de l'utilisation de diurétiques et d'inhibiteurs de l'ECA/ARA-II, car nous pouvons aggraver la fonction rénale et/ou augmenter les taux de potassium (parmi les diurétiques, la spironolactone et l'éplérénone sont \"épargnant le potassium\")."}, "2": {"exist": true, "char_ranges": [[0, 449]], "word_ranges": [[0, 58]], "text": "Chez un patient souffrant de CVRF multiples avec insuffisance cardiaque et rénale concomitante (probablement d'étiologie multiple : hypoperfusion rénale, artériosclérose...), nous devons être très prudents lors de l'utilisation de diurétiques et d'inhibiteurs de l'ECA/ARA-II, car nous pouvons aggraver la fonction rénale et/ou augmenter les taux de potassium (parmi les diurétiques, la spironolactone et l'éplérénone sont \"épargnant le potassium\")."}, "3": {"exist": true, "char_ranges": [[0, 449]], "word_ranges": [[0, 58]], "text": "Chez un patient souffrant de CVRF multiples avec insuffisance cardiaque et rénale concomitante (probablement d'étiologie multiple : hypoperfusion rénale, artériosclérose...), nous devons être très prudents lors de l'utilisation de diurétiques et d'inhibiteurs de l'ECA/ARA-II, car nous pouvons aggraver la fonction rénale et/ou augmenter les taux de potassium (parmi les diurétiques, la spironolactone et l'éplérénone sont \"épargnant le potassium\")."}, "4": {"exist": false, "char_ranges": [], "word_ranges": [], "text": ""}, "5": {"exist": true, "char_ranges": [[0, 449]], "word_ranges": [[0, 58]], "text": "Chez un patient souffrant de CVRF multiples avec insuffisance cardiaque et rénale concomitante (probablement d'étiologie multiple : hypoperfusion rénale, artériosclérose...), nous devons être très prudents lors de l'utilisation de diurétiques et d'inhibiteurs de l'ECA/ARA-II, car nous pouvons aggraver la fonction rénale et/ou augmenter les taux de potassium (parmi les diurétiques, la spironolactone et l'éplérénone sont \"épargnant le potassium\")."}}} +{"id": 427, "year": 2018, "question_id_specific": 95, "full_question": "Une femme de 45 ans s'est présentée au service des urgences avec des symptômes de confusion accompagnés de dysphonie. À l'examen, elle présentait une masse palpable au niveau du cou et un test sanguin a révélé un taux de calcium plasmatique de 15 mg/dl (normal jusqu'à 10,2 mg/dl). Au vu de ces résultats, lequel des diagnostics suivants doit être suspecté ?", "full_answer": "Masse palpable dans le cou : suspecter une pathologie thyroïdienne/parathyroïdienne. Elle est associée à une hypercalcémie, ce qui exclut l'option d'un cancer médullaire de la thyroïde (qui n'affecte pas le niveau de la calcémie). Pour penser à une MEN de type I, d'autres pathologies (tumeurs hypophysaires, pancréatiques ou duodénales) doivent être associées. La présence d'une calcémie supérieure à 13 mgr/dl, associée à une dysphonie, oriente vers une tumeur maligne (calcémie plus élevée et infiltration), ce qui nous amène à l'option 1.", "type": "ENDOCRINOLOGIE", "options": {"1": "Carcinome de la parathyroïde.", "2": "Carcinome médullaire de la thyroïde.", "3": "MEN type I.", "4": "Adénome parathyroïdien.", "5": null}, "correct_option": 1, "explanations": {"1": {"exist": true, "char_ranges": [[362, 542]], "word_ranges": [[52, 81]], "text": "La présence d'une calcémie supérieure à 13 mgr/dl, associée à une dysphonie, oriente vers une tumeur maligne (calcémie plus élevée et infiltration), ce qui nous amène à l'option 1."}, "2": {"exist": true, "char_ranges": [[85, 230]], "word_ranges": [[10, 34]], "text": "Elle est associée à une hypercalcémie, ce qui exclut l'option d'un cancer médullaire de la thyroïde (qui n'affecte pas le niveau de la calcémie)."}, "3": {"exist": true, "char_ranges": [[231, 361]], "word_ranges": [[34, 52]], "text": "Pour penser à une MEN de type I, d'autres pathologies (tumeurs hypophysaires, pancréatiques ou duodénales) doivent être associées."}, "4": {"exist": false, "char_ranges": [], "word_ranges": [], "text": ""}, "5": {"exist": false, "char_ranges": [], "word_ranges": [], "text": ""}}} +{"id": 462, "year": 2018, "question_id_specific": 151, "full_question": "Un garçon de 13 ans présente des céphalées subaiguës et une diplopie. L'examen neurologique révèle une paralysie du regard vertical et l'IRM montre une lésion prenant le contraste dans la région pinéale et obstruant l'aqueduc de Sylvian. Le diagnostic le plus probable est le suivant :", "full_answer": "La diplopie est généralement causée par un strabisme aigu ou secondaire. La paralysie du regard vertical est due à une lésion du mésencéphale. Dans ce cas, l'IRM situe déjà l'origine des symptômes dans la glande pinéale. La tumeur la plus fréquente dans cette région est une tumeur germinale (réponse 3).", "type": "OPHTALMOLOGIE (ECTOPIQUE)", "options": {"1": "Glioblastome.", "2": "Médulloblastome.", "3": "Tumeur des cellules germinales.", "4": "Méningiome.", "5": null}, "correct_option": 3, "explanations": {"1": {"exist": false, "char_ranges": [], "word_ranges": [], "text": ""}, "2": {"exist": false, "char_ranges": [], "word_ranges": [], "text": ""}, "3": {"exist": true, "char_ranges": [[143, 304]], "word_ranges": [[23, 50]], "text": "Dans ce cas, l'IRM situe déjà l'origine des symptômes dans la glande pinéale. La tumeur la plus fréquente dans cette région est une tumeur germinale (réponse 3)."}, "4": {"exist": false, "char_ranges": [], "word_ranges": [], "text": ""}, "5": {"exist": false, "char_ranges": [], "word_ranges": [], "text": ""}}} +{"id": 557, "year": 2022, "question_id_specific": 168, "full_question": "Un homme de 58 ans, obèse, asymptomatique, buveur modéré, présente une première glycémie de référence de 153 mg/j avec une glycosurie négative. Au cours des semaines suivantes, il a eu deux autres glycémies de référence de 118 et 136 mg/dl. Laquelle des mesures suivantes est la plus appropriée pour confirmer le diagnostic de diabète sucré ?", "full_answer": "Critères de diagnostic du diabète selon ISPAD 2018 *Symptômes classiques du diabète avec une glycémie > 200mgr/dl ou : * Glycémie à jeun ≥ 126mgr/dl (à jeun au moins 8h) ou * Glycémie à 2h SOG ≥ 200mgr/dl (à 2 reprises) ou * HbA1C ≥ 6,5%.", "type": "ENDOCRINOLOGIE", "options": {"1": "Effectuer une courbe de glycémie avec 75 g de glucose.", "2": "Il répond déjà aux critères de diagnostic du diabète sucré.", "3": "Demander une mesure de l'insuline basale ou du peptide C.", "4": "Demander un dosage de l'hémoglobine glycosylée.", "5": null}, "correct_option": 2, "explanations": {"1": {"exist": false, "char_ranges": [], "word_ranges": [], "text": ""}, "2": {"exist": true, "char_ranges": [[0, 238]], "word_ranges": [[0, 46]], "text": "Critères de diagnostic du diabète selon ISPAD 2018 *Symptômes classiques du diabète avec une glycémie > 200mgr/dl ou : * Glycémie à jeun ≥ 126mgr/dl (à jeun au moins 8h) ou * Glycémie à 2h SOG ≥ 200mgr/dl (à 2 reprises) ou * HbA1C ≥ 6,5%."}, "3": {"exist": false, "char_ranges": [], "word_ranges": [], "text": ""}, "4": {"exist": false, "char_ranges": [], "word_ranges": [], "text": ""}, "5": {"exist": false, "char_ranges": [], "word_ranges": [], "text": ""}}} +{"id": 35, "year": 2011, "question_id_specific": 68, "full_question": "Un homme de 56 ans, sans antécédents personnels ou familiaux, a été vu en consultation pour avoir des difficultés à marcher sur sa jambe droite depuis six mois. Il n'a signalé aucun autre symptôme. L'examen a montré une faiblesse de 4/5 pour la flexion dorsale et l'éversion du pied et de 4/5 pour la flexion et l'inversion du pied, avec une augmentation des réflexes musculaires et de Babinski, le reste étant normal. Le diagnostic syndromique serait le suivant :", "full_answer": "Mauvaise réponse 1 : Elle n'explique pas le pyramidalisme. Réponse 2 correcte : Une réponse non spécifique, mais probablement la bonne réponse. Mauvaise réponse 3 : On manquerait de données pour penser à un syndrome hémimédullaire de Brown Sequard. Cela expliquerait le pyramidalisme, mais il manquerait des données évocatrices de cette affection, comme l'hypoesthésie controlatérale. Mauvaise réponse 4 : elle pourrait expliquer la difficulté de dorsiflexion et de flexion plantaire du pied, mais pas le pyramidalisme. Mauvaise réponse 5 : de la même manière que la précédente, elle pourrait expliquer une partie du tableau clinique, mais pas le pyramidalisme.", "type": "NEUROLOGIE ET NEUROCHIRURGIE", "options": {"1": "Mononeuropathie du nerf péronier commun droit.", "2": "Atteinte focale du premier motoneurone et atteinte probable du deuxième motoneurone.", "3": "Atteinte hémimédullaire droite (Brown-Sequard).", "4": "Mononeuropathie multiple avec atteinte du nerf péronier commun et du nerf tibial postérieur droit.", "5": "Plexopathie lombaire droite."}, "correct_option": 2, "explanations": {"1": {"exist": true, "char_ranges": [[21, 58]], "word_ranges": [[4, 9]], "text": "Elle n'explique pas le pyramidalisme."}, "2": {"exist": false, "char_ranges": [], "word_ranges": [], "text": ""}, "3": {"exist": true, "char_ranges": [[165, 384]], "word_ranges": [[26, 55]], "text": "On manquerait de données pour penser à un syndrome hémimédullaire de Brown Sequard. Cela expliquerait le pyramidalisme, mais il manquerait des données évocatrices de cette affection, comme l'hypoesthésie controlatérale."}, "4": {"exist": true, "char_ranges": [[406, 519]], "word_ranges": [[59, 76]], "text": "elle pourrait expliquer la difficulté de dorsiflexion et de flexion plantaire du pied, mais pas le pyramidalisme."}, "5": {"exist": true, "char_ranges": [[541, 661]], "word_ranges": [[80, 99]], "text": "de la même manière que la précédente, elle pourrait expliquer une partie du tableau clinique, mais pas le pyramidalisme."}}} +{"id": 271, "year": 2016, "question_id_specific": 170, "full_question": "Un homme de 80 ans se présente à votre cabinet pour une évaluation en vue d'une cholécystectomie programmée par laparoscopie. Il a des antécédents d'hypertension artérielle traitée depuis 10 ans. Il n'a pas de maladie cardiaque ou pulmonaire. Pas de douleur thoracique. Elle mène une vie active et se rend tous les jours à la salle de sport, où elle alterne entre la natation et la marche sur tapis roulant pendant au moins une heure. Traitement habituel : nébivolol 5 mg toutes les 24 heures et hydrochlorothiazide 12,5 mg par jour. Examen physique : poids 73 kg ; taille 179 cm ; tension artérielle 138/80 mmHg ; fréquence cardiaque 60 battements/minute. Lequel des éléments suivants est l'approche préopératoire la plus appropriée ?", "full_answer": "C'est une question tout à fait gratuite, posée par un homme en meilleure santé que moi, qui va subir une intervention chirurgicale à faible risque de façon programmée. La réponse est évidemment 4. S'il y avait eu une cinquième option, elle aurait été qu'il n'y a rien à faire, ce qui aurait pu susciter quelques doutes chez l'étudiant, mais comme cette option n'existe pas, la question est un jeu d'enfant. Bien qu'il existe de nombreux protocoles préopératoires, presque autant qu'il y a d'hôpitaux, la question facilite les choses en plaçant un patient très âgé, 80 ans. La tranche d'âge à partir de laquelle il est considéré comme obligatoire de demander un ECG varie entre 45, 50, 60 ou même 70 ans. Par exemple : > 50 ans : toujours, sauf si un ECG antérieur est disponible < 50 ans : uniquement en cas de maladie cardiaque connue ou suspectée (CVRF), de diabète sévère, d'hyperthyroïdie ou de BPCO sévère.", "type": "ANESTHÉSIOLOGIE ET SOINS INTENSIFS", "options": {"1": "Effectuer un test d'effort.", "2": "Effectuer une échocardiographie", "3": "Effectuer un scanner avec du thallium et du dipyridamole.", "4": "Effectuer un électrocardiogramme.", "5": null}, "correct_option": 4, "explanations": {"1": {"exist": false, "char_ranges": [], "word_ranges": [], "text": ""}, "2": {"exist": false, "char_ranges": [], "word_ranges": [], "text": ""}, "3": {"exist": false, "char_ranges": [], "word_ranges": [], "text": ""}, "4": {"exist": true, "char_ranges": [[407, 703]], "word_ranges": [[70, 121]], "text": "Bien qu'il existe de nombreux protocoles préopératoires, presque autant qu'il y a d'hôpitaux, la question facilite les choses en plaçant un patient très âgé, 80 ans. La tranche d'âge à partir de laquelle il est considéré comme obligatoire de demander un ECG varie entre 45, 50, 60 ou même 70 ans."}, "5": {"exist": false, "char_ranges": [], "word_ranges": [], "text": ""}}} +{"id": 306, "year": 2016, "question_id_specific": 207, "full_question": "Un patient de 34 ans jouant au tennis a reçu une balle de tennis dans l'orbite gauche. À l'examen, il présente un large hématome palpébral, une hyposphagmie, une diplopie de la vision supérieure avec limitation de la version supérieure du globe oculaire. Que suspecteriez-vous ?", "full_answer": "Dans l'énoncé, ils décrivent une diplopie de la vision supérieure avec limitation de l'ersion supérieure du globe oculaire. La fonction mécanique du droit inférieur gauche étant compromise, elle ne permettrait pas d'achever le mouvement supraductionnel, générant une diplopie verticale, ce qui, ajouté à l'antécédent traumatique, nous fait suspecter une fracture du plancher de l'orbite avec piégeage musculaire du droit inférieur. La fracture du toit est beaucoup moins fréquente et le piégeage du droit supérieur entraînerait une limitation de l'infraduction.", "type": "OPHTHALMOLOGIE", "options": {"1": "Fracture de la paroi inférieure du plancher de l'orbite avec coincement du muscle droit inférieur.", "2": "Fracture de l'arcade zygomatique.", "3": "Fracture de la paroi supérieure de l'orbite avec coincement du muscle droit supérieur.", "4": "Fracture dentoalvéolaire.", "5": null}, "correct_option": 1, "explanations": {"1": {"exist": true, "char_ranges": [[124, 431]], "word_ranges": [[18, 60]], "text": "La fonction mécanique du droit inférieur gauche étant compromise, elle ne permettrait pas d'achever le mouvement supraductionnel, générant une diplopie verticale, ce qui, ajouté à l'antécédent traumatique, nous fait suspecter une fracture du plancher de l'orbite avec piégeage musculaire du droit inférieur."}, "2": {"exist": false, "char_ranges": [], "word_ranges": [], "text": ""}, "3": {"exist": true, "char_ranges": [[432, 561]], "word_ranges": [[60, 79]], "text": "La fracture du toit est beaucoup moins fréquente et le piégeage du droit supérieur entraînerait une limitation de l'infraduction."}, "4": {"exist": false, "char_ranges": [], "word_ranges": [], "text": ""}, "5": {"exist": false, "char_ranges": [], "word_ranges": [], "text": ""}}} +{"id": 328, "year": 2016, "question_id_specific": 83, "full_question": "Un homme de 49 ans a consulté pour une polyurie et une polydipsie intenses et une perte de poids involontaire de 10 kg. On lui a diagnostiqué un diabète sucré en raison d'une glycémie plasmatique de 322 mg/dL et d'une hémoglobine glycosylée de 9,8 %. Son médecin lui a donné des recommandations diététiques, lui a conseillé de faire de l'exercice physique et a commencé un traitement à base de metformine 850 mg/12 heures et de glimépiride 6 mg/jour. Au cours des semaines suivantes, les contrôles glycémiques ont été progressivement réduits. Après 4 mois, la glycémie est de 94 mg/dL et l'HbA1c de 5,9 %. Le patient se plaint d'épisodes fréquents de vertiges, de douleurs épigastriques, de troubles de la vision, de sueurs et de tremblements, qui s'améliorent avec l'alimentation et surviennent principalement en fin de matinée et en fin d'après-midi. Quelle modification proposeriez-vous à son traitement ?", "full_answer": "Interrompre la sulfonylurée en raison du risque d'hypoglycémie.", "type": "ENDOCRINOLOGIE", "options": {"1": "Révisez la répartition des glucides dans votre alimentation.", "2": "Arrêter la metformine.", "3": "Interrompre la sulfonylurée.", "4": "Remplacer la metformine par un inhibiteur de la DPP4.", "5": null}, "correct_option": 3, "explanations": {"1": {"exist": false, "char_ranges": [], "word_ranges": [], "text": ""}, "2": {"exist": false, "char_ranges": [], "word_ranges": [], "text": ""}, "3": {"exist": true, "char_ranges": [[0, 63]], "word_ranges": [[0, 8]], "text": "Interrompre la sulfonylurée en raison du risque d'hypoglycémie."}, "4": {"exist": false, "char_ranges": [], "word_ranges": [], "text": ""}, "5": {"exist": false, "char_ranges": [], "word_ranges": [], "text": ""}}} +{"id": 223, "year": 2014, "question_id_specific": 62, "full_question": "Chez un patient diabétique de 60 ans présentant un angor stable secondaire à une cardiopathie ischémique due à une obstruction subtotale du tiers moyen de l'artère coronaire descendante antérieure, quel traitement proposeriez-vous ?", "full_answer": "Angor stable chez le diabétique : cette question est un clin d'œil à l'étude BARI-2D (NEJM 2009), qui a montré que la prise en charge médicale n'était pas inférieure à la revascularisation (d'ailleurs, les nouvelles recommandations reflètent la tendance à ne pas ouvrir systématiquement toutes les sténoses coronaires). La réponse 1 va dans ce sens, même si les statines, les IEC et les antiplaquettaires me manquent, et que j'ai trop de vasodilatateurs (les nitrates sont de classe IIb, si ma mémoire est bonne). La seconde est absurde : s'asseoir pour éviter l'angine de poitrine ? C'est comme couper la forêt pour éviter les incendies. Les points 3 à 5 concernent la revascularisation : comme il s'agit d'un vaisseau unique et non d'une LAD proximale, la revascularisation devrait être percutanée, ce qui exclut le point 3. Entre 4 et 5, le cinquième est plus complet : il faudrait non seulement dilater, mais aussi implanter un stent. En appliquant les règles générales de réponse du MIR, si deux options sont similaires à un détail près, la bonne est l'une d'entre elles, nous cocherions donc 5 (et c'est ce qui serait fait dans n'importe quel hôpital de nos jours : une artère suboccluse est dilatée directement). D'autre part, et bien que le point 1 soit incomplet, je suppose que l'auteur a voulu être intelligent et montrer qu'il a lu les derniers articles. Conclusion : je mettrais 5... mais je ne serais pas surpris si c'était 1.", "type": "CARDIOLOGIE", "options": {"1": "Médecin avec vasodilatateurs et bêta-bloquants pour prévenir l'angine de poitrine.", "2": "Expectorant avec repos strict car la diminution de la demande en oxygène du myocarde devrait réduire l'angine.", "3": "Chirurgie pour revasculariser le myocarde ischémique en contournant l'artère mammaire gauche distale de la lésion dans l'artère coronaire malade.", "4": "Dilatation de la lésion de l'artère coronaire par cathétérisme thérapeutique.", "5": "Cathétérisme thérapeutique pour dilater la lésion artérielle malade et implantation d'un stent dans la zone dilatée."}, "correct_option": 5, "explanations": {"1": {"exist": true, "char_ranges": [[939, 1219]], "word_ranges": [[155, 203]], "text": "En appliquant les règles générales de réponse du MIR, si deux options sont similaires à un détail près, la bonne est l'une d'entre elles, nous cocherions donc 5 (et c'est ce qui serait fait dans n'importe quel hôpital de nos jours : une artère suboccluse est dilatée directement)."}, "2": {"exist": true, "char_ranges": [[514, 583]], "word_ranges": [[83, 95]], "text": "La seconde est absurde : s'asseoir pour éviter l'angine de poitrine ?"}, "3": {"exist": true, "char_ranges": [[690, 826]], "word_ranges": [[113, 135]], "text": "comme il s'agit d'un vaisseau unique et non d'une LAD proximale, la revascularisation devrait être percutanée, ce qui exclut le point 3."}, "4": {"exist": true, "char_ranges": [[827, 938]], "word_ranges": [[135, 155]], "text": "Entre 4 et 5, le cinquième est plus complet : il faudrait non seulement dilater, mais aussi implanter un stent."}, "5": {"exist": true, "char_ranges": [[939, 1219]], "word_ranges": [[155, 203]], "text": "En appliquant les règles générales de réponse du MIR, si deux options sont similaires à un détail près, la bonne est l'une d'entre elles, nous cocherions donc 5 (et c'est ce qui serait fait dans n'importe quel hôpital de nos jours : une artère suboccluse est dilatée directement)."}}} +{"id": 468, "year": 2020, "question_id_specific": 122, "full_question": "Un homme de 70 ans consulte pour des douleurs lombaires non irradiantes d'une durée d'une semaine, d'intensité progressive, accompagnées d'une limitation marquée de la mobilité de la colonne vertébrale et de fièvre. La radiographie du rachis lombaire ne montre aucune anomalie. Quel diagnostic doit être éliminé en premier et avec quelle technique ?", "full_answer": "Nous soupçonnons une spondylodiscite - la fièvre fébrile et la mobilité limitée de la colonne vertébrale au cours de l'évolution aiguë en sont la clé - et l'examen indiqué est l'IRM. \"L'examen le plus démonstratif est l'imagerie par résonance magnétique (IRM), qui est positive même dans les deux premières semaines du tableau, ce qui facilite le diagnostic précoce, avec une sensibilité de 90 %\". Un aplatissement vertébral serait visible sur une simple radiographie, de même qu'une hyperplasie vertébrale ankylosante et de nombreuses métastases. De plus, il n'y a pas de données dans les antécédents qui indiqueraient des métastases osseuses (antécédents oncologiques, perte de poids, etc.).", "type": "CHIRURGIE ORTHOPÉDIQUE ET TRAUMATOLOGIE", "options": {"1": "Écrasement vertébral par tomodensitométrie (CT).", "2": "Métastases osseuses avec la scintigraphie osseuse au technétium.", "3": "Hyperostose vertébrale ankylosante avec radiographie du rachis dorsolombaire.", "4": "Spondylodiscite pyogénique avec imagerie par résonance magnétique (IRM).", "5": null}, "correct_option": 4, "explanations": {"1": {"exist": true, "char_ranges": [[398, 547]], "word_ranges": [[64, 83]], "text": "Un aplatissement vertébral serait visible sur une simple radiographie, de même qu'une hyperplasie vertébrale ankylosante et de nombreuses métastases."}, "2": {"exist": true, "char_ranges": [[557, 693]], "word_ranges": [[85, 105]], "text": "il n'y a pas de données dans les antécédents qui indiqueraient des métastases osseuses (antécédents oncologiques, perte de poids, etc.)."}, "3": {"exist": true, "char_ranges": [[398, 547]], "word_ranges": [[64, 83]], "text": "Un aplatissement vertébral serait visible sur une simple radiographie, de même qu'une hyperplasie vertébrale ankylosante et de nombreuses métastases."}, "4": {"exist": true, "char_ranges": [[0, 182]], "word_ranges": [[0, 31]], "text": "Nous soupçonnons une spondylodiscite - la fièvre fébrile et la mobilité limitée de la colonne vertébrale au cours de l'évolution aiguë en sont la clé - et l'examen indiqué est l'IRM."}, "5": {"exist": false, "char_ranges": [], "word_ranges": [], "text": ""}}} +{"id": 96, "year": 2012, "question_id_specific": 170, "full_question": "Un homme de 60 ans a signalé il y a 10 jours l'apparition de lésions vésiculeuses sur le dos de ses mains après une exposition au soleil. L'affection s'accompagnait d'une fragilité cutanée. L'histopathologie montre une cloque sous-épidermique avec des dépôts de PAS+ dans et autour des vaisseaux dermiques superficiels. Le diagnostic le plus plausible est le suivant :", "full_answer": "La porphyrie cutanée tardive (PCT) est la porphyrie la plus courante et peut se présenter sous trois formes cliniques principales : familiale, sporadique et toxique. Cliniquement, elle se caractérise par une fragilité cutanée marquée avec l'apparition d'érosions, de vésicules et d'ampoules après un traumatisme mineur. Les sites les plus fréquents sont le dos des mains et le visage, c'est-à-dire les zones les plus exposées à la lumière. Sur le plan clinique, le diagnostic différentiel le plus important doit être établi avec la pseudoporphyrie ou la dermatite bulleuse phototoxique, dans laquelle les vésicules et les érosions cutanées sont causées par une photosensibilité induite par des médicaments tels que les tétracyclines, les sulfamides, etc. Cependant, contrairement à la porphyrie, les niveaux de porphyrine dans le sérum, les fèces et l'urine sont normaux. Le diagnostic différentiel histopathologique doit être établi avec les maladies présentant des dépôts hyalins dermiques, telles que : la hyalinose cutanée, dans laquelle, contrairement à la porphyrie, les dépôts hyalins PAS-positifs sont étendus et occupent massivement le derme profond et le tissu cellulaire sous-cutané ; le milium colloïdal dans lequel la matière hyaline, généralement PAS-négative, étend le derme papillaire, formant des fentes et des fissures caractéristiques ; et la protéinose lipoïdique, dans laquelle la matière se dépose dans tout le derme, a une distribution non seulement périvasculaire mais plus étendue que dans la porphyrie, et affecte généralement les glandes sudoripares.", "type": "DERMATOLOGIE, VÉNÉRÉOLOGIE ET CHIRURGIE PLASTIQUE", "options": {"1": "Pemphigus bénin familial.", "2": "Pemphigoïde.", "3": "Porphyrie cutanée tardive.", "4": "Pemphigus vulgaire.", "5": "Syndrome de la peau échaudée."}, "correct_option": 3, "explanations": {"1": {"exist": false, "char_ranges": [], "word_ranges": [], "text": ""}, "2": {"exist": false, "char_ranges": [], "word_ranges": [], "text": ""}, "3": {"exist": true, "char_ranges": [[0, 439]], "word_ranges": [[0, 67]], "text": "La porphyrie cutanée tardive (PCT) est la porphyrie la plus courante et peut se présenter sous trois formes cliniques principales : familiale, sporadique et toxique. Cliniquement, elle se caractérise par une fragilité cutanée marquée avec l'apparition d'érosions, de vésicules et d'ampoules après un traumatisme mineur. Les sites les plus fréquents sont le dos des mains et le visage, c'est-à-dire les zones les plus exposées à la lumière."}, "4": {"exist": false, "char_ranges": [], "word_ranges": [], "text": ""}, "5": {"exist": false, "char_ranges": [], "word_ranges": [], "text": ""}}} +{"id": 603, "year": 2022, "question_id_specific": 112, "full_question": "Une femme de 61 ans, administrative, aux antécédents de surpoids, d'hypertension, de dyslipidémie et de syndrome métabolique, consulte pour des douleurs dans les deux fesses, la région trochantérienne gauche, la face latérale de la cuisse gauche jusqu'au genou et la jambe gauche jusqu'au tiers moyen. La douleur apparaît lorsque le membre inférieur est soulevé avec le genou en extension, mais est soulagée lorsque le genou est fléchi. Quelle est la première suspicion clinique ?", "full_answer": "La symptomatologie la plus courante d'un patient atteint de coxarthrose est la suivante : douleur à la hanche qui limite la marche, gêne nocturne et même au repos, raideur et blocage de la hanche. L'arthrose coxo-fémorale se manifeste par un test de Lasegue négatif. Mauvaise réponse 2. Les lombalgies de nature neuropathique se manifestent généralement par une radiculopathie unilatérale et répartie selon les dermatomes. Les facteurs de risque reconnus les plus courants sont : les femmes, l'obésité, le tabagisme et le mode de vie sédentaire ou la position assise prolongée. La lombosciatique est généralement associée à la fesse, à l'arrière des cuisses et même à l'aine. Le cas clinique décrit la manœuvre ou le test de Lasegue, un test provocateur qui met en évidence une irritation radiculaire dans la région lombo-sacrée (réponse correcte 3). La claudication due à une sténose canalaire se manifeste typiquement par une douleur qui s'aggrave lors de l'extension du tronc (marche, station debout) et qui est soulagée lors de la flexion (position assise, sommeil en position fœtale...) Mauvaise réponse 4.", "type": "TRAUMATOLOGIE", "options": {"1": "Arthrite goutteuse de la hanche gauche.", "2": "Arthrose coxo-fémorale gauche.", "3": "Lombalgie irradiée / lombosciatique.", "4": "Claudication due à une sténose du canal.", "5": null}, "correct_option": 3, "explanations": {"1": {"exist": false, "char_ranges": [], "word_ranges": [], "text": ""}, "2": {"exist": true, "char_ranges": [[197, 286]], "word_ranges": [[34, 47]], "text": "L'arthrose coxo-fémorale se manifeste par un test de Lasegue négatif. Mauvaise réponse 2."}, "3": {"exist": true, "char_ranges": [[676, 850]], "word_ranges": [[106, 134]], "text": "Le cas clinique décrit la manœuvre ou le test de Lasegue, un test provocateur qui met en évidence une irritation radiculaire dans la région lombo-sacrée (réponse correcte 3)."}, "4": {"exist": true, "char_ranges": [[851, 1111]], "word_ranges": [[134, 174]], "text": "La claudication due à une sténose canalaire se manifeste typiquement par une douleur qui s'aggrave lors de l'extension du tronc (marche, station debout) et qui est soulagée lors de la flexion (position assise, sommeil en position fœtale...) Mauvaise réponse 4."}, "5": {"exist": false, "char_ranges": [], "word_ranges": [], "text": ""}}} +{"id": 574, "year": 2022, "question_id_specific": 100, "full_question": "Une femme de 80 ans, ayant des antécédents d'hypertension artérielle et de diabète sucré, s'est soudainement présentée avec un trouble du langage. À son arrivée à l'hôpital, une fibrillation auriculaire inconnue a été observée et, à l'examen, elle présentait un discours fluide avec une tendance à l'argot, une mauvaise compréhension du langage, une incapacité à répéter et une dysnomie. Sur le plan sémiologique, il présentait un tableau compatible avec.. :", "full_answer": "Il s'agit d'une question de sémiologie qui permet de rappeler les types d'aphasie. Les aspects suivants doivent être pris en compte : l'émission du langage (fluent, non fluent, non émission), la compréhension (oui ou non), la répétition (oui ou non) et la nomination. L'aphasie de Wernicke ou aphasie sensitive a un langage fluide, ne comprend pas, ne répète pas et ne nomme pas, comme le patient de la question. L'aphasie transcorticale sensorielle diffère de l'aphasie sensorielle en ce que les patients sont capables de répéter. L'aphasie de conduction diffère de l'aphasie sensorielle en ce que la compréhension est préservée, toutes choses égales par ailleurs. Enfin, dans l'aphasie de Broca, le langage est non fluent, la compréhension est préservée, le patient ne répète pas et ne nomme pas.", "type": "NEUROLOGIE", "options": {"1": "Aphasie de Broca.", "2": "Aphasie de conduction.", "3": "Aphasie transcorticale sensorielle.", "4": "Aphasie de Wernicke.", "5": null}, "correct_option": 4, "explanations": {"1": {"exist": true, "char_ranges": [[666, 798]], "word_ranges": [[104, 127]], "text": "Enfin, dans l'aphasie de Broca, le langage est non fluent, la compréhension est préservée, le patient ne répète pas et ne nomme pas."}, "2": {"exist": true, "char_ranges": [[532, 665]], "word_ranges": [[85, 104]], "text": "L'aphasie de conduction diffère de l'aphasie sensorielle en ce que la compréhension est préservée, toutes choses égales par ailleurs."}, "3": {"exist": true, "char_ranges": [[413, 531]], "word_ranges": [[69, 85]], "text": "L'aphasie transcorticale sensorielle diffère de l'aphasie sensorielle en ce que les patients sont capables de répéter."}, "4": {"exist": true, "char_ranges": [[268, 412]], "word_ranges": [[43, 69]], "text": "L'aphasie de Wernicke ou aphasie sensitive a un langage fluide, ne comprend pas, ne répète pas et ne nomme pas, comme le patient de la question."}, "5": {"exist": false, "char_ranges": [], "word_ranges": [], "text": ""}}} +{"id": 67, "year": 2012, "question_id_specific": 111, "full_question": "Un homme de 45 ans, ayant des antécédents de lithiase urique et de coliques néphrétiques expulsives répétées depuis 25 ans, s'est présenté aux urgences en se plaignant de palpitations et d'une douleur lombaire droite intense depuis 2 heures. L'analyse d'urine a révélé un pH de 5,5 et une leucocyturie sans protéinurie. L'électrocardiogramme confirme une FA inconnue jusqu'alors. La biochimie sanguine montre une Creat de 0,9, un Ca de 11 et une LDH de 950. Quelle est la manœuvre diagnostique la plus utile à effectuer ?", "full_answer": "A priori, ce que vous nous dites ne correspond pas à une colique néphrétique. L'urine est normale (sans hématurie, causée par la lithiase au fur et à mesure de son évolution), la créatine est normale (elle peut monter à 1,9 dans la colique néphritique). L'augmentation des LDH n'est pas spécifique et peut être retrouvée dans les lésions parenchymateuses du foie, l'IAM ou l'embolie pulmonaire, ainsi que dans de nombreux autres cas moins fréquents (tumeurs, myopathies, etc.). Dans ce cas, nous avons un facteur de risque important d'embolie, à savoir la fibrillation auriculaire, qui, de plus, s'agissant d'un homme de 45 ans, sans antécédents, ne semble pas répondre à une maladie cardiaque structurelle sous-jacente, ainsi qu'une douleur lombaire droite, qui pourrait correspondre à une irritation de la plèvre au niveau de la colonne lombaire droite. La tomodensitométrie spiralée avec contraste a acquis une grande valeur dans le diagnostic de la TEP, en raison de sa rapidité et de sa bonne sensibilité, supérieure à 83-92 % dans la plupart des études, et selon certaines, elle peut atteindre 100 % pour les thrombus centraux, situés dans les artères pulmonaires principales, lobaires ou segmentaires. Sa spécificité est supérieure à 90 %. Les ganglions lymphatiques peuvent produire des faux positifs. Il peut également fournir un diagnostic alternatif si la TEP n'est pas confirmée, ce qui en fait un test très utile. L'échographie Doppler serait l'examen le plus utile pour diagnostiquer la source des emboles, et pourrait donc constituer une réponse valable, en fonction de ce que l'on entend par \"utile\". Il s'agit d'une technique non invasive qui a démontré une bonne sensibilité et une bonne spécificité dans le diagnostic de la TVP par rapport à la phlébographie. Elle permet également de diagnostiquer d'autres pathologies qui pourraient justifier les symptômes du patient. Ses limites sont l'évaluation des thrombus infrapoplités et, dans de nombreux cas, chez les patients qui ne sont pas minces, des veines iliaques et caves.", "type": "ANESTHÉSIOLOGIE, SOINS INTENSIFS ET MÉDECINE D'URGENCE", "options": {"1": "Radiographie abdominale simple.", "2": "Urographie intraveineuse.", "3": "CT spiralé avec contraste.", "4": "Échographie abdominale.", "5": "Échographie Doppler."}, "correct_option": 3, "explanations": {"1": {"exist": false, "char_ranges": [], "word_ranges": [], "text": ""}, "2": {"exist": false, "char_ranges": [], "word_ranges": [], "text": ""}, "3": {"exist": true, "char_ranges": [[856, 1246]], "word_ranges": [[134, 197]], "text": "La tomodensitométrie spiralée avec contraste a acquis une grande valeur dans le diagnostic de la TEP, en raison de sa rapidité et de sa bonne sensibilité, supérieure à 83-92 % dans la plupart des études, et selon certaines, elle peut atteindre 100 % pour les thrombus centraux, situés dans les artères pulmonaires principales, lobaires ou segmentaires. Sa spécificité est supérieure à 90 %."}, "4": {"exist": false, "char_ranges": [], "word_ranges": [], "text": ""}, "5": {"exist": false, "char_ranges": [], "word_ranges": [], "text": ""}}} +{"id": 77, "year": 2012, "question_id_specific": 31, "full_question": "Un garçon de 17 ans atteint d'une maladie de Crohn avec atteinte colique depuis 2 ans, sous traitement d'entretien par azathioprine, consulte pour l'apparition depuis 5 jours de nodules sous-cutanés rouge violacé, chauds, douloureux, bilatéraux dans la région prétibiale, associés à une augmentation du nombre de selles et à des douleurs abdominales. L'approche la plus appropriée dans ce cas est la suivante :", "full_answer": "Il semble que le patient présente des lésions d'érythème noueux associées à une poussée de sa maladie, ce qui est fréquent. Le traitement et le contrôle de la maladie améliorent généralement les lésions cutanées. Les lésions ne doivent pas être biopsiées.", "type": "SYSTÈME DIGESTIF", "options": {"1": "Recommandez un repos relatif et des compresses chaudes sur les deux jambes et ajoutez un traitement antidépresseur.", "2": "Effectuez des biopsies sur des zones de peau éloignées des zones blessées et prescrivez des analgésiques opioïdes dès le début.", "3": "Effectuez des biopsies sur des zones de peau éloignées des zones blessées et prescrivez des analgésiques opioïdes dès le début.", "4": "Suspecter une ischémie bilatérale des membres inférieurs d'origine médicamenteuse.", "5": "Adaptation du traitement de la maladie."}, "correct_option": 5, "explanations": {"1": {"exist": false, "char_ranges": [], "word_ranges": [], "text": ""}, "2": {"exist": true, "char_ranges": [[0, 255]], "word_ranges": [[0, 41]], "text": "Il semble que le patient présente des lésions d'érythème noueux associées à une poussée de sa maladie, ce qui est fréquent. Le traitement et le contrôle de la maladie améliorent généralement les lésions cutanées. Les lésions ne doivent pas être biopsiées."}, "3": {"exist": true, "char_ranges": [[0, 255]], "word_ranges": [[0, 41]], "text": "Il semble que le patient présente des lésions d'érythème noueux associées à une poussée de sa maladie, ce qui est fréquent. Le traitement et le contrôle de la maladie améliorent généralement les lésions cutanées. Les lésions ne doivent pas être biopsiées."}, "4": {"exist": false, "char_ranges": [], "word_ranges": [], "text": ""}, "5": {"exist": true, "char_ranges": [[0, 255]], "word_ranges": [[0, 41]], "text": "Il semble que le patient présente des lésions d'érythème noueux associées à une poussée de sa maladie, ce qui est fréquent. Le traitement et le contrôle de la maladie améliorent généralement les lésions cutanées. Les lésions ne doivent pas être biopsiées."}}} +{"id": 440, "year": 2018, "question_id_specific": 97, "full_question": "Une femme de 24 ans consulte pour des douleurs abdominales basses qui durent depuis deux semaines et qui se sont aggravées lors du dernier rapport sexuel. Elle déclare avoir un partenaire récent avec lequel elle n'utilise qu'occasionnellement un préservatif. À l'examen, elle est en bon état général. Température 38,6°C, leucocytes 16 000/uL (85% de neutrophiles) ; CRP 30 mg/L. L'examen au spéculum montre des pertes vaginales abondantes et anormales et une mobilisation douloureuse du col de l'utérus. Indiquer la réponse FAUX :", "full_answer": "Une maladie inflammatoire pelvienne est suspectée, étant donné les symptômes de la patiente. Selon le protocole SEGO 2006 pour les maladies inflammatoires pelviennes, la réponse 2 est la réponse fausse et, par conséquent, celle qui doit être cochée. En effet, l'hystérosalpingographie peut propager l'infection à la cavité pelvienne et abdominale. Toutes les autres réponses sont correctes.", "type": "GYNÉCOLOGIE ET OBSTÉTRIQUE", "options": {"1": "Nous commencerons un traitement antibiotique dès que nous soupçonnerons le diagnostic.", "2": "Nous indiquons une hystérosalpingographie pour l'évaluation des trompes comme examen complémentaire préférentiel.", "3": "Un retard dans le diagnostic et le traitement augmente la probabilité de séquelles.", "4": "La présence d'un abcès serait un critère d'hospitalisation.", "5": null}, "correct_option": 2, "explanations": {"1": {"exist": false, "char_ranges": [], "word_ranges": [], "text": ""}, "2": {"exist": true, "char_ranges": [[167, 347]], "word_ranges": [[23, 50]], "text": "la réponse 2 est la réponse fausse et, par conséquent, celle qui doit être cochée. En effet, l'hystérosalpingographie peut propager l'infection à la cavité pelvienne et abdominale."}, "3": {"exist": false, "char_ranges": [], "word_ranges": [], "text": ""}, "4": {"exist": false, "char_ranges": [], "word_ranges": [], "text": ""}, "5": {"exist": false, "char_ranges": [], "word_ranges": [], "text": ""}}} +{"id": 203, "year": 2013, "question_id_specific": 47, "full_question": "Une femme de 55 ans, opérée de l'appendicite il y a 24 ans, s'est présentée à l'hôpital car, bien qu'elle se portât bien auparavant, elle a commencé à vomir à plusieurs reprises après le petit-déjeuner, il y a environ 12 heures, en même temps qu'une distension abdominale. Elle a eu des selles diarrhéiques quelques heures après le début des symptômes. L'examen a révélé une distension abdominale, un tympanisme et une augmentation des bruits intestinaux, mais pas d'irritation péritonéale. Les tests de laboratoire n'ont révélé aucune anomalie. La radiographie abdominale a montré une dilatation des anses de l'intestin grêle, mais aucun gaz n'a pu être observé dans l'ampoule rectale. En ce qui concerne ce cas, il est vrai que :", "full_answer": "Il s'agit de la description typique d'un tableau d'obstruction intestinale par des ponts ou des adhérences dus à une intervention chirurgicale antérieure. La bonne réponse est 5 et non 1 car la plupart de ces situations se résolvent spontanément avec un traitement conservateur.", "type": "CHIRURGIE GÉNÉRALE", "options": {"1": "Il s'agit d'un cas d'obstruction de l'intestin grêle nécessitant une laparotomie d'urgence.", "2": "Il s'agit vraisemblablement d'un cas d'obstruction au niveau du gros intestin.", "3": "Le fait que le patient ait présenté des selles diarrhéiques nous permet d'exclure le diagnostic d'occlusion intestinale. D'autres examens d'imagerie sont nécessaires pour assurer le diagnostic.", "4": "Une coloscopie décompressive doit être indiquée.", "5": "La prise en charge initiale doit être conservatrice, avec un traitement au sérum, une aspiration nasogastrique et une surveillance clinique, analytique et radiographique régulière."}, "correct_option": 5, "explanations": {"1": {"exist": true, "char_ranges": [[0, 278]], "word_ranges": [[0, 43]], "text": "Il s'agit de la description typique d'un tableau d'obstruction intestinale par des ponts ou des adhérences dus à une intervention chirurgicale antérieure. La bonne réponse est 5 et non 1 car la plupart de ces situations se résolvent spontanément avec un traitement conservateur."}, "2": {"exist": false, "char_ranges": [], "word_ranges": [], "text": ""}, "3": {"exist": false, "char_ranges": [], "word_ranges": [], "text": ""}, "4": {"exist": false, "char_ranges": [], "word_ranges": [], "text": ""}, "5": {"exist": true, "char_ranges": [[0, 278]], "word_ranges": [[0, 43]], "text": "Il s'agit de la description typique d'un tableau d'obstruction intestinale par des ponts ou des adhérences dus à une intervention chirurgicale antérieure. La bonne réponse est 5 et non 1 car la plupart de ces situations se résolvent spontanément avec un traitement conservateur."}}} +{"id": 158, "year": 2012, "question_id_specific": 133, "full_question": "Garçon obèse de 12 ans (87 kg). Il signale des douleurs mécaniques à la cuisse et au genou droits depuis 4 mois. L'examen clinique a révélé une différence de longueur des membres inférieurs de 1 cm et une attitude du membre inférieur droit en rotation externe avec perte de la rotation interne. Le patient doit être suspecté d'être atteint :", "full_answer": "On nous présente une patiente adolescente en surpoids avec des douleurs à la cuisse et au genou, un décalage de longueur de 1 cm et le membre en rotation externe. Ce tableau devrait nous amener automatiquement à poser le diagnostic d'épiphysiolyse capitis femoris (épiphysiolyse fémorale proximale, le fait de le lire en latin est un peu un malentendu mais ne doit pas nous induire en erreur). Il faut penser à celle-ci car c'est la plus fréquente et la plus possible (bonne réponse 2). Les autres options ne sont pas impossibles mais elles sont moins fréquentes et on ne nous demanderait pas cela si on voulait que nous choisissions une autre option. S'il s'agissait de séquelles de la dysplasie de la hanche, elles ne se manifesteraient pas à 4 mois et l'âge d'apparition serait plus précoce. Les complications de la dysplasie sont les luxations et subluxations qui peuvent nécessiter une intervention chirurgicale et la nécrose avasculaire de la tête (fausse réponse 5). La maladie de Perthes se manifeste chez les jeunes enfants (4-9 ans) par des épisodes de boiterie et de douleur à la hanche et au genou (fausse réponse 4). Chez un enfant, une fracture au niveau de la physe, et non au niveau du col du fémur, serait la plus typique, ce qui nous permet d'écarter la réponse 3. Pour que nous puissions envisager cette option, il faudrait que l'on nous ait présenté un kyste osseux, ce qui justifierait une faiblesse dans la région du col du fémur. Quant à l'arthrite chronique juvénile, il s'agit d'une option rhumatoïde entourée d'options traumatiques. Il n'est pas fait mention de facteurs rhumatologiques dans une analyse analytique, ni d'un tableau polyarticulaire. On pourrait peut-être y penser car ils mentionnent une douleur au genou droit, mais ce tableau qu'ils nous présentent est limité au membre inférieur droit et ils ne mentionnent pas d'autres articulations, de fièvre ou d'autres symptômes systémiques d'accompagnement, donc ce n'est pas une option à évaluer, surtout avec les autres options qu'ils nous présentent et qui seraient plus probables compte tenu du tableau qu'ils nous présentent (option 1 fausse). En outre, la douleur au genou dans ce cas fait référence à une douleur référée dans la hanche, sans affecter le genou.", "type": "TRAUMATOLOGIE ET ORTHOPÉDIE", "options": {"1": "Arthrite chronique juvénile.", "2": "Epiphysiolyse de la tête du fémur.", "3": "Fracture du col du fémur due à une surcharge.", "4": "Maladie de Perthes.", "5": "Séquelles de la dysplasie développementale de la hanche."}, "correct_option": 2, "explanations": {"1": {"exist": true, "char_ranges": [[1675, 2132]], "word_ranges": [[278, 348]], "text": "On pourrait peut-être y penser car ils mentionnent une douleur au genou droit, mais ce tableau qu'ils nous présentent est limité au membre inférieur droit et ils ne mentionnent pas d'autres articulations, de fièvre ou d'autres symptômes systémiques d'accompagnement, donc ce n'est pas une option à évaluer, surtout avec les autres options qu'ils nous présentent et qui seraient plus probables compte tenu du tableau qu'ils nous présentent (option 1 fausse)."}, "2": {"exist": true, "char_ranges": [[394, 486]], "word_ranges": [[66, 83]], "text": "Il faut penser à celle-ci car c'est la plus fréquente et la plus possible (bonne réponse 2)."}, "3": {"exist": true, "char_ranges": [[1130, 1282]], "word_ranges": [[190, 220]], "text": "Chez un enfant, une fracture au niveau de la physe, et non au niveau du col du fémur, serait la plus typique, ce qui nous permet d'écarter la réponse 3."}, "4": {"exist": true, "char_ranges": [[974, 1129]], "word_ranges": [[161, 190]], "text": "La maladie de Perthes se manifeste chez les jeunes enfants (4-9 ans) par des épisodes de boiterie et de douleur à la hanche et au genou (fausse réponse 4)."}, "5": {"exist": true, "char_ranges": [[795, 973]], "word_ranges": [[135, 161]], "text": "Les complications de la dysplasie sont les luxations et subluxations qui peuvent nécessiter une intervention chirurgicale et la nécrose avasculaire de la tête (fausse réponse 5)."}}} +{"id": 46, "year": 2011, "question_id_specific": 155, "full_question": "Un garçon d'un an dont la mère vient de recevoir un diagnostic de tuberculose pulmonaire bacillifère, après avoir commencé le traitement adéquat. L'enfant est asymptomatique, l'examen est normal, le test tuberculinique (TP) est négatif et la radiographie pulmonaire est normale. Quelle est la meilleure conduite à tenir pour l'enfant ?", "full_answer": "La bonne réponse est 5. La mère doit être isolée pendant les 2 à 4 premières semaines du traitement, après quoi elle ne devrait plus être contagieuse. Si le résultat d'un nouveau TP est toujours négatif, l'enfant ne doit plus recevoir de traitement. S'il est positif, il faut refaire une radiographie (ou un scanner des poumons pour les enfants plus jeunes).", "type": "PÉDIATRIE", "options": {"1": "Ne le séparez de la mère que jusqu'�� la fin de son traitement (au moins 6 mois).", "2": "Effectuer une tomodensitométrie à haute résolution du poumon et, si elle est normale, répéter la PT à 8-12 semaines.", "3": "Chimioprophylaxie à l'isoniazide (INH) jusqu'à ce que le frottis de la mère soit négatif.", "4": "Chimioprophylaxie par INH pendant 6 à 9 mois. Nouveau TP et radiographie pulmonaire à la fin du traitement pour décider de la fin ou non du traitement.", "5": "Chimioprophylaxie par INH, 8-12 semaines. Répéter le TP. Si négatif, arrêter le traitement. Si le résultat est positif, effectuer une radiographie du thorax pour déterminer s'il y a ou non une maladie."}, "correct_option": 5, "explanations": {"1": {"exist": false, "char_ranges": [], "word_ranges": [], "text": ""}, "2": {"exist": false, "char_ranges": [], "word_ranges": [], "text": ""}, "3": {"exist": false, "char_ranges": [], "word_ranges": [], "text": ""}, "4": {"exist": false, "char_ranges": [], "word_ranges": [], "text": ""}, "5": {"exist": true, "char_ranges": [[24, 358]], "word_ranges": [[5, 61]], "text": "La mère doit être isolée pendant les 2 à 4 premières semaines du traitement, après quoi elle ne devrait plus être contagieuse. Si le résultat d'un nouveau TP est toujours négatif, l'enfant ne doit plus recevoir de traitement. S'il est positif, il faut refaire une radiographie (ou un scanner des poumons pour les enfants plus jeunes)."}}} +{"id": 593, "year": 2022, "question_id_specific": 146, "full_question": "Une femme de 40 ans, sans antécédents personnels ou familiaux, a présenté une diarrhée aqueuse très abondante durant plusieurs jours, accompagnée d'une perte de poids de 10 kg, de bouffées vasomotrices et d'une rougeur du visage, ainsi que d'une lipothymie et de douleurs abdominales avec coliques, et d'une hypokaliémie dans les données de laboratoire. Une échographie abdominale a montré une masse de 1,5 cm de diamètre dans la queue du pancréas. Parmi les éléments suivants, sa première suspicion diagnostique est la suivante :", "full_answer": "peptide intestinal vasoactif (VIP). VIPoma-diarrhée. 3.", "type": "ONCOLOGIE", "options": {"1": "Insulinome.", "2": "PPoma.", "3": "VIPoma.", "4": "Glucagonomie.", "5": null}, "correct_option": 3, "explanations": {"1": {"exist": false, "char_ranges": [], "word_ranges": [], "text": ""}, "2": {"exist": false, "char_ranges": [], "word_ranges": [], "text": ""}, "3": {"exist": true, "char_ranges": [[0, 55]], "word_ranges": [[0, 6]], "text": "peptide intestinal vasoactif (VIP). VIPoma-diarrhée. 3."}, "4": {"exist": false, "char_ranges": [], "word_ranges": [], "text": ""}, "5": {"exist": false, "char_ranges": [], "word_ranges": [], "text": ""}}} +{"id": 395, "year": 2016, "question_id_specific": 136, "full_question": "Un homme de 41 ans se présente aux urgences après trois jours de gonflement et de douleur au genou droit, d'impuissance fonctionnelle et de fièvre. Deux semaines auparavant, il avait eu une diarrhée spontanée. L'examen révèle un épanchement articulaire ; une arthrocentèse est donc pratiquée et 50 cc de liquide trouble sont obtenus, avec une viscosité réduite et les paramètres analytiques suivants : leucocytes 40.000/microL (85% de neutrophiles), glucose 40 mg/dL, absence de cristaux, coloration de Gram : aucun micro-organisme n'a été observé.000/microL (85 % de neutrophiles), glucose 40 mg/dL, absence de cristaux, coloration de Gram : aucun micro-organisme n'est observé. Laquelle des affirmations suivantes concernant ce patient est \"CORRECTE\" ?", "full_answer": "Une coloration de Gram négative n'exclut jamais la présence d'une infection.", "type": "TRAUMATOLOGIE ET ORTHOPÉDIE", "options": {"1": "Un traitement à base de cloxacilline et de ceftriaxone doit être mis en place en attendant le résultat de la culture du liquide.", "2": "Une arthrocentèse quotidienne est conseillée pour soulager les symptômes et prévenir la destruction des articulations.", "3": "Si la culture est négative, il s'agit probablement d'une arthrite réactive.", "4": "Une coloration de Gram négative exclut la possibilité d'une arthrite septique.", "5": null}, "correct_option": 4, "explanations": {"1": {"exist": false, "char_ranges": [], "word_ranges": [], "text": ""}, "2": {"exist": false, "char_ranges": [], "word_ranges": [], "text": ""}, "3": {"exist": false, "char_ranges": [], "word_ranges": [], "text": ""}, "4": {"exist": true, "char_ranges": [[0, 76]], "word_ranges": [[0, 11]], "text": "Une coloration de Gram négative n'exclut jamais la présence d'une infection."}, "5": {"exist": false, "char_ranges": [], "word_ranges": [], "text": ""}}} +{"id": 424, "year": 2018, "question_id_specific": 91, "full_question": "Une femme de 27 ans consulte pour une aménorrhée secondaire qui dure depuis un an. Elle ne signale aucun antécédent iatrogène. Sur le plan analytique, la concentration d'œstradiol est inférieure à la normale et la valeur de la prolactine est de 12 ng/mL. Indiquez lequel des examens suivants vous demanderiez en première intention pour tenter d'identifier l'origine du trouble :", "full_answer": "Jeune femme présentant une aménorrhée secondaire, sans antécédents particuliers. Les valeurs de prolactine sont normales (des valeurs supérieures à 20 ngr/ml sont considérées comme une hyperprolectinémie), ce qui exclut l'hyperprolactinémie comme cause de l'aménorrhée. Elles nous indiquent que le taux d'œstrogènes est bas. Dans ce cas, il faut suspecter une insuffisance ovarienne précoce. Pour la diagnostiquer, il faut demander un taux basal de FSH (en cas d'insuffisance ovarienne prématurée, ce taux est élevé).", "type": "ENDOCRINOLOGIE", "options": {"1": "IRM hypothalamohypophysaire.", "2": "Caryotype.", "3": "FSH basale.", "4": "Stimulation de la gonadotrophine chorionique.", "5": null}, "correct_option": 3, "explanations": {"1": {"exist": false, "char_ranges": [], "word_ranges": [], "text": ""}, "2": {"exist": false, "char_ranges": [], "word_ranges": [], "text": ""}, "3": {"exist": true, "char_ranges": [[270, 517]], "word_ranges": [[34, 73]], "text": "Elles nous indiquent que le taux d'œstrogènes est bas. Dans ce cas, il faut suspecter une insuffisance ovarienne précoce. Pour la diagnostiquer, il faut demander un taux basal de FSH (en cas d'insuffisance ovarienne prématurée, ce taux est élevé)."}, "4": {"exist": false, "char_ranges": [], "word_ranges": [], "text": ""}, "5": {"exist": false, "char_ranges": [], "word_ranges": [], "text": ""}}} +{"id": 169, "year": 2013, "question_id_specific": 87, "full_question": "Femme de 70 ans, hypertendue et diabétique, ayant travaillé toute sa vie comme vendeuse. Elle consulte pour une lésion de la malléole interne de la jambe droite avec une douleur intense tout au long de la journée qui s'aggrave la nuit. L'examen révèle des varices non compliquées aux deux extrémités et des pouls fémoraux et poplités normaux avec absence de pouls distaux. Quel examen faut-il pratiquer en premier ?", "full_answer": "En tout cas, en tant que médecin de famille, je commence généralement par les tests les moins invasifs et les plus rentables, et dans le cas décrit, un problème d'ischémie des membres inférieurs, il s'agit de l'indice cheville-brachiale.", "type": "CARDIOLOGIE ET CHIRURGIE CARDIOVASCULAIRE", "options": {"1": "Echographie Doppler veineuse des membres inférieurs.", "2": "Imagerie par angiorésonance des membres inférieurs.", "3": "Artériographie.", "4": "Indice brachial de la cheville.", "5": "AngioTC."}, "correct_option": 4, "explanations": {"1": {"exist": false, "char_ranges": [], "word_ranges": [], "text": ""}, "2": {"exist": false, "char_ranges": [], "word_ranges": [], "text": ""}, "3": {"exist": false, "char_ranges": [], "word_ranges": [], "text": ""}, "4": {"exist": true, "char_ranges": [[45, 237]], "word_ranges": [[9, 38]], "text": "je commence généralement par les tests les moins invasifs et les plus rentables, et dans le cas décrit, un problème d'ischémie des membres inférieurs, il s'agit de l'indice cheville-brachiale."}, "5": {"exist": false, "char_ranges": [], "word_ranges": [], "text": ""}}} +{"id": 95, "year": 2012, "question_id_specific": 170, "full_question": "Un homme de 60 ans a signalé il y a 10 jours l'apparition de lésions vésiculeuses sur le dos de ses mains après une exposition au soleil. L'affection s'accompagnait d'une fragilité cutanée. L'histopathologie montre une cloque sous-épidermique avec des dépôts de PAS+ dans et autour des vaisseaux dermiques superficiels. Le diagnostic le plus plausible est le suivant :", "full_answer": "Le fait que le tableau soit photoinduit, l'histologie et l'âge rendent le diagnostic de PCT plus probable (3). Les autres cas ne correspondent pas au tableau clinique ou à l'histologie.", "type": "DERMATOLOGIE, VÉNÉRÉOLOGIE ET CHIRURGIE PLASTIQUE", "options": {"1": "Pemphigus bénin familial.", "2": "Pemphigoïde.", "3": "Porphyrie cutanée tardive.", "4": "Pemphigus vulgaire.", "5": "Syndrome de la peau échaudée."}, "correct_option": 3, "explanations": {"1": {"exist": true, "char_ranges": [[111, 185]], "word_ranges": [[18, 30]], "text": "Les autres cas ne correspondent pas au tableau clinique ou à l'histologie."}, "2": {"exist": true, "char_ranges": [[111, 185]], "word_ranges": [[18, 30]], "text": "Les autres cas ne correspondent pas au tableau clinique ou à l'histologie."}, "3": {"exist": true, "char_ranges": [[0, 110]], "word_ranges": [[0, 18]], "text": "Le fait que le tableau soit photoinduit, l'histologie et l'âge rendent le diagnostic de PCT plus probable (3)."}, "4": {"exist": true, "char_ranges": [[111, 185]], "word_ranges": [[18, 30]], "text": "Les autres cas ne correspondent pas au tableau clinique ou à l'histologie."}, "5": {"exist": true, "char_ranges": [[111, 185]], "word_ranges": [[18, 30]], "text": "Les autres cas ne correspondent pas au tableau clinique ou à l'histologie."}}} +{"id": 537, "year": 2021, "question_id_specific": 1, "full_question": "Patient de 44 ans présentant une lésion à croissance rapide (16x8x12 cm) touchant les tissus mous du tiers proximal de la cuisse droite. Il a indiqué qu'il avait commencé à remarquer la masse, qui avait une consistance dure et adhérait aux plans profonds, au cours des six derniers mois. Auparavant, elle était une sportive assidue et avait même terminé un marathon. À l'examen, une circulation collatérale a été observée, mais aucun ganglion lymphatique inguinal droit n'était palpable. Après biopsie, il a été confirmé qu'il s'agissait d'un sarcome pléomorphe indifférencié de haut grade, qui a été découvert à proximité du petit trochanter sur les examens d'imagerie. Le scanner d'extension était négatif. Quelle serait l'approche thérapeutique ?", "full_answer": "En ce qui concerne les marges de résection, nous avons : intralésionnelle → marginale (option 3) → étendue → radicale (options 1 et 4). Il s'agit d'une tumeur de haut grade, pour laquelle une résection étendue incluant des marges tumorales libres est indiquée. Il n'a pas été démontré que l'amputation radicale améliorait la survie et elle est réservée à des cas particuliers (invasion des structures neurovasculaires, fracture pathologique...). Par conséquent, nous écartons les options 1 et 4 ; l'option 3 est également écartée, car il s'agit d'une marge plus petite que la marge élargie ; par élimination, il nous reste l'option 2.", "type": "TRAUMATOLOGIE", "options": {"1": "Désarticulation de la hanche.", "2": "Traitement à l'ifosfamide et au tamoxifène pendant 6 cycles avant la chirurgie de résection pour réduire la taille de la lésion.", "3": "Chirurgie marginale en retirant la pseudocapsule que ces tumeurs forment souvent en réponse à leur croissance rapide.", "4": "Hémipelvectomie droite modifiée.", "5": null}, "correct_option": 2, "explanations": {"1": {"exist": true, "char_ranges": [[261, 496]], "word_ranges": [[43, 78]], "text": "Il n'a pas été démontré que l'amputation radicale améliorait la survie et elle est réservée à des cas particuliers (invasion des structures neurovasculaires, fracture pathologique...). Par conséquent, nous écartons les options 1 et 4 ;"}, "2": {"exist": false, "char_ranges": [], "word_ranges": [], "text": ""}, "3": {"exist": true, "char_ranges": [[497, 591]], "word_ranges": [[78, 95]], "text": "l'option 3 est également écartée, car il s'agit d'une marge plus petite que la marge élargie ;"}, "4": {"exist": true, "char_ranges": [[261, 496]], "word_ranges": [[43, 78]], "text": "Il n'a pas été démontré que l'amputation radicale améliorait la survie et elle est réservée à des cas particuliers (invasion des structures neurovasculaires, fracture pathologique...). Par conséquent, nous écartons les options 1 et 4 ;"}, "5": {"exist": false, "char_ranges": [], "word_ranges": [], "text": ""}}} +{"id": 363, "year": 2016, "question_id_specific": 97, "full_question": "Un patient de 35 ans qui, en raison d'une hématémèse due à un ulcère, reçoit une transfusion de deux concentrés de globules rouges. 5-10 minutes après le début de la transfusion, il commence à avoir de la fièvre, des frissons, une hypotension et des douleurs dans la région lombaire. Quel serait le diagnostic le plus probable ?", "full_answer": "Les réactions transfusionnelles hémolytiques aiguës peuvent survenir en quelques minutes. Cliniquement, elles se caractérisent par des frissons, de la fièvre, de l'urticaire, de la tachycardie, des nausées et des vomissements, des douleurs lombaires, des saignements dus à la coagulation intravasculaire disséminée (CIVD). ..... On nous parle d'un patient qui, quelques minutes après avoir commencé une transfusion, a présenté un état aigu et très grave (avec choc). Une telle situation aiguë et grave ne peut que correspondre à l'option 4.", "type": "HÉMATOLOGIE", "options": {"1": "Contamination bactérienne du sang.", "2": "Réaction fébrile secondaire à une transfusion.", "3": "Réaction fébrile due à la contamination des globules rouges par le plasma.", "4": "Réaction transfusionnelle hémolytique.", "5": null}, "correct_option": 4, "explanations": {"1": {"exist": false, "char_ranges": [], "word_ranges": [], "text": ""}, "2": {"exist": false, "char_ranges": [], "word_ranges": [], "text": ""}, "3": {"exist": false, "char_ranges": [], "word_ranges": [], "text": ""}, "4": {"exist": true, "char_ranges": [[329, 540]], "word_ranges": [[43, 79]], "text": "On nous parle d'un patient qui, quelques minutes après avoir commencé une transfusion, a présenté un état aigu et très grave (avec choc). Une telle situation aiguë et grave ne peut que correspondre à l'option 4."}, "5": {"exist": false, "char_ranges": [], "word_ranges": [], "text": ""}}} +{"id": 339, "year": 2016, "question_id_specific": 33, "full_question": "Une femme de 67 ans a été diagnostiquée avec un carcinome canalaire infiltrant du sein sans antécédents familiaux de néoplasie. Quelles sont les études complémentaires à réaliser sur la tumeur pour ses implications cliniques et thérapeutiques ?", "full_answer": "Tous les carcinomes mammaires doivent être accompagnés d'une étude des récepteurs hormonaux et de HER-2 en raison des implications pronostiques de la tumeur, ainsi que de l'adéquation du traitement, car en fonction du résultat, certains médicaments seront appliqués ou d'autres. L'étude des parents et du gène BCRA n'est pas indiquée car elle n'a pas d'antécédents familiaux de néoplasie.", "type": "GYNÉCOLOGIE ET OBSTÉTRIQUE", "options": {"1": "Étude phénotypique complète par cytométrie de flux.", "2": "Étude des récepteurs hormonaux et de HER2.", "3": "Étude des récepteurs hormonaux, de l'e-cadhérine et étude des parents au premier degré.", "4": "Dépistage BRCA 1-2 et dépistage des parents au premier degré.", "5": null}, "correct_option": 2, "explanations": {"1": {"exist": false, "char_ranges": [], "word_ranges": [], "text": ""}, "2": {"exist": true, "char_ranges": [[0, 198]], "word_ranges": [[0, 29]], "text": "Tous les carcinomes mammaires doivent être accompagnés d'une étude des récepteurs hormonaux et de HER-2 en raison des implications pronostiques de la tumeur, ainsi que de l'adéquation du traitement,"}, "3": {"exist": true, "char_ranges": [[279, 388]], "word_ranges": [[40, 58]], "text": "L'étude des parents et du gène BCRA n'est pas indiquée car elle n'a pas d'antécédents familiaux de néoplasie."}, "4": {"exist": true, "char_ranges": [[279, 388]], "word_ranges": [[40, 58]], "text": "L'étude des parents et du gène BCRA n'est pas indiquée car elle n'a pas d'antécédents familiaux de néoplasie."}, "5": {"exist": false, "char_ranges": [], "word_ranges": [], "text": ""}}} +{"id": 144, "year": 2012, "question_id_specific": 142, "full_question": "Une jeune fille de 13 ans nous a consultés pour perte de poids, asthénie, aménorrhée secondaire et constipation. L'examen a révélé une température de 35ºC, un poids inférieur au 3e percentile, une bradycardie et une hypotension orthostatique. Les analyses de laboratoire révèlent une anémie avec une légère leucopénie, un cholestérol de 230 mg/dl et, à l'électrocardiogramme, une onde T aplatie avec une élévation du segment ST et un QTc prolongé. Quel processus suspecteriez-vous et que feriez-vous ensuite ?", "full_answer": "La bonne réponse est 1. Les symptômes et les signes décrits sont les résultats biologiques associés à l'anorexie mentale. Les perturbations de l'ECG sont dues à des perturbations électrolytiques.", "type": "PÉDIATRIE", "options": {"1": "Je soupçonnerais une anorexie mentale et m'informerais sur sa consommation, son image corporelle, etc.", "2": "Je soupçonnerais une hypothyroïdie et demanderais une TSH et des hormones thyroïdiennes.", "3": "Je soupçonnerais une cardiomyopathie et demanderais un échocardiogramme.", "4": "Je soupçonnerais une maladie cœliaque et demanderais des anticorps IgA anti-transglutaminase.", "5": "Je soupçonnerais un panhypopituitarisme et demanderais une tomodensitométrie crânienne."}, "correct_option": 1, "explanations": {"1": {"exist": true, "char_ranges": [[24, 121]], "word_ranges": [[5, 19]], "text": "Les symptômes et les signes décrits sont les résultats biologiques associés à l'anorexie mentale."}, "2": {"exist": false, "char_ranges": [], "word_ranges": [], "text": ""}, "3": {"exist": false, "char_ranges": [], "word_ranges": [], "text": ""}, "4": {"exist": false, "char_ranges": [], "word_ranges": [], "text": ""}, "5": {"exist": false, "char_ranges": [], "word_ranges": [], "text": ""}}} +{"id": 356, "year": 2016, "question_id_specific": 166, "full_question": "Une femme enceinte de 10 semaines s'est présentée au service des urgences avec une tension artérielle de 160/105 mmHg. Elle était en bon état général et ne souffrait que de légers maux de tête, raison pour laquelle sa tension artérielle a été prise. Après 4 heures de repos, la tension artérielle était de 150/95 mmHg. L'hémogramme était normal et la protéinurie négative. De quel type d'hypertension souffre-t-il ?", "full_answer": "Hypertension chronique. Hypertension détectée avant la grossesse ou avant la 20e semaine de grossesse et/ou persistant jusqu'à 12 semaines après l'accouchement.", "type": "GYNÉCOLOGIE ET OBSTÉTRIQUE", "options": {"1": "Pré-éclampsie modérée.", "2": "Hypertension induite par la grossesse.", "3": "L'hypertension chronique.", "4": "Éclampsie.", "5": null}, "correct_option": 3, "explanations": {"1": {"exist": false, "char_ranges": [], "word_ranges": [], "text": ""}, "2": {"exist": false, "char_ranges": [], "word_ranges": [], "text": ""}, "3": {"exist": true, "char_ranges": [[24, 160]], "word_ranges": [[2, 21]], "text": "Hypertension détectée avant la grossesse ou avant la 20e semaine de grossesse et/ou persistant jusqu'à 12 semaines après l'accouchement."}, "4": {"exist": false, "char_ranges": [], "word_ranges": [], "text": ""}, "5": {"exist": false, "char_ranges": [], "word_ranges": [], "text": ""}}} +{"id": 495, "year": 2020, "question_id_specific": 64, "full_question": "Une femme de 65 ans, sans comorbidité, a subi une chirurgie conservatrice du sein gauche avec biopsie sélective du ganglion lymphatique sentinelle. L'anatomie pathologique de l'échantillon a montré un carcinome canalaire infiltrant de grade III de 11 mm, deux ganglions sentinelles négatifs, récepteurs aux œstrogènes : 0%, récepteurs à la progestérone : 0%, Ki 67 : 70%, HER2 : négatif. L'étude d'extension est négative. Le traitement adjuvant serait le suivant :", "full_answer": "Question typique, chirurgie conservatrice signifie radiothérapie, bien sûr. Il s'agit d'un cancer de haut grade, sans récepteurs hormonaux ni her2 (le redoutable triple négatif). J'étais là quand il n'y avait pas de taxanes ou d'herceptine et j'ai participé aux essais qui ont montré que de plus en plus de patients sont guéris, malheureusement pas encore tous. Aujourd'hui, les anthracyclines (pas de maladie cardiaque) et les taxanes avant la RT. Ma spécialité est magnifique, et elle le sera encore plus dans les années à venir.", "type": "ONCOLOGIE MÉDICALE", "options": {"1": "Chimiothérapie à base d'anthracycline et de taxane, radiothérapie et trastuzumab.", "2": "Chimiothérapie à base d'anthracyclines et de taxanes, radiothérapie et tamoxifène.", "3": "Chimiothérapie et radiothérapie à base d'anthracycline et de taxane.", "4": "Chimiothérapie à base d'anthracyclines et de taxanes, radiothérapie et inhibiteurs de l'aromatase, car elle est ménopausée.", "5": null}, "correct_option": 3, "explanations": {"1": {"exist": false, "char_ranges": [], "word_ranges": [], "text": ""}, "2": {"exist": false, "char_ranges": [], "word_ranges": [], "text": ""}, "3": {"exist": true, "char_ranges": [[362, 448]], "word_ranges": [[56, 69]], "text": "Aujourd'hui, les anthracyclines (pas de maladie cardiaque) et les taxanes avant la RT."}, "4": {"exist": false, "char_ranges": [], "word_ranges": [], "text": ""}, "5": {"exist": false, "char_ranges": [], "word_ranges": [], "text": ""}}} +{"id": 355, "year": 2016, "question_id_specific": 166, "full_question": "Une femme enceinte de 10 semaines s'est présentée au service des urgences avec une tension artérielle de 160/105 mmHg. Elle était en bon état général et ne souffrait que de légers maux de tête, raison pour laquelle sa tension artérielle a été prise. Après 4 heures de repos, la tension artérielle était de 150/95 mmHg. L'hémogramme était normal et la protéinurie négative. De quel type d'hypertension souffre-t-il ?", "full_answer": "La bonne réponse est 3. Dans son protocole de 2006 intitulé \"Troubles hypertensifs de la grossesse\", la SEGO fait référence à l'hypertension chronique, bien qu'elle ait été diagnostiquée pendant la grossesse. Cela s'explique par le fait qu'elle a été diagnostiquée à 10 semaines de gestation. Au-delà de 20 semaines de gestation, nous parlerions de pré-éclampsie (si HTA + protéinurie ou indice de pulsatilité de l'artère utérine supérieur au 95e percentile), d'hypertension induite par la grossesse (seulement HTA) ou d'éclampsie (une forme de pré-éclampsie sévère).", "type": "GYNÉCOLOGIE ET OBSTÉTRIQUE", "options": {"1": "Pré-éclampsie modérée.", "2": "Hypertension induite par la grossesse.", "3": "L'hypertension chronique.", "4": "Éclampsie.", "5": null}, "correct_option": 3, "explanations": {"1": {"exist": true, "char_ranges": [[293, 567]], "word_ranges": [[45, 84]], "text": "Au-delà de 20 semaines de gestation, nous parlerions de pré-éclampsie (si HTA + protéinurie ou indice de pulsatilité de l'artère utérine supérieur au 95e percentile), d'hypertension induite par la grossesse (seulement HTA) ou d'éclampsie (une forme de pré-éclampsie sévère)."}, "2": {"exist": true, "char_ranges": [[293, 567]], "word_ranges": [[45, 84]], "text": "Au-delà de 20 semaines de gestation, nous parlerions de pré-éclampsie (si HTA + protéinurie ou indice de pulsatilité de l'artère utérine supérieur au 95e percentile), d'hypertension induite par la grossesse (seulement HTA) ou d'éclampsie (une forme de pré-éclampsie sévère)."}, "3": {"exist": true, "char_ranges": [[24, 292]], "word_ranges": [[5, 45]], "text": "Dans son protocole de 2006 intitulé \"Troubles hypertensifs de la grossesse\", la SEGO fait référence à l'hypertension chronique, bien qu'elle ait été diagnostiquée pendant la grossesse. Cela s'explique par le fait qu'elle a été diagnostiquée à 10 semaines de gestation."}, "4": {"exist": true, "char_ranges": [[293, 567]], "word_ranges": [[45, 84]], "text": "Au-delà de 20 semaines de gestation, nous parlerions de pré-éclampsie (si HTA + protéinurie ou indice de pulsatilité de l'artère utérine supérieur au 95e percentile), d'hypertension induite par la grossesse (seulement HTA) ou d'éclampsie (une forme de pré-éclampsie sévère)."}, "5": {"exist": false, "char_ranges": [], "word_ranges": [], "text": ""}}} +{"id": 94, "year": 2012, "question_id_specific": 31, "full_question": "Un garçon de 17 ans atteint d'une maladie de Crohn avec atteinte colique depuis 2 ans, sous traitement d'entretien par azathioprine, consulte pour l'apparition depuis 5 jours de nodules sous-cutanés rouge violacé, chauds, douloureux, bilatéraux dans la région prétibiale, associés à une augmentation du nombre de selles et à des douleurs abdominales. L'approche la plus appropriée dans ce cas est la suivante :", "full_answer": "Il s'agit vraisemblablement d'un érythème noueux. Environ 10 % des cas d'érythème noueux sont associés à une maladie inflammatoire de l'intestin, qu'il s'agisse de la colite ulcéreuse ou de la maladie de Crohn. Comme nous l'avons mentionné, dans la plupart des cas, l'érythème noueux est spontanément résolutif. Lorsqu'il est associé à une maladie inflammatoire de l'intestin, l'érythème noueux disparaît généralement avec le traitement de la poussée intestinale et réapparaît avec les récidives de la maladie. Les mesures locales comprennent l'élévation des jambes et le repos au lit. Dans les cas particulièrement graves, l'administration orale d'iodure de potassium s'est avérée utile. Bien que l'aspirine et d'autres AINS soient efficaces contre l'érythème noueux, leur utilisation chez les patients atteints d'une maladie inflammatoire de l'intestin doit être limitée, car ils peuvent aggraver l'état de l'intestin, et les antidépresseurs n'ont pas été rapportés comme étant utiles dans cette pathologie.", "type": "DERMATOLOGIE, VÉNÉRÉOLOGIE ET CHIRURGIE PLASTIQUE", "options": {"1": "Recommandez un repos relatif et des compresses chaudes sur les deux jambes et ajoutez un traitement antidépresseur.", "2": "Effectuez des biopsies sur des zones de peau éloignées des zones blessées et prescrivez des analgésiques opioïdes dès le début.", "3": "Suspecter l'existence d'une lésion tumorale intestinale maligne comme facteur déclenchant du processus cutané.", "4": "Suspecter une ischémie bilatérale des membres inférieurs d'origine médicamenteuse.", "5": "Adaptation du traitement des maladies intestinales."}, "correct_option": 5, "explanations": {"1": {"exist": true, "char_ranges": [[689, 1009]], "word_ranges": [[100, 145]], "text": "Bien que l'aspirine et d'autres AINS soient efficaces contre l'érythème noueux, leur utilisation chez les patients atteints d'une maladie inflammatoire de l'intestin doit être limitée, car ils peuvent aggraver l'état de l'intestin, et les antidépresseurs n'ont pas été rapportés comme étant utiles dans cette pathologie."}, "2": {"exist": false, "char_ranges": [], "word_ranges": [], "text": ""}, "3": {"exist": false, "char_ranges": [], "word_ranges": [], "text": ""}, "4": {"exist": false, "char_ranges": [], "word_ranges": [], "text": ""}, "5": {"exist": true, "char_ranges": [[312, 510]], "word_ranges": [[47, 75]], "text": "Lorsqu'il est associé à une maladie inflammatoire de l'intestin, l'érythème noueux disparaît généralement avec le traitement de la poussée intestinale et réapparaît avec les récidives de la maladie."}}} +{"id": 39, "year": 2011, "question_id_specific": 134, "full_question": "Femme de 60 ans chez qui on a diagnostiqué un cancer du sein traité par chirurgie et radiothérapie il y a 10 ans. Elle présente actuellement des métastases dans la colonne vertébrale dorsale et l'articulation sacro-iliaque, pour lesquelles elle a reçu un traitement mensuel par acide zolédronique i.v. au cours des deux dernières années. Elle s'est présentée à la clinique avec une exposition osseuse spontanée d'environ 2 cm au niveau de l'os mandibulaire. Il n'y avait aucun signe d'infection aiguë et la gencive autour de l'exposition osseuse était tout à fait normale. Le scanner de la mandibule montre une zone de sclérose osseuse. Quel est le diagnostic ?", "full_answer": "J'ai trouvé cette question difficile, alors excluons-la. 1 ne peut pas être, parce qu'il y aurait des signes infectieux. 2 est possible car les bisphosphonates provoquent une nécrose mandibulaire. 3 me semble peu probable car la radiothérapie que la patiente a reçue aurait été dirigée vers le sein et non vers la mâchoire. J'exclus le 4, car il ne s'agit pas d'un cas d'atteinte des racines dentaires. Le 5 est possible ; après tout, la patiente a déjà des métastases osseuses. J'hésite donc entre le 2 et le 5, lequel dois-je choisir ? Je ne sais pas ce que signifie une sclérose osseuse au scanner, mais une métastase osseuse serait plus destructrice qu'une sclérose. D'autre part, en recourant à la psychologie inversée, qu'est-ce que l'auteur de la question voulait que je sache ? Que les bisphosphonates peuvent créer une nécrose mandibulaire ? Ou qu'une femme ayant plusieurs métastases osseuses peut également avoir une métastase dans la mâchoire ? Cette dernière hypothèse me semble trop simple et me ferait plutôt pencher pour la 2, mais elle n'est pas claire pour moi. Quoi qu'il en soit, entre deux réponses possibles, il faut toujours répondre, et entre 2 et 5, il faut cocher 2, qui est le plus probable.", "type": "OTORHINOLARYNGOLOGIE ET CHIRURGIE MAXILLO-FACIALE", "options": {"1": "Abcès odontogène.", "2": "Ostéonécrose due aux bisphosphonates.", "3": "Ostéoradionécrose.", "4": "Racine dentaire incluse.", "5": "Métastases du carcinome mammaire."}, "correct_option": 2, "explanations": {"1": {"exist": true, "char_ranges": [[57, 120]], "word_ranges": [[7, 19]], "text": "1 ne peut pas être, parce qu'il y aurait des signes infectieux."}, "2": {"exist": true, "char_ranges": [[602, 670]], "word_ranges": [[105, 114]], "text": "mais une métastase osseuse serait plus destructrice qu'une sclérose."}, "3": {"exist": true, "char_ranges": [[197, 323]], "word_ranges": [[29, 53]], "text": "3 me semble peu probable car la radiothérapie que la patiente a reçue aurait été dirigée vers le sein et non vers la mâchoire."}, "4": {"exist": true, "char_ranges": [[324, 478]], "word_ranges": [[53, 81]], "text": "J'exclus le 4, car il ne s'agit pas d'un cas d'atteinte des racines dentaires. Le 5 est possible ; après tout, la patiente a déjà des métastases osseuses."}, "5": {"exist": true, "char_ranges": [[602, 670]], "word_ranges": [[105, 114]], "text": "mais une métastase osseuse serait plus destructrice qu'une sclérose."}}} +{"id": 252, "year": 2014, "question_id_specific": 83, "full_question": "Quelle maladie devrions-nous suspecter chez un jeune patient qui rapporte des antécédents fréquents de dysphagie aux solides et aux liquides avec des impactions alimentaires répétées, sans symptômes de brûlures d'estomac et chez qui l'endoscopie buccale montre de multiples anneaux œsophagiens concentriques avec une muqueuse normale (aspect trachéalisé) ?", "full_answer": "L'aspect endoscopique de l'œsophage trachéalisé est très typique de l'œsophagite à éosinophiles, qui se manifeste par une dysphagie et des impactions alimentaires répétées.", "type": "SYSTÈME DIGESTIF", "options": {"1": "Œsophagite herpétique.", "2": "Oesophagite à éosinophiles.", "3": "Oesophagite à candidose.", "4": "Oesophagite à cytomégalovirus.", "5": "Adénocarcinome de l'œsophage."}, "correct_option": 2, "explanations": {"1": {"exist": false, "char_ranges": [], "word_ranges": [], "text": ""}, "2": {"exist": true, "char_ranges": [[0, 172]], "word_ranges": [[0, 23]], "text": "L'aspect endoscopique de l'œsophage trachéalisé est très typique de l'œsophagite à éosinophiles, qui se manifeste par une dysphagie et des impactions alimentaires répétées."}, "3": {"exist": false, "char_ranges": [], "word_ranges": [], "text": ""}, "4": {"exist": false, "char_ranges": [], "word_ranges": [], "text": ""}, "5": {"exist": false, "char_ranges": [], "word_ranges": [], "text": ""}}} +{"id": 57, "year": 2011, "question_id_specific": 77, "full_question": "Chez une femme obèse de 42 ans, les taux de cortisol dans le sang, l'urine et la salive sont déterminés et sont élevés dans tous les tests. Après avoir effectué le test de suppression nocturne avec la dexaméthasone, les taux de cortisol dans le sang restent élevés, de sorte que le syndrome de Cushing indépendant de l'ACTH est suspecté, quel test supplémentaire demanderiez-vous pour confirmer le diagnostic à ce stade ?", "full_answer": "Quoi qu'il en soit, je pense que le cas clinique est mal formulé, puisqu'avant d'effectuer la tomodensitométrie, il faut mesurer l'ACTH et, une fois que l'ACTH est supprimée, il faut effectuer la tomodensitométrie. Mais comme la question est \"que feriez-vous pour confirmer le diagnostic de suspicion de syndrome de Cushing indépendant de l'ACTH\", la réponse est le scanner.", "type": "ENDOCRINOLOGIE", "options": {"1": "Un cathétérisme du sinus pétrosal.", "2": "Un scanner avec de la somatostatine marquée (Octreoscan).", "3": "Un scanner Sesta-MIBI.", "4": "Une échographie abdominale.", "5": "Une tomodensitométrie des surrénales."}, "correct_option": 5, "explanations": {"1": {"exist": false, "char_ranges": [], "word_ranges": [], "text": ""}, "2": {"exist": false, "char_ranges": [], "word_ranges": [], "text": ""}, "3": {"exist": false, "char_ranges": [], "word_ranges": [], "text": ""}, "4": {"exist": false, "char_ranges": [], "word_ranges": [], "text": ""}, "5": {"exist": false, "char_ranges": [], "word_ranges": [], "text": ""}}} +{"id": 595, "year": 2022, "question_id_specific": 88, "full_question": "Un garçon de 23 mois présente de la fièvre, une hypotension, des vomissements répétés, des signes méningés, des pétéchies et des ecchymoses. L'hémogramme montre 25 000 leucocytes (75 % de neutrophiles, 10 % de globules blancs) et 12 000 plaquettes/microL et l'étude de la coagulation montre une activité prothrombinique de 35 %. Quelle option n'est PAS appropriée dans sa prise en charge ?", "full_answer": "Patient présentant une suspicion de septicémie à méningocoques. La stabilisation liquidienne (hypotension) selon la méthode ABCDE est essentielle. Une hémoculture peut être obtenue en même temps que le test sanguin initial. Une antibiothérapie précoce (céfotaxime) est essentielle. Cependant, chez un patient instable comme le nôtre, une ponction lombaire n'est pas indiquée.", "type": "PÉDIATRIE", "options": {"1": "Administration de céfotaxime par voie intraveineuse.", "2": "Réalisation d'une ponction lombaire.", "3": "Prélèvement d'hémoculture.", "4": "Liquides intraveineux.", "5": null}, "correct_option": 2, "explanations": {"1": {"exist": true, "char_ranges": [[224, 281]], "word_ranges": [[31, 37]], "text": "Une antibiothérapie précoce (céfotaxime) est essentielle."}, "2": {"exist": true, "char_ranges": [[282, 375]], "word_ranges": [[37, 51]], "text": "Cependant, chez un patient instable comme le nôtre, une ponction lombaire n'est pas indiquée."}, "3": {"exist": true, "char_ranges": [[147, 223]], "word_ranges": [[18, 31]], "text": "Une hémoculture peut être obtenue en même temps que le test sanguin initial."}, "4": {"exist": true, "char_ranges": [[0, 146]], "word_ranges": [[0, 18]], "text": "Patient présentant une suspicion de septicémie à méningocoques. La stabilisation liquidienne (hypotension) selon la méthode ABCDE est essentielle."}, "5": {"exist": false, "char_ranges": [], "word_ranges": [], "text": ""}}} +{"id": 98, "year": 2012, "question_id_specific": 171, "full_question": "Le patient présente depuis 2 semaines une éruption prurigineuse constituée de multiples papules polygonales érythémato-violacées à surface aplatie, localisées principalement sur la face antérieure des poignets, la zone prétibiale et la zone lombaire. Il présente également des lésions réticulées blanchâtres sur la muqueuse jugale. Quel est le diagnostic le plus probable ?", "full_answer": "Le lichen plan est une dermatose très fréquente d'étiologie inconnue, caractérisée par l'apparition de papules polygonales prurigineuses, de couleur violacée et à la surface brillante, situées de préférence sur les zones distales des extrémités. Ces papules présentent une distribution symétrique et sont préférentiellement localisées sur la face antérieure des poignets (Figure 1) et des chevilles. L'atteinte de la muqueuse buccale est très fréquente et environ 75 % des patients atteints de lichen plan présentent une réticulation blanchâtre asymptomatique affectant la muqueuse jugale ou linguale.", "type": "DERMATOLOGIE, VÉNÉRÉOLOGIE ET CHIRURGIE PLASTIQUE", "options": {"1": "Pityriasis versicolor.", "2": "Mycosis fungoides.", "3": "Psoriasis en gouttes.", "4": "Pityriasis rosea de Gibert.", "5": "Lichen plan."}, "correct_option": 5, "explanations": {"1": {"exist": false, "char_ranges": [], "word_ranges": [], "text": ""}, "2": {"exist": false, "char_ranges": [], "word_ranges": [], "text": ""}, "3": {"exist": false, "char_ranges": [], "word_ranges": [], "text": ""}, "4": {"exist": false, "char_ranges": [], "word_ranges": [], "text": ""}, "5": {"exist": true, "char_ranges": [[0, 399]], "word_ranges": [[0, 55]], "text": "Le lichen plan est une dermatose très fréquente d'étiologie inconnue, caractérisée par l'apparition de papules polygonales prurigineuses, de couleur violacée et à la surface brillante, situées de préférence sur les zones distales des extrémités. Ces papules présentent une distribution symétrique et sont préférentiellement localisées sur la face antérieure des poignets (Figure 1) et des chevilles."}}} +{"id": 118, "year": 2012, "question_id_specific": 117, "full_question": "Un garçon asymptomatique de 14 ans, dont le père vient d'être diagnostiqué avec une tuberculose pulmonaire, subit un test de Mantoux dont le résultat est négatif. Quelle est l'approche appropriée ?", "full_answer": "Chez les jeunes contacts, la chimioprophylaxie doit être entamée même si le test de Mantoux est négatif et répétée après 2 ou 3 mois pour décider de poursuivre le traitement jusqu'à 6 mois (s'il se révèle positif) ou de l'arrêter.", "type": "INFECTOLOGIE", "options": {"1": "Rassurez-le en lui disant qu'il n'est pas en danger.", "2": "Effectuer une radiographie pulmonaire.", "3": "Commencer une chimioprophylaxie et répéter le test cutané après 3 mois.", "4": "Répétez le test cutané après un mois.", "5": "Une étude des expectorations doit être réalisée avant de commencer la chimioprophylaxie par l'isoniazide pendant un an."}, "correct_option": 3, "explanations": {"1": {"exist": false, "char_ranges": [], "word_ranges": [], "text": ""}, "2": {"exist": false, "char_ranges": [], "word_ranges": [], "text": ""}, "3": {"exist": true, "char_ranges": [[0, 230]], "word_ranges": [[0, 40]], "text": "Chez les jeunes contacts, la chimioprophylaxie doit être entamée même si le test de Mantoux est négatif et répétée après 2 ou 3 mois pour décider de poursuivre le traitement jusqu'à 6 mois (s'il se révèle positif) ou de l'arrêter."}, "4": {"exist": false, "char_ranges": [], "word_ranges": [], "text": ""}, "5": {"exist": false, "char_ranges": [], "word_ranges": [], "text": ""}}} +{"id": 388, "year": 2016, "question_id_specific": 233, "full_question": "Un garçon de 13 ans s'est présenté en pédiatrie pour une sensation de brûlures d'estomac pendant et après les repas et une dysphagie occasionnelle, qui augmentait avec l'ingestion d'aliments solides tels que la viande. Une hernie hiatale a été diagnostiquée chez sa mère. Les antécédents personnels du patient comprenaient une allergie aux pollens, aux acariens et au latex. L'examen physique ne révèle rien de significatif, à l'exception de lésions eczématiformes dans les creux poplités et antécubitaux. Parmi les examens complémentaires suivants, lesquels NE réaliseriez-vous PAS en fonction des diagnostics les plus probables ?", "full_answer": "On nous présente un cas d'œsophagite à éosinophiles (adolescent de sexe masculin, présentant des brûlures d'estomac et une dysphagie, et ayant des antécédents de dermatite atopique). De tous les tests proposés, l'option 3 serait la moins indiquée dans ce cas.", "type": "PÉDIATRIE", "options": {"1": "Endoscopie gastro-intestinale supérieure.", "2": "pHmétrie œsophagienne sur 24 heures.", "3": "Test respiratoire au carbone 13 pour l'Helicobacter pylori.", "4": "Étude barytée oesophago-gastro-duodénale.", "5": null}, "correct_option": 3, "explanations": {"1": {"exist": false, "char_ranges": [], "word_ranges": [], "text": ""}, "2": {"exist": false, "char_ranges": [], "word_ranges": [], "text": ""}, "3": {"exist": true, "char_ranges": [[0, 259]], "word_ranges": [[0, 40]], "text": "On nous présente un cas d'œsophagite à éosinophiles (adolescent de sexe masculin, présentant des brûlures d'estomac et une dysphagie, et ayant des antécédents de dermatite atopique). De tous les tests proposés, l'option 3 serait la moins indiquée dans ce cas."}, "4": {"exist": false, "char_ranges": [], "word_ranges": [], "text": ""}, "5": {"exist": false, "char_ranges": [], "word_ranges": [], "text": ""}}} +{"id": 421, "year": 2018, "question_id_specific": 78, "full_question": "Un homme de 52 ans a été adressé au service gastro-intestinal pour hématochézie, ténesme et diminution du diamètre des selles. Une série d'examens a été réalisée et un diagnostic d'adénocarcinome du sigma sans métastase à distance a été posé. Le patient a subi une intervention chirurgicale et a été adressé au service d'oncologie médicale pour évaluer un traitement complémentaire par chimiothérapie. Parmi les facteurs suivants, lequel est associé à un mauvais pronostic après une résection chirurgicale et doit être pris en compte lors de la planification d'un traitement par chimiothérapie ?", "full_answer": "À cette occasion, on nous demande quels sont les facteurs qui influencent le plus l'utilisation de la chimiothérapie adjuvante dans le cancer colorectal. Pour ce faire, nous devons savoir quels sont les facteurs de mauvais pronostic dans cette pathologie, où l'obstruction ou la perforation intestinale due à la lésion, l'invasion des organes adjacents, l'élévation des marqueurs tumoraux tels que l'ACE et l'histologie du néoplasme se distinguent. La taille de la lésion n'est pas aussi importante que le degré d'invasion de la paroi intestinale ou d'autres structures adjacentes, un critère qui doit être pris en compte pour la stadification à l'aide des classifications TNM ou Astler-Coller, qui guident l'indication d'un traitement adjuvant après l'approche chirurgicale.", "type": "CHIRURGIE GÉNÉRALE", "options": {"1": "La présence d'une anémie au moment du diagnostic.", "2": "L'existence d'antécédents familiaux de cancer colorectal.", "3": "La taille de la lésion primaire et la différenciation histologique.", "4": "Perforation ou adhésion de la tumeur aux organes adjacents.", "5": null}, "correct_option": 4, "explanations": {"1": {"exist": false, "char_ranges": [], "word_ranges": [], "text": ""}, "2": {"exist": false, "char_ranges": [], "word_ranges": [], "text": ""}, "3": {"exist": true, "char_ranges": [[449, 498]], "word_ranges": [[66, 75]], "text": "La taille de la lésion n'est pas aussi importante"}, "4": {"exist": true, "char_ranges": [[18, 319]], "word_ranges": [[3, 49]], "text": "on nous demande quels sont les facteurs qui influencent le plus l'utilisation de la chimiothérapie adjuvante dans le cancer colorectal. Pour ce faire, nous devons savoir quels sont les facteurs de mauvais pronostic dans cette pathologie, où l'obstruction ou la perforation intestinale due à la lésion,"}, "5": {"exist": false, "char_ranges": [], "word_ranges": [], "text": ""}}} +{"id": 500, "year": 2020, "question_id_specific": 45, "full_question": "Une jeune femme de 15 ans présentant un retard de la ménarche et une petite taille n'a pas de déficience intellectuelle. Lequel des tests génétiques suivants serait systématiquement utilisé pour diagnostiquer cette patiente ?", "full_answer": "Chez une femme présentant un retard de puberté et une petite taille, il faut toujours envisager et exclure le syndrome de Turner, dont le diagnostic est établi par caryotypage (45X0).", "type": "PÉDIATRIE", "options": {"1": "Séquençage en masse (NGS).", "2": "POISSONS.", "3": "Microréseaux d'ADN et/ou d'ARN.", "4": "Caryotype.", "5": null}, "correct_option": 4, "explanations": {"1": {"exist": false, "char_ranges": [], "word_ranges": [], "text": ""}, "2": {"exist": false, "char_ranges": [], "word_ranges": [], "text": ""}, "3": {"exist": false, "char_ranges": [], "word_ranges": [], "text": ""}, "4": {"exist": true, "char_ranges": [[0, 183]], "word_ranges": [[0, 30]], "text": "Chez une femme présentant un retard de puberté et une petite taille, il faut toujours envisager et exclure le syndrome de Turner, dont le diagnostic est établi par caryotypage (45X0)."}, "5": {"exist": false, "char_ranges": [], "word_ranges": [], "text": ""}}} +{"id": 611, "year": 2022, "question_id_specific": 116, "full_question": "Une femme de 95 ans vivant dans une maison de retraite, indépendante pour les activités de base de la vie quotidienne, sort se promener dans le jardin. Elle a des antécédents d'hypertension, de dyslipidémie, d'ostéoporose et de troubles cognitifs légers. Elle fait une chute en se levant la nuit pour aller aux toilettes. La radiographie montre une fracture sous-capitale déplacée de la hanche droite. Quel est le traitement recommandé ?", "full_answer": "On nous rapporte le cas d'une patiente de 95 ans présentant une fracture intracapsulaire (sous-capitale) déplacée de la hanche. Dans ce cas, les options chirurgicales peuvent être l'utilisation de vis canulées ou l'arthroplastie de la hanche. Les vis perforées sont réservées aux fractures non déplacées (réponse incorrecte 2) et l'option la plus indiquée serait l'arthroplastie, dans ce cas l'hémiarthroplastie (réponse correcte 1). Les broches trochantériennes sont réservées aux fractures extracapsulaires (Mauvaise réponse 2) et le traitement conservateur n'est envisagé que chez les patients présentant une très faible demande fonctionnelle ou une contre-indication chirurgicale (Mauvaise réponse 4).", "type": "TRAUMATOLOGIE", "options": {"1": "Hémiarthroplastie de la hanche.", "2": "Fixation du clou trochantérien.", "3": "Fixation par vis canulées.", "4": "Conservateur : la vie en fauteuil roulant.", "5": null}, "correct_option": 1, "explanations": {"1": {"exist": true, "char_ranges": [[243, 433]], "word_ranges": [[36, 62]], "text": "Les vis perforées sont réservées aux fractures non déplacées (réponse incorrecte 2) et l'option la plus indiquée serait l'arthroplastie, dans ce cas l'hémiarthroplastie (réponse correcte 1)."}, "2": {"exist": true, "char_ranges": [[243, 529]], "word_ranges": [[36, 73]], "text": "Les vis perforées sont réservées aux fractures non déplacées (réponse incorrecte 2) et l'option la plus indiquée serait l'arthroplastie, dans ce cas l'hémiarthroplastie (réponse correcte 1). Les broches trochantériennes sont réservées aux fractures extracapsulaires (Mauvaise réponse 2)"}, "3": {"exist": false, "char_ranges": [], "word_ranges": [], "text": ""}, "4": {"exist": true, "char_ranges": [[533, 705]], "word_ranges": [[74, 96]], "text": "le traitement conservateur n'est envisagé que chez les patients présentant une très faible demande fonctionnelle ou une contre-indication chirurgicale (Mauvaise réponse 4)."}, "5": {"exist": false, "char_ranges": [], "word_ranges": [], "text": ""}}} +{"id": 542, "year": 2022, "question_id_specific": 38, "full_question": "Une femme de 25 ans a des antécédents d'asthme bronchique allergique dû à une sensibilisation aux pollens de graminées et à une dermatite atopique. Elle a soudainement présenté une hypotension, une tachycardie, une urticaire et un bronchospasme après avoir ingéré des cacahuètes et a été emmenée au service des urgences. Quel serait l'ordre des médicaments à administrer et quels tests de laboratoire pourraient être demandés pour une évaluation ultérieure ?", "full_answer": "Le patient souffre d'un choc anaphylactique consécutif à l'ingestion de cacahuètes. Le traitement de premier choix pour l'anaphylaxie et le choc anaphylactique est l'adrénaline intramusculaire 1mg/ml (dose de 0,01 mg/kg, chez les adultes 0,5 mg au maximum). Il est suivi d'un traitement symptomatique visant à stabiliser les voies respiratoires avec 100 % d'O2 à haut débit, à assurer un accès veineux de gros calibre, à remplacer les liquides et à assurer une surveillance continue (HR, BP, SatO2 , diurèse). Comme traitement adjuvant, un bronchodilatateur inhalé tel que le SABA, la dexchlorphéniramine intramusculaire ou des corticostéroïdes tels que l'hydrocortisone IV ou la méthylprednisolone IV peuvent être administrés. L'adrénaline peut être répétée aux mêmes doses toutes les 5 minutes, au maximum 3 fois, et si après 15 minutes il n'y a pas de réponse, une perfusion d'adrénaline IV doit être mise en place, de Glucagon (en cas de traitement par bêta-bloquants), d'Atropine (en cas de bradycardie prolongée) ou de Vasopresseurs (en cas d'hypotension réfractaire). Le marqueur de laboratoire à demander en cas de suspicion d'anaphylaxie est la tryptase sérique dans la première heure pour une mesure de référence et une autre après 4 à 6 heures pour observer sa courbe d'élévation. Il ne s'agit pas d'un marqueur d'urgence, mais il est très utile pour les études allergologiques futures.", "type": "ALLERGOLOGIE", "options": {"1": "Adrénaline intramusculaire, suivie d'un bronchodilatateur inhalé de type SABA et de dexchlorphéniramine intramusculaire. Demander ensuite le dosage de la tryptase sérique.", "2": "Un bronchodilatateur inhalé de type SABA, suivi de dexchlorphéniramine intramusculaire et d'adrénaline intramusculaire. Demander ensuite le dosage de la tryptase sérique.", "3": "Dexchlorphéniramine intramusculaire, suivi d'un bronchodilatateur de type SABA inhalé et d'adrénaline intramusculaire. Demander ensuite une protéine cationique d'éosinophiles.", "4": "Adrénaline intramusculaire, suivie d'un bronchodilatateur de type SABA inhalé et de dexchlorphéniramine intramusculaire. Demander ensuite une protéine cationique d'éosinophiles.", "5": null}, "correct_option": 1, "explanations": {"1": {"exist": true, "char_ranges": [[1075, 1291]], "word_ranges": [[162, 199]], "text": "Le marqueur de laboratoire à demander en cas de suspicion d'anaphylaxie est la tryptase sérique dans la première heure pour une mesure de référence et une autre après 4 à 6 heures pour observer sa courbe d'élévation."}, "2": {"exist": false, "char_ranges": [], "word_ranges": [], "text": ""}, "3": {"exist": false, "char_ranges": [], "word_ranges": [], "text": ""}, "4": {"exist": true, "char_ranges": [[1075, 1291]], "word_ranges": [[162, 199]], "text": "Le marqueur de laboratoire à demander en cas de suspicion d'anaphylaxie est la tryptase sérique dans la première heure pour une mesure de référence et une autre après 4 à 6 heures pour observer sa courbe d'élévation."}, "5": {"exist": false, "char_ranges": [], "word_ranges": [], "text": ""}}} +{"id": 79, "year": 2012, "question_id_specific": 36, "full_question": "Laquelle des données cliniques suivantes, relevées lors de l'anamnèse d'un patient présentant une diarrhée de plus de 4 semaines d'évolution, peut être retrouvée chez des personnes ne présentant pas de pathologie organique :", "full_answer": "Bien que les mucosités aient tendance à faire un peu plus peur en consultation, elles entrent dans les critères de Rome II pour le diagnostic du syndrome de l'intestin irritable. Tous les autres sont des signes typiques d'organicité, même si je pense que la question est mal formulée, car toutes ces données (surtout l'âge) peuvent être retrouvées chez des patients qui n'ont finalement rien, mais dans ces cas-là, il faut toujours rechercher l'organicité.", "type": "SYSTÈME DIGESTIF", "options": {"1": "Perte de poids avec ou sans rectorragie.", "2": "Persistance de la diarrhée pendant la nuit.", "3": "Début des symptômes après l'âge de 50 ans.", "4": "Présence de dermatite et/ou d'arthrite à l'examen.", "5": "Expulsion de mucus dans plus de 25 % des selles."}, "correct_option": 5, "explanations": {"1": {"exist": true, "char_ranges": [[179, 233]], "word_ranges": [[30, 38]], "text": "Tous les autres sont des signes typiques d'organicité,"}, "2": {"exist": true, "char_ranges": [[179, 233]], "word_ranges": [[30, 38]], "text": "Tous les autres sont des signes typiques d'organicité,"}, "3": {"exist": true, "char_ranges": [[179, 233]], "word_ranges": [[30, 38]], "text": "Tous les autres sont des signes typiques d'organicité,"}, "4": {"exist": true, "char_ranges": [[179, 233]], "word_ranges": [[30, 38]], "text": "Tous les autres sont des signes typiques d'organicité,"}, "5": {"exist": true, "char_ranges": [[0, 178]], "word_ranges": [[0, 30]], "text": "Bien que les mucosités aient tendance à faire un peu plus peur en consultation, elles entrent dans les critères de Rome II pour le diagnostic du syndrome de l'intestin irritable."}}} +{"id": 524, "year": 2021, "question_id_specific": 54, "full_question": "Une femme de 86 ans souffre de fibrillation auriculaire non valvulaire. Son score CHADS2 est de 3 points. Dans la littérature, des patients similaires sous warfarine ont un risque d'AVC de 2,2 % contre 5,2 % chez les patients sans warfarine. Quel serait le nombre nécessaire à traiter (NNT) pour prévenir un AVC embolique avec un traitement anticoagulant ?", "full_answer": "En moyenne, le traitement de 100 patients comme cette femme par la warfarine réduit le nombre d'accidents vasculaires cérébraux de 3 (de 5,2 patients non traités à 3,2 patients traités). Par conséquent, si vous traitez 33,3 patients comme cette femme avec de la warfarine, vous réduisez le nombre d'accidents vasculaires cérébraux de 1 : le NNT est de 33,3.", "type": "STATISTIQUES", "options": {"1": "3.", "2": "19,2.", "3": "33,3.", "4": "49,5.", "5": null}, "correct_option": 3, "explanations": {"1": {"exist": false, "char_ranges": [], "word_ranges": [], "text": ""}, "2": {"exist": false, "char_ranges": [], "word_ranges": [], "text": ""}, "3": {"exist": true, "char_ranges": [[0, 357]], "word_ranges": [[0, 59]], "text": "En moyenne, le traitement de 100 patients comme cette femme par la warfarine réduit le nombre d'accidents vasculaires cérébraux de 3 (de 5,2 patients non traités à 3,2 patients traités). Par conséquent, si vous traitez 33,3 patients comme cette femme avec de la warfarine, vous réduisez le nombre d'accidents vasculaires cérébraux de 1 : le NNT est de 33,3."}, "4": {"exist": false, "char_ranges": [], "word_ranges": [], "text": ""}, "5": {"exist": false, "char_ranges": [], "word_ranges": [], "text": ""}}} +{"id": 319, "year": 2016, "question_id_specific": 141, "full_question": "Garçon de 20 ans, qui a consulté pour des douleurs lombo-sacrées avec un rythme inflammatoire de 4 mois d'évolution. Il souffre également de thalgies bilatérales et d'une raideur matinale pendant une heure. Au cours des deux derniers mois, il a présenté des symptômes diarrhéiques avec une perte de poids de 4 kg. Quelle est l'approche diagnostique la plus correcte ?", "full_answer": "Le tableau clinique est typique de la spondylarthrite et, avec les symptômes digestifs, il est probablement associé à une maladie inflammatoire de l'intestin.", "type": "RHEUMATOLOGIE", "options": {"1": "Compte tenu de l'âge du patient, il est très probable qu'il souffre de lombalgies non spécifiques et de tendinites aux pieds. Si la diarrhée persiste, un bilan digestif doit être effectué.", "2": "J'effectuerais une étude digestive pour exclure une pathologie tumorale. La douleur lombaire peut être due à une pathologie viscérale.", "3": "Le tableau clinique est très évocateur d'une spondylarthrite. Une maladie inflammatoire de l'intestin doit être exclue.", "4": "Je demanderais une IRM lombaire pour exclure une hernie discale et, si la diarrhée persiste, une étude digestive.", "5": null}, "correct_option": 3, "explanations": {"1": {"exist": false, "char_ranges": [], "word_ranges": [], "text": ""}, "2": {"exist": false, "char_ranges": [], "word_ranges": [], "text": ""}, "3": {"exist": true, "char_ranges": [[0, 158]], "word_ranges": [[0, 23]], "text": "Le tableau clinique est typique de la spondylarthrite et, avec les symptômes digestifs, il est probablement associé à une maladie inflammatoire de l'intestin."}, "4": {"exist": false, "char_ranges": [], "word_ranges": [], "text": ""}, "5": {"exist": false, "char_ranges": [], "word_ranges": [], "text": ""}}} +{"id": 400, "year": 2016, "question_id_specific": 147, "full_question": "Un garçon de 13 ans présente des douleurs dorsales depuis plusieurs mois, uniquement en se levant et en marchant. Quel diagnostic faut-il envisager ?", "full_answer": "La douleur est de nature mécanique, et la cause la plus fréquente des douleurs dorsales mécaniques chez les adolescents est la surcharge mécanique. Il n'y a pas de données dans la déclaration pour suggérer d'autres étiologies.", "type": "TRAUMATOLOGIE ET ORTHOPÉDIE", "options": {"1": "Maladie de Scheuermann.", "2": "Scoliose thoracique.", "3": "Tumeur maligne du corps vertébral.", "4": "Il s'agit d'une surcharge mécanique.", "5": null}, "correct_option": 4, "explanations": {"1": {"exist": true, "char_ranges": [[148, 226]], "word_ranges": [[23, 36]], "text": "Il n'y a pas de données dans la déclaration pour suggérer d'autres étiologies."}, "2": {"exist": true, "char_ranges": [[148, 226]], "word_ranges": [[23, 36]], "text": "Il n'y a pas de données dans la déclaration pour suggérer d'autres étiologies."}, "3": {"exist": true, "char_ranges": [[148, 226]], "word_ranges": [[23, 36]], "text": "Il n'y a pas de données dans la déclaration pour suggérer d'autres étiologies."}, "4": {"exist": true, "char_ranges": [[0, 147]], "word_ranges": [[0, 23]], "text": "La douleur est de nature mécanique, et la cause la plus fréquente des douleurs dorsales mécaniques chez les adolescents est la surcharge mécanique."}, "5": {"exist": false, "char_ranges": [], "word_ranges": [], "text": ""}}} +{"id": 196, "year": 2013, "question_id_specific": 164, "full_question": "Un garçon de 4 ans consulte pour l'apparition, depuis trois jours, de lésions cutanées sur les jambes et les fesses. Ses parents ont indiqué que son pédiatre avait diagnostiqué une infection des voies respiratoires supérieures dix jours plus tôt. Au cours des 12 dernières heures, il a présenté des douleurs abdominales intenses avec coliques et a eu deux selles diarrhéiques. Afebrile. Pas de perte de poids. L'examen physique révèle de nombreuses pétéchies et des lésions purpuriques palpables, principalement sur les fesses et les membres inférieurs. Bon état général, bien qu'elle souffre de douleurs abdominales intenses. La palpation abdominale est difficile à évaluer en raison de la douleur diffuse. Pas de viscéro-mégalie. Aucun autre élément intéressant à l'examen. Lequel des éléments suivants ne renforce PAS votre suspicion diagnostique ?", "full_answer": "Il décrit un purpura de Schönlein-Henoch qui est un phénomène vasculaire, de sorte que les lésions ne sont pas dues à une plaquettopénie. L'œdème du scrotum (parfois même la torsion de la capside de Morgani du testicule) n'est pas exceptionnel chez les hommes atteints de S Schönlein-Henoch.", "type": "PÉDIATRIE", "options": {"1": "Arthrite du genou et de la cheville.", "2": "Hématurie.", "3": "La plaquettopénie.", "4": "Sang occulte dans les selles.", "5": "Œdème du scrotum."}, "correct_option": 3, "explanations": {"1": {"exist": false, "char_ranges": [], "word_ranges": [], "text": ""}, "2": {"exist": false, "char_ranges": [], "word_ranges": [], "text": ""}, "3": {"exist": false, "char_ranges": [], "word_ranges": [], "text": ""}, "4": {"exist": false, "char_ranges": [], "word_ranges": [], "text": ""}, "5": {"exist": true, "char_ranges": [[0, 291]], "word_ranges": [[0, 47]], "text": "Il décrit un purpura de Schönlein-Henoch qui est un phénomène vasculaire, de sorte que les lésions ne sont pas dues à une plaquettopénie. L'œdème du scrotum (parfois même la torsion de la capside de Morgani du testicule) n'est pas exceptionnel chez les hommes atteints de S Schönlein-Henoch."}}} +{"id": 509, "year": 2021, "question_id_specific": 150, "full_question": "Un homme de 83 ans a consulté pour des douleurs abdominales récurrentes dans la fosse iliaque gauche, accompagnées d'une diarrhée allant jusqu'à 6 selles par jour sans produits pathologiques, d'une durée de 10 à 15 jours. Il ne signale ni fièvre ni malaise général. Une analyse sanguine récente n'a pas révélé de leucocytose et la recherche de sang occulte dans les selles s'est révélée négative. L'examen a révélé une légère douleur à la palpation de la fosse iliaque gauche sans irritation péritonéale. Une coloscopie réalisée il y a trois ans a révélé des diverticules dans tout le côlon, plus nombreux dans le côlon gauche, sans autres lésions associées. Quelle est la meilleure approche à adopter parmi les suivantes ?", "full_answer": "Ils se réfèrent à un concept relativement nouveau qui est apparu assez récemment et qui n'est pas présent dans toutes les directives cliniques, à savoir la maladie diverticulaire symptomatique non compliquée. Dans ces cas, un essai thérapeutique avec la rifaximine est recommandé, le vrai étant le numéro 1. L'absence de réactifs de phase aiguë dans l'analyse de laboratoire suggère que le patient n'avait pas de diverticulite aiguë au moment de la consultation, ce qui nous amènerait à demander un examen d'imagerie pour l'exclure.", "type": "DIGESTIF", "options": {"1": "Appliquer un traitement antibiotique empirique.", "2": "Demander une coloscopie préférentielle.", "3": "Demander une échographie abdominale urgente.", "4": "Adressez-vous à un chirurgien pour évaluer la sigmoïdectomie.", "5": null}, "correct_option": 1, "explanations": {"1": {"exist": true, "char_ranges": [[0, 307]], "word_ranges": [[0, 48]], "text": "Ils se réfèrent à un concept relativement nouveau qui est apparu assez récemment et qui n'est pas présent dans toutes les directives cliniques, à savoir la maladie diverticulaire symptomatique non compliquée. Dans ces cas, un essai thérapeutique avec la rifaximine est recommandé, le vrai étant le numéro 1."}, "2": {"exist": true, "char_ranges": [[308, 532]], "word_ranges": [[48, 83]], "text": "L'absence de réactifs de phase aiguë dans l'analyse de laboratoire suggère que le patient n'avait pas de diverticulite aiguë au moment de la consultation, ce qui nous amènerait à demander un examen d'imagerie pour l'exclure."}, "3": {"exist": true, "char_ranges": [[308, 532]], "word_ranges": [[48, 83]], "text": "L'absence de réactifs de phase aiguë dans l'analyse de laboratoire suggère que le patient n'avait pas de diverticulite aiguë au moment de la consultation, ce qui nous amènerait à demander un examen d'imagerie pour l'exclure."}, "4": {"exist": false, "char_ranges": [], "word_ranges": [], "text": ""}, "5": {"exist": false, "char_ranges": [], "word_ranges": [], "text": ""}}} +{"id": 549, "year": 2022, "question_id_specific": 128, "full_question": "Une femme de 75 ans ayant des antécédents d'insuffisance cardiaque avec une fraction d'éjection de 25 % est venue pour un bilan de santé et était stable en classe fonctionnelle III de la NYHA. Elle a un défibrillateur cardiaque implanté. Son traitement actuel consiste en lisinopril, carvedilol et spironolactone aux doses maximales tolérées. À l'examen physique, la tension artérielle est de 118/74 mmHg et la fréquence cardiaque de 78 bpm. L'auscultation cardiaque révèle un troisième tonus, l'auscultation pulmonaire est normale et il n'y a pas d'œdème. Laquelle des mesures suivantes est la plus appropriée dans sa prise en charge ?", "full_answer": "Arrêter le lisinopril et commencer le sacubitril/valsartan.", "type": "CARDIOLOGIE", "options": {"1": "Ajouter le sacubitrile/valsartan.", "2": "Ajouter l'ivabradine.", "3": "Arrêter le lisinopril et commencer le sacubitril/valsartan.", "4": "Arrêter le carvédilol et commencer l'ivabradine.", "5": null}, "correct_option": 3, "explanations": {"1": {"exist": false, "char_ranges": [], "word_ranges": [], "text": ""}, "2": {"exist": false, "char_ranges": [], "word_ranges": [], "text": ""}, "3": {"exist": true, "char_ranges": [[0, 59]], "word_ranges": [[0, 7]], "text": "Arrêter le lisinopril et commencer le sacubitril/valsartan."}, "4": {"exist": false, "char_ranges": [], "word_ranges": [], "text": ""}, "5": {"exist": false, "char_ranges": [], "word_ranges": [], "text": ""}}} +{"id": 221, "year": 2014, "question_id_specific": 48, "full_question": "Un homme de 20 ans souffrant d'ataxie, de maux de tête et d'une masse kystique solide dans l'hémisphère cérébelleux droit a été opéré et une lésion a été réséquée. Histologiquement, la lésion présentait des cellules avec de longs et minces processus cytoplasmiques, un motif fasciculaire et microkystique, de nombreux vaisseaux et des fibres de Rosenthal. Le diagnostic anatomopathologique le plus probable est le suivant :", "full_answer": "Une question qui ne semble pas avoir de deuxième intention, car il n'y a pas d'éléments qui prêtent à confusion. L'énoncé résume de façon concise la description de l'astrocytome pilocytique. Les fibres de Rosenthal peuvent prêter à confusion car elles ne sont pas très spécifiques et font penser, par exemple, au xanthoastrocytome pléomorphe dans lequel elles peuvent également être trouvées, mais la morphologie cellulaire décrite (processus cytoplasmiques longs et fins, qui sont à l'origine du nom \"pilocytique\") maintient la première option comme étant la bonne. Une bonne description de cette entité, expliquant ce qui la distingue des autres tumeurs gliales, peut être trouvée dans Practical Surgical Neuropathology. A Diagnostic Approach. A. Perry, D.J. Brat. p82-88. Churchill Livingstone Elsevier. 2010. (Philadephie).", "type": "ANATOMIE PATHOLOGIQUE", "options": {"1": "Astrocytome pilocytique.", "2": "Xanthoastrocytome pléomorphe.", "3": "Neurocytome central.", "4": "Liponeurocytome.", "5": "Maladie à prions."}, "correct_option": 1, "explanations": {"1": {"exist": true, "char_ranges": [[191, 566]], "word_ranges": [[30, 85]], "text": "Les fibres de Rosenthal peuvent prêter à confusion car elles ne sont pas très spécifiques et font penser, par exemple, au xanthoastrocytome pléomorphe dans lequel elles peuvent également être trouvées, mais la morphologie cellulaire décrite (processus cytoplasmiques longs et fins, qui sont à l'origine du nom \"pilocytique\") maintient la première option comme étant la bonne."}, "2": {"exist": true, "char_ranges": [[191, 566]], "word_ranges": [[30, 85]], "text": "Les fibres de Rosenthal peuvent prêter à confusion car elles ne sont pas très spécifiques et font penser, par exemple, au xanthoastrocytome pléomorphe dans lequel elles peuvent également être trouvées, mais la morphologie cellulaire décrite (processus cytoplasmiques longs et fins, qui sont à l'origine du nom \"pilocytique\") maintient la première option comme étant la bonne."}, "3": {"exist": false, "char_ranges": [], "word_ranges": [], "text": ""}, "4": {"exist": false, "char_ranges": [], "word_ranges": [], "text": ""}, "5": {"exist": false, "char_ranges": [], "word_ranges": [], "text": ""}}} +{"id": 606, "year": 2022, "question_id_specific": 113, "full_question": "Un homme de 35 ans, facteur, sans antécédents, s'est présenté aux urgences pour des douleurs cervicales aiguës depuis 24 heures, sans antécédents traumatiques, irradiant dans le bras gauche jusqu'à la main et accompagnées d'une paresthésie du bord radial de l'avant-bras. Il n'y a pas de perte objective de force, le patient conserve la mobilité du cou bien qu'il soit douloureux et il y a une contracture des muscles paravertébraux. La première approche est la suivante :", "full_answer": "La réponse 1 est correcte. 75 % des patients atteints de radiculopathie s'améliorent avec un traitement non chirurgical, qui est le traitement de choix dans les phases initiales. L'IRM est le traitement de choix pour certains \"signaux d'alarme\" : fièvre, perte de poids, douleur nocturne, persistance des symptômes malgré le traitement conservateur et perte de force (réponse 3 incorrecte).", "type": "TRAUMATOLOGIE", "options": {"1": "Traitement conservateur à base d'anti-inflammatoires non stéroïdiens, de chaleur locale et de repos relatif.", "2": "Appel urgent au neurochirurgien pour une évaluation chirurgicale.", "3": "Demande préférentielle pour l'IRM et l'électromyogramme.", "4": "Orientation préférentielle vers les cliniques ambulatoires spécialisées dans les traumatismes.", "5": null}, "correct_option": 1, "explanations": {"1": {"exist": true, "char_ranges": [[0, 178]], "word_ranges": [[0, 28]], "text": "La réponse 1 est correcte. 75 % des patients atteints de radiculopathie s'améliorent avec un traitement non chirurgical, qui est le traitement de choix dans les phases initiales."}, "2": {"exist": false, "char_ranges": [], "word_ranges": [], "text": ""}, "3": {"exist": true, "char_ranges": [[179, 390]], "word_ranges": [[28, 59]], "text": "L'IRM est le traitement de choix pour certains \"signaux d'alarme\" : fièvre, perte de poids, douleur nocturne, persistance des symptômes malgré le traitement conservateur et perte de force (réponse 3 incorrecte)."}, "4": {"exist": false, "char_ranges": [], "word_ranges": [], "text": ""}, "5": {"exist": false, "char_ranges": [], "word_ranges": [], "text": ""}}} +{"id": 436, "year": 2018, "question_id_specific": 223, "full_question": "Une femme bolivienne de 35 ans, enceinte de trois mois, qui vit en Espagne depuis 10 ans avec son partenaire, qui est espagnol et n'a jamais voyagé en Amérique latine. Cette femme a deux autres enfants avec son compagnon, nés en Espagne, et un frère qui vit en Bolivie et un autre en Espagne. Pendant sa grossesse, on a diagnostiqué chez elle une infection à Trypanosoma cruzi. Lequel des énoncés suivants est FAUX ?", "full_answer": "La voie sexuelle n'est pas un mode de transmission de Trypanosoma cruzi. Dans notre environnement, en l'absence de vecteur de transmission, le mode de transmission le plus important est la voie verticale de la mère à l'enfant. La femme était déjà infectée par le parasite lorsqu'elle est arrivée en Espagne, il peut donc être correct de suggérer que ses frères et sœurs subissent un test sérologique de T. cruzi en raison du risque que sa mère ait transmis l'infection verticalement à l'ensemble de ses frères et sœurs. De même, ayant eu deux grossesses au cours de ses années de séjour en Espagne, il devrait être suggéré que ses enfants soient également testés pour la transmission verticale probable.", "type": "MALADIES INFECTIEUSES ET MICROBIOLOGIE", "options": {"1": "recommander à leurs enfants de subir un test de dépistage de T. cruzi en raison du risque de transmission transplacentaire.", "2": "Informer la patiente du risque de transmission de T. cruzi et de l'attitude à adopter pendant la grossesse, l'accouchement et le post-partum.", "3": "Recommander à son mari de subir un test de dépistage de T. cruzi en raison du risque de transmission sexuelle de la maladie.", "4": "Recommander que les frères et sœurs soient testés pour T. cruzi en raison du risque de transmission vectorielle.", "5": null}, "correct_option": 3, "explanations": {"1": {"exist": true, "char_ranges": [[529, 703]], "word_ranges": [[89, 117]], "text": "ayant eu deux grossesses au cours de ses années de séjour en Espagne, il devrait être suggéré que ses enfants soient également testés pour la transmission verticale probable."}, "2": {"exist": false, "char_ranges": [], "word_ranges": [], "text": ""}, "3": {"exist": true, "char_ranges": [[0, 72]], "word_ranges": [[0, 12]], "text": "La voie sexuelle n'est pas un mode de transmission de Trypanosoma cruzi."}, "4": {"exist": true, "char_ranges": [[227, 519]], "word_ranges": [[37, 87]], "text": "La femme était déjà infectée par le parasite lorsqu'elle est arrivée en Espagne, il peut donc être correct de suggérer que ses frères et sœurs subissent un test sérologique de T. cruzi en raison du risque que sa mère ait transmis l'infection verticalement à l'ensemble de ses frères et sœurs."}, "5": {"exist": false, "char_ranges": [], "word_ranges": [], "text": ""}}} +{"id": 220, "year": 2014, "question_id_specific": 126, "full_question": "Un homme de 42 ans a été examiné pour des œdèmes. Les premiers examens ont montré la présence d'une protéinurie de plus de 10 grammes/24 heures. Laquelle des mesures suivantes n'est PAS indiquée dans le traitement de ce patient ?", "full_answer": "Le patient dans ce cas est atteint du syndrome néphrotique, jusqu'ici tout va bien. En ce qui concerne le régime alimentaire, pour réduire l'œdème chez ces patients, il est recommandé qu'il soit pauvre en protéines et en sel, ce qui signifie que les options 1 et 3 sont correctes. Les inhibiteurs de l'ECA sont également administrés comme traitement primaire du syndrome néphrotique, car ils diminuent l'excrétion urinaire des protéines, tout comme les ARA II et les diurétiques de l'anse, de sorte que les options 2 et 5 seraient également correctes. Par conséquent, en écartant simplement cette option, nous nous retrouvons avec l'option 4 comme fausse, et donc la bonne réponse. D'ailleurs, elle devrait se démarquer dès le départ comme l'option à cocher, puisque l'une des causes du syndrome néphrotique est précisément l'utilisation d'AINS, alors comment ces derniers pourraient-ils être indiqués dans le traitement ?", "type": "NEPHROLOGIE", "options": {"1": "Régime pauvre en protéines.", "2": "Administration de diurétiques de l'anse.", "3": "Restriction du sel alimentaire.", "4": "Administration d'AINS.", "5": "Administration d'inhibiteurs de l'enzyme de conversion de l'angiotensine."}, "correct_option": 4, "explanations": {"1": {"exist": true, "char_ranges": [[126, 280]], "word_ranges": [[21, 49]], "text": "pour réduire l'œdème chez ces patients, il est recommandé qu'il soit pauvre en protéines et en sel, ce qui signifie que les options 1 et 3 sont correctes."}, "2": {"exist": true, "char_ranges": [[281, 551]], "word_ranges": [[49, 90]], "text": "Les inhibiteurs de l'ECA sont également administrés comme traitement primaire du syndrome néphrotique, car ils diminuent l'excrétion urinaire des protéines, tout comme les ARA II et les diurétiques de l'anse, de sorte que les options 2 et 5 seraient également correctes."}, "3": {"exist": true, "char_ranges": [[126, 280]], "word_ranges": [[21, 49]], "text": "pour réduire l'œdème chez ces patients, il est recommandé qu'il soit pauvre en protéines et en sel, ce qui signifie que les options 1 et 3 sont correctes."}, "4": {"exist": true, "char_ranges": [[759, 845]], "word_ranges": [[122, 133]], "text": "puisque l'une des causes du syndrome néphrotique est précisément l'utilisation d'AINS,"}, "5": {"exist": true, "char_ranges": [[281, 551]], "word_ranges": [[49, 90]], "text": "Les inhibiteurs de l'ECA sont également administrés comme traitement primaire du syndrome néphrotique, car ils diminuent l'excrétion urinaire des protéines, tout comme les ARA II et les diurétiques de l'anse, de sorte que les options 2 et 5 seraient également correctes."}}} +{"id": 483, "year": 2020, "question_id_specific": 174, "full_question": "Une femme de 22 ans présente un ACS avec un rythme jugé compatible avec la défibrillation. Des compressions thoraciques et des ventilations sont effectuées, un choc de défibrillateur a été délivré et une ligne veineuse périphérique a été canulée. Laquelle des actions suivantes serait la bonne ?", "full_answer": "Question controversée, puisque l'amiodarone (la dose est la bonne, 300 mg) est administrée après le troisième choc, et non après la première défibrillation, comme l'indique la déclaration. Mais, attention, c'est aussi ce que disent les lignes directrices. Je suis sûr que plus d'un d'entre nous a administré de l'amiodarone dès qu'il disposait d'un accès veineux en cas de rythme choquable. Les autres options sont incorrectes.", "type": "SOINS INTENSIFS", "options": {"1": "Administrer 150 mg d'amiodarone par voie intraveineuse.", "2": "Administrer 300 mg d'amiodarone par voie intraveineuse.", "3": "Administrer 1 mg d'atropine par voie intraveineuse.", "4": "Administrer 2 mg d'atropine par voie intraveineuse.", "5": null}, "correct_option": 2, "explanations": {"1": {"exist": false, "char_ranges": [], "word_ranges": [], "text": ""}, "2": {"exist": true, "char_ranges": [[31, 74]], "word_ranges": [[3, 11]], "text": "l'amiodarone (la dose est la bonne, 300 mg)"}, "3": {"exist": false, "char_ranges": [], "word_ranges": [], "text": ""}, "4": {"exist": false, "char_ranges": [], "word_ranges": [], "text": ""}, "5": {"exist": false, "char_ranges": [], "word_ranges": [], "text": ""}}} +{"id": 113, "year": 2012, "question_id_specific": 93, "full_question": "Une femme de 35 ans en bonne santé consulte pour une asthénie. L'hémogramme montre une anémie microcytaire et hypochromatique (hémoglobine 7 g/dl, volume corpusculaire moyen 68 fl, hémoglobine corpusculaire moyenne 24 pg) avec un intervalle de distribution des globules rouges élevé (ADE 20%), une diminution des réticulocytes (0,3%, 30000/l absolu), une diminution du contenu en hémoglobine dans les réticulocytes (17 pg) et une thrombocytose discrète (500000 plaquettes). Avec les données de l'hémogramme, quelle serait votre suspicion diagnostique même sans avoir la biochimie du patient ?", "full_answer": "Avec les données fournies, la première chose qui vient à l'esprit est qu'il s'agit d'une anémie arégénérative, car la moelle osseuse n'a pas répondu par une exaltation de l'érythropoïèse, ce qui élimine les options qui parlent d'anémie régénérative. L'élément suivant qui indique la cause est qu'elle est microcytaire et hypochrome, ce qui est caractéristique de l'anémie ferriprive, car la carence en acide folique entraîne une anémie mégaloblastique et l'hémorragie active n'entraîne pas de microcytose et d'hypochromie, mais une normocytose et une normochromie. Nous penchons presque pour la réponse 5, l'anémie centrale arégénérative due à une carence en fer... mais il reste la réponse 1, \"trait thalassémique\", qui nous met mal à l'aise et nous fait douter de la facilité apparente de notre choix... jusqu'à ce que nous nous souvenions que dans la thalassémie, l'EAD est normale. Cette jeune fille en âge de procréer souffre sans aucun doute d'une anémie ferriprive, ce qui est assez courant. Si vous avez des doutes, vous pouvez consulter \"Haematology. Manuel de base raisonné\" de Jesús San Miguel.", "type": "HÉMATOLOGIE", "options": {"1": "Trait thalassémique.", "2": "Anémie centrale, régénérative, due à une carence en acide folique.", "3": "Anémie hémolytique périphérique, régénérative.", "4": "Anémie périphérique, régénérative, due à une hémorragie active aiguë.", "5": "Anémie centrale, régénérative, anémie ferriprive."}, "correct_option": 5, "explanations": {"1": {"exist": true, "char_ranges": [[846, 885]], "word_ranges": [[130, 136]], "text": "dans la thalassémie, l'EAD est normale."}, "2": {"exist": true, "char_ranges": [[115, 249]], "word_ranges": [[18, 38]], "text": "la moelle osseuse n'a pas répondu par une exaltation de l'érythropoïèse, ce qui élimine les options qui parlent d'anémie régénérative."}, "3": {"exist": true, "char_ranges": [[115, 249]], "word_ranges": [[18, 38]], "text": "la moelle osseuse n'a pas répondu par une exaltation de l'érythropoïèse, ce qui élimine les options qui parlent d'anémie régénérative."}, "4": {"exist": false, "char_ranges": [], "word_ranges": [], "text": ""}, "5": {"exist": true, "char_ranges": [[272, 564]], "word_ranges": [[41, 82]], "text": "indique la cause est qu'elle est microcytaire et hypochrome, ce qui est caractéristique de l'anémie ferriprive, car la carence en acide folique entraîne une anémie mégaloblastique et l'hémorragie active n'entraîne pas de microcytose et d'hypochromie, mais une normocytose et une normochromie."}}} +{"id": 59, "year": 2011, "question_id_specific": 51, "full_question": "Patient de 47 ans consultant le service des urgences pour une perte de connaissance soudaine Lequel des éléments suivants a un meilleur rendement diagnostique ?", "full_answer": "Il est clair que c'est la seule réponse qui englobe toutes les causes de perte de conscience soudaine et qui permet de poser un diagnostic d'une certaine manière.", "type": "ANESTHÉSIOLOGIE ET SOINS INTENSIFS", "options": {"1": "Antécédents médicaux.", "2": "Électrocardiogramme.", "3": "Enregistrement Holter.", "4": "EEG.", "5": "Tomodensitométrie crânienne."}, "correct_option": 1, "explanations": {"1": {"exist": true, "char_ranges": [[0, 162]], "word_ranges": [[0, 28]], "text": "Il est clair que c'est la seule réponse qui englobe toutes les causes de perte de conscience soudaine et qui permet de poser un diagnostic d'une certaine manière."}, "2": {"exist": false, "char_ranges": [], "word_ranges": [], "text": ""}, "3": {"exist": false, "char_ranges": [], "word_ranges": [], "text": ""}, "4": {"exist": false, "char_ranges": [], "word_ranges": [], "text": ""}, "5": {"exist": false, "char_ranges": [], "word_ranges": [], "text": ""}}} +{"id": 412, "year": 2018, "question_id_specific": 54, "full_question": "Elizabeth a trois fils et une fille (Maria), tous en bonne santé. Un frère et un oncle maternel d'Isabel sont décédés de la maladie de Duchenne, une maladie récessive liée au chromosome X. Maria souhaite maintenant être enceinte et veut connaître le risque de transmission de la maladie à sa progéniture. Maria souhaite maintenant tomber enceinte et veut connaître le risque de transmission de la maladie à sa descendance. Sur la base des données recueillies, quelle est la probabilité que Maria soit porteuse de la maladie de Duchenne ?", "full_answer": "Sa grand-mère maternelle était porteuse de la mutation d'une copie du gène sur un chromosome X (on ne nous dit pas qu'elle a souffert de la maladie, mais un de ses fils, l'oncle maternel d'Isabel, en a souffert). La mère d'Isabel continue d'être porteuse du gène, car l'un des frères d'Isabel a été atteint de la maladie, mais pas son père. Isabel a donc 50 % de chances d'avoir hérité du gène muté de sa mère (comme ses enfants sont tous en bonne santé, nous ne savons pas si elle est porteuse), et sa fille a donc 25 % de chances d'avoir hérité du gène muté de sa grand-mère.", "type": "BIOSTATISTIQUE", "options": {"1": "2/3.", "2": "1/2.", "3": "1/4.", "4": "Moins de 1/4.", "5": null}, "correct_option": 3, "explanations": {"1": {"exist": false, "char_ranges": [], "word_ranges": [], "text": ""}, "2": {"exist": false, "char_ranges": [], "word_ranges": [], "text": ""}, "3": {"exist": true, "char_ranges": [[0, 577]], "word_ranges": [[0, 109]], "text": "Sa grand-mère maternelle était porteuse de la mutation d'une copie du gène sur un chromosome X (on ne nous dit pas qu'elle a souffert de la maladie, mais un de ses fils, l'oncle maternel d'Isabel, en a souffert). La mère d'Isabel continue d'être porteuse du gène, car l'un des frères d'Isabel a été atteint de la maladie, mais pas son père. Isabel a donc 50 % de chances d'avoir hérité du gène muté de sa mère (comme ses enfants sont tous en bonne santé, nous ne savons pas si elle est porteuse), et sa fille a donc 25 % de chances d'avoir hérité du gène muté de sa grand-mère."}, "4": {"exist": false, "char_ranges": [], "word_ranges": [], "text": ""}, "5": {"exist": false, "char_ranges": [], "word_ranges": [], "text": ""}}} +{"id": 347, "year": 2016, "question_id_specific": 159, "full_question": "Une femme de 69 ans est venue à la clinique pour se plaindre de saignements génitaux depuis plusieurs mois. Elle refuse le traitement hormonal substitutif et l'anticoagulation. Elle a fourni une cytologie cervicovaginale normale. L'examen physique général et génital n'a rien révélé d'intéressant. IMC de 38 kg/m2. Indiquez l'approche la plus appropriée :", "full_answer": "Biopsie de l'endomètre. Péri ou ménopause : - En raison de la prévalence élevée de la pathologie organique au cours de cette période, il est nécessaire de réaliser systématiquement une biopsie de l'endomètre (Cornier ou hystéroscopie).", "type": "GYNÉCOLOGIE ET OBSTÉTRIQUE", "options": {"1": "Prescrire de la progestérone cyclique.", "2": "Biopsie de l'endomètre.", "3": "Biopsies cervicales aléatoires.", "4": "Évaluation hormonale avec FSH, LH et œstradiol.", "5": null}, "correct_option": 2, "explanations": {"1": {"exist": false, "char_ranges": [], "word_ranges": [], "text": ""}, "2": {"exist": true, "char_ranges": [[24, 235]], "word_ranges": [[3, 36]], "text": "Péri ou ménopause : - En raison de la prévalence élevée de la pathologie organique au cours de cette période, il est nécessaire de réaliser systématiquement une biopsie de l'endomètre (Cornier ou hystéroscopie)."}, "3": {"exist": false, "char_ranges": [], "word_ranges": [], "text": ""}, "4": {"exist": false, "char_ranges": [], "word_ranges": [], "text": ""}, "5": {"exist": false, "char_ranges": [], "word_ranges": [], "text": ""}}} +{"id": 325, "year": 2016, "question_id_specific": 56, "full_question": "Un patient de 54 ans a été admis pour une fièvre de 38°C au cours des cinq jours précédents et une dyspnée au repos (NYHA lV) apparue 6 heures avant son arrivée à l'hôpital. Au service des urgences, l'examen est compatible avec une insuffisance cardiaque et l'ECG montre un bloc auriculo-ventriculaire complet avec une fréquence ventriculaire d'échappement de 45 bpm. Les signes d'insuffisance cardiaque sont réfractaires au traitement médical et l'échocardiographie transœsophagienne montre une valve aortique avec un orifice régurgitant effectif de 0,5 cm2. Les cultures en série sont positives pour Streptococcus gallolyticus. Indiquez le meilleur plan d'action :", "full_answer": "La bonne réponse est 1, car les indications de la chirurgie cardiaque comprennent le développement d'une insuffisance cardiaque ou de nouveaux troubles de la conduction, et dans ce cas, elle présente les deux.", "type": "LES SOINS CRITIQUES, PALLIATIFS ET D'URGENCE", "options": {"1": "Chirurgie cardiaque pour le remplacement de la valve aortique par une prothèse mécanique avec antibiothérapie selon l'antibiogramme.", "2": "Antibiothérapie selon l'antibiogramme et implantation d'une pompe à ballonnet intra-aortique et d'un stimulateur cardiaque temporaire pour une durée maximale de 3 semaines, après quoi un stimulateur cardiaque permanent sera implanté.", "3": "Implantation d'un stimulateur cardiaque transitoire, antibiothérapie selon l'antibiogramme et implantation percutanée d'une prothèse valvulaire aortique.", "4": "Implantation urgente d'un stimulateur cardiaque permanent avec une antibiothérapie conforme à l'antibiogramme pendant 6 semaines.", "5": null}, "correct_option": 1, "explanations": {"1": {"exist": true, "char_ranges": [[28, 209]], "word_ranges": [[6, 33]], "text": "les indications de la chirurgie cardiaque comprennent le développement d'une insuffisance cardiaque ou de nouveaux troubles de la conduction, et dans ce cas, elle présente les deux."}, "2": {"exist": false, "char_ranges": [], "word_ranges": [], "text": ""}, "3": {"exist": false, "char_ranges": [], "word_ranges": [], "text": ""}, "4": {"exist": false, "char_ranges": [], "word_ranges": [], "text": ""}, "5": {"exist": false, "char_ranges": [], "word_ranges": [], "text": ""}}} +{"id": 335, "year": 2016, "question_id_specific": 175, "full_question": "Un homme de 88 ans est venu consulter parce qu'il était tombé trois fois au cours des six derniers mois. Aucune des chutes n'a été accompagnée de vertiges ou de syncope. Une chute s'est produite alors qu'il marchait dans le jardin. Ses antécédents médicaux comprennent une hypertension artérielle sans modification posturale de la pression artérielle, de la goutte, de l'arthrose et une dépression. Lequel des éléments suivants est le plus susceptible de contribuer aux chutes de ce patient ?", "full_answer": "Paroxétine ; question compliquée car l'hydrochlorothiazide et le lisinopril peuvent également provoquer des chutes chez les personnes âgées, mais elles sont généralement accompagnées de vertiges.", "type": "PHARMACOLOGIE", "options": {"1": "Allopurinol.", "2": "Hydrochlorothiazide.", "3": "Lisinopril.", "4": "Paroxétine.", "5": null}, "correct_option": 4, "explanations": {"1": {"exist": false, "char_ranges": [], "word_ranges": [], "text": ""}, "2": {"exist": true, "char_ranges": [[37, 195]], "word_ranges": [[5, 25]], "text": "l'hydrochlorothiazide et le lisinopril peuvent également provoquer des chutes chez les personnes âgées, mais elles sont généralement accompagnées de vertiges."}, "3": {"exist": true, "char_ranges": [[37, 195]], "word_ranges": [[5, 25]], "text": "l'hydrochlorothiazide et le lisinopril peuvent également provoquer des chutes chez les personnes âgées, mais elles sont généralement accompagnées de vertiges."}, "4": {"exist": true, "char_ranges": [[37, 195]], "word_ranges": [[5, 25]], "text": "l'hydrochlorothiazide et le lisinopril peuvent également provoquer des chutes chez les personnes âgées, mais elles sont généralement accompagnées de vertiges."}, "5": {"exist": false, "char_ranges": [], "word_ranges": [], "text": ""}}} +{"id": 19, "year": 2011, "question_id_specific": 109, "full_question": "Une femme de 32 ans s'est rendue à Cuba, où elle est arrivée il y a trois jours. Le lendemain de son retour, elle consulte son médecin généraliste pour une forte fièvre, une arthromyalgie sévère et des céphalées qui ont débuté avant le voyage de retour et se voit prescrire du paracétamol. Trois jours plus tard, sans amélioration, il a présenté le matin une éruption maculo-papuleuse prurigineuse généralisée, plus intense sur les membres inférieurs, où elle s'est transformée en pétéchies, pour laquelle il s'est rendu au service des urgences. Aucune donnée pertinente n'a été observée dans le CD, à l'exception des plaquettes 75000/mm3 (htco 36%, leucocytes 4100 79% neutrophiles). Quel est le diagnostic le plus probable ?", "full_answer": "Dans ce cas, je pense que la bonne réponse est 5. La réponse 1 ne devrait pas l'être car, bien que le tableau clinique soit évocateur, il n'y a pas de lymphocytose. Le tableau clinique n'indique pas non plus qu'il s'agit d'un exanthème allergique. Et il n'y a certainement pas de symptômes ou de signes cliniques compatibles avec le paludisme ou la fièvre typhoïde. Outre les signes cliniques, c'est l'hémopénie plaquettaire qui indique la présence d'une dengue.", "type": "INFECTIEUX", "options": {"1": "Mononucléose infectieuse.", "2": "Éruption allergique.", "3": "Paludisme à P. falciparum.", "4": "Fièvre typhoïde.", "5": "Dengue."}, "correct_option": 5, "explanations": {"1": {"exist": false, "char_ranges": [], "word_ranges": [], "text": ""}, "2": {"exist": true, "char_ranges": [[165, 247]], "word_ranges": [[32, 44]], "text": "Le tableau clinique n'indique pas non plus qu'il s'agit d'un exanthème allergique."}, "3": {"exist": true, "char_ranges": [[248, 365]], "word_ranges": [[44, 64]], "text": "Et il n'y a certainement pas de symptômes ou de signes cliniques compatibles avec le paludisme ou la fièvre typhoïde."}, "4": {"exist": true, "char_ranges": [[248, 365]], "word_ranges": [[44, 64]], "text": "Et il n'y a certainement pas de symptômes ou de signes cliniques compatibles avec le paludisme ou la fièvre typhoïde."}, "5": {"exist": true, "char_ranges": [[50, 164], [366, 462]], "word_ranges": [[11, 32], [64, 77]], "text": "La réponse 1 ne devrait pas l'être car, bien que le tableau clinique soit évocateur, il n'y a pas de lymphocytose. Outre les signes cliniques, c'est l'hémopénie plaquettaire qui indique la présence d'une dengue."}}} +{"id": 401, "year": 2016, "question_id_specific": 147, "full_question": "Un garçon de 13 ans présente des douleurs dorsales depuis plusieurs mois, uniquement en se levant et en marchant. Quel diagnostic faut-il envisager ?", "full_answer": "Une tumeur maligne serait également douloureuse au repos. En ce qui concerne la maladie de Scheuermann et la scoliose, il convient de nous fournir des données physiques, car le patient présente un examen spécifique qui nous permettrait de poser le diagnostic.", "type": "TRAUMATOLOGIE ET ORTHOPÉDIE", "options": {"1": "Maladie de Scheuermann.", "2": "Scoliose thoracique.", "3": "Tumeur maligne du corps vertébral.", "4": "Il s'agit d'une surcharge mécanique.", "5": null}, "correct_option": 4, "explanations": {"1": {"exist": true, "char_ranges": [[58, 259]], "word_ranges": [[8, 41]], "text": "En ce qui concerne la maladie de Scheuermann et la scoliose, il convient de nous fournir des données physiques, car le patient présente un examen spécifique qui nous permettrait de poser le diagnostic."}, "2": {"exist": true, "char_ranges": [[58, 259]], "word_ranges": [[8, 41]], "text": "En ce qui concerne la maladie de Scheuermann et la scoliose, il convient de nous fournir des données physiques, car le patient présente un examen spécifique qui nous permettrait de poser le diagnostic."}, "3": {"exist": true, "char_ranges": [[0, 57]], "word_ranges": [[0, 8]], "text": "Une tumeur maligne serait également douloureuse au repos."}, "4": {"exist": false, "char_ranges": [], "word_ranges": [], "text": ""}, "5": {"exist": false, "char_ranges": [], "word_ranges": [], "text": ""}}} +{"id": 607, "year": 2022, "question_id_specific": 115, "full_question": "Un homme de 27 ans, sportif régulier, a signalé une douleur à la jambe droite après une course continue. Il a consulté un kinésithérapeute à plusieurs reprises, qui a diagnostiqué une surcharge du mollet. Plusieurs mois se sont écoulés, son état ne s'est pas amélioré et il signale une douleur intense après l'activité physique, qui s'estompe au repos dans les heures qui suivent l'exercice. Quel test peut aider à établir le diagnostic ?", "full_answer": "Suspicion clinique de syndrome du compartiment chronique (SCC). Le diagnostic est établi en obtenant les pressions des compartiments au repos, pendant l'exercice et après l'exercice.", "type": "TRAUMATOLOGIE", "options": {"1": "Tomographie par émission de positons au 18 FDG.", "2": "Détermination de la pression du compartiment aval immédiatement après l'activité.", "3": "Échographie Doppler pour exclure un trouble circulatoire du membre inférieur.", "4": "Spectrométrie par résonance magnétique.", "5": null}, "correct_option": 2, "explanations": {"1": {"exist": false, "char_ranges": [], "word_ranges": [], "text": ""}, "2": {"exist": true, "char_ranges": [[0, 182]], "word_ranges": [[0, 25]], "text": "Suspicion clinique de syndrome du compartiment chronique (SCC). Le diagnostic est établi en obtenant les pressions des compartiments au repos, pendant l'exercice et après l'exercice."}, "3": {"exist": false, "char_ranges": [], "word_ranges": [], "text": ""}, "4": {"exist": false, "char_ranges": [], "word_ranges": [], "text": ""}, "5": {"exist": false, "char_ranges": [], "word_ranges": [], "text": ""}}} +{"id": 453, "year": 2018, "question_id_specific": 150, "full_question": "Un homme de 25 ans consulte pour des tremblements. L'examen révèle une dysarthrie et une dystonie. Il a des antécédents familiaux de maladie psychiatrique et de troubles du mouvement. Quel test diagnostique vous semble le plus approprié ?", "full_answer": "Si l'on parle d'un jeune homme de 20 à 30 ans, avec des tremblements et une dystonie, qu'il ait ou non des antécédents familiaux, et surtout si l'une des options parle de cuivre dans les urines, on me dit de penser à la maladie de Wilson. Je ne pense pas à la maladie de Parkinson juvénile, ni à une clinique d'épilepsie. C'est pourquoi la bonne réponse est 4.", "type": "NEUROLOGIE", "options": {"1": "DATS CAN.", "2": "Étude de la conduction nerveuse.", "3": "Électroencéphalogramme.", "4": "Cuivre dans l'urine de 24.", "5": null}, "correct_option": 4, "explanations": {"1": {"exist": true, "char_ranges": [[0, 360]], "word_ranges": [[0, 68]], "text": "Si l'on parle d'un jeune homme de 20 à 30 ans, avec des tremblements et une dystonie, qu'il ait ou non des antécédents familiaux, et surtout si l'une des options parle de cuivre dans les urines, on me dit de penser à la maladie de Wilson. Je ne pense pas à la maladie de Parkinson juvénile, ni à une clinique d'épilepsie. C'est pourquoi la bonne réponse est 4."}, "2": {"exist": false, "char_ranges": [], "word_ranges": [], "text": ""}, "3": {"exist": true, "char_ranges": [[0, 360]], "word_ranges": [[0, 68]], "text": "Si l'on parle d'un jeune homme de 20 à 30 ans, avec des tremblements et une dystonie, qu'il ait ou non des antécédents familiaux, et surtout si l'une des options parle de cuivre dans les urines, on me dit de penser à la maladie de Wilson. Je ne pense pas à la maladie de Parkinson juvénile, ni à une clinique d'épilepsie. C'est pourquoi la bonne réponse est 4."}, "4": {"exist": true, "char_ranges": [[0, 360]], "word_ranges": [[0, 68]], "text": "Si l'on parle d'un jeune homme de 20 à 30 ans, avec des tremblements et une dystonie, qu'il ait ou non des antécédents familiaux, et surtout si l'une des options parle de cuivre dans les urines, on me dit de penser à la maladie de Wilson. Je ne pense pas à la maladie de Parkinson juvénile, ni à une clinique d'épilepsie. C'est pourquoi la bonne réponse est 4."}, "5": {"exist": false, "char_ranges": [], "word_ranges": [], "text": ""}}} +{"id": 529, "year": 2021, "question_id_specific": 166, "full_question": "Un patient de 18 ans consulte pour des œdèmes. Un examen sanguin complet a révélé une protéinurie de 8 g/jour sans microhématurie, une hypoalbuminémie et une hypercholestérolémie avec une fonction rénale normale. Des corticostéroïdes ont été administrés de manière empirique. Après un mois, le tableau clinique a complètement disparu. Quelle est votre hypothèse diagnostique ?", "full_answer": "Nous décrivons un jeune patient présentant les critères du syndrome néphrotique avec une excellente réponse aux corticostéroïdes. Sans biopsie, la présentation clinique et l'évolution suggèrent une néphropathie avec des changements minimes, qui se présente habituellement avec une fonction rénale normale ou légèrement altérée en même temps que le syndrome néphrotique, et qui dans 85-90% des cas se résout avec un traitement aux stéroïdes. L'âge est la seule donnée qui pose un peu problème, car bien qu'il s'agisse de la cause la plus fréquente de syndrome néphrotique idiopathique chez les enfants et les adolescents, il est généralement recommandé d'effectuer une biopsie avant le traitement chez les plus de 16 ans, et dans tous les cas, il n'y a pas beaucoup de discussions à ce sujet.", "type": "NEPHROLOGIE", "options": {"1": "Amyloïdose.", "2": "Néphropathie à IgA ou maladie de Berger.", "3": "Syndrome d'Alport.", "4": "Néphropathie avec changements minimes.", "5": null}, "correct_option": 4, "explanations": {"1": {"exist": false, "char_ranges": [], "word_ranges": [], "text": ""}, "2": {"exist": false, "char_ranges": [], "word_ranges": [], "text": ""}, "3": {"exist": false, "char_ranges": [], "word_ranges": [], "text": ""}, "4": {"exist": true, "char_ranges": [[0, 369]], "word_ranges": [[0, 50]], "text": "Nous décrivons un jeune patient présentant les critères du syndrome néphrotique avec une excellente réponse aux corticostéroïdes. Sans biopsie, la présentation clinique et l'évolution suggèrent une néphropathie avec des changements minimes, qui se présente habituellement avec une fonction rénale normale ou légèrement altérée en même temps que le syndrome néphrotique,"}, "5": {"exist": false, "char_ranges": [], "word_ranges": [], "text": ""}}} +{"id": 258, "year": 2014, "question_id_specific": 96, "full_question": "Un homme de 62 ans atteint de diabète de type 2 depuis 10 ans est traité par metformine et sitagliptine. Il fait peu d'exercice physique et suit un régime alimentaire adéquat. Au cours des six derniers mois, il a perdu du poids et est devenu plus asthénique. Ses contrôles glycémiques se sont détériorés, passant d'une glycémie de base de 110-140 mg/dl à 170-200 mg/dl, tout comme son hémoglobine glyquée, qui est passée de 7,1 à 8,5 %. La mesure thérapeutique la plus appropriée à prendre est la suivante :", "full_answer": "Ajouter une dose basale d'insuline au traitement. Question attendue sur un sujet très important : le diabète et son traitement. Patient mal contrôlé par la metformine et la sitagliptine avec des symptômes cardinaux discrets (perte de poids, asthénie). La meilleure option est l'insuline pour contrôler les symptômes cardinaux et diminuer l'HbA1c à < 7%.", "type": "ENDOCRINOLOGIE", "options": {"1": "Augmenter l'apport en protéines et en glucides à chaîne longue dans le régime alimentaire pour améliorer l'asthénie et la perte de poids.", "2": "Ajouter une dose basale d'insuline au traitement.", "3": "Combiner le traitement avec l'ascarbose.", "4": "Remplacer la sitagliptine par la pioglitazone.", "5": "Remplacer la metformine par le glimépiride."}, "correct_option": 2, "explanations": {"1": {"exist": false, "char_ranges": [], "word_ranges": [], "text": ""}, "2": {"exist": true, "char_ranges": [[128, 353]], "word_ranges": [[20, 54]], "text": "Patient mal contrôlé par la metformine et la sitagliptine avec des symptômes cardinaux discrets (perte de poids, asthénie). La meilleure option est l'insuline pour contrôler les symptômes cardinaux et diminuer l'HbA1c à < 7%."}, "3": {"exist": false, "char_ranges": [], "word_ranges": [], "text": ""}, "4": {"exist": false, "char_ranges": [], "word_ranges": [], "text": ""}, "5": {"exist": false, "char_ranges": [], "word_ranges": [], "text": ""}}} +{"id": 375, "year": 2016, "question_id_specific": 128, "full_question": "Patient de 45 ans examiné pour une possible méningite, avec fièvre, maux de tête et vomissements depuis 2 jours. Une IRM cérébrale et une ponction lombaire ont été réalisées. Vingt heures plus tard, en se levant pour aller aux toilettes, il se plaint de céphalées sévères, très marquées lorsqu'il s'assoit mais qui disparaissent lorsqu'il s'allonge. Elle n'a plus de fièvre ni de vomissements et est incapable de marcher. Quelle est la cause la plus probable de ce mal de tête ?", "full_answer": "Le tableau qu'ils décrivent est celui d'une céphalée intense avec orthostatisme qui disparaît en position allongée, même s'il est vrai que la présentation la plus typique se situe entre les 24 et 48 premières heures et que, dans le cas présent, elle est un peu plus précoce. Néanmoins, les autres réponses sont improbables : la première ne peut l'être parce qu'on ne nous a pas dit comment était la céphalée initiale ; la troisième, bien qu'elle puisse être due à l'heure d'apparition (comme je l'ai dit, un peu plus tôt que prévu), la réalité est que la céphalée avec orthostatisme est si typique qu'elle serait la cause la plus probable ; la dernière ne peut l'être non plus parce que l'HSA ne suit pas ce modèle en ce qui concerne l'orthostatisme.", "type": "NEUROLOGIE", "options": {"1": "La méningite reste la cause première de votre mal de tête, car il s'agit du même type de mal de tête qu'au début des symptômes.", "2": "Il s'agit d'une céphalée post-ponction lombaire.", "3": "Il est nécessaire de rechercher une autre cause à ce mal de tête, car il n'est pas typique d'un syndrome de ponction post-lombaire ou d'une méningite.", "4": "Il ne s'agit probablement pas d'une méningite virale, mais d'une hémorragie sous-arachnoïdienne. C'est pourquoi le mal de tête initial disparaît presque complètement en position allongée.", "5": null}, "correct_option": 2, "explanations": {"1": {"exist": true, "char_ranges": [[325, 418]], "word_ranges": [[54, 72]], "text": "la première ne peut l'être parce qu'on ne nous a pas dit comment était la céphalée initiale ;"}, "2": {"exist": true, "char_ranges": [[0, 115]], "word_ranges": [[0, 16]], "text": "Le tableau qu'ils décrivent est celui d'une céphalée intense avec orthostatisme qui disparaît en position allongée,"}, "3": {"exist": true, "char_ranges": [[419, 640]], "word_ranges": [[72, 111]], "text": "la troisième, bien qu'elle puisse être due à l'heure d'apparition (comme je l'ai dit, un peu plus tôt que prévu), la réalité est que la céphalée avec orthostatisme est si typique qu'elle serait la cause la plus probable ;"}, "4": {"exist": true, "char_ranges": [[641, 750]], "word_ranges": [[111, 131]], "text": "la dernière ne peut l'être non plus parce que l'HSA ne suit pas ce modèle en ce qui concerne l'orthostatisme."}, "5": {"exist": false, "char_ranges": [], "word_ranges": [], "text": ""}}} +{"id": 352, "year": 2016, "question_id_specific": 161, "full_question": "Une femme de 27 ans a été adressée au service de gynécologie pour évaluation, signalant une dyspareunie depuis 8 mois, ainsi qu'une dyschésie et une rectorragie occasionnelle coïncidant avec les menstruations depuis 3-4 mois. Elle signale également une dysménorrhée depuis des années, qu'elle contrôle bien avec de l'ibuprofène. Elle essaie de tomber enceinte depuis 16 mois, mais n'y est pas encore parvenue. Lors de l'examen gynécologique, seule une douleur est notée à la pression du fornix vaginal postérieur. Quel examen vous permettrait, selon vous, de poser un diagnostic de certitude sur sa pathologie ?", "full_answer": "La bonne réponse est 2. Nous avons affaire à une suspicion d'endométriose, en raison des symptômes et des signes mentionnés dans l'énoncé. On nous demande un diagnostic de certitude, et l'étalon-or pour le diagnostic de cette maladie est la laparoscopie diagnostique.", "type": "GYNÉCOLOGIE ET OBSTÉTRIQUE", "options": {"1": "Échographie transvaginale.", "2": "La laparoscopie diagnostique.", "3": "Imagerie par résonance magnétique.", "4": "Coloscopie.", "5": null}, "correct_option": 2, "explanations": {"1": {"exist": false, "char_ranges": [], "word_ranges": [], "text": ""}, "2": {"exist": true, "char_ranges": [[24, 267]], "word_ranges": [[5, 41]], "text": "Nous avons affaire à une suspicion d'endométriose, en raison des symptômes et des signes mentionnés dans l'énoncé. On nous demande un diagnostic de certitude, et l'étalon-or pour le diagnostic de cette maladie est la laparoscopie diagnostique."}, "3": {"exist": false, "char_ranges": [], "word_ranges": [], "text": ""}, "4": {"exist": false, "char_ranges": [], "word_ranges": [], "text": ""}, "5": {"exist": false, "char_ranges": [], "word_ranges": [], "text": ""}}} +{"id": 575, "year": 2022, "question_id_specific": 102, "full_question": "Femme de 29 ans ayant des antécédents de crises fébriles et grande fumeuse. Elle consulte depuis 2 ans pour des épisodes fréquents et stéréotypés qui débutent par une sensation épigastrique ascendante et une odeur désagréable, suivies d'une déconnexion de son environnement. Les membres de la famille qui sont avec elle observent qu'elle fait des mouvements de succion et des mouvements répétés d'ouverture et de fermeture de la main gauche. Au bout de deux minutes, les mouvements disparaissent, mais elle a du mal à réagir de manière appropriée et ne se souvient que partiellement de ce qui s'est passé. Lequel des diagnostics suivants est le plus probable ?", "full_answer": "Le cas clinique est une description classique d'une crise temporale médiane. L'aura de sensation épigastrique ascendante et de perturbation olfactive et la description ultérieure de la crise avec altération de l'état de conscience (déconnexion de l'environnement) avec automatismes oraux et manuels. La classification actuelle est celle de l'ILAE de 2017, qui décrirait cette crise comme une crise focale avec altération du niveau de conscience, mais dans le MIR, on continue d'utiliser l'ancienne terminologie (complexe, c'est ainsi que l'on appelait l'altération du niveau de conscience).", "type": "NEUROLOGIE", "options": {"1": "Crises complexes du lobe temporal focal.", "2": "Événements ischémiques transitoires dans le territoire carotidien droit.", "3": "Crises motrices focales simples.", "4": "Épisodes d'amnésie globale transitoire.", "5": null}, "correct_option": 1, "explanations": {"1": {"exist": true, "char_ranges": [[300, 590]], "word_ranges": [[41, 84]], "text": "La classification actuelle est celle de l'ILAE de 2017, qui décrirait cette crise comme une crise focale avec altération du niveau de conscience, mais dans le MIR, on continue d'utiliser l'ancienne terminologie (complexe, c'est ainsi que l'on appelait l'altération du niveau de conscience)."}, "2": {"exist": false, "char_ranges": [], "word_ranges": [], "text": ""}, "3": {"exist": false, "char_ranges": [], "word_ranges": [], "text": ""}, "4": {"exist": false, "char_ranges": [], "word_ranges": [], "text": ""}, "5": {"exist": false, "char_ranges": [], "word_ranges": [], "text": ""}}} +{"id": 278, "year": 2016, "question_id_specific": 31, "full_question": "Une femme de 20 ans avec une tumeur ovarienne kystique solide de 15 cm détectée par échographie après avoir présenté des symptômes abdominaux non spécifiques. Lors de l'étude anatomopathologique de l'échantillon correspondant, des dents, des poils, des zones d'épithélium intestinal, des zones d'épithélium malpighien (15 %) et d'épithélium bronchique, ainsi que des éléments neuroectodermiques et embryonnaires ont été trouvés dans plusieurs des préparations histologiques. En ce qui concerne ce cas, veuillez indiquer le bon diagnostic :", "full_answer": "Après lecture de la question et des réponses possibles, nous avons deux options tumorales et deux options non tumorales. A aucun moment il n'est fait mention de caractéristiques cellulaires atypiques, ce qui permet d'écarter les lésions tumorales, le doute subsistant entre les réponses 2 et 3. Dans ce cas, il faudrait opter pour le TÉRATÔME IMMATUREL, puisqu'il est fait mention de la présence de restes embryonnaires.", "type": "ANATOMIE PATHOLOGIQUE", "options": {"1": "Tératocarcinome.", "2": "Tératome immature.", "3": "Tératome kystique mature.", "4": "Dysgerminome.", "5": null}, "correct_option": 2, "explanations": {"1": {"exist": true, "char_ranges": [[121, 247]], "word_ranges": [[19, 37]], "text": "A aucun moment il n'est fait mention de caractéristiques cellulaires atypiques, ce qui permet d'écarter les lésions tumorales,"}, "2": {"exist": true, "char_ranges": [[308, 420]], "word_ranges": [[49, 66]], "text": "il faudrait opter pour le TÉRATÔME IMMATUREL, puisqu'il est fait mention de la présence de restes embryonnaires."}, "3": {"exist": true, "char_ranges": [[308, 420]], "word_ranges": [[49, 66]], "text": "il faudrait opter pour le TÉRATÔME IMMATUREL, puisqu'il est fait mention de la présence de restes embryonnaires."}, "4": {"exist": true, "char_ranges": [[121, 247]], "word_ranges": [[19, 37]], "text": "A aucun moment il n'est fait mention de caractéristiques cellulaires atypiques, ce qui permet d'écarter les lésions tumorales,"}, "5": {"exist": false, "char_ranges": [], "word_ranges": [], "text": ""}}} +{"id": 225, "year": 2014, "question_id_specific": 64, "full_question": "Un patient de 82 ans présente des vertiges soudains et fréquents, sans prodromes, de courte durée et qui s'estompent spontanément, sans jamais présenter de syncope. L'examen physique et l'électrocardiogramme de base sont normaux. Un électrocardiogramme ambulatoire de 24 heures montre de courtes phases d'absence d'ondes P pré-QRS avec un rythme d'échappement de la jonction auriculo-ventriculaire avec un QRS étroit à 40 bpm et une onde d'apparition du segment ST correspondant à une onde P rétrograde. Aucune période d'asystolie de plus de 3 secondes n'est détectée. Compte tenu de ce qui précède, vous diriez :", "full_answer": "Quels sont les résultats de l'examen Holter ? En cas de défaillance du nœud sinusal, le nœud AV prend le relais et stimule le ventricule. Chez 10 % des personnes, il existe une conduction rétrograde, de sorte que le battement généré dans le nœud AV stimule à la fois les ventricules et les oreillettes, ce qui donne une P rétrograde, comme indiqué dans l'étude. En ce sens, il n'y a pas de problème. De même, il n'y a pas de pauses de plus de 3 secondes qui pourraient provoquer une syncope cardiogénique (dont il n'a d'ailleurs jamais souffert). L'option 1 est exclue : il ne s'agit pas d'une BAV complète. On ne peut pas non plus répondre à l'option 2 : exclure, que dit-on pour exclure... C'est comme jamais/toujours. L'option 3 n'a pas beaucoup de sens : oui, on pourrait suspendre ceux qui altèrent la conduction (bêtabloquants, antagonistes calciques), mais les médicaments qui la facilitent, tant qu'on ne lui donne pas une perfusion d'atropine à emporter chez lui... Dans l'option 4, dire qu'il a un bloc sino-auriculaire, c'est un peu du vent : sans étude électrophysiologique, on ne peut pas le dire, il n'a pas forcément de bloc SA, et cela ne suffit pas à indiquer l'implantation d'un pacemaker. Donc, finalement, je cocherais l'option 5.", "type": "CARDIOLOGIE", "options": {"1": "Le patient souffre d'un BAV du troisième degré et nécessite l'implantation d'un stimulateur cardiaque.", "2": "L'absence de périodes d'asystolie > 3 secondes exclut une cause cardiaque des vertiges.", "3": "Un traitement médicamenteux visant à augmenter la conduction dans le nœud AV serait indiqué.", "4": "Le patient présente un dysfonctionnement sinusal avec bloc sino-auriculaire et nécessite l'implantation d'un stimulateur cardiaque en raison des symptômes.", "5": "Les altérations détectées dans l'électrocardiogramme ambulatoire sont typiques des patients de cet âge et il n'y a pas d'indication pour une intervention thérapeutique."}, "correct_option": 5, "explanations": {"1": {"exist": true, "char_ranges": [[547, 607]], "word_ranges": [[98, 110]], "text": "L'option 1 est exclue : il ne s'agit pas d'une BAV complète."}, "2": {"exist": true, "char_ranges": [[608, 720]], "word_ranges": [[110, 129]], "text": "On ne peut pas non plus répondre à l'option 2 : exclure, que dit-on pour exclure... C'est comme jamais/toujours."}, "3": {"exist": true, "char_ranges": [[721, 974]], "word_ranges": [[129, 168]], "text": "L'option 3 n'a pas beaucoup de sens : oui, on pourrait suspendre ceux qui altèrent la conduction (bêtabloquants, antagonistes calciques), mais les médicaments qui la facilitent, tant qu'on ne lui donne pas une perfusion d'atropine à emporter chez lui..."}, "4": {"exist": true, "char_ranges": [[975, 1207]], "word_ranges": [[168, 209]], "text": "Dans l'option 4, dire qu'il a un bloc sino-auriculaire, c'est un peu du vent : sans étude électrophysiologique, on ne peut pas le dire, il n'a pas forcément de bloc SA, et cela ne suffit pas à indiquer l'implantation d'un pacemaker."}, "5": {"exist": false, "char_ranges": [], "word_ranges": [], "text": ""}}} +{"id": 370, "year": 2016, "question_id_specific": 125, "full_question": "Un ancien fumeur, chez qui on a déjà diagnostiqué une BPCO, se présente pour un bilan de santé. L'examen physique révèle l'existence d'un hypokratisme digital d'apparition récente. Quelle est l'explication la plus acceptable de cette constatation dans le contexte clinique décrit ?", "full_answer": "Dans le cas d'une hypokratite digitale d'apparition récente chez un patient fumeur, la première option à exclure est le carcinome broncho-pulmonaire. D'autres pathologies peuvent également survenir, mais pas de manière aussi soudaine.", "type": "PNEUMOLOGIE ET CHIRURGIE THORACIQUE", "options": {"1": "Carcinome pulmonaire.", "2": "Bronchectasie.", "3": "Fibrose pulmonaire.", "4": "Maladie cardiaque cyanosée.", "5": null}, "correct_option": 1, "explanations": {"1": {"exist": true, "char_ranges": [[0, 149]], "word_ranges": [[0, 21]], "text": "Dans le cas d'une hypokratite digitale d'apparition récente chez un patient fumeur, la première option à exclure est le carcinome broncho-pulmonaire."}, "2": {"exist": true, "char_ranges": [[150, 234]], "word_ranges": [[21, 32]], "text": "D'autres pathologies peuvent également survenir, mais pas de manière aussi soudaine."}, "3": {"exist": true, "char_ranges": [[150, 234]], "word_ranges": [[21, 32]], "text": "D'autres pathologies peuvent également survenir, mais pas de manière aussi soudaine."}, "4": {"exist": true, "char_ranges": [[150, 234]], "word_ranges": [[21, 32]], "text": "D'autres pathologies peuvent également survenir, mais pas de manière aussi soudaine."}, "5": {"exist": false, "char_ranges": [], "word_ranges": [], "text": ""}}} +{"id": 332, "year": 2016, "question_id_specific": 88, "full_question": "Les tests de routine effectués chez une femme de 59 ans, fumeuse de 20 cigarettes/jour depuis 25 ans, montrent une hypercalcémie de 11,3 mg/dl avec un taux de phosphore de 3,4 mg/dl. Il ne serait PAS efficace d'emblée :", "full_answer": "Hydroxyproline urinaire : L'excrétion urinaire d'hydroxyproline reflète la dégradation du collagène osseux, mais elle est également influencée par le métabolisme d'autres tissus (cartilage, peau) et par l'absorption de produits riches en collagène tels que la viande ou les gélatines. En raison de la diversité de son origine tissulaire et de son schéma métabolique, elle est mal corrélée à la résorption osseuse.", "type": "ENDOCRINOLOGIE", "options": {"1": "Déterminer les taux sériques de PTH.", "2": "Déterminer les taux sériques de vitamine D.", "3": "Détermination de l'hydroxyprolinurie.", "4": "Une radiographie thoracique simple.", "5": null}, "correct_option": 3, "explanations": {"1": {"exist": false, "char_ranges": [], "word_ranges": [], "text": ""}, "2": {"exist": false, "char_ranges": [], "word_ranges": [], "text": ""}, "3": {"exist": true, "char_ranges": [[0, 284]], "word_ranges": [[0, 39]], "text": "Hydroxyproline urinaire : L'excrétion urinaire d'hydroxyproline reflète la dégradation du collagène osseux, mais elle est également influencée par le métabolisme d'autres tissus (cartilage, peau) et par l'absorption de produits riches en collagène tels que la viande ou les gélatines."}, "4": {"exist": false, "char_ranges": [], "word_ranges": [], "text": ""}, "5": {"exist": false, "char_ranges": [], "word_ranges": [], "text": ""}}} +{"id": 78, "year": 2012, "question_id_specific": 33, "full_question": "Une femme ménopausée de 55 ans consulte pour asthénie et dyspnée à l'effort. Dans l'anamnèse, elle signale de légères épigastralgies et des brûlures d'estomac occasionnelles. Pas de métrorragies. Examens de laboratoire : Hb6 g/dL, MCV 69 fl, sidérémie 13 microgrammes/dL, ferritine 4 ngr/mL. Endoscopie gastro-intestinale : petite hernie hiatale coulissante sans signe d'oesophagite. Quelle est l'approche la plus correcte ?", "full_answer": "Nous avons un cas d'anémie ferriprive chronique, sans saignement apparent. La première cause d'anémie ferriprive est le saignement occulte d'origine digestive. L'existence de symptômes cliniques évocateurs d'une pathologie du tube digestif supérieur nous a conduit à débuter l'étude par une endoscopie digestive haute, mais aucune pathologie ne justifie l'anémie et il faut donc poursuivre les investigations, ce qui exclut les réponses 1 et 2. L'absence de métrorragies exclut la réponse 5 tant que l'origine digestive du saignement n'est pas écartée. Entre 3 et 4, les lésions du gros intestin sont beaucoup plus fréquentes et doivent être éliminées avant les lésions du petit intestin. En fait, les recommandations relatives à l'endoscopie par capsule dans les études sur l'anémie sont limitées.", "type": "SYSTÈME DIGESTIF", "options": {"1": "Administrer du fer par voie orale et surveiller l'évolution de l'anémie.", "2": "Traiter avec des inhibiteurs de la pompe à protons et évaluer après trois mois.", "3": "Recommander une coloscopie complète.", "4": "Réaliser une étude endoscopique de la capsule.", "5": "Demander une évaluation gynécologique."}, "correct_option": 3, "explanations": {"1": {"exist": true, "char_ranges": [[160, 444]], "word_ranges": [[21, 64]], "text": "L'existence de symptômes cliniques évocateurs d'une pathologie du tube digestif supérieur nous a conduit à débuter l'étude par une endoscopie digestive haute, mais aucune pathologie ne justifie l'anémie et il faut donc poursuivre les investigations, ce qui exclut les réponses 1 et 2."}, "2": {"exist": true, "char_ranges": [[160, 444]], "word_ranges": [[21, 64]], "text": "L'existence de symptômes cliniques évocateurs d'une pathologie du tube digestif supérieur nous a conduit à débuter l'étude par une endoscopie digestive haute, mais aucune pathologie ne justifie l'anémie et il faut donc poursuivre les investigations, ce qui exclut les réponses 1 et 2."}, "3": {"exist": true, "char_ranges": [[553, 688]], "word_ranges": [[80, 103]], "text": "Entre 3 et 4, les lésions du gros intestin sont beaucoup plus fréquentes et doivent être éliminées avant les lésions du petit intestin."}, "4": {"exist": true, "char_ranges": [[553, 688]], "word_ranges": [[80, 103]], "text": "Entre 3 et 4, les lésions du gros intestin sont beaucoup plus fréquentes et doivent être éliminées avant les lésions du petit intestin."}, "5": {"exist": true, "char_ranges": [[445, 552]], "word_ranges": [[64, 80]], "text": "L'absence de métrorragies exclut la réponse 5 tant que l'origine digestive du saignement n'est pas écartée."}}} +{"id": 397, "year": 2016, "question_id_specific": 140, "full_question": "Vous évaluez une patiente de 66 ans qui présente des douleurs à l'aine accentuées par une station debout prolongée quelques jours par mois. La radiographie simple des hanches montre un rétrécissement de l'espace articulaire fémoro-acétabulaire, une sclérose et des ostéophytes. Quelle est votre approche ?", "full_answer": "Il présente une coxarthrose (aucun autre examen d'imagerie n'est nécessaire) et les options conservatrices sont épuisées avant d'opter pour un traitement chirurgical. Les opioïdes faibles ne sont pas le traitement initial de choix.", "type": "TRAUMATOLOGIE ET ORTHOPÉDIE", "options": {"1": "Je pose un diagnostic de coxarthrose et j'envoie le traumatologue poser une prothèse de hanche.", "2": "Initiation d'un traitement avec des opioïdes faibles qui ont démontré leur capacité à stopper la progression de la maladie.", "3": "J'ai commencé le traitement avec du paracétamol, j'ai expliqué que l'évolution est très variable et que l'indication d'une intervention chirurgicale dépend de la fonctionnalité et du contrôle de la douleur.", "4": "En raison des caractéristiques radiologiques décrites, j'ai besoin d'une IRM de la hanche avant de prendre une décision thérapeutique.", "5": null}, "correct_option": 3, "explanations": {"1": {"exist": false, "char_ranges": [], "word_ranges": [], "text": ""}, "2": {"exist": true, "char_ranges": [[0, 231]], "word_ranges": [[0, 33]], "text": "Il présente une coxarthrose (aucun autre examen d'imagerie n'est nécessaire) et les options conservatrices sont épuisées avant d'opter pour un traitement chirurgical. Les opioïdes faibles ne sont pas le traitement initial de choix."}, "3": {"exist": true, "char_ranges": [[0, 231]], "word_ranges": [[0, 33]], "text": "Il présente une coxarthrose (aucun autre examen d'imagerie n'est nécessaire) et les options conservatrices sont épuisées avant d'opter pour un traitement chirurgical. Les opioïdes faibles ne sont pas le traitement initial de choix."}, "4": {"exist": false, "char_ranges": [], "word_ranges": [], "text": ""}, "5": {"exist": false, "char_ranges": [], "word_ranges": [], "text": ""}}} +{"id": 275, "year": 2016, "question_id_specific": 74, "full_question": "Un homme de 67 ans, atteint d'une cirrhose du foie et buveur actif, a été admis pour une distension abdominale progressive accompagnée d'un malaise diffus, évoluant depuis deux semaines. Il n'a pas signalé de fièvre ni d'autres symptômes. L'examen a révélé une matité changeante à la percussion abdominale, sans œdème. Une paracentèse diagnostique a été effectuée et un liquide légèrement trouble a été trouvé, avec 2 300 cellules/mL, dont 30 % de lymphocytes, 60 % de polymorphonucléaires et 10 % de globules rouges. Quelle est la première mesure thérapeutique immédiate que vous prescrirez à ce patient ?", "full_answer": "Il s'agit d'une péritonite bactérienne spontanée (PBS), définie par la présence de > 250 polymorphonucléaires/mL dans le liquide ascitique (dans ce cas, il y en a 1 380, soit 60 % de 2 300). C'est une indication pour commencer les céphalosporines de 3ème génération (ceftriaxone ou céfotaxime) et l'albumine. Les autres sont des traitements qui peuvent attendre une amélioration (on ne parle pas d'ascite de tension).", "type": "SYSTÈME DIGESTIF", "options": {"1": "Restriction de la consommation de sel et de liquide.", "2": "Traitement diurétique par spironolactone orale.", "3": "Traitement par une céphalosporine de troisième génération.", "4": "Paracentèse évacuatrice.", "5": null}, "correct_option": 3, "explanations": {"1": {"exist": true, "char_ranges": [[309, 417]], "word_ranges": [[49, 66]], "text": "Les autres sont des traitements qui peuvent attendre une amélioration (on ne parle pas d'ascite de tension)."}, "2": {"exist": true, "char_ranges": [[309, 417]], "word_ranges": [[49, 66]], "text": "Les autres sont des traitements qui peuvent attendre une amélioration (on ne parle pas d'ascite de tension)."}, "3": {"exist": true, "char_ranges": [[0, 308]], "word_ranges": [[0, 49]], "text": "Il s'agit d'une péritonite bactérienne spontanée (PBS), définie par la présence de > 250 polymorphonucléaires/mL dans le liquide ascitique (dans ce cas, il y en a 1 380, soit 60 % de 2 300). C'est une indication pour commencer les céphalosporines de 3ème génération (ceftriaxone ou céfotaxime) et l'albumine."}, "4": {"exist": true, "char_ranges": [[309, 417]], "word_ranges": [[49, 66]], "text": "Les autres sont des traitements qui peuvent attendre une amélioration (on ne parle pas d'ascite de tension)."}, "5": {"exist": false, "char_ranges": [], "word_ranges": [], "text": ""}}} +{"id": 418, "year": 2018, "question_id_specific": 77, "full_question": "Une femme de 80 ans s'est présentée aux urgences avec des douleurs abdominales débutant dans l'épigastre et irradiant ensuite vers la fosse iliaque gauche. Elle était accompagnée d'une fièvre de 37,5 ºC. Un scanner abdominal a été réalisé, montrant une inflammation des parois du sigma et un abcès mésentérique de 2 cm. Le traitement de choix est le suivant :", "full_answer": "Ils nous interrogent sur la prise en charge d'une pathologie très fréquente, la diverticulite. Et on nous demande quelles sont les différentes options thérapeutiques en fonction du degré d'avancement de la maladie. La bonne réponse est 1, car elle ne remplit pas les critères pour nécessiter un drainage par ponction guidée (l'abcès ne dépasse pas 5-6 cm). La colostomie de décharge dans ce cas n'a aucun sens, car il ne s'agit pas de résoudre un état obstructif. Le drainage par chirurgie laparoscopique n'est pas indiqué dans ce cas. Il pourrait être indiqué en l'absence de possibilité de drainage percutané après échec du traitement conservateur, bien qu'il s'agisse d'une option très controversée à l'heure actuelle. La sigmoïdectomie et l'anastomose ne sont pas non plus nécessaires. Dans un grand nombre de cas, après la résolution de l'affection, les poussées sont réduites et peuvent être évitées grâce à un régime et à des habitudes alimentaires hygiéniques.", "type": "CHIRURGIE GÉNÉRALE", "options": {"1": "Admission dans le service avec régime alimentaire absolu et traitement antibiotique à large spectre.", "2": "Décharge de la colostomie.", "3": "Drainage par chirurgie laparoscopique.", "4": "Chirurgie urgente avec sigmoïdectomie et anastomose colorectale.", "5": null}, "correct_option": 1, "explanations": {"1": {"exist": true, "char_ranges": [[790, 968]], "word_ranges": [[124, 153]], "text": "Dans un grand nombre de cas, après la résolution de l'affection, les poussées sont réduites et peuvent être évitées grâce à un régime et à des habitudes alimentaires hygiéniques."}, "2": {"exist": true, "char_ranges": [[357, 463]], "word_ranges": [[57, 77]], "text": "La colostomie de décharge dans ce cas n'a aucun sens, car il ne s'agit pas de résoudre un état obstructif."}, "3": {"exist": true, "char_ranges": [[239, 356]], "word_ranges": [[37, 57]], "text": "car elle ne remplit pas les critères pour nécessiter un drainage par ponction guidée (l'abcès ne dépasse pas 5-6 cm)."}, "4": {"exist": true, "char_ranges": [[722, 789]], "word_ranges": [[114, 124]], "text": "La sigmoïdectomie et l'anastomose ne sont pas non plus nécessaires."}, "5": {"exist": false, "char_ranges": [], "word_ranges": [], "text": ""}}} +{"id": 200, "year": 2013, "question_id_specific": 75, "full_question": "Une femme de 75 ans consulte pour insomnie. Elle signale que depuis des années, elle se réveille la nuit avec une sensation de picotement et de brûlure dans les jambes et parfois dans les bras ; ces symptômes se manifestent également le soir. Elle constate une amélioration lorsqu'elle bouge ses jambes, mais les symptômes réapparaissent lorsqu'elle se repose, de sorte qu'elle ne parvient plus à s'endormir. Ce tableau pourrait permettre de diagnostiquer un syndrome des jambes sans repos. Lequel des énoncés suivants est FAUX ?", "full_answer": "Le diagnostic est basé sur des critères cliniques, de sorte que la biopsie musculaire n'est pratiquée que pour exclure d'autres maladies, et non pour confirmer celle-ci. Un profil de fer, des tests d'hormones thyroïdiennes et de vitamines doivent être effectués afin d'exclure d'autres maladies présentant des symptômes similaires. Le traitement est symptomatique et est mis en place lorsqu'il interfère avec le sommeil ou la qualité de vie. Les agonistes dopaminergiques tels que le pramipexole et le ropirinol constituent le traitement de premier choix.", "type": "NEUROLOGIE", "options": {"1": "Le diagnostic de cette maladie repose sur des critères cliniques.", "2": "Un test sanguin de base comprenant un profil de fer, des hormones thyroïdiennes, de la B 12 et de l'acide folique doit être effectué.", "3": "Le traitement serait indiqué chez les patients qui présentent des troubles du sommeil ou de la qualité de vie, mais ne modifie pas l'évolution de la maladie.", "4": "Le diagnostic est confirmé par une biopsie musculaire.", "5": "Le pramipexole et le ropirinol sont utilisés dans le traitement."}, "correct_option": 4, "explanations": {"1": {"exist": false, "char_ranges": [], "word_ranges": [], "text": ""}, "2": {"exist": true, "char_ranges": [[170, 331]], "word_ranges": [[26, 48]], "text": "Un profil de fer, des tests d'hormones thyroïdiennes et de vitamines doivent être effectués afin d'exclure d'autres maladies présentant des symptômes similaires."}, "3": {"exist": true, "char_ranges": [[332, 555]], "word_ranges": [[48, 83]], "text": "Le traitement est symptomatique et est mis en place lorsqu'il interfère avec le sommeil ou la qualité de vie. Les agonistes dopaminergiques tels que le pramipexole et le ropirinol constituent le traitement de premier choix."}, "4": {"exist": true, "char_ranges": [[0, 169]], "word_ranges": [[0, 26]], "text": "Le diagnostic est basé sur des critères cliniques, de sorte que la biopsie musculaire n'est pratiquée que pour exclure d'autres maladies, et non pour confirmer celle-ci."}, "5": {"exist": true, "char_ranges": [[332, 555]], "word_ranges": [[48, 83]], "text": "Le traitement est symptomatique et est mis en place lorsqu'il interfère avec le sommeil ou la qualité de vie. Les agonistes dopaminergiques tels que le pramipexole et le ropirinol constituent le traitement de premier choix."}}} +{"id": 469, "year": 2020, "question_id_specific": 137, "full_question": "Femme de 93 ans ayant des antécédents de BPCO, d'hypertension, de dyslipidémie, de fibrillation auriculaire et de cardiopathie ischémique. Elle vit seule. Elle a une fraction d'éjection ventriculaire de 53 %, une créatinine de 1,5 mg/dl et une coagulation normale. Elle reçoit un traitement à base de bronchodilatateurs inhalés et d'AAS 100 mg par jour. Il fait une chute dans la rue, après laquelle il lui est impossible de se lever ou de s'asseoir, avec une douleur intense dans la hanche droite lors des mouvements, un raccourcissement sévère et une rotation externe de la jambe. Quand et comment doit-il être traité ?", "full_answer": "\"Une nouvelle approche du traitement de ces patients est en cours de développement, qui vise à coordonner l'activité des différents secteurs impliqués dès le départ, en évitant le transfert du patient d'un secteur à l'autre à la fin du traitement de chaque partie. Cette idée a donné naissance aux \"unités orthogériatriques\", qui combinent l'activité chirurgicale du chirurgien avec celle de la rééducation immédiate du patient et les contrôles et la surveillance de la médecine interne. Dès le début du processus, des assistants sociaux et diverses aides vitales et de soutien sont mis en place pour assurer une récupération aussi rapide et complète que possible. ... Dans la plupart des cas, la chirurgie doit être pratiquée le plus tôt possible, dès que le patient est médicalement stabilisé. Il existe une contre-indication à la chirurgie chez les patients présentant une comorbidité médicale instable, mais plusieurs études ont montré que retarder l'opération au-delà de 48 heures après l'admission augmente la mortalité. C'est pourquoi le moment idéal pour opérer ces patients est après la stabilisation de leurs pathologies médicales, l'objectif étant que cette stabilisation intervienne dans les 48 heures. Il n'y a pas de données suggérant qu'il présente une comorbidité instable à l'heure actuelle, d'où l'intérêt d'une chirurgie dans les 48 heures et d'une prise en charge orthogériatrique.", "type": "CHIRURGIE ORTHOPÉDIQUE ET TRAUMATOLOGIE", "options": {"1": "Admission pour échographie transthoracique, évaluation par un cardiologue, puis réduction et fixation à l'aide d'une plaque vissée.", "2": "Admission pour évaluation par la néphrologie et la cardiologie, puis la traumatologie décidera de l'utilisation d'une arthroplastie ou d'un clou trochantérien.", "3": "Admission en médecine interne / gériatrie, stabilisation progressive des comorbidités, traitement orthopédique conservateur.", "4": "Réduction et fixation de la fracture dans les 48 heures, sédation très précoce, prise en charge orthogériatrique.", "5": null}, "correct_option": 4, "explanations": {"1": {"exist": false, "char_ranges": [], "word_ranges": [], "text": ""}, "2": {"exist": false, "char_ranges": [], "word_ranges": [], "text": ""}, "3": {"exist": false, "char_ranges": [], "word_ranges": [], "text": ""}, "4": {"exist": true, "char_ranges": [[1027, 1401]], "word_ranges": [[158, 214]], "text": "C'est pourquoi le moment idéal pour opérer ces patients est après la stabilisation de leurs pathologies médicales, l'objectif étant que cette stabilisation intervienne dans les 48 heures. Il n'y a pas de données suggérant qu'il présente une comorbidité instable à l'heure actuelle, d'où l'intérêt d'une chirurgie dans les 48 heures et d'une prise en charge orthogériatrique."}, "5": {"exist": false, "char_ranges": [], "word_ranges": [], "text": ""}}} +{"id": 378, "year": 2016, "question_id_specific": 132, "full_question": "Un patient de 72 ans présente depuis un an des pertes de mémoire progressives et des difficultés à trouver les mots. Quel est l'examen le plus utile pour caractériser le type de déficit cognitif qu'il présente ?", "full_answer": "La caractérisation des processus de déficience cognitive continue d'être clinique, les tests d'imagerie (IRM ou PET) étant parfois un outil de soutien. La première option serait la bonne car elle permet une mesure standardisée de chacun des domaines cognitifs, ce qui nous permettra d'encadrer les déficits du patient dans un syndrome spécifique et de suivre la détérioration avec des scanners répétés dans le temps.", "type": "NEUROLOGIE", "options": {"1": "Évaluation neuropsychologique.", "2": "Imagerie par résonance magnétique du cerveau.", "3": "Tomographie par émission de positrons.", "4": "Électroencéphalogramme.", "5": null}, "correct_option": 1, "explanations": {"1": {"exist": true, "char_ranges": [[0, 259]], "word_ranges": [[0, 39]], "text": "La caractérisation des processus de déficience cognitive continue d'être clinique, les tests d'imagerie (IRM ou PET) étant parfois un outil de soutien. La première option serait la bonne car elle permet une mesure standardisée de chacun des domaines cognitifs,"}, "2": {"exist": false, "char_ranges": [], "word_ranges": [], "text": ""}, "3": {"exist": true, "char_ranges": [[0, 259]], "word_ranges": [[0, 39]], "text": "La caractérisation des processus de déficience cognitive continue d'être clinique, les tests d'imagerie (IRM ou PET) étant parfois un outil de soutien. La première option serait la bonne car elle permet une mesure standardisée de chacun des domaines cognitifs,"}, "4": {"exist": false, "char_ranges": [], "word_ranges": [], "text": ""}, "5": {"exist": false, "char_ranges": [], "word_ranges": [], "text": ""}}} +{"id": 582, "year": 2022, "question_id_specific": 208, "full_question": "Un homme de 53 ans sans antécédents qui, après un mauvais mouvement à la salle de sport, a ressenti une douleur lombaire qui, au fil des heures, a irradié vers l'arrière de la jambe droite et atteint le bord latéral du pied. À l'examen, il présentait une manœuvre de Lasègue positive à 40o, un réflexe ischio-jambier aboli et était incapable de se tenir sur la pointe des pieds. Le diagnostic étiologique le plus probable est le suivant :", "full_answer": "Il décrit l'atteinte de la racine S1 comme l'incapacité de marcher sur la pointe des pieds et l'absence de réflexe d'Achille, en plus de la distribution de la douleur le long de la jambe. Parmi nos options, la cause la plus probable de l'atteinte S1 est la hernie discale L5-S1.", "type": "NEUROLOGIE", "options": {"1": "Hernie discale L2-L3 droite.", "2": "Hernie discale L3-L4 droite.", "3": "Hernie discale L4-L5 droite.", "4": "Hernie discale L5-S1 droite.", "5": null}, "correct_option": 4, "explanations": {"1": {"exist": false, "char_ranges": [], "word_ranges": [], "text": ""}, "2": {"exist": false, "char_ranges": [], "word_ranges": [], "text": ""}, "3": {"exist": false, "char_ranges": [], "word_ranges": [], "text": ""}, "4": {"exist": true, "char_ranges": [[0, 278]], "word_ranges": [[0, 50]], "text": "Il décrit l'atteinte de la racine S1 comme l'incapacité de marcher sur la pointe des pieds et l'absence de réflexe d'Achille, en plus de la distribution de la douleur le long de la jambe. Parmi nos options, la cause la plus probable de l'atteinte S1 est la hernie discale L5-S1."}, "5": {"exist": false, "char_ranges": [], "word_ranges": [], "text": ""}}} +{"id": 160, "year": 2013, "question_id_specific": 117, "full_question": "Un homme de 35 ans présente une hématurie après plusieurs années d'infections respiratoires, l'analyse sanguine révèle une créatinine de 1 mg/dl sans autre altération et l'urine contient des globules rouges à 50/champ dont 80 % sont dysmorphiques, avec une protéinurie de 0,8 gramme en 24 heures. Quel est le diagnostic le plus probable ?", "full_answer": "La bonne réponse est : 3. néphropathie à IgA. Étant donné que la présentation clinique de cette néphropathie est très peu spécifique, il faut s'appuyer sur l'épidémiologie pour identifier la maladie responsable. Comme il s'agit d'un jeune homme, avec des antécédents d'infections respiratoires, une relation est établie entre ces infections et la pathologie rénale. Sur cette base, nous pensons à une néphropathie à IgA qui, en plus d'être la glomérulonéphrite la plus fréquente, se présente dans la plupart des cas de la manière décrite.", "type": "NEPHROLOGIE", "options": {"1": "Néphropathie à changement minime.", "2": "Glomérulonéphrite membraneuse.", "3": "Néphropathie à IgA.", "4": "Glomérulonéphrite proliférative et diffuse.", "5": "Glomérulosclérose focale et segmentaire primaire."}, "correct_option": 3, "explanations": {"1": {"exist": false, "char_ranges": [], "word_ranges": [], "text": ""}, "2": {"exist": false, "char_ranges": [], "word_ranges": [], "text": ""}, "3": {"exist": true, "char_ranges": [[212, 538]], "word_ranges": [[32, 84]], "text": "Comme il s'agit d'un jeune homme, avec des antécédents d'infections respiratoires, une relation est établie entre ces infections et la pathologie rénale. Sur cette base, nous pensons à une néphropathie à IgA qui, en plus d'être la glomérulonéphrite la plus fréquente, se présente dans la plupart des cas de la manière décrite."}, "4": {"exist": false, "char_ranges": [], "word_ranges": [], "text": ""}, "5": {"exist": false, "char_ranges": [], "word_ranges": [], "text": ""}}} +{"id": 312, "year": 2016, "question_id_specific": 217, "full_question": "Une femme de 84 ans s'est présentée avec une perte de vision de l'œil gauche depuis 4 jours, accompagnée d'une métamorphopsie. La macula présente d'abondants exsudats durs, deux petites hémorragies profondes et un décollement de rétine neurosensoriel localisé. L'œil controlatéral présente d'abondants drusen mous. Parmi les diagnostics suivants, lequel vous semble le plus probable ?", "full_answer": "En principe, il s'agit d'une question facile, puisqu'elle traite d'une maladie qui revient fréquemment dans les examens MIR. La description est typique de l'option 2 : il s'agit d'une DMLA exsudative (bien que, comme nous l'avons dit à la question 215, avec la nouvelle classification, il s'agirait d'une DMLA avancée dans sa variante néovasculaire). Quoi qu'il en soit, nous avons un patient âgé qui présente une perte de vision et une métamorphopsie dans un œil. L'examen révèle des lésions dans la macula. Cela nous permet d'exclure un décollement postérieur aigu du vitré, qui n'entraîne ni perte de vision, ni métamorphopsie, ni lésions maculaires. Il en va de même pour la neuropathie optique ischémique antérieure non artéritique, car bien qu'elle entraîne une perte de vision et qu'elle soit typique des personnes âgées, elle ne provoque pas de métamorphopsie. Il n'y a pas de lésions dans la macula, mais un œdème de la papille. Dans l'obstruction de l'artère centrale de la rétine, la métamorphopsie n'est pas particulièrement caractéristique (la perte visuelle et l'âge avancé le sont), mais les lésions du fond de l'œil varient. En cas d'occlusion artérielle, il existe un œdème intracellulaire qui se traduit par une absence d'hémorragies et un fond d'œil blanchâtre et pâle. Il affecte aussi globalement toute la rétine et une tache rouge cerise peut apparaître dans la macula. Dans ce cas, les lésions sont totalement différentes. Elles décrivent des exsudats durs, des hémorragies profondes et un décollement neurosensoriel localisé de la rétine. Tout cela dans la macula. Ce sont les caractéristiques de la DMLA néovasculaire (ou exsudative). À titre d'indice, l'autre œil présente d'abondants drusen mous. Les drusen sont des manifestations typiques de la DMLA. Vous avez donc une DMLA dans les deux yeux et, dans l'œil droit, elle s'est récemment compliquée par l'apparition d'une néovascularisation.", "type": "OPHTHALMOLOGIE", "options": {"1": "Décollement postérieur aigu du vitré.", "2": "Dégénérescence maculaire liée à l'âge (DMLA) exsudative.", "3": "Obstruction de l'artère centrale de la rétine.", "4": "Neuropathie optique ischémique antérieure non artéritique.", "5": null}, "correct_option": 2, "explanations": {"1": {"exist": true, "char_ranges": [[465, 653]], "word_ranges": [[75, 103]], "text": "L'examen révèle des lésions dans la macula. Cela nous permet d'exclure un décollement postérieur aigu du vitré, qui n'entraîne ni perte de vision, ni métamorphopsie, ni lésions maculaires."}, "2": {"exist": true, "char_ranges": [[1446, 1659]], "word_ranges": [[230, 261]], "text": "Elles décrivent des exsudats durs, des hémorragies profondes et un décollement neurosensoriel localisé de la rétine. Tout cela dans la macula. Ce sont les caractéristiques de la DMLA néovasculaire (ou exsudative)."}, "3": {"exist": true, "char_ranges": [[1141, 1445]], "word_ranges": [[182, 230]], "text": "En cas d'occlusion artérielle, il existe un œdème intracellulaire qui se traduit par une absence d'hémorragies et un fond d'œil blanchâtre et pâle. Il affecte aussi globalement toute la rétine et une tache rouge cerise peut apparaître dans la macula. Dans ce cas, les lésions sont totalement différentes."}, "4": {"exist": true, "char_ranges": [[654, 868]], "word_ranges": [[103, 137]], "text": "Il en va de même pour la neuropathie optique ischémique antérieure non artéritique, car bien qu'elle entraîne une perte de vision et qu'elle soit typique des personnes âgées, elle ne provoque pas de métamorphopsie."}, "5": {"exist": false, "char_ranges": [], "word_ranges": [], "text": ""}}} +{"id": 24, "year": 2011, "question_id_specific": 115, "full_question": "Un homme de 68 ans est examiné pour une fièvre fébrile51e, une perte de poids et une dyspnée d'effort. Le PE révèle un nouveau souffle diastolique aortique et des stigmates d'embolie périphérique. L'échocardiographie montre une végétation de 1 cm sur la valve aortique et deux hémocultures isolent Streptococcus bovis sensible à la pénicilline. Après le début du traitement de l'endocardite, lequel des examens suivants pourrait nous aider à poser le diagnostic ?", "full_answer": "Un autre cadeau d'une question, l'association de l'endocardite S.bovis et du cancer du côlon. Au fait, il est amusant de constater que la radiographie thoracique est considérée comme un examen à effectuer sur le site ..... Êtes-vous en train de me dire qu'un patient fiévreux et dyspnéique subira un échocardiogramme avant une simple radiographie du thorax ? Et d'un autre côté... Pourquoi les personnes qui rédigent le MIR ne posent-elles pas des questions plus courtes sans autant de va-et-vient ? Si ce qu'ils veulent, c'est demander le test à effectuer sur un patient atteint d'endocardite due à S. bovis, ils pourraient poser la question directement sans nous parler des grains de beauté du patient ?", "type": "INFECTIEUX", "options": {"1": "Test de tolérance au glucose.", "2": "Radiographie pulmonaire.", "3": "Endoscopie gastro-intestinale supérieure.", "4": "Coloscopie.", "5": "Échographie abdominale."}, "correct_option": 4, "explanations": {"1": {"exist": false, "char_ranges": [], "word_ranges": [], "text": ""}, "2": {"exist": false, "char_ranges": [], "word_ranges": [], "text": ""}, "3": {"exist": false, "char_ranges": [], "word_ranges": [], "text": ""}, "4": {"exist": true, "char_ranges": [[32, 93]], "word_ranges": [[5, 14]], "text": "l'association de l'endocardite S.bovis et du cancer du côlon."}, "5": {"exist": false, "char_ranges": [], "word_ranges": [], "text": ""}}} +{"id": 260, "year": 2014, "question_id_specific": 99, "full_question": "Une femme de 45 ans s'est présentée à la clinique, référée par la chirurgie, avec un diagnostic de tumeur neuroendocrine posé après une pancréatectomie partielle pour une tumeur de 2 cm dans la queue du pancréas. La tumeur avait été détectée par hasard lors d'un scanner abdominal demandé pour compléter l'étude d'un simple kyste hépatique. L'interrogatoire de la patiente met en évidence des antécédents de règles irrégulières, d'aménorrhée depuis 6 mois et de coliques néphrétiques à répétition depuis l'âge de 20 ans, pour lesquelles elle a eu recours à la lithotritie à plusieurs reprises. Quelle est votre hypothèse diagnostique ?", "full_answer": "Très belle question clinique de MEN. Tumeur neuroendocrine pancréatique + adénome hypophysaire (prolactinome) + hyperparathyroïdie (colique néphrétique du sujet jeune) : MEN 1. Difficulté moyenne car il faut connaître la triade MEN et reconnaître les différentes tumeurs.", "type": "ENDOCRINOLOGIE", "options": {"1": "Néoplasie endocrinienne multiple de type 1 ou syndrome de Wermer.", "2": "Néoplasie endocrinienne multiple de type 2 A ou syndrome de Sipple.", "3": "Néoplasie endocrinienne multiple de type 2B.", "4": "Somatostatinome.", "5": "Une tumeur neuroendocrine produisant de la PTH."}, "correct_option": 1, "explanations": {"1": {"exist": true, "char_ranges": [[37, 176]], "word_ranges": [[6, 23]], "text": "Tumeur neuroendocrine pancréatique + adénome hypophysaire (prolactinome) + hyperparathyroïdie (colique néphrétique du sujet jeune) : MEN 1."}, "2": {"exist": false, "char_ranges": [], "word_ranges": [], "text": ""}, "3": {"exist": false, "char_ranges": [], "word_ranges": [], "text": ""}, "4": {"exist": false, "char_ranges": [], "word_ranges": [], "text": ""}, "5": {"exist": false, "char_ranges": [], "word_ranges": [], "text": ""}}} +{"id": 211, "year": 2014, "question_id_specific": 87, "full_question": "Une femme de 78 ans s'est présentée aux urgences pour une douleur dans la fosse iliaque gauche depuis 24 heures, associée à de la fièvre et à des vomissements occasionnels. L'examen révèle une douleur sélective à la palpation de la fosse iliaque gauche avec une sensation d'occupation, de défense et de décompression positive. En cas de suspicion de diverticulite aiguë, laquelle des affirmations suivantes est correcte ?", "full_answer": "1 est incorrect car les examens d'imagerie de choix seraient l'échographie ou le scanner ; 3 est incorrect car l'approche laparoscopique est indiquée dès le départ ; 4 est incorrect car l'indication dans le premier épisode concerne les personnes de moins de 50 ans, dans les autres cas elle est indiquée après le deuxième épisode ou en cas de symptômes persistants ou de suspicion de néoplasie ; et 5 est incorrect car la chirurgie en cas de perforation avec péritonite doit toujours réséquer le segment de côlon affecté pour contrôler le foyer de l'infection.", "type": "CHIRURGIE GÉNÉRALE", "options": {"1": "L'examen complémentaire le plus sûr et le plus performant est le lavement baryté.", "2": "En cas d'abcès pelvien contenu, un drainage percutané sous guidage tomodensitométrique ou échographique est indiqué.", "3": "Si une intervention chirurgicale est nécessaire après la résolution de l'épisode aigu, l'approche laparoscopique est contre-indiquée.", "4": "En cas de diverticulite aiguë non compliquée, une sigmoïdectomie élective est indiquée après la guérison du premier épisode aigu.", "5": "En cas de péritonite généralisée, la technique chirurgicale la plus appropriée consiste à réaliser une colostomie de dérivation sans résection du segment sigmoïde affecté."}, "correct_option": 2, "explanations": {"1": {"exist": true, "char_ranges": [[0, 90]], "word_ranges": [[0, 15]], "text": "1 est incorrect car les examens d'imagerie de choix seraient l'échographie ou le scanner ;"}, "2": {"exist": false, "char_ranges": [], "word_ranges": [], "text": ""}, "3": {"exist": true, "char_ranges": [[91, 165]], "word_ranges": [[15, 27]], "text": "3 est incorrect car l'approche laparoscopique est indiquée dès le départ ;"}, "4": {"exist": true, "char_ranges": [[166, 395]], "word_ranges": [[27, 67]], "text": "4 est incorrect car l'indication dans le premier épisode concerne les personnes de moins de 50 ans, dans les autres cas elle est indiquée après le deuxième épisode ou en cas de symptômes persistants ou de suspicion de néoplasie ;"}, "5": {"exist": true, "char_ranges": [[399, 560]], "word_ranges": [[68, 94]], "text": "5 est incorrect car la chirurgie en cas de perforation avec péritonite doit toujours réséquer le segment de côlon affecté pour contrôler le foyer de l'infection."}}} +{"id": 162, "year": 2013, "question_id_specific": 121, "full_question": "Un patient de 52 ans atteint d'une maladie rénale chronique de stade V, secondaire à une polykystose rénale autosomique dominante, reçoit une greffe de rein d'un donneur cadavérique. Un de ses frères, âgé de 34 ans, vient lui rendre visite et lui dit qu'il ne sait pas s'il est atteint de la maladie parce qu'il n'a jamais vu de médecin depuis son adolescence. Quelle attitude ou quel test vous semble le plus approprié à recommander à ce moment-là ?", "full_answer": "La bonne réponse est : 2. l'échographie abdomino-pelvienne. L'échographie abdomino-pelvienne est sans aucun doute le test le plus spécifique et le plus sensible pour le diagnostic de la polykystose rénale et, surtout, le test le moins coûteux pour déterminer si le frère ou la sœur du patient a hérité de la polykystose rénale.", "type": "NEPHROLOGIE", "options": {"1": "Étude génétique mutationnelle et de liaison.", "2": "Échographie abdomino-pelvienne.", "3": "Tomographie axiale hélicoïdale avec contraste iodé.", "4": "IRM abdominale et cérébrale.", "5": "Examens cliniques réguliers."}, "correct_option": 2, "explanations": {"1": {"exist": false, "char_ranges": [], "word_ranges": [], "text": ""}, "2": {"exist": true, "char_ranges": [[60, 327]], "word_ranges": [[8, 53]], "text": "L'échographie abdomino-pelvienne est sans aucun doute le test le plus spécifique et le plus sensible pour le diagnostic de la polykystose rénale et, surtout, le test le moins coûteux pour déterminer si le frère ou la sœur du patient a hérité de la polykystose rénale."}, "3": {"exist": false, "char_ranges": [], "word_ranges": [], "text": ""}, "4": {"exist": false, "char_ranges": [], "word_ranges": [], "text": ""}, "5": {"exist": false, "char_ranges": [], "word_ranges": [], "text": ""}}} +{"id": 207, "year": 2014, "question_id_specific": 181, "full_question": "Une femme de 43 ans a été adressée au service de pathologie de l'appareil génital inférieur de l'hôpital pour avoir présenté une cytologie cervico-vaginale rapportée comme \"L-SIL\" (Low Grade Intraepithelial Lesion). Elle a signalé des infections vaginales répétées (trichomonase,...) et a eu plus de 5 partenaires sexuels dans sa vie. La meilleure stratégie de soins pour cette femme est la suivante :", "full_answer": "La meilleure réponse à cette question est le numéro 5. La plupart des lésions identifiées comme L-SIL sont associées à une infection par le HPV : il est donc conseillé d'arrêter de fumer, car c'est un facteur de progression de l'infection par le HPV. D'autre part, selon les dernières études, 70 % des cytologies signalées comme LSIL vont se résorber, 15 % vont persister et 15 % vont présenter une cytologie plus sévère, sans que l'on puisse différencier s'il s'agit d'une progression biologique ou d'une lésion cachée dans la cytologie initiale. C'est pourquoi il est préférable de revenir pour un contrôle à 6 mois. Compte tenu des antécédents personnels de la femme (plus de 5 partenaires sexuels et infections répétées) et de son âge, l'idéal serait de l'orienter vers une colposcopie. Cependant, la réponse 5 indique un contrôle à 6 mois, mais ne précise pas quel contrôle (il pourrait s'agir d'une colposcopie ou d'une cytologie).", "type": "GYNÉCOLOGIE ET OBSTÉTRIQUE", "options": {"1": "Conisation à l'aide d'une boucle de diathermie.", "2": "Curetage endocervical avec curetage de Kevorkian.", "3": "Biopsie de l'endomètre à l'aide d'une canule de Cornier.", "4": "Hystérectomie sans annexectomie.", "5": "Conseillez-lui d'arrêter de fumer et de prendre des contraceptifs, et de se faire examiner dans six mois."}, "correct_option": 5, "explanations": {"1": {"exist": false, "char_ranges": [], "word_ranges": [], "text": ""}, "2": {"exist": false, "char_ranges": [], "word_ranges": [], "text": ""}, "3": {"exist": false, "char_ranges": [], "word_ranges": [], "text": ""}, "4": {"exist": false, "char_ranges": [], "word_ranges": [], "text": ""}, "5": {"exist": true, "char_ranges": [[55, 618]], "word_ranges": [[10, 104]], "text": "La plupart des lésions identifiées comme L-SIL sont associées à une infection par le HPV : il est donc conseillé d'arrêter de fumer, car c'est un facteur de progression de l'infection par le HPV. D'autre part, selon les dernières études, 70 % des cytologies signalées comme LSIL vont se résorber, 15 % vont persister et 15 % vont présenter une cytologie plus sévère, sans que l'on puisse différencier s'il s'agit d'une progression biologique ou d'une lésion cachée dans la cytologie initiale. C'est pourquoi il est préférable de revenir pour un contrôle à 6 mois."}}} +{"id": 584, "year": 2022, "question_id_specific": 73, "full_question": "Une femme de 51 ans, ménopausée depuis un an et demi, a consulté pour des taches vaginales d'une durée de deux semaines. Elle signale que le spotting est inférieur à la durée d'une période menstruelle. L'échographie montre un endomètre de 7 mm. Cochez la bonne option :", "full_answer": "L'endomètre est épaissi chez une femme ménopausée (certaines lignes directrices fixent la limite à 5 et d'autres à 3 mm). Le premier examen complémentaire à réaliser sera une biopsie de l'endomètre. Si celle-ci n'est plus décisive, l'hystéroscopie sera envisagée en deuxième intention.", "type": "OBSTÉTRIQUE ET GYNÉCOLOGIE", "options": {"1": "L'endomètre est épaissi et, compte tenu des symptômes de la patiente, une biopsie de l'endomètre est effectuée à l'aide d'une canule de Cornier.", "2": "L'endomètre n'étant pas épaissi, on lui prescrit de l'acide tranexamique et un suivi ambulatoire.", "3": "Il existe une indication pour une hystéroscopie diagnostique sans nécessité de biopsie de l'endomètre.", "4": "L'endomètre n'est pas épaissi, mais comme elle présente des signes cliniques de métrorragie, une biopsie de l'endomètre est indiquée.", "5": null}, "correct_option": 1, "explanations": {"1": {"exist": true, "char_ranges": [[0, 198]], "word_ranges": [[0, 31]], "text": "L'endomètre est épaissi chez une femme ménopausée (certaines lignes directrices fixent la limite à 5 et d'autres à 3 mm). Le premier examen complémentaire à réaliser sera une biopsie de l'endomètre."}, "2": {"exist": false, "char_ranges": [], "word_ranges": [], "text": ""}, "3": {"exist": false, "char_ranges": [], "word_ranges": [], "text": ""}, "4": {"exist": false, "char_ranges": [], "word_ranges": [], "text": ""}, "5": {"exist": false, "char_ranges": [], "word_ranges": [], "text": ""}}} +{"id": 138, "year": 2012, "question_id_specific": 165, "full_question": "Un enfant se présente à la consultation externe parce qu'il porte la main à son oreille, sans fièvre ni signes inflammatoires aigus, et qu'à l'examen, on observe un tympan rosé/ambreux. Dans ses antécédents, il mentionne qu'il y a un mois, il a présenté un épisode d'otalgie, de fièvre accompagnée d'irritabilité et qu'il a été traité avec de l'amoxicilline à 40 mg/kg/jour. Quel est le diagnostic le plus probable ?", "full_answer": "La lecture du cas permet d'écarter les options suivantes : enfant avec otalgie, mais sans fièvre ni signes inflammatoires aigus : ce n'est ni 1 (otite moyenne aiguë récidivante) ni 5 (mastoïdite). L'otoscopie montre un tympan rosé, ambré : pas de squames, donc pas de 4 (otite moyenne chronique cholestéatomateuse) ; pas de perforations ni d'adhérences, donc pas de 3 (otite moyenne chronique simple). L'otoscopie est caractéristique de l'otite moyenne séreuse (réponse 2 correcte). Il est bien connu des ORL, des pédiatres et des médecins de famille que dans le mois qui suit une otite moyenne aiguë, l'oreille moyenne est occupée par un contenu muqueux dont il faut surveiller l'évolution. Je ne pense pas que cela puisse être contesté non plus.", "type": "OTORHINOLARYNGOLOGIE ET CHIRURGIE MAXILLO-FACIALE", "options": {"1": "Otite moyenne aiguë récurrente.", "2": "Otite moyenne séreuse.", "3": "Otite moyenne chronique simple.", "4": "Otite moyenne chronique cholestéatomateuse.", "5": "Mastoïdite."}, "correct_option": 2, "explanations": {"1": {"exist": true, "char_ranges": [[59, 196]], "word_ranges": [[10, 32]], "text": "enfant avec otalgie, mais sans fièvre ni signes inflammatoires aigus : ce n'est ni 1 (otite moyenne aiguë récidivante) ni 5 (mastoïdite)."}, "2": {"exist": true, "char_ranges": [[402, 482]], "word_ranges": [[64, 74]], "text": "L'otoscopie est caractéristique de l'otite moyenne séreuse (réponse 2 correcte)."}, "3": {"exist": true, "char_ranges": [[317, 401]], "word_ranges": [[51, 64]], "text": "pas de perforations ni d'adhérences, donc pas de 3 (otite moyenne chronique simple)."}, "4": {"exist": true, "char_ranges": [[197, 316]], "word_ranges": [[32, 51]], "text": "L'otoscopie montre un tympan rosé, ambré : pas de squames, donc pas de 4 (otite moyenne chronique cholestéatomateuse) ;"}, "5": {"exist": true, "char_ranges": [[59, 196]], "word_ranges": [[10, 32]], "text": "enfant avec otalgie, mais sans fièvre ni signes inflammatoires aigus : ce n'est ni 1 (otite moyenne aiguë récidivante) ni 5 (mastoïdite)."}}} +{"id": 136, "year": 2012, "question_id_specific": 132, "full_question": "Un homme de 60 ans s'est présenté au service des urgences pour une crise de coma. Il était un ancien fumeur depuis 3 ans et n'avait pas d'autres antécédents. Un scanner montre de multiples métastases. Quelle est la cause la plus probable ?", "full_answer": "Plutôt facile, n'est-ce pas ? Le cancer du poumon est la cause la plus fréquente de métastases cérébrales et, dans 20 à 30 % des cas, c'est à la suite de celles-ci que la tumeur primaire est diagnostiquée (comme ici).", "type": "ONCOLOGIE", "options": {"1": "Cancer de la tête et du cou.", "2": "Cancer du poumon.", "3": "Cancer de la vessie.", "4": "Cancer du côlon.", "5": "Cancer du pancréas."}, "correct_option": 2, "explanations": {"1": {"exist": false, "char_ranges": [], "word_ranges": [], "text": ""}, "2": {"exist": true, "char_ranges": [[30, 217]], "word_ranges": [[5, 40]], "text": "Le cancer du poumon est la cause la plus fréquente de métastases cérébrales et, dans 20 à 30 % des cas, c'est à la suite de celles-ci que la tumeur primaire est diagnostiquée (comme ici)."}, "3": {"exist": false, "char_ranges": [], "word_ranges": [], "text": ""}, "4": {"exist": false, "char_ranges": [], "word_ranges": [], "text": ""}, "5": {"exist": false, "char_ranges": [], "word_ranges": [], "text": ""}}} +{"id": 518, "year": 2021, "question_id_specific": 105, "full_question": "Une femme de 24 ans, ayant des antécédents de migraine, sous propranolol et contraceptifs oraux, présente une dyspnée sévère, un enrouement, une éruption cutanée, des nausées et des vomissements 30 minutes après avoir pris du métamizole. Sa tension artérielle est de 90/40 mmHg et sa SatO2 de 90 %. Le traitement initial le plus correct serait d'administrer :", "full_answer": "Il s'agit d'une réaction anaphylactique au métamizole, pour laquelle il faut agir rapidement, faute de quoi le patient risque d'être victime d'un arrêt cardiaque. Il est important d'évaluer la gravité des réactions allergiques, car dans ce cas, un traitement précoce est essentiel pour éviter une mortalité élevée s'il n'est pas appliqué. Le traitement de choix est l'adrénaline IM (ne passer à l'IV que si l'anaphylaxie est réfractaire), qui s'est avérée augmenter le taux de survie et peut être répétée toutes les 5 à 15 minutes si les symptômes ne s'estompent pas. Comme traitement adjuvant, on peut administrer de la dexchlorphéniramine. Dans ce cas particulier, comme le patient a déjà été traité avec du propranolol, cela indique que la réponse correcte est 4, qui inclut l'administration de glucagon, qui est administré parce que les patients prenant des bêta-bloquants peuvent être résistants au traitement à l'adrénaline et développer une hypotension réfractaire et une bradycardie prolongée. La dose chez l'adulte est de 1 à 2 mg, IV ou IM, qui peut être répétée en 5 minutes ou suivie d'une perfusion à 5-15 mcg/min.", "type": "SOINS CRITIQUES ET D'URGENCE", "options": {"1": "Adrénaline.", "2": "Adrénaline et dexchlorphéniramine.", "3": "Adrénaline, dexchlorphéniramine et méthylprednisolone.", "4": "Adrénaline, dexchlorphéniramine et glucagon.", "5": null}, "correct_option": 4, "explanations": {"1": {"exist": false, "char_ranges": [], "word_ranges": [], "text": ""}, "2": {"exist": false, "char_ranges": [], "word_ranges": [], "text": ""}, "3": {"exist": false, "char_ranges": [], "word_ranges": [], "text": ""}, "4": {"exist": true, "char_ranges": [[642, 1001]], "word_ranges": [[100, 153]], "text": "Dans ce cas particulier, comme le patient a déjà été traité avec du propranolol, cela indique que la réponse correcte est 4, qui inclut l'administration de glucagon, qui est administré parce que les patients prenant des bêta-bloquants peuvent être résistants au traitement à l'adrénaline et développer une hypotension réfractaire et une bradycardie prolongée."}, "5": {"exist": false, "char_ranges": [], "word_ranges": [], "text": ""}}} +{"id": 581, "year": 2022, "question_id_specific": 205, "full_question": "Un homme de 61 ans, fumeur et hypertendu, se rend aux urgences pour une perte soudaine de force et des picotements dans la main droite qui durent environ 15 minutes, avec un rétablissement presque complet par la suite. ECG : rythme sinusal à 93 bpm. Par rapport au diagnostic le plus probable, cochez la réponse FAUX :", "full_answer": "Il s'agit d'un AIT (accident ischémique transitoire). L'étude étiologique de base comprend un écho-Doppler des troncs supra-aortiques +/- des artères cérébrales, un échocardiogramme et des tests sanguins ciblés.", "type": "NEUROLOGIE", "options": {"1": "La cause probable est une embolie artério-artérielle due à une plaque carotidienne délogée.", "2": "L'échographie Doppler des troncs supra-aortiques a peu de valeur diagnostique.", "3": "Un scanner crânien est nécessaire pour évaluer l'impact sur le parenchyme cérébral.", "4": "Le traitement chirurgical est indiqué si les examens d'imagerie révèlent une sténose carotidienne >70%.", "5": null}, "correct_option": 2, "explanations": {"1": {"exist": false, "char_ranges": [], "word_ranges": [], "text": ""}, "2": {"exist": false, "char_ranges": [], "word_ranges": [], "text": ""}, "3": {"exist": false, "char_ranges": [], "word_ranges": [], "text": ""}, "4": {"exist": false, "char_ranges": [], "word_ranges": [], "text": ""}, "5": {"exist": false, "char_ranges": [], "word_ranges": [], "text": ""}}} +{"id": 543, "year": 2022, "question_id_specific": 39, "full_question": "Un patient de 35 ans a consulté pour confirmer une suspicion d'allergie à Anisakis simplex. Il avait auparavant présenté un tableau clinique d'anisakiase gastro-intestinale après avoir mangé du poisson. Les tests ont révélé des taux d'IgE spécifiques de 10 KU/l contre Anisakis simplex. Parmi les propositions suivantes, quelle serait la recommandation diététique pour ce patient ?", "full_answer": "Anisakis simplex est un parasite que l'on trouve fréquemment dans les poissons. Pour éviter les réactions à ce parasite, il convient de suivre un régime sans poisson frais ni cru (anchois au vinaigre, poisson fumé, salé, sushi, etc. Le poisson surgelé ou le poisson frais préalablement congelé à -20ºC pendant au moins 5 jours avant d'être cuisiné peut être consommé, à condition qu'il soit cuit à des températures élevées >60ºC (rôti, frit, etc.), en évitant les préparations crues, grillées ou cuites au four à micro-ondes. Il est recommandé de consommer de préférence les queues car les larves du parasite sont généralement proches des viscères de la tête.", "type": "ALLERGOLOGIE", "options": {"1": "Ne pas consommer de poissons, de crustacés, de mollusques ou de céphalopodes.", "2": "Vous pouvez manger du poisson frais cuit sur le gril.", "3": "Vous pouvez manger du poisson surgelé du commerce.", "4": "Vous pouvez manger du poisson frais mariné ou fumé à froid.", "5": null}, "correct_option": 3, "explanations": {"1": {"exist": false, "char_ranges": [], "word_ranges": [], "text": ""}, "2": {"exist": false, "char_ranges": [], "word_ranges": [], "text": ""}, "3": {"exist": false, "char_ranges": [], "word_ranges": [], "text": ""}, "4": {"exist": false, "char_ranges": [], "word_ranges": [], "text": ""}, "5": {"exist": false, "char_ranges": [], "word_ranges": [], "text": ""}}} +{"id": 2, "year": 2011, "question_id_specific": 36, "full_question": "Une cholécystite calcaire aiguë a été diagnostiquée chez un patient de 87 ans ayant des antécédents de bronchite chronique et d'insuffisance cardiaque. Après 4 jours d'hospitalisation et un traitement par régime absolu et sérum thérapeutique à base de pipéracilline-tazobactam, le patient continue à avoir de la fièvre, des douleurs abdominales persistantes et une leucocytose. L'approche la plus appropriée à ce stade serait la suivante :", "full_answer": "Dans cette question, on insiste sur l'âge et la comorbidité du patient, ainsi que sur le temps d'évolution, donc la réponse n'est probablement pas 1. En tout état de cause, l'idéal serait de drainer la vésicule biliaire qui n'est pas remontée, donc la réponse que je donnerais serait 2.", "type": "DIGESTIF", "options": {"1": "Traitement chirurgical urgent.", "2": "Drainage biliaire par cholécystostomie percutanée.", "3": "Remplacer la pipéracilline-tazobactam par le métronidazole-cefotaxime.", "4": "Remplacer la pipéracilline-tazobactam par l'amikacine-clindamycine.", "5": "Ajouter au traitement un aminoglycoside tel que la gentamicine."}, "correct_option": 2, "explanations": {"1": {"exist": true, "char_ranges": [[0, 149]], "word_ranges": [[0, 25]], "text": "Dans cette question, on insiste sur l'âge et la comorbidité du patient, ainsi que sur le temps d'évolution, donc la réponse n'est probablement pas 1."}, "2": {"exist": true, "char_ranges": [[151, 287]], "word_ranges": [[25, 49]], "text": "En tout état de cause, l'idéal serait de drainer la vésicule biliaire qui n'est pas remontée, donc la réponse que je donnerais serait 2."}, "3": {"exist": false, "char_ranges": [], "word_ranges": [], "text": ""}, "4": {"exist": false, "char_ranges": [], "word_ranges": [], "text": ""}, "5": {"exist": false, "char_ranges": [], "word_ranges": [], "text": ""}}} +{"id": 122, "year": 2012, "question_id_specific": 119, "full_question": "Une jeune femme de 17 ans s'est présentée au service des urgences avec des symptômes aigus de forte fièvre, de douleur pharyngée et d'adénopathie cervicale. Une pharyngite aiguë avait déjà été diagnostiquée et traitée à l'amoxicilline. Elle a ensuite présenté une éruption cutanée maculaire généralisée. Les examens de laboratoire ont révélé une légère leucocytose et la présence de leucocytes activés, une légère thrombopénie et une légère augmentation des transaminases. Quel serait le diagnostic le plus probable de ce tableau clinique ?", "full_answer": "La bonne réponse est la première. Il s'agit d'un tableau typique de la mononucléose infectieuse. Bien qu'un tableau similaire puisse se produire en cas de toxoplasmose aiguë, moins de 1 % des infections aiguës à toxoplasme se présentent sous la forme d'une mononucléose. La mononucléose infectieuse d'Epstein-Barr se caractérise par l'apparition d'une éruption maculaire après un traitement à l'amoxicilline.", "type": "MICROBIOLOGIE", "options": {"1": "Il s'agit d'un tableau typique de la mononucléose infectieuse.", "2": "Infection par le virus de la varicelle et du zona.", "3": "Toxoplasmose aiguë.", "4": "Maladie de Lyme.", "5": "Infection par le virus de l'herpès 8."}, "correct_option": 1, "explanations": {"1": {"exist": true, "char_ranges": [[271, 408]], "word_ranges": [[43, 59]], "text": "La mononucléose infectieuse d'Epstein-Barr se caractérise par l'apparition d'une éruption maculaire après un traitement à l'amoxicilline."}, "2": {"exist": false, "char_ranges": [], "word_ranges": [], "text": ""}, "3": {"exist": true, "char_ranges": [[97, 270]], "word_ranges": [[15, 43]], "text": "Bien qu'un tableau similaire puisse se produire en cas de toxoplasmose aiguë, moins de 1 % des infections aiguës à toxoplasme se présentent sous la forme d'une mononucléose."}, "4": {"exist": false, "char_ranges": [], "word_ranges": [], "text": ""}, "5": {"exist": false, "char_ranges": [], "word_ranges": [], "text": ""}}} +{"id": 36, "year": 2011, "question_id_specific": 130, "full_question": "Une femme de 85 ans consulte pour fatigue et faiblesse, surtout le matin. Elle est parfois instable en marchant et doit s'asseoir pour retrouver son équilibre. À deux reprises, elle a dû s'asseoir pour éviter de tomber, mais elle nie tout symptôme de vertige. Elle souffre d'hypertension, d'incontinence urinaire et d'arthrose. Son traitement est composé d'hydrochlorothiazide (25 mg/j), d'oxybutynine (10 mg/j), de lisinopril (10 mg/j), de calcium (1500 mg/j) et de paracétamol (3 g/j). A l'examen, tension artérielle 115/70 mm Hg, pouls 80 bpm. Ses mouvements sont lents. Elle présente un tremblement modéré des mains. Elle est capable de se lever de la chaise lentement mais sans avoir besoin de s'appuyer sur ses bras. Elle marche légèrement penchée en avant, avec un faible balancement des bras. Elle se tourne lentement mais sans perdre l'équilibre. N'est pas capable de se tenir sur un pied. Parmi les actions possibles suivantes, laquelle effectueriez-vous en premier ?", "full_answer": "La réponse correcte est de commencer le traitement par la L-dopa, car le tableau clinique décrit est celui d'un parkinsonisme, dont le diagnostic est exclusivement clinique. Les autres réponses peuvent être faites pour le diagnostic différentiel, mais la réalisation d'examens complémentaires ne justifie pas de retarder le début du traitement.", "type": "NEUROLOGIE ET NEUROCHIRURGIE", "options": {"1": "Évaluer l'acuité visuelle.", "2": "Réaliser une IRM.", "3": "Étude avec table basculante.", "4": "Mesurer la tension artérielle en position couchée et debout.", "5": "Essai thérapeutique avec la L-dopa."}, "correct_option": 5, "explanations": {"1": {"exist": true, "char_ranges": [[174, 344]], "word_ranges": [[26, 50]], "text": "Les autres réponses peuvent être faites pour le diagnostic différentiel, mais la réalisation d'examens complémentaires ne justifie pas de retarder le début du traitement."}, "2": {"exist": true, "char_ranges": [[174, 344]], "word_ranges": [[26, 50]], "text": "Les autres réponses peuvent être faites pour le diagnostic différentiel, mais la réalisation d'examens complémentaires ne justifie pas de retarder le début du traitement."}, "3": {"exist": true, "char_ranges": [[174, 344]], "word_ranges": [[26, 50]], "text": "Les autres réponses peuvent être faites pour le diagnostic différentiel, mais la réalisation d'examens complémentaires ne justifie pas de retarder le début du traitement."}, "4": {"exist": true, "char_ranges": [[174, 344]], "word_ranges": [[26, 50]], "text": "Les autres réponses peuvent être faites pour le diagnostic différentiel, mais la réalisation d'examens complémentaires ne justifie pas de retarder le début du traitement."}, "5": {"exist": true, "char_ranges": [[0, 173]], "word_ranges": [[0, 26]], "text": "La réponse correcte est de commencer le traitement par la L-dopa, car le tableau clinique décrit est celui d'un parkinsonisme, dont le diagnostic est exclusivement clinique."}}} +{"id": 338, "year": 2016, "question_id_specific": 31, "full_question": "Une femme de 20 ans présentant une tumeur ovarienne kystique solide de 15 cm détectée par échographie après avoir présenté des symptômes abdominaux non spécifiques. Lors de l'étude histopathologique de l'échantillon correspondant, des dents, des poils, des zones d'épithélium intestinal, des zones d'épithélium malpighien (15 %) et d'épithélium bronchique, ainsi que des éléments neuroectodermiques et embryonnaires ont été trouvés dans plusieurs des préparations histologiques. En ce qui concerne ce cas, veuillez indiquer le bon diagnostic :", "full_answer": "Tératome kystique mature (fréquent dans l'ovaire de la femme adulte). Masse avec un grand kyste occupant la majeure partie de la masse ; dans sa cavité, on trouve du matériel sébacé et des poils. La paroi, généralement épaisse de quelques millimètres, a la structure de la peau, avec sa surface épidermique tournée vers la cavité. En raison de ce développement prédominant des structures cutanées, on parle souvent de kyste dermoïde. Les tissus qui composent la tumeur sont bien différenciés (de type mature ou adulte) ; outre les structures cutanées, de nombreux autres tissus peuvent être observés, notamment dans un épaississement ou un éperon éminent dans la cavité, dans lequel on trouve souvent des dents, du cartilage et de l'os.", "type": "GYNÉCOLOGIE ET OBSTÉTRIQUE", "options": {"1": "Tératocarcinome.", "2": "Tératome immature.", "3": "Tératome kystique mature.", "4": "Dysgerminome.", "5": null}, "correct_option": 3, "explanations": {"1": {"exist": false, "char_ranges": [], "word_ranges": [], "text": ""}, "2": {"exist": false, "char_ranges": [], "word_ranges": [], "text": ""}, "3": {"exist": true, "char_ranges": [[434, 736]], "word_ranges": [[70, 119]], "text": "Les tissus qui composent la tumeur sont bien différenciés (de type mature ou adulte) ; outre les structures cutanées, de nombreux autres tissus peuvent être observés, notamment dans un épaississement ou un éperon éminent dans la cavité, dans lequel on trouve souvent des dents, du cartilage et de l'os."}, "4": {"exist": false, "char_ranges": [], "word_ranges": [], "text": ""}, "5": {"exist": false, "char_ranges": [], "word_ranges": [], "text": ""}}} +{"id": 432, "year": 2018, "question_id_specific": 117, "full_question": "Un homme de 85 ans porteur d'un cathéter veineux périphérique qui, une semaine après avoir été hospitalisé pour un accident vasculaire cérébral, a commencé à présenter des frissons et de la fièvre. Des hémocultures ont été prélevées et la microbiologie a indiqué que des cocci gram-positifs se développaient en grappes. Dans l'attente de l'antibiogramme, quel est le traitement antibiotique le plus approprié ?", "full_answer": "La cause la plus fréquente d'infection de cathéter est une bactérie qui colonise la peau. La coloration de Gram nous indique qu'il s'agit d'un staphylocoque, la plupart des espèces de ce genre acquises à l'hôpital sont résistantes à la méthicilline, elles sont donc considérées comme résistantes à la cloxacilline, à la céfazoline et il n'y a pas de données cliniques sur l'efficacité du linézolide pour le traitement de l'infection systémique liée au cathéter. Ceci, ajouté à l'expérience de l'utilisation de la vancomycine dans ce contexte, est considéré comme le choix empirique de la vancomycine.", "type": "MALADIES INFECTIEUSES ET MICROBIOLOGIE", "options": {"1": "Céfazoline.", "2": "Cloxacilline.", "3": "Vancomycine.", "4": "Linezolid.", "5": null}, "correct_option": 3, "explanations": {"1": {"exist": true, "char_ranges": [[90, 330]], "word_ranges": [[15, 52]], "text": "La coloration de Gram nous indique qu'il s'agit d'un staphylocoque, la plupart des espèces de ce genre acquises à l'hôpital sont résistantes à la méthicilline, elles sont donc considérées comme résistantes à la cloxacilline, à la céfazoline"}, "2": {"exist": true, "char_ranges": [[90, 330]], "word_ranges": [[15, 52]], "text": "La coloration de Gram nous indique qu'il s'agit d'un staphylocoque, la plupart des espèces de ce genre acquises à l'hôpital sont résistantes à la méthicilline, elles sont donc considérées comme résistantes à la cloxacilline, à la céfazoline"}, "3": {"exist": true, "char_ranges": [[477, 600]], "word_ranges": [[76, 94]], "text": "l'expérience de l'utilisation de la vancomycine dans ce contexte, est considéré comme le choix empirique de la vancomycine."}, "4": {"exist": true, "char_ranges": [[334, 461]], "word_ranges": [[53, 73]], "text": "il n'y a pas de données cliniques sur l'efficacité du linézolide pour le traitement de l'infection systémique liée au cathéter."}, "5": {"exist": false, "char_ranges": [], "word_ranges": [], "text": ""}}} +{"id": 513, "year": 2021, "question_id_specific": 38, "full_question": "Un patient de 72 ans s'est présenté aux urgences pour un angio-œdème lingual sévère. Il ne présentait pas d'urticaire associée et n'avait pas d'antécédents d'allergie médicamenteuse ou alimentaire. Ses antécédents personnels comprenaient un diabète sucré de type 2, une dyslipidémie, une hypertension, une hypothyroïdie et la maladie de Parkinson. Il suit un traitement régulier à la metformine, à la simvastatine, à l'énalapril, à la thyroxine et à la lévodopa. Parmi les médicaments suivants, lequel est le plus susceptible d'être à l'origine du tableau clinique décrit ?", "full_answer": "Parmi les médicaments proposés, les options les moins probables seraient la 3, la metformine provoquant très rarement un angio-œdème lingual (en mars 2020, un seul cas avait été rapporté dans le monde) et la 4, l'angio-œdème étant classé parmi les effets indésirables de la lévodopa parmi les très rares (fréquence inférieure à 0,1 %). Parmi les deux options restantes, les médicaments les plus fréquemment et classiquement associés à l'angio-œdème lingual sont les inhibiteurs de l'ECA (fréquence autour de 0,2%), la simvastatine étant également associée à moins de 0,1%.", "type": "SOINS CRITIQUES ET D'URGENCE", "options": {"1": "Enalapril.", "2": "Simvastatine.", "3": "Metformine ou lévodopa également.", "4": "Levodopa.", "5": null}, "correct_option": 1, "explanations": {"1": {"exist": true, "char_ranges": [[370, 514]], "word_ranges": [[59, 79]], "text": "les médicaments les plus fréquemment et classiquement associés à l'angio-œdème lingual sont les inhibiteurs de l'ECA (fréquence autour de 0,2%),"}, "2": {"exist": true, "char_ranges": [[515, 572]], "word_ranges": [[79, 88]], "text": "la simvastatine étant également associée à moins de 0,1%."}, "3": {"exist": true, "char_ranges": [[79, 201]], "word_ranges": [[12, 32]], "text": "la metformine provoquant très rarement un angio-œdème lingual (en mars 2020, un seul cas avait été rapporté dans le monde)"}, "4": {"exist": true, "char_ranges": [[211, 335]], "word_ranges": [[35, 54]], "text": "l'angio-œdème étant classé parmi les effets indésirables de la lévodopa parmi les très rares (fréquence inférieure à 0,1 %)."}, "5": {"exist": false, "char_ranges": [], "word_ranges": [], "text": ""}}} +{"id": 580, "year": 2022, "question_id_specific": 198, "full_question": "Une femme de 35 ans a été adressée aux urgences pour un traumatisme crânien consécutif à une chute en scooter, sans perte de connaissance ni amnésie. L'examen révèle un traumatisme crânien fermé avec commotion cérébrale et de légères céphalées sans nausées ni vomissements, l'examen neurologique est normal et l'indice de Glasgow est de 15. L'examen neurologique est normal et l'indice de Glasgow est de 15. Dans cette situation, quel examen d'imagerie est indiqué en premier lieu ?", "full_answer": "Après un traumatisme crânien chez un patient qui n'est pas anticoagulé ou sous antiplaquettaire, qui n'a pas de facteurs de risque connus d'hémorragie et qui ne présente pas de données alarmantes à l'anamnèse ou à l'examen physique, il n'est pas nécessaire d'effectuer un test d'imagerie crânienne.", "type": "NEUROLOGIE", "options": {"1": "Tomodensitométrie crânienne sans contraste.", "2": "Tomodensitométrie crânienne avec contraste.", "3": "Aucune preuve par imagerie.", "4": "Radiographie simple du crâne.", "5": null}, "correct_option": 3, "explanations": {"1": {"exist": true, "char_ranges": [[0, 298]], "word_ranges": [[0, 46]], "text": "Après un traumatisme crânien chez un patient qui n'est pas anticoagulé ou sous antiplaquettaire, qui n'a pas de facteurs de risque connus d'hémorragie et qui ne présente pas de données alarmantes à l'anamnèse ou à l'examen physique, il n'est pas nécessaire d'effectuer un test d'imagerie crânienne."}, "2": {"exist": true, "char_ranges": [[0, 298]], "word_ranges": [[0, 46]], "text": "Après un traumatisme crânien chez un patient qui n'est pas anticoagulé ou sous antiplaquettaire, qui n'a pas de facteurs de risque connus d'hémorragie et qui ne présente pas de données alarmantes à l'anamnèse ou à l'examen physique, il n'est pas nécessaire d'effectuer un test d'imagerie crânienne."}, "3": {"exist": true, "char_ranges": [[0, 298]], "word_ranges": [[0, 46]], "text": "Après un traumatisme crânien chez un patient qui n'est pas anticoagulé ou sous antiplaquettaire, qui n'a pas de facteurs de risque connus d'hémorragie et qui ne présente pas de données alarmantes à l'anamnèse ou à l'examen physique, il n'est pas nécessaire d'effectuer un test d'imagerie crânienne."}, "4": {"exist": true, "char_ranges": [[0, 298]], "word_ranges": [[0, 46]], "text": "Après un traumatisme crânien chez un patient qui n'est pas anticoagulé ou sous antiplaquettaire, qui n'a pas de facteurs de risque connus d'hémorragie et qui ne présente pas de données alarmantes à l'anamnèse ou à l'examen physique, il n'est pas nécessaire d'effectuer un test d'imagerie crânienne."}, "5": {"exist": false, "char_ranges": [], "word_ranges": [], "text": ""}}} +{"id": 248, "year": 2014, "question_id_specific": 118, "full_question": "Un homme de 34 ans consulte pour fièvre et malaise général. Ses antécédents médicaux comprennent des relations homosexuelles depuis quatre mois avec un nouveau partenaire. Deux mois avant la consultation actuelle, il avait présent�� une lésion ulcéreuse indolore sur le gland du pénis, ainsi qu'une lymphadénopathie inguinale bilatérale, qui s'est résorbée d'elle-même. Des études sérologiques ont été demandées et ont donné les résultats suivants : VIH négatif, RPR 1/320, HAART 1/128. Quel traitement proposeriez-vous à ce patient ?", "full_answer": "Réponse directe à la question. Il s'agit d'une syphilis de stade secondaire (à la fois en termes d'antécédents et de sérologie) et le traitement consiste donc en une dose unique de pénicilline benzathine 2,4 MU.", "type": "LES MALADIES INFECTIEUSES", "options": {"1": "Aucun.", "2": "Pénicilline G intraveineuse, 24 MU par jour pendant 14 jours.", "3": "Pénicilline benzathine 2,4 MU par voie intramusculaire, 3 doses au cours de trois semaines consécutives.", "4": "Ceftriaxone 2 grammes par voie intramusculaire en une seule dose.", "5": "Pénicilline benzathine 2,4 MU par voie intramusculaire en une seule dose."}, "correct_option": 5, "explanations": {"1": {"exist": false, "char_ranges": [], "word_ranges": [], "text": ""}, "2": {"exist": false, "char_ranges": [], "word_ranges": [], "text": ""}, "3": {"exist": false, "char_ranges": [], "word_ranges": [], "text": ""}, "4": {"exist": false, "char_ranges": [], "word_ranges": [], "text": ""}, "5": {"exist": true, "char_ranges": [[31, 211]], "word_ranges": [[5, 35]], "text": "Il s'agit d'une syphilis de stade secondaire (à la fois en termes d'antécédents et de sérologie) et le traitement consiste donc en une dose unique de pénicilline benzathine 2,4 MU."}}} +{"id": 222, "year": 2014, "question_id_specific": 197, "full_question": "Un homosexuel séropositif (+) de 30 ans est bénévole dans un centre pour malades du SIDA. D'après ses antécédents médicaux, il a reçu l'anatoxine diphtérique (Td) il y a 6 ans, le vaccin ROR dans l'enfance et l'adolescence, et l'hépatite B il y a 3 ans. Il est actuellement asymptomatique et son taux de CD4 est supérieur à 200 cls/microlitre. Quels vaccins devrions-nous recommander ?", "full_answer": "Les recommandations de vaccination pour les adultes séropositifs comprennent l'hépatite B, la grippe, le ROR, le pneumocoque, le Td et le Tdap, et pour certains adultes, l'hépatite A, ou les combinaisons A et B, la méningite bactérienne, le HPV et le méningocoque (Source [1]). Si l'on tient compte du fait que la personne a reçu certaines vaccinations, que son taux de CD4 est supérieur à 100 (s'il est inférieur, il y a une contre-indication) et qu'elle travaille dans un centre d'aide aux malades du sida (assimilé aux \"travailleurs de la santé\"), les réponses 2, 4 et 5 sont erronées. Les quatre vaccins de l'option 1 peuvent être administrés. Un peu plus de détails : Vaccin contre le tétanos et la diphtérie : toutes les personnes devraient être vaccinées contre ces deux maladies. Vaccin contre l'hépatite B : les virus de l'hépatite B et du VIH partagent les mêmes voies de transmission, de sorte qu'il est fréquent que les patients infectés par le VIH soient également infectés par le virus de l'hépatite B. Il est donc important de savoir si l'on peut administrer le vaccin au patient. Il est donc important de savoir si la personne infectée par le VIH présente des marqueurs d'infection par le virus de l'hépatite B et de la vacciner si ce n'est pas le cas. Vaccin contre l'hépatite A : L'hépatite A chez une personne atteinte d'hépatite B, d'hépatite C ou d'une autre maladie du foie peut être très grave. Ces infections étant plus fréquentes chez les patients infectés par le VIH, la vaccination contre l'hépatite A est recommandée. Vaccination contre la grippe : la grippe chez un patient ne présentant pas de réponse immunitaire est plus grave et présente un risque plus élevé de complications, c'est pourquoi une vaccination annuelle contre cette maladie est indiquée. Si l'état immunitaire du patient est gravement altéré, la réponse à la vaccination est diminuée. Dans ce cas, il est essentiel de vacciner toutes les personnes vivant avec la personne infectée par le VIH pour éviter qu'elles ne transmettent la maladie. Vaccin antipneumococcique : les infections à pneumocoques (en particulier la pneumonie) sont jusqu'à 10 fois plus fréquentes chez les personnes infectées par le VIH que chez les adultes non infectés par le VIH, c'est pourquoi la vaccination est recommandée, bien que chez les patients dont la numération est inférieure à 200/mm3, la réponse protectrice ne soit généralement pas suffisante. Ils doivent recevoir une deuxième dose à l'âge de 3 à 5 ans. Les enfants infectés par le VIH doivent être vaccinés avec le vaccin conjugué antipneumococcique, avec le nombre de doses correspondant à leur âge. Vaccin contre l'Haemophilus influenzae de type b : cette bactérie est à l'origine de pneumonies et de méningites, en particulier chez les enfants. Bien qu'elle ne représente qu'une petite partie des causes d'infection chez les adultes infectés par le VIH, la vaccination est recommandée. Vaccination contre la rougeole, la rubéole et les oreillons (ROR) : ces trois maladies peuvent s'aggraver chez les patients infectés par le VIH, en particulier la rougeole, et il convient donc de les vacciner tous, à condition que l'immunodépression ne soit pas grave (numération inférieure à 200/mm3).", "type": "EPIDEMIOLOGIE", "options": {"1": "Grippe saisonnière, pneumocoque, méningite tétravalente et hépatite A.", "2": "Grippe saisonnière, Td, pneumocoque et méningite tétravalente.", "3": "Méningite tétravalente, pneumocoque et grippe saisonnière.", "4": "Td, méningite tétravalente, pneumococcique.", "5": "Triple virus, grippe saisonnière, pneumocoque."}, "correct_option": 1, "explanations": {"1": {"exist": true, "char_ranges": [[589, 2427]], "word_ranges": [[99, 400]], "text": "Les quatre vaccins de l'option 1 peuvent être administrés. Un peu plus de détails : Vaccin contre le tétanos et la diphtérie : toutes les personnes devraient être vaccinées contre ces deux maladies. Vaccin contre l'hépatite B : les virus de l'hépatite B et du VIH partagent les mêmes voies de transmission, de sorte qu'il est fréquent que les patients infectés par le VIH soient également infectés par le virus de l'hépatite B. Il est donc important de savoir si l'on peut administrer le vaccin au patient. Il est donc important de savoir si la personne infectée par le VIH présente des marqueurs d'infection par le virus de l'hépatite B et de la vacciner si ce n'est pas le cas. Vaccin contre l'hépatite A : L'hépatite A chez une personne atteinte d'hépatite B, d'hépatite C ou d'une autre maladie du foie peut être très grave. Ces infections étant plus fréquentes chez les patients infectés par le VIH, la vaccination contre l'hépatite A est recommandée. Vaccination contre la grippe : la grippe chez un patient ne présentant pas de réponse immunitaire est plus grave et présente un risque plus élevé de complications, c'est pourquoi une vaccination annuelle contre cette maladie est indiquée. Si l'état immunitaire du patient est gravement altéré, la réponse à la vaccination est diminuée. Dans ce cas, il est essentiel de vacciner toutes les personnes vivant avec la personne infectée par le VIH pour éviter qu'elles ne transmettent la maladie. Vaccin antipneumococcique : les infections à pneumocoques (en particulier la pneumonie) sont jusqu'à 10 fois plus fréquentes chez les personnes infectées par le VIH que chez les adultes non infectés par le VIH, c'est pourquoi la vaccination est recommandée, bien que chez les patients dont la numération est inférieure à 200/mm3, la réponse protectrice ne soit généralement pas suffisante."}, "2": {"exist": true, "char_ranges": [[278, 588]], "word_ranges": [[44, 99]], "text": "Si l'on tient compte du fait que la personne a reçu certaines vaccinations, que son taux de CD4 est supérieur à 100 (s'il est inférieur, il y a une contre-indication) et qu'elle travaille dans un centre d'aide aux malades du sida (assimilé aux \"travailleurs de la santé\"), les réponses 2, 4 et 5 sont erronées."}, "3": {"exist": false, "char_ranges": [], "word_ranges": [], "text": ""}, "4": {"exist": true, "char_ranges": [[278, 588]], "word_ranges": [[44, 99]], "text": "Si l'on tient compte du fait que la personne a reçu certaines vaccinations, que son taux de CD4 est supérieur à 100 (s'il est inférieur, il y a une contre-indication) et qu'elle travaille dans un centre d'aide aux malades du sida (assimilé aux \"travailleurs de la santé\"), les réponses 2, 4 et 5 sont erronées."}, "5": {"exist": true, "char_ranges": [[278, 588]], "word_ranges": [[44, 99]], "text": "Si l'on tient compte du fait que la personne a reçu certaines vaccinations, que son taux de CD4 est supérieur à 100 (s'il est inférieur, il y a une contre-indication) et qu'elle travaille dans un centre d'aide aux malades du sida (assimilé aux \"travailleurs de la santé\"), les réponses 2, 4 et 5 sont erronées."}}} +{"id": 244, "year": 2014, "question_id_specific": 114, "full_question": "Un garçon de 16 ans a présenté une amygdalite pustuleuse, une fièvre atteignant 38,5 °C, une lymphadénopathie cervicale douloureuse, une éruption maculaire non prurigineuse sur le thorax et une légère hépatosplénomégalie, depuis 4 à 5 jours. Le test de Paul-Bunnell et les Ig M pour le virus d'Epstein-Barr étaient positifs. Au cours de son admission, il a présenté une fièvre continue atteignant 40 ºC, une pancytopénie, une hépatite ictérique et une coagulopathie d'intensité progressive. Une semaine après son admission, il a été transféré à l'unité de soins intensifs en raison d'une confusion et d'une insuffisance respiratoire. Les hémocultures et les cultures d'urine sont négatives, le LCR est normal et la radiographie du thorax ne montre pas d'infiltrats. La procalcitonine est normale, mais la CRP et la ferritine sont très élevées. Parmi les affirmations suivantes, quelle serait l'approche diagnostique et thérapeutique la plus correcte ?", "full_answer": "Question difficile dont la réponse correcte peut donner lieu à une discussion. Le patient est un jeune homme de 16 ans qui est admis pour un cas assez clair de mononucléose due à une infection par le virus EBV qui se complique au cours de l'admission. Les données dont nous disposons sont les suivantes : - Pancytopénie. - Hépatopathie. - Coagulopathie. - Absence de foyer infectieux : hémocultures négatives, cultures urinaires négatives, radiographie normale, LCR normal. - CRP et ferritine élevées / procalcitonine normale. Les complications possibles qui auraient pu survenir (et qui sont prises en compte dans les réponses) sont une septicémie ou une mauvaise évolution de la mononucléose. Les données cliniques pourraient être compatibles avec un processus septique, mais les principaux foyers infectieux ont été raisonnablement écartés et l'un des marqueurs les plus sensibles et spécifiques de la bactériémie (la procalcitonine) se situe dans la plage normale, ce qui devrait attirer notre attention sur ce point. D'autre part, une complication de la mononucléose EBV dont souffre notre patient doit être évaluée. Parmi les complications possibles, citons l'anémie hémolytique, la méningocéphalite (LCR normal) ou le syndrome de Guillain-Barré. Le syndrome de Guillain-Barré, qui n'est pas très cohérent avec le tableau clinique qui nous a été présenté. Une autre complication possible, d'une gravité exceptionnelle, est le syndrome hémophagocytaire ; il s'agit d'une entité rare qui survient chez certains individus prédisposés et qui se caractérise par une activation et une prolifération incontrôlées des histiocytes et des lymphocytes T, entraînant un état d'hypercytokinémie. Les critères de diagnostic, révisés en 2004, comprennent une fièvre élevée persistante, une hépatosplénomégalie, des cytopénies, une hypertriglycéridémie, une hyperferritinémie et une hypofibrinogénémie. Une hyperferritinémie >3000 dans un contexte clinique évocateur nécessite un traitement du patient (principalement avec des immunosuppresseurs et des corticostéroïdes) car la mortalité est très élevée et un traitement précoce est essentiel. Le diagnostic définitif est posé par l'étude microscopique d'une biopsie de l'OM. Bien que cette question ne quantifie pas la valeur de la ferritinémie, il s'agit d'un fait remarquable et étant donné que le tableau clinique est évocateur et que les critères diagnostiques sont remplis, il me semble que la réponse 5 serait la plus appropriée.", "type": "LES MALADIES INFECTIEUSES", "options": {"1": "Il souffre d'une septicémie bactérienne d'origine indéterminée et doit recevoir de la ceftriaxone et des soins de soutien.", "2": "Il souffre d'une septicémie bactérienne d'origine indéterminée et doit recevoir de la vancomycine, de la ceftacidime et des soins de soutien.", "3": "Il s'agit d'une mononucléose infectieuse sévère et des glucocorticoïdes doivent être administrés.", "4": "Il s'agit d'une mononucléose infectieuse grave et un traitement à l'acyclovir doit être instauré.", "5": "Je procéderais à une biopsie/aspiration de la moelle osseuse et, si l'hémophagocytose est confirmée, je mettrais en place un traitement par immunosuppresseurs."}, "correct_option": 5, "explanations": {"1": {"exist": true, "char_ranges": [[695, 1021]], "word_ranges": [[110, 158]], "text": "Les données cliniques pourraient être compatibles avec un processus septique, mais les principaux foyers infectieux ont été raisonnablement écartés et l'un des marqueurs les plus sensibles et spécifiques de la bactériémie (la procalcitonine) se situe dans la plage normale, ce qui devrait attirer notre attention sur ce point."}, "2": {"exist": true, "char_ranges": [[695, 1021]], "word_ranges": [[110, 158]], "text": "Les données cliniques pourraient être compatibles avec un processus septique, mais les principaux foyers infectieux ont été raisonnablement écartés et l'un des marqueurs les plus sensibles et spécifiques de la bactériémie (la procalcitonine) se situe dans la plage normale, ce qui devrait attirer notre attention sur ce point."}, "3": {"exist": true, "char_ranges": [[1022, 1361]], "word_ranges": [[158, 207]], "text": "D'autre part, une complication de la mononucléose EBV dont souffre notre patient doit être évaluée. Parmi les complications possibles, citons l'anémie hémolytique, la méningocéphalite (LCR normal) ou le syndrome de Guillain-Barré. Le syndrome de Guillain-Barré, qui n'est pas très cohérent avec le tableau clinique qui nous a été présenté."}, "4": {"exist": true, "char_ranges": [[1022, 1361]], "word_ranges": [[158, 207]], "text": "D'autre part, une complication de la mononucléose EBV dont souffre notre patient doit être évaluée. Parmi les complications possibles, citons l'anémie hémolytique, la méningocéphalite (LCR normal) ou le syndrome de Guillain-Barré. Le syndrome de Guillain-Barré, qui n'est pas très cohérent avec le tableau clinique qui nous a été présenté."}, "5": {"exist": true, "char_ranges": [[1894, 2216]], "word_ranges": [[274, 318]], "text": "Une hyperferritinémie >3000 dans un contexte clinique évocateur nécessite un traitement du patient (principalement avec des immunosuppresseurs et des corticostéroïdes) car la mortalité est très élevée et un traitement précoce est essentiel. Le diagnostic définitif est posé par l'étude microscopique d'une biopsie de l'OM."}}} +{"id": 420, "year": 2018, "question_id_specific": 78, "full_question": "Un homme de 52 ans a été adressé au service gastro-intestinal pour hématochézie, ténesme et diminution du diamètre des selles. Une série d'examens a été réalisée et un diagnostic d'adénocarcinome du sigma sans métastase à distance a été posé. Le patient a subi une intervention chirurgicale et a été adressé au service d'oncologie médicale pour évaluer un traitement complémentaire par chimiothérapie. Parmi les facteurs suivants, lequel est associé à un mauvais pronostic après une résection chirurgicale et doit être pris en compte lors de la planification d'un traitement par chimiothérapie ?", "full_answer": "Ils ne posent pas de questions sur les facteurs qui affectent le pronostic du cancer et impliquent donc un changement dans la stratégie QT. Cette question est simple. L'option 4 décrit un stade T4, soit en raison d'une invasion des organes adjacents, soit en raison d'une perforation. Les tumeurs de stade T4 ont un risque de récidive beaucoup plus élevé. En fait, des études en cours proposent une HIPEC prophylactique pour les tumeurs de stade T4 et une chirurgie de contrôle + HIPEC pour les tumeurs du côlon perforées.", "type": "CHIRURGIE GÉNÉRALE", "options": {"1": "La présence d'une anémie au moment du diagnostic.", "2": "L'existence d'antécédents familiaux de cancer colorectal.", "3": "La taille de la lésion primaire et la différenciation histologique.", "4": "Perforation ou adhésion de la tumeur aux organes adjacents.", "5": null}, "correct_option": 4, "explanations": {"1": {"exist": false, "char_ranges": [], "word_ranges": [], "text": ""}, "2": {"exist": false, "char_ranges": [], "word_ranges": [], "text": ""}, "3": {"exist": false, "char_ranges": [], "word_ranges": [], "text": ""}, "4": {"exist": true, "char_ranges": [[167, 522]], "word_ranges": [[28, 89]], "text": "L'option 4 décrit un stade T4, soit en raison d'une invasion des organes adjacents, soit en raison d'une perforation. Les tumeurs de stade T4 ont un risque de récidive beaucoup plus élevé. En fait, des études en cours proposent une HIPEC prophylactique pour les tumeurs de stade T4 et une chirurgie de contrôle + HIPEC pour les tumeurs du côlon perforées."}, "5": {"exist": false, "char_ranges": [], "word_ranges": [], "text": ""}}} +{"id": 321, "year": 2016, "question_id_specific": 144, "full_question": "Un homme de 50 ans, chez qui on a diagnostiqué une polyangéite avec granulomatose il y a 10 ans et qui n'a pas été traité depuis 5 ans, se présente à un contrôle programmé et se trouve asymptomatique. La radiographie du thorax, les analyses de sang et le sédiment urinaire sont normaux, à l'exception d'anticorps anti-neutrophiles cytoplasmiques (ANCA) positifs au titre 1/320, avec une spécificité anti-protéinase 3, qui étaient négatifs auparavant. Quelle est l'approche thérapeutique la plus conseillée ?", "full_answer": "Dans la vascularite ANCA, quelle qu'elle soit, il a été démontré que ces anticorps sont liés à l'activité de la maladie, mais à aucun moment il n'est obligatoire de modifier le traitement en raison de leur taux. Dans ce cas, si le patient est asymptomatique et qu'il existe des preuves objectives de l'absence d'activité, une surveillance étroite doit être effectuée sans reprendre ou modifier le traitement.", "type": "RHEUMATOLOGIE", "options": {"1": "Instaurer un traitement aux corticostéroïdes.", "2": "Initier le traitement par cyclophosphamide.", "3": "Initier le traitement par mycophénolate mofétil.", "4": "Attente vigilante.", "5": null}, "correct_option": 4, "explanations": {"1": {"exist": true, "char_ranges": [[0, 211]], "word_ranges": [[0, 37]], "text": "Dans la vascularite ANCA, quelle qu'elle soit, il a été démontré que ces anticorps sont liés à l'activité de la maladie, mais à aucun moment il n'est obligatoire de modifier le traitement en raison de leur taux."}, "2": {"exist": true, "char_ranges": [[0, 211]], "word_ranges": [[0, 37]], "text": "Dans la vascularite ANCA, quelle qu'elle soit, il a été démontré que ces anticorps sont liés à l'activité de la maladie, mais à aucun moment il n'est obligatoire de modifier le traitement en raison de leur taux."}, "3": {"exist": true, "char_ranges": [[0, 211]], "word_ranges": [[0, 37]], "text": "Dans la vascularite ANCA, quelle qu'elle soit, il a été démontré que ces anticorps sont liés à l'activité de la maladie, mais à aucun moment il n'est obligatoire de modifier le traitement en raison de leur taux."}, "4": {"exist": true, "char_ranges": [[225, 408]], "word_ranges": [[40, 66]], "text": "si le patient est asymptomatique et qu'il existe des preuves objectives de l'absence d'activité, une surveillance étroite doit être effectuée sans reprendre ou modifier le traitement."}, "5": {"exist": false, "char_ranges": [], "word_ranges": [], "text": ""}}} +{"id": 516, "year": 2021, "question_id_specific": 103, "full_question": "Un patient de 56 ans est trouvé dans le coma à son domicile. Il a des antécédents d'hypertension artérielle et de diabète sucré. Il n'a pas d'habitudes toxiques ni d'autre maladie chronique. Il est traité par irbésartan et empaglifozine. Il ne présente aucun signe de malnutrition. Il a une pression artérielle de 110/60 mmHg, une fréquence cardiaque de 110 bpm, une SatO2 de 90%, une glycémie capillaire de 120 mg/dl et une fréquence respiratoire de 7 rpm. Quelle approche thérapeutique initiale vous semble la plus appropriée ?", "full_answer": "Nous avons trouvé un patient souffrant d'hypoventilation. La glycémie est correcte (nous excluons le coma hyperosmolaire et l'hypoglycémie, options 2 et 4), donc, bien qu'il n'y ait pas d'antécédents de consommation de toxiques, nous devons nous tourner vers les plus fréquents et que nous pouvons antagoniser avec des antidotes : les benzodiazépines (flumazénil) et les opiacés (naloxone). Si nous suspectons une intoxication à l'éthylène glycol, nous administrerons de la thiamine, mais il n'y a pas de données qui nous permettent de suspecter cette intoxication.", "type": "SOINS CRITIQUES ET D'URGENCE", "options": {"1": "Administrer de la naloxone, du flumazénil et de la thiamine.", "2": "Administrer de la naloxone, du flumazénil et du glucose hypertonique.", "3": "Administrer de la naloxone et du flumazénil.", "4": "Administrer de la thiamine et du glucose hypertonique.", "5": null}, "correct_option": 3, "explanations": {"1": {"exist": false, "char_ranges": [], "word_ranges": [], "text": ""}, "2": {"exist": false, "char_ranges": [], "word_ranges": [], "text": ""}, "3": {"exist": true, "char_ranges": [[163, 390]], "word_ranges": [[23, 57]], "text": "bien qu'il n'y ait pas d'antécédents de consommation de toxiques, nous devons nous tourner vers les plus fréquents et que nous pouvons antagoniser avec des antidotes : les benzodiazépines (flumazénil) et les opiacés (naloxone)."}, "4": {"exist": true, "char_ranges": [[391, 565]], "word_ranges": [[57, 84]], "text": "Si nous suspectons une intoxication à l'éthylène glycol, nous administrerons de la thiamine, mais il n'y a pas de données qui nous permettent de suspecter cette intoxication."}, "5": {"exist": false, "char_ranges": [], "word_ranges": [], "text": ""}}} +{"id": 44, "year": 2011, "question_id_specific": 153, "full_question": "Bébé garçon, âgé de 28 jours, né à terme. Grossesse et accouchement normaux. Il est allaité depuis sa naissance. Il a consulté parce qu'il y a 8 jours, il a commencé à vomir, d'abord sporadiquement et depuis 5 jours après chaque tétée. Il a faim en permanence. Les vomissements sont des vomissements de nourriture \"en rafale\". À la palpation, l'abdomen est mou et dépressible, sans viscéro-mégalie. L'analyse des gaz du sang montre un pH de 7,49, un bicarbonate de 30 mEq/l, une pCO2 de 53 mmHg, un excès de base de +8 mEq/l. Ions : Na 137 mEq/l, K 3,1 mEq/l, Cl 94 mEq/l. Compte tenu du diagnostic le plus probable dans le tableau clinique décrit, quel est l'examen complémentaire de choix pour confirmer ce diagnostic clinique ?", "full_answer": "La bonne réponse est 3. Question facile si la pathologie est suspectée, ce qui me semble également facile car la description clinique est celle d'une sténose hypertrophique du pylore.", "type": "PÉDIATRIE", "options": {"1": "Radiographie abdominale simple.", "2": "pHmétrie.", "3": "Échographie abdominale.", "4": "Détermination des électrolytes dans la sueur.", "5": "Oesophagogastroscopie."}, "correct_option": 3, "explanations": {"1": {"exist": false, "char_ranges": [], "word_ranges": [], "text": ""}, "2": {"exist": false, "char_ranges": [], "word_ranges": [], "text": ""}, "3": {"exist": true, "char_ranges": [[110, 183]], "word_ranges": [[19, 29]], "text": "la description clinique est celle d'une sténose hypertrophique du pylore."}, "4": {"exist": false, "char_ranges": [], "word_ranges": [], "text": ""}, "5": {"exist": false, "char_ranges": [], "word_ranges": [], "text": ""}}} +{"id": 54, "year": 2011, "question_id_specific": 72, "full_question": "Une étudiante de 22 ans, sans antécédents pathologiques ni utilisation de médicaments autres que des anovulatoires, s'est présentée au service des urgences en raison d'une détérioration de son état général et d'un besoin de respirer profondément. Elle a signalé une perte de poids au cours des 2 ou 3 derniers jours, une polydipsie, une polyurie et des nausées. Pas de toux ni de sensation fébrile. Examen : aspect de gravité, tension artérielle 100/60 mmHg, respiration profonde et rapide (28rpm), niveau de conscience préservé, muqueuses sèches. Pas de fièvre. NFS : glycémie 420 mg/dL, Na+ 131 mEq/L, K+ normal, pH 7,08, bicarbonate 8 mEq/L et cétonurie (+++) Quelle réponse vous semble la plus correcte ?", "full_answer": "Début de DM avec acidocétose. La priorité est d'administrer de l'insuline par voie intraveineuse et une thérapie sérique. Bicarbonate si pH < 7. L'administration d'insuline ne doit jamais être retardée.", "type": "ENDOCRINOLOGIE", "options": {"1": "Début de diabète sucré de type 2, avec dyspnée probablement due à une pneumonie ou à une thromboembolie, car elle prend des médicaments anovulatoires.", "2": "Il s'agit d'une acidocétose diabétique. Elle doit être traitée avec de l'insuline intraveineuse, une thérapie sérique, des mesures générales et la recherche d'une cause précipitante.", "3": "Début d'un diabète de type 1 avec acidocétose. Traiter avec du bicarbonate et, après correction de l'acidose, ajouter de l'insuline par voie intraveineuse.", "4": "Cela ressemble à une acidocétose diabétique, mais il pourrait s'agir d'un problème d'alcoolisme. Le taux d'alcoolémie doit être déterminé avant de commencer un traitement à l'insuline.", "5": "Traitez-le avec de l'insuline sous-cutanée rapide, une thérapie sérique et demandez-lui de boire beaucoup."}, "correct_option": 2, "explanations": {"1": {"exist": false, "char_ranges": [], "word_ranges": [], "text": ""}, "2": {"exist": true, "char_ranges": [[30, 202]], "word_ranges": [[5, 30]], "text": "La priorité est d'administrer de l'insuline par voie intraveineuse et une thérapie sérique. Bicarbonate si pH < 7. L'administration d'insuline ne doit jamais être retardée."}, "3": {"exist": false, "char_ranges": [], "word_ranges": [], "text": ""}, "4": {"exist": false, "char_ranges": [], "word_ranges": [], "text": ""}, "5": {"exist": false, "char_ranges": [], "word_ranges": [], "text": ""}}} +{"id": 553, "year": 2022, "question_id_specific": 177, "full_question": "Une patiente de 33 ans, résidant à Valence, consulte pour une plaque érythémateuse-orange sur la joue droite, consécutive à une piqûre d'insecte, qui s'est lentement étendue jusqu'à 1,5 cm et qui, au cours des dernières semaines, s'est ulcérée et recouverte d'une croûte squameuse. La lésion ne provoque qu'une discrète gêne si elle se gratte. La patiente a des antécédents de psoriasis en plaques traité par adalimumab sous-cutané. Elle est en bon état de santé général et ne prend pas d'autres médicaments ni ne déclare d'allergies médicamenteuses. Indiquez le traitement que vous utiliseriez :", "full_answer": "Le fait que le lieu de résidence soit mentionné nous donne de nombreux indices. Une \"morsure\" qui se transforme en lésion sur une zone découverte, avec une surface croûteuse et qui ne gêne pas ou peu... eh bien, dans le bassin méditerranéen, c'est la leishmaniose cutanée ou leishmaniose du bouton d'Orient. Et s'il est vrai que la lésion est < 4 cm et unique, on nous dit que le patient est traité par adalimumab, un anticorps monoclonal anti-TNF. A l'exception de l'antimoniate de méglumine, aucun des autres traitements proposés dans les différentes options n'est indiqué dans la leishmaniose, c'est donc clair. La seule chose qui est plus discutable est que, comme il s'agit d'un patient immunodéprimé, un traitement systémique serait probablement plus approprié. Je ne sais pas si c'est donc contestable, mais au moins discutable.", "type": "DERMATOLOGIE", "options": {"1": "Isotrétinoïne par voie orale.", "2": "Méglumine antimoniate intralésionnelle.", "3": "Corticostéroïdes systémiques.", "4": "Amoxicilline par voie orale.", "5": null}, "correct_option": 2, "explanations": {"1": {"exist": true, "char_ranges": [[449, 614]], "word_ranges": [[78, 101]], "text": "A l'exception de l'antimoniate de méglumine, aucun des autres traitements proposés dans les différentes options n'est indiqué dans la leishmaniose, c'est donc clair."}, "2": {"exist": true, "char_ranges": [[449, 614]], "word_ranges": [[78, 101]], "text": "A l'exception de l'antimoniate de méglumine, aucun des autres traitements proposés dans les différentes options n'est indiqué dans la leishmaniose, c'est donc clair."}, "3": {"exist": true, "char_ranges": [[449, 614]], "word_ranges": [[78, 101]], "text": "A l'exception de l'antimoniate de méglumine, aucun des autres traitements proposés dans les différentes options n'est indiqué dans la leishmaniose, c'est donc clair."}, "4": {"exist": true, "char_ranges": [[449, 614]], "word_ranges": [[78, 101]], "text": "A l'exception de l'antimoniate de méglumine, aucun des autres traitements proposés dans les différentes options n'est indiqué dans la leishmaniose, c'est donc clair."}, "5": {"exist": false, "char_ranges": [], "word_ranges": [], "text": ""}}} +{"id": 608, "year": 2022, "question_id_specific": 115, "full_question": "Un homme de 27 ans, sportif régulier, a signalé une douleur à la jambe droite après une course continue. Il a consulté un kinésithérapeute à plusieurs reprises, qui a diagnostiqué une surcharge du mollet. Plusieurs mois se sont écoulés, son état ne s'est pas amélioré et il signale une douleur intense après l'activité physique, qui s'estompe au repos dans les heures qui suivent l'exercice. Quel test peut aider à établir le diagnostic ?", "full_answer": "On nous présente un cas de syndrome des loges chronique. Pour le diagnostic, il est nécessaire de mesurer la pression dans les compartiments affectés au repos et après l'activité (réponse 2 correcte).", "type": "TRAUMATOLOGIE", "options": {"1": "Tomographie par émission de positons au 18 FDG.", "2": "Détermination de la pression du compartiment aval immédiatement après l'activité.", "3": "Échographie Doppler pour exclure un trouble circulatoire du membre inférieur.", "4": "Spectrométrie par résonance magnétique.", "5": null}, "correct_option": 2, "explanations": {"1": {"exist": false, "char_ranges": [], "word_ranges": [], "text": ""}, "2": {"exist": true, "char_ranges": [[0, 200]], "word_ranges": [[0, 32]], "text": "On nous présente un cas de syndrome des loges chronique. Pour le diagnostic, il est nécessaire de mesurer la pression dans les compartiments affectés au repos et après l'activité (réponse 2 correcte)."}, "3": {"exist": false, "char_ranges": [], "word_ranges": [], "text": ""}, "4": {"exist": false, "char_ranges": [], "word_ranges": [], "text": ""}, "5": {"exist": false, "char_ranges": [], "word_ranges": [], "text": ""}}} +{"id": 294, "year": 2016, "question_id_specific": 44, "full_question": "Amalia est une fillette de 3 ans qui a été opérée pour une tétralogie de Fallot. Elle a également présenté un dysfonctionnement thymique pendant la période néonatale et une hypocalcémie, ainsi qu'une voix nasillarde et un retard psychomoteur. Quel test génétique vous semble le plus approprié pour parvenir à un diagnostic étiologique ?", "full_answer": "Question dans laquelle le tableau clinique doit être reconnu : anomalies cardiovasculaires - tétralogie de Fallot - ; aplasie/hypoplasie thymique - entraînant une immunodéficience primaire des lymphocytes T ; aplasie/hypoplasie parathyroïdienne - d'où hypocalcémie - ; anomalies palatines - voix nasillarde ; retard psychomoteur. A mon avis, rien qu'avec les mots \"dysfonctionnement thymique\", hypocalcémie et cardiopathie, il faudrait s'orienter directement vers le syndrome de DiGeorge. En effet, l'anomalie génétique est une délétion 22q11.2 (si on le sait, tant mieux, mais le tableau clinique suffirait). La seule option avec laquelle il pourrait y avoir un doute est le syndrome de Noonan, qui peut également être associé à une cardiopathie, mais qui se caractérise surtout par une petite taille, une cardiopathie et des altérations osseuses ; en outre, dans la majorité des cas, il est causé par une altération du gène PTPN11, et non par les gènes mentionnés dans la question.", "type": "GÉNÉTIQUE ET IMMUNOLOGIE", "options": {"1": "Étude de la délétion 22q11 (syndrome de Di George ou de Velo-cardio-facial).", "2": "Étude de l'expansion du gène FMR1, responsable du syndrome de l'X fragile.", "3": "Séquençage des gènes liés au HRAS (syndrome de Noonan).", "4": "Étude génétique du syndrome de Williams Beuren.", "5": null}, "correct_option": 1, "explanations": {"1": {"exist": true, "char_ranges": [[342, 488]], "word_ranges": [[47, 65]], "text": "rien qu'avec les mots \"dysfonctionnement thymique\", hypocalcémie et cardiopathie, il faudrait s'orienter directement vers le syndrome de DiGeorge."}, "2": {"exist": false, "char_ranges": [], "word_ranges": [], "text": ""}, "3": {"exist": true, "char_ranges": [[700, 849]], "word_ranges": [[101, 124]], "text": "peut également être associé à une cardiopathie, mais qui se caractérise surtout par une petite taille, une cardiopathie et des altérations osseuses ;"}, "4": {"exist": false, "char_ranges": [], "word_ranges": [], "text": ""}, "5": {"exist": false, "char_ranges": [], "word_ranges": [], "text": ""}}} +{"id": 156, "year": 2012, "question_id_specific": 230, "full_question": "Un patient de 40 ans se présente aux urgences avec une stomatite aphteuse, une conjonctivite, une urétrite, une balanite et des douleurs articulaires. Parmi les diagnostics suivants, lequel est le plus probable ?", "full_answer": "Compte tenu de ce tableau clinique et en l'absence de tests complémentaires, le diagnostic le plus probable est celui d'une arthrite réactionnelle. Le seul problème est que la déclaration fait référence à des douleurs articulaires et non à l'arthrite... Le syndrome de Reiter (triade conjonctivite, urétrite et arthrite) est l'une des formes sous lesquelles l'arthrite réactionnelle peut se manifester. Dans la maladie de Behçet, le muguet buccal et génital peut se manifester, mais l'urétrite n'est pas une caractéristique. L'affection oculaire caractéristique est l'uvéite.", "type": "RHEUMATOLOGIE", "options": {"1": "Behçet.", "2": "Syndrome d'arthrite réactive.", "3": "Syndrome de Sweet.", "4": "Carence en vitamine A.", "5": "Infection gonococcique."}, "correct_option": 2, "explanations": {"1": {"exist": true, "char_ranges": [[403, 524]], "word_ranges": [[59, 78]], "text": "Dans la maladie de Behçet, le muguet buccal et génital peut se manifester, mais l'urétrite n'est pas une caractéristique."}, "2": {"exist": true, "char_ranges": [[254, 402]], "word_ranges": [[39, 59]], "text": "Le syndrome de Reiter (triade conjonctivite, urétrite et arthrite) est l'une des formes sous lesquelles l'arthrite réactionnelle peut se manifester."}, "3": {"exist": false, "char_ranges": [], "word_ranges": [], "text": ""}, "4": {"exist": false, "char_ranges": [], "word_ranges": [], "text": ""}, "5": {"exist": false, "char_ranges": [], "word_ranges": [], "text": ""}}} +{"id": 82, "year": 2012, "question_id_specific": 45, "full_question": "Un homme de 60 ans se plaint de douleurs précordiales après un effort moyen. La coronarographie montre une sténose significative dans les segments proximaux des trois vaisseaux principaux avec un bon lit distal. La fonction ventriculaire est déprimée (<30%). Quelle est la meilleure option thérapeutique ?", "full_answer": "On nous parle d'un patient souffrant d'une maladie coronarienne à trois vaisseaux et, ce qui est très important, d'un dysfonctionnement ventriculaire sévère. Dans ces conditions, le traitement de choix serait chirurgical (chirurgie de revascularisation myocardique ou by-pass). Dans le cas d'un patient dont la fonction ventriculaire gauche est préservée, le choix du traitement chirurgical par rapport au traitement percutané serait plus discutable, car aujourd'hui, grâce aux stents à élution de médicaments, les différences en termes de morbidité et de mortalité entre la chirurgie de pontage et l'angioplastie sont minimes.", "type": "CARDIOLOGIE ET CHIRURGIE VASCULAIRE", "options": {"1": "Revascularisation percutanée.", "2": "Traitement médical.", "3": "Chirurgie de revascularisation du myocarde.", "4": "Implantation d'une contre-pulsion par ballonnet.", "5": "Transplantation cardiaque."}, "correct_option": 3, "explanations": {"1": {"exist": false, "char_ranges": [], "word_ranges": [], "text": ""}, "2": {"exist": false, "char_ranges": [], "word_ranges": [], "text": ""}, "3": {"exist": true, "char_ranges": [[0, 277]], "word_ranges": [[0, 37]], "text": "On nous parle d'un patient souffrant d'une maladie coronarienne à trois vaisseaux et, ce qui est très important, d'un dysfonctionnement ventriculaire sévère. Dans ces conditions, le traitement de choix serait chirurgical (chirurgie de revascularisation myocardique ou by-pass)."}, "4": {"exist": false, "char_ranges": [], "word_ranges": [], "text": ""}, "5": {"exist": false, "char_ranges": [], "word_ranges": [], "text": ""}}} +{"id": 485, "year": 2020, "question_id_specific": 49, "full_question": "Un homme de 60 ans traité à la carbamazépine pour une épilepsie présente de la fièvre (38,8º C), une odynophagie, une conjonctivite, des lésions cutanées brunâtres réparties sur une grande partie du corps, d'aspect similaire à un œil de bœuf et accompagnées d'un décollement de l'épiderme (plus de 30 %) au moindre contact. Il est très probable qu'il présente un tableau clinique de :", "full_answer": "Il s'agit d'un patient sévère avec des lésions cibles et le signe de Nikolsky sur > 30% de la surface corporelle. Avec les antécédents de traitement à la carbamazépine, une nécrolyse épidermique toxique est suspectée (réponse 4). Si le décollement de l'épiderme est inférieur à 10 %, on soupçonne une maladie de Stevens-Johnson (les cas intermédiaires sont qualifiés de chevauchement, car il s'agit d'un spectre complet). Nous recommandons d'ailleurs l'admission immédiate dans un service de grands brûlés.", "type": "DERMATOLOGIE", "options": {"1": "Urticaire aiguë d'origine pharmacologique.", "2": "Erythème mineur exsudatif secondaire à des médicaments.", "3": "Syndrome pharmacologique de Stevens-Johnson.", "4": "Nécrolyse épidermique toxique.", "5": null}, "correct_option": 4, "explanations": {"1": {"exist": false, "char_ranges": [], "word_ranges": [], "text": ""}, "2": {"exist": false, "char_ranges": [], "word_ranges": [], "text": ""}, "3": {"exist": true, "char_ranges": [[230, 327]], "word_ranges": [[37, 53]], "text": "Si le décollement de l'épiderme est inférieur à 10 %, on soupçonne une maladie de Stevens-Johnson"}, "4": {"exist": true, "char_ranges": [[0, 229]], "word_ranges": [[0, 37]], "text": "Il s'agit d'un patient sévère avec des lésions cibles et le signe de Nikolsky sur > 30% de la surface corporelle. Avec les antécédents de traitement à la carbamazépine, une nécrolyse épidermique toxique est suspectée (réponse 4)."}, "5": {"exist": false, "char_ranges": [], "word_ranges": [], "text": ""}}} +{"id": 219, "year": 2014, "question_id_specific": 124, "full_question": "Une femme de 72 ans souffrant de diabète de type 2 et d'insuffisance rénale chronique de stade 5. En cas de suspicion de thromboembolie pulmonaire, indiquez quel test diagnostique est contre-indiqué :", "full_answer": "Une question très simple et très utile pour la prise en charge de l'ETP aux urgences. Il n'est pas nécessaire de se creuser la tête : les patients souffrant d'insuffisance rénale avancée (stade 5) sont contre-indiqués pour tout examen d'imagerie nécessitant l'administration d'un produit de contraste par voie intraveineuse, tel que l'angiographie par tomodensitométrie. Tous les autres examens n'en ont pas besoin et seraient donc plus qu'indiqués en présence d'une PTE.", "type": "NEPHROLOGIE", "options": {"1": "Echocardiographie du cœur.", "2": "Scintigraphie pulmonaire.", "3": "Angiographie pulmonaire par tomodensitométrie.", "4": "Électrocardiogramme.", "5": "Radiographie pulmonaire."}, "correct_option": 3, "explanations": {"1": {"exist": false, "char_ranges": [], "word_ranges": [], "text": ""}, "2": {"exist": false, "char_ranges": [], "word_ranges": [], "text": ""}, "3": {"exist": true, "char_ranges": [[134, 370]], "word_ranges": [[26, 54]], "text": "les patients souffrant d'insuffisance rénale avancée (stade 5) sont contre-indiqués pour tout examen d'imagerie nécessitant l'administration d'un produit de contraste par voie intraveineuse, tel que l'angiographie par tomodensitométrie."}, "4": {"exist": false, "char_ranges": [], "word_ranges": [], "text": ""}, "5": {"exist": false, "char_ranges": [], "word_ranges": [], "text": ""}}} +{"id": 578, "year": 2022, "question_id_specific": 107, "full_question": "Un homme de 55 ans souffre d'une hémorragie sous-arachnoïdienne due à une rupture d'anévrisme de l'artère cérébrale moyenne. Il a été traité chirurgicalement. Après une bonne évolution au cours de laquelle il a récupéré la capacité de marcher, un mois après l'hémorragie, il subit une détérioration progressive, devenant hébété et ayant des difficultés à obéir aux ordres. Cochez la bonne réponse :", "full_answer": "Le vasospasme en tant que complication d'une hémorragie sous-arachnoïdienne survient généralement tôt, dans la semaine qui suit l'hémorragie. La détérioration progressive avec altération du niveau de conscience nous amène à suspecter l'une des complications qui peuvent survenir dans cette situation : l'hydrocéphalie. La technique diagnostique de choix dans ce cas est la neuro-imagerie avec la tomodensitométrie crânienne.", "type": "NEUROLOGIE", "options": {"1": "C'est au moment de l'évolution que le vasospasme est le plus fréquent.", "2": "Cette détérioration progressive est probablement secondaire à une nouvelle hémorragie de l'anévrisme.", "3": "La détérioration subaiguë est très probablement due à l'hydrocéphalie.", "4": "La technique diagnostique de choix est l'angiographie cérébrale.", "5": null}, "correct_option": 3, "explanations": {"1": {"exist": false, "char_ranges": [], "word_ranges": [], "text": ""}, "2": {"exist": false, "char_ranges": [], "word_ranges": [], "text": ""}, "3": {"exist": true, "char_ranges": [[0, 318]], "word_ranges": [[0, 42]], "text": "Le vasospasme en tant que complication d'une hémorragie sous-arachnoïdienne survient généralement tôt, dans la semaine qui suit l'hémorragie. La détérioration progressive avec altération du niveau de conscience nous amène à suspecter l'une des complications qui peuvent survenir dans cette situation : l'hydrocéphalie."}, "4": {"exist": false, "char_ranges": [], "word_ranges": [], "text": ""}, "5": {"exist": false, "char_ranges": [], "word_ranges": [], "text": ""}}} +{"id": 199, "year": 2013, "question_id_specific": 74, "full_question": "Chez un patient présentant au cours de la deuxième décennie de sa vie un parkinsonisme progressif, des tremblements, une dystonie et des troubles du comportement, il faut toujours rechercher une maladie dont le traitement approprié peut améliorer les symptômes neurologiques et arrêter l'évolution clinique. De quelle maladie s'agit-il ?", "full_answer": "La clé de la question est \"l'arrêt de l'évolution clinique\", car la maladie de Hungtinton, la maladie de Tourette et la maladie de Parkinson n'ont pas de traitement permettant d'arrêter l'évolution clinique. La chorée de Sydenham est typique des jeunes enfants ayant des antécédents d'infection streptococcique et sa résolution est généralement spontanée. D'autre part, dans les cas cliniques, il est recommandé d'exclure la maladie de Wilson chez les jeunes patients présentant des troubles du mouvement et des troubles neuropsychiatriques.", "type": "NEUROLOGIE", "options": {"1": "La maladie de Huntington.", "2": "La maladie de Wilson.", "3": "Le syndrome de Gilles de la Tourette.", "4": "Sydenham Korea.", "5": "Maladie de Parkinson d'apparition juvénile."}, "correct_option": 2, "explanations": {"1": {"exist": true, "char_ranges": [[0, 207]], "word_ranges": [[0, 32]], "text": "La clé de la question est \"l'arrêt de l'évolution clinique\", car la maladie de Hungtinton, la maladie de Tourette et la maladie de Parkinson n'ont pas de traitement permettant d'arrêter l'évolution clinique."}, "2": {"exist": true, "char_ranges": [[370, 541]], "word_ranges": [[54, 79]], "text": "dans les cas cliniques, il est recommandé d'exclure la maladie de Wilson chez les jeunes patients présentant des troubles du mouvement et des troubles neuropsychiatriques."}, "3": {"exist": true, "char_ranges": [[0, 207]], "word_ranges": [[0, 32]], "text": "La clé de la question est \"l'arrêt de l'évolution clinique\", car la maladie de Hungtinton, la maladie de Tourette et la maladie de Parkinson n'ont pas de traitement permettant d'arrêter l'évolution clinique."}, "4": {"exist": true, "char_ranges": [[208, 355]], "word_ranges": [[32, 52]], "text": "La chorée de Sydenham est typique des jeunes enfants ayant des antécédents d'infection streptococcique et sa résolution est généralement spontanée."}, "5": {"exist": true, "char_ranges": [[0, 207]], "word_ranges": [[0, 32]], "text": "La clé de la question est \"l'arrêt de l'évolution clinique\", car la maladie de Hungtinton, la maladie de Tourette et la maladie de Parkinson n'ont pas de traitement permettant d'arrêter l'évolution clinique."}}} +{"id": 331, "year": 2016, "question_id_specific": 87, "full_question": "Chez un patient hospitalisé incapable de s'alimenter par voie orale pendant plus de 6 jours, dans laquelle des situations cliniques suivantes doit-on obligatoirement recourir à la nutrition parentérale ?", "full_answer": "L'iléus paralytique est un défaut de propulsion intestinale qui survient de façon aiguë en l'absence d'obstruction mécanique ; il est secondaire à diverses causes et affecte la contractilité intestinale. Il est secondaire à diverses causes et affecte la contractilité intestinale, nécessitant une administration parentérale.", "type": "ENDOCRINOLOGIE", "options": {"1": "Accident vasculaire cérébral cardioembolique avec dysphagie neurologique complète.", "2": "Cachexie due à l'empyème chronique chez les patients immunodéprimés.", "3": "accident vasculaire cérébral paralytique prolongé.", "4": "Maladie d'Alzheimer avancée avec risque sévère d'aspiration bronchique.", "5": null}, "correct_option": 3, "explanations": {"1": {"exist": false, "char_ranges": [], "word_ranges": [], "text": ""}, "2": {"exist": false, "char_ranges": [], "word_ranges": [], "text": ""}, "3": {"exist": true, "char_ranges": [[0, 324]], "word_ranges": [[0, 44]], "text": "L'iléus paralytique est un défaut de propulsion intestinale qui survient de façon aiguë en l'absence d'obstruction mécanique ; il est secondaire à diverses causes et affecte la contractilité intestinale. Il est secondaire à diverses causes et affecte la contractilité intestinale, nécessitant une administration parentérale."}, "4": {"exist": false, "char_ranges": [], "word_ranges": [], "text": ""}, "5": {"exist": false, "char_ranges": [], "word_ranges": [], "text": ""}}} +{"id": 205, "year": 2014, "question_id_specific": 214, "full_question": "Un garçon de 24 ans présente, trois jours après un contact sexuel à risque, de nombreuses petites lésions pustuleuses très prurigineuses qui évoluent vers de minuscules érosions touchant l'ensemble du gland du pénis et la surface interne du prépuce. Lequel des diagnostics suivants est le plus probable ?", "full_answer": "Le chancre est une lésion solitaire. La syphilis secondaire se présente généralement sous la forme d'une éruption généralisée ou de lésions caractéristiques sur les paumes et les plantes (ongles syphilitiques) avec une période de latence plus longue. La balanite à Trichomonas est toujours associée à une urétrite, qui n'est pas mentionnée dans la question. La réponse est donc 1. Cependant, le tableau clinique suggère un herpès simplex plutôt qu'une candidose, et 5 fait également référence à des champignons (bien que les dermatophytes ne provoquent pas de balanite) .... De notre point de vue, même si nous considérons que 1 est correct, la question est trompeuse et pourrait faire l'objet d'une contestation. De plus, la candidose génitale n'est pas nécessairement sexuellement transmissible.", "type": "DERMATOLOGIE", "options": {"1": "Candidose génitale.", "2": "Chancre.", "3": "Syphilis secondaire.", "4": "Balanite à Trichomonas.", "5": "Infection fongique dermatophyte."}, "correct_option": 1, "explanations": {"1": {"exist": false, "char_ranges": [], "word_ranges": [], "text": ""}, "2": {"exist": true, "char_ranges": [[0, 36]], "word_ranges": [[0, 6]], "text": "Le chancre est une lésion solitaire."}, "3": {"exist": true, "char_ranges": [[37, 250]], "word_ranges": [[6, 37]], "text": "La syphilis secondaire se présente généralement sous la forme d'une éruption généralisée ou de lésions caractéristiques sur les paumes et les plantes (ongles syphilitiques) avec une période de latence plus longue."}, "4": {"exist": true, "char_ranges": [[251, 357]], "word_ranges": [[37, 54]], "text": "La balanite à Trichomonas est toujours associée à une urétrite, qui n'est pas mentionnée dans la question."}, "5": {"exist": false, "char_ranges": [], "word_ranges": [], "text": ""}}} +{"id": 441, "year": 2018, "question_id_specific": 98, "full_question": "Une femme de 21 ans se présente à la clinique avec une étude hormonale montrant une FSH de 0,29 mUl/mL (normale en phase folliculaire = 3,50-12,50 mUl/mL) et une LH < 0,1 mUl/mL (normale en phase folliculaire 2,40-12,60 mUl/mL). Un diagnostic différentiel doit être établi entre les situations cliniques suivantes SAUF :", "full_answer": "En effet, en cas d'insuffisance ovarienne précoce, la FSH et la LH sont élevées. Ceci est confirmé par le protocole SEGO appelé Ménoguie précoce. Les autres diagnostics sont cohérents avec ces paramètres analytiques.", "type": "GYNÉCOLOGIE ET OBSTÉTRIQUE", "options": {"1": "Administration de contraceptifs oraux combinés.", "2": "Insuffisance ovarienne prématurée.", "3": "Troubles du comportement alimentaire.", "4": "Craniopharyngiome.", "5": null}, "correct_option": 2, "explanations": {"1": {"exist": true, "char_ranges": [[145, 216]], "word_ranges": [[23, 33]], "text": "précoce. Les autres diagnostics sont cohérents avec ces paramètres analytiques."}, "2": {"exist": true, "char_ranges": [[0, 145]], "word_ranges": [[0, 24]], "text": "En effet, en cas d'insuffisance ovarienne précoce, la FSH et la LH sont élevées. Ceci est confirmé par le protocole SEGO appelé Ménoguie précoce."}, "3": {"exist": true, "char_ranges": [[145, 216]], "word_ranges": [[23, 33]], "text": "précoce. Les autres diagnostics sont cohérents avec ces paramètres analytiques."}, "4": {"exist": true, "char_ranges": [[145, 216]], "word_ranges": [[23, 33]], "text": "précoce. Les autres diagnostics sont cohérents avec ces paramètres analytiques."}, "5": {"exist": false, "char_ranges": [], "word_ranges": [], "text": ""}}} +{"id": 302, "year": 2016, "question_id_specific": 111, "full_question": "Chez un patient atteint de polyarthrite rhumatoïde sous méthotrexate, prednisone et indométhacine qui présente de manière aiguë des œdèmes et une augmentation de la créatinine plasmatique avec un sédiment urinaire peu expressif et une protéinurie inférieure à 100 mg/24 h, la cause la plus probable est la suivante :", "full_answer": "Une insuffisance rénale aiguë pré-rénale due aux AINS semble l'option la plus plausible dans ce cas. Une néphrite tubulo-interstitielle induite par le méthotrexate est moins probable en raison de l'absence d'éruption cutanée, de fièvre ou d'éosinophilie, qui, bien que rares dans la MIR, sont souvent présentes. Bien que la GMN et l'amylose rénale soient relativement fréquentes dans la PR, le sédiment urinaire fade et l'absence de protéinurie rendent ces options peu probables.", "type": "NEPHROLOGIE", "options": {"1": "Amyloïdose rénale.", "2": "Glomérulonéphrite secondaire à la polyarthrite rhumatoïde.", "3": "Insuffisance rénale due aux anti-inflammatoires non stéroïdiens.", "4": "Néphrite interstitielle due au méthotrexate.", "5": null}, "correct_option": 3, "explanations": {"1": {"exist": true, "char_ranges": [[312, 479]], "word_ranges": [[46, 72]], "text": "Bien que la GMN et l'amylose rénale soient relativement fréquentes dans la PR, le sédiment urinaire fade et l'absence de protéinurie rendent ces options peu probables."}, "2": {"exist": false, "char_ranges": [], "word_ranges": [], "text": ""}, "3": {"exist": true, "char_ranges": [[0, 100]], "word_ranges": [[0, 16]], "text": "Une insuffisance rénale aiguë pré-rénale due aux AINS semble l'option la plus plausible dans ce cas."}, "4": {"exist": true, "char_ranges": [[101, 311]], "word_ranges": [[16, 46]], "text": "Une néphrite tubulo-interstitielle induite par le méthotrexate est moins probable en raison de l'absence d'éruption cutanée, de fièvre ou d'éosinophilie, qui, bien que rares dans la MIR, sont souvent présentes."}, "5": {"exist": false, "char_ranges": [], "word_ranges": [], "text": ""}}} +{"id": 517, "year": 2021, "question_id_specific": 104, "full_question": "Un homme de 34 ans est impliqué dans un accident de la route et est pris en charge sur le lieu de l'accident. Il est pâle, en sueur, a un pouls filant avec une pression artérielle de 90/50 mmHg, une fréquence cardiaque de 127 bpm, une fréquence respiratoire de 28 rpm et une saturation en oxygène de 92%. Il a été perfusé avec 20 ml/kg de cristalloïdes. Pendant le transfert à l'hôpital, les signes vitaux s'améliorent transitoirement puis se détériorent à l'arrivée à l'hôpital. Cochez la bonne réponse :", "full_answer": "Chez un patient polytraumatisé, la première cause de choc est la perte de sang (choc hémorragique). Les directives à suivre doivent toujours inclure la reconstitution du volume sanguin (de préférence avec des produits sanguins le plus tôt possible) et le contrôle de la focalisation. Dans ce cas, compte tenu de la situation vitale compromise, le besoin de transfusion est très urgent, et l'indication est donc une transfusion sanguine d'urgence (O Rh négatif).", "type": "SOINS CRITIQUES ET D'URGENCE", "options": {"1": "Nécessite une transfusion de sang de groupe spécifique.", "2": "Nécessite une transfusion sanguine d'urgence (O Rh négatif).", "3": "Une éventuelle transfusion sanguine doit être préparée à l'aide de tests de compatibilité croisée.", "4": "Une nouvelle perfusion de 20 ml/kg de cristalloïde doit être tentée à nouveau.", "5": null}, "correct_option": 2, "explanations": {"1": {"exist": false, "char_ranges": [], "word_ranges": [], "text": ""}, "2": {"exist": true, "char_ranges": [[284, 461]], "word_ranges": [[44, 72]], "text": "Dans ce cas, compte tenu de la situation vitale compromise, le besoin de transfusion est très urgent, et l'indication est donc une transfusion sanguine d'urgence (O Rh négatif)."}, "3": {"exist": false, "char_ranges": [], "word_ranges": [], "text": ""}, "4": {"exist": false, "char_ranges": [], "word_ranges": [], "text": ""}, "5": {"exist": false, "char_ranges": [], "word_ranges": [], "text": ""}}} +{"id": 520, "year": 2021, "question_id_specific": 38, "full_question": "Un patient de 72 ans s'est présenté aux urgences pour un angio-œdème lingual sévère. Il ne présentait pas d'urticaire associée et n'avait pas d'antécédents d'allergie médicamenteuse ou alimentaire. Ses antécédents personnels comprenaient un diabète sucré de type 2, une dyslipidémie, une hypertension, une hypothyroïdie et la maladie de Parkinson. Il suit un traitement régulier à la metformine, à la simvastatine, à l'énalapril, à la thyroxine et à la lévodopa. Parmi les médicaments suivants, lequel est le plus susceptible d'être à l'origine du tableau clinique décrit ?", "full_answer": "L'œdème de Quincke est un type d'urticaire profonde qui peut être d'origine médicamenteuse (non IgE) et dans ce cas, les inhibiteurs de l'ECA (tels que l'énalapril) sont considérés comme la cause pharmacologique la plus fréquente, donc dans ce cas ce serait la bonne réponse (si nous parlons en termes probabilistes).", "type": "DERMATOLOGIE", "options": {"1": "Enalapril.", "2": "Simvastatine.", "3": "Metformine ou lévodopa également.", "4": "Levodopa.", "5": null}, "correct_option": 1, "explanations": {"1": {"exist": true, "char_ranges": [[0, 317]], "word_ranges": [[0, 50]], "text": "L'œdème de Quincke est un type d'urticaire profonde qui peut être d'origine médicamenteuse (non IgE) et dans ce cas, les inhibiteurs de l'ECA (tels que l'énalapril) sont considérés comme la cause pharmacologique la plus fréquente, donc dans ce cas ce serait la bonne réponse (si nous parlons en termes probabilistes)."}, "2": {"exist": false, "char_ranges": [], "word_ranges": [], "text": ""}, "3": {"exist": false, "char_ranges": [], "word_ranges": [], "text": ""}, "4": {"exist": false, "char_ranges": [], "word_ranges": [], "text": ""}, "5": {"exist": false, "char_ranges": [], "word_ranges": [], "text": ""}}} +{"id": 379, "year": 2016, "question_id_specific": 133, "full_question": "Une jeune fille de 20 ans se présente au service des urgences car elle a remarqué, après s'être levée, en se regardant dans le miroir, une faiblesse dans toute la moitié droite de son visage (y compris au niveau du front, en fermant la paupière et en souriant). Elle est accompagnée d'une dysgueusie, avec une sensation de goût métallique des aliments, ainsi que d'une hyperacousie et d'une douleur mastoïdienne ipsilatérale. À l'examen, il n'y a aucun signe de déficit de force ou de déficit sensoriel dans les extrémités, ni de troubles de la parole et du langage. Dans ce cas, laquelle des affirmations suivantes est correcte ?", "full_answer": "Ils décrivent un tableau de paralysie faciale périphérique typique, avec dysgueusie, hyperacousie et douleur mastoïdienne. Dans ce cas, le traitement est un corticostéroïde. La première réponse n'est pas correcte car, bien qu'elle se situe dans la fourchette d'âge pour l'apparition de la sclérose en plaques, la présentation est normalement progressive sur une période de plusieurs jours et l'atteinte nucléaire du 7e nerf crânien (qui pourrait simuler une paralysie faciale périphérique) ne provoquerait pas de dysgueusie, d'hyperacousie ou de douleur mastoïdienne. La troisième réponse suggère qu'il s'agit d'une affection vasculaire et donc que la paralysie faciale est centrale, ce qui l'exclut. La dernière réponse n'est pas correcte car la plupart des paralysies faciales périphériques idiopathiques (paralysie de Bell) guérissent sans séquelles.", "type": "NEUROLOGIE", "options": {"1": "Le diagnostic le plus probable est celui d'une plaque de démyélinisation dans l'hémiprotubérance controlatérale, l'examen complémentaire le plus nécessaire étant une IRM crânienne.", "2": "Les corticostéroïdes oraux sont le traitement de choix pour le patient.", "3": "La fibrinolyse intraveineuse doit être envisagée si la durée d'évolution est inférieure à 3 heures.", "4": "L'image est très probablement irréversible.", "5": null}, "correct_option": 2, "explanations": {"1": {"exist": true, "char_ranges": [[174, 567]], "word_ranges": [[23, 80]], "text": "La première réponse n'est pas correcte car, bien qu'elle se situe dans la fourchette d'âge pour l'apparition de la sclérose en plaques, la présentation est normalement progressive sur une période de plusieurs jours et l'atteinte nucléaire du 7e nerf crânien (qui pourrait simuler une paralysie faciale périphérique) ne provoquerait pas de dysgueusie, d'hyperacousie ou de douleur mastoïdienne."}, "2": {"exist": true, "char_ranges": [[0, 173]], "word_ranges": [[0, 23]], "text": "Ils décrivent un tableau de paralysie faciale périphérique typique, avec dysgueusie, hyperacousie et douleur mastoïdienne. Dans ce cas, le traitement est un corticostéroïde."}, "3": {"exist": true, "char_ranges": [[568, 700]], "word_ranges": [[80, 100]], "text": "La troisième réponse suggère qu'il s'agit d'une affection vasculaire et donc que la paralysie faciale est centrale, ce qui l'exclut."}, "4": {"exist": true, "char_ranges": [[701, 853]], "word_ranges": [[100, 120]], "text": "La dernière réponse n'est pas correcte car la plupart des paralysies faciales périphériques idiopathiques (paralysie de Bell) guérissent sans séquelles."}, "5": {"exist": false, "char_ranges": [], "word_ranges": [], "text": ""}}} +{"id": 600, "year": 2022, "question_id_specific": 184, "full_question": "Une femme de 70 ans a signalé une douleur et une raideur au niveau de la ceinture scapulaire et du bassin depuis un mois, prédominant en se levant et s'améliorant avec le mouvement, sans fièvre, ni maux de tête, ni malaise général. L'examen physique révèle une raideur et une douleur aux mouvements des épaules et des hanches. Les analyses sanguines révèlent une élévation de l'ESR et de la protéine C-réactive. Quel est le diagnostic le plus probable et quel traitement prescrivez-vous ?", "full_answer": "Dans ce cas, il décrit les symptômes cardinaux de la polymyalgie rhumatismale. Patient âgé de plus de 50 ans, présentant une douleur et une raideur au niveau des deux ceintures, associées à une élévation des réactifs de phase aiguë. Il/elle présente tous les critères indispensables : âge de 50 ans ou plus, omalgie bilatérale et augmentation des réactifs de phase aiguë. Parmi les critères optionnels, raideur, douleur ou limitation de la hanche. Le traitement initial consiste en l'administration de corticostéroïdes à doses moyennes.", "type": "RHEUMATOLOGIE", "options": {"1": "La polyarthrite rhumatoïde est traitée par le méthotrexate.", "2": "Fibromyalgie, commencerait le traitement par la duloxétine.", "3": "Polymyalgie rhumatismale, commencer le traitement avec des corticostéroïdes.", "4": "Spondylarthrite ankylosante, commencer le traitement par des anti-inflammatoires non stéroïdiens.", "5": null}, "correct_option": 3, "explanations": {"1": {"exist": false, "char_ranges": [], "word_ranges": [], "text": ""}, "2": {"exist": false, "char_ranges": [], "word_ranges": [], "text": ""}, "3": {"exist": true, "char_ranges": [[0, 370]], "word_ranges": [[0, 61]], "text": "Dans ce cas, il décrit les symptômes cardinaux de la polymyalgie rhumatismale. Patient âgé de plus de 50 ans, présentant une douleur et une raideur au niveau des deux ceintures, associées à une élévation des réactifs de phase aiguë. Il/elle présente tous les critères indispensables : âge de 50 ans ou plus, omalgie bilatérale et augmentation des réactifs de phase aiguë."}, "4": {"exist": false, "char_ranges": [], "word_ranges": [], "text": ""}, "5": {"exist": false, "char_ranges": [], "word_ranges": [], "text": ""}}} +{"id": 415, "year": 2018, "question_id_specific": 72, "full_question": "Un homme de 45 ans, ayant des antécédents de maladie fébrile et de douleur thoracique, se présente à l'hôpital avec une dyspnée et une tachypnée. À l'examen physique, sa tension artérielle est basse, la pression veineuse jugulaire est élevée avec un sinus descendant proéminent et il a un pouls paradoxal. Quel diagnostic suspecteriez-vous ?", "full_answer": "C'est l'examen classique : il avait de la fièvre et des douleurs, il avait une péricardite avec épanchement, et maintenant cela provoque une tamponnade. D'où l'hypotension (son cœur ne peut pas se remplir correctement de sang et pomper), l'engorgement jugulaire et la dépendance à la respiration pour atteindre la précharge et le débit cardiaque (c'est-à-dire le pouls paradoxal).", "type": "CARDIOLOGIE ET CHIRURGIE CARDIOVASCULAIRE", "options": {"1": "Cardiomyopathie dilatée.", "2": "Épanchement péricardique avec tamponnade cardiaque.", "3": "Péricardite constrictive.", "4": "Maladie cardiaque ischémique.", "5": null}, "correct_option": 2, "explanations": {"1": {"exist": false, "char_ranges": [], "word_ranges": [], "text": ""}, "2": {"exist": true, "char_ranges": [[27, 380]], "word_ranges": [[4, 58]], "text": "il avait de la fièvre et des douleurs, il avait une péricardite avec épanchement, et maintenant cela provoque une tamponnade. D'où l'hypotension (son cœur ne peut pas se remplir correctement de sang et pomper), l'engorgement jugulaire et la dépendance à la respiration pour atteindre la précharge et le débit cardiaque (c'est-à-dire le pouls paradoxal)."}, "3": {"exist": false, "char_ranges": [], "word_ranges": [], "text": ""}, "4": {"exist": false, "char_ranges": [], "word_ranges": [], "text": ""}, "5": {"exist": false, "char_ranges": [], "word_ranges": [], "text": ""}}} +{"id": 560, "year": 2022, "question_id_specific": 180, "full_question": "Une femme de 26 ans, chez qui on a diagnostiqué un lupus érythémateux disséminé et qui est traitée à l'hydroxychloroquine, consulte pour une sensation de faiblesse généralisée qui s'est progressivement développée au cours des 15 derniers jours. L'examen physique a révélé une pâleur cutanée et l'analyse sanguine a révélé une Hb de 7,4 g/dL, une Hct de 31 % et une MCV de 108. Lequel des tests suivants serait le plus utile pour décider de la marche à suivre ?", "full_answer": "Ils rapportent le cas d'une patiente atteinte de LED et traitée à l'hydroxychloroquine. Elle a un taux d'hémoglobine de 7,4 et un VCM supérieur à 100. Si l'on ajoute la pâleur et la faiblesse, nous aurions affaire à une anémie macrocytaire. A mon avis, l'alternative la plus appropriée serait l'alternative 2, car il a été rapporté que l'hydroxyloroquine diminue l'absorption de l'acide folique, comme elle a une complication du LED, dans ce cas, nous pourrions avoir affaire à une anémie hémolytique auto-immune avec consommation d'acide folique. Pour cela, nous devrions d'abord déterminer le taux de coombs direct.", "type": "HÉMATOLOGIE", "options": {"1": "Haptoglobine.", "2": "Test de Coombs.", "3": "Vitamine B12.", "4": "Anticorps antinucléaires.", "5": null}, "correct_option": 2, "explanations": {"1": {"exist": false, "char_ranges": [], "word_ranges": [], "text": ""}, "2": {"exist": true, "char_ranges": [[241, 617]], "word_ranges": [[41, 97]], "text": "A mon avis, l'alternative la plus appropriée serait l'alternative 2, car il a été rapporté que l'hydroxyloroquine diminue l'absorption de l'acide folique, comme elle a une complication du LED, dans ce cas, nous pourrions avoir affaire à une anémie hémolytique auto-immune avec consommation d'acide folique. Pour cela, nous devrions d'abord déterminer le taux de coombs direct."}, "3": {"exist": false, "char_ranges": [], "word_ranges": [], "text": ""}, "4": {"exist": false, "char_ranges": [], "word_ranges": [], "text": ""}, "5": {"exist": false, "char_ranges": [], "word_ranges": [], "text": ""}}} +{"id": 358, "year": 2016, "question_id_specific": 232, "full_question": "Une femme de 24 ans consulte après avoir constaté une lymphadénopathie inguinale. L'interrogatoire n'a pas révélé de gêne locale ni d'éléments évocateurs d'une infection sexuellement transmissible. L'examen révèle deux adénopathies, une dans chaque aine, de 1 cm de diamètre, molles, mobiles, non douloureuses. Aucune lésion cutanée n'est observée au niveau des membres inférieurs, de l'anus ou du périnée. Quel examen vous semble indispensable ?", "full_answer": "La bonne réponse est 3. Dans la région inguinale, les ganglions lymphatiques peuvent mesurer jusqu'à 1,5 cm et sont normaux. De plus, le reste des données de l'examen inguinal (pas de douleur, mobile, souple, etc.) corrobore cela. D'autre part, il n'y a pas de signes ou de symptômes suggérant une infection sexuellement transmissible.", "type": "GYNÉCOLOGIE ET OBSTÉTRIQUE", "options": {"1": "Une sérologie pour la syphilis, car il s'agit très probablement d'une infection à Treponema pallidum.", "2": "Un examen gynécologique pour écarter la possibilité d'un cancer de l'ovaire.", "3": "D'après les caractéristiques cliniques, il semble que les ganglions lymphatiques soient normaux et qu'il n'y ait pas lieu de procéder à d'autres examens.", "4": "Un test de Paul-Bunell doit être effectué pour exclure une mononucléose infectieuse.", "5": null}, "correct_option": 3, "explanations": {"1": {"exist": true, "char_ranges": [[231, 335]], "word_ranges": [[37, 53]], "text": "D'autre part, il n'y a pas de signes ou de symptômes suggérant une infection sexuellement transmissible."}, "2": {"exist": false, "char_ranges": [], "word_ranges": [], "text": ""}, "3": {"exist": true, "char_ranges": [[24, 230]], "word_ranges": [[5, 37]], "text": "Dans la région inguinale, les ganglions lymphatiques peuvent mesurer jusqu'à 1,5 cm et sont normaux. De plus, le reste des données de l'examen inguinal (pas de douleur, mobile, souple, etc.) corrobore cela."}, "4": {"exist": false, "char_ranges": [], "word_ranges": [], "text": ""}, "5": {"exist": false, "char_ranges": [], "word_ranges": [], "text": ""}}} +{"id": 414, "year": 2018, "question_id_specific": 71, "full_question": "Homme de 84 ans souffrant d'insuffisance cardiaque chronique de grade fonctionnel II secondaire à une cardiopathie ischémique avec dysfonction systolique sévère non revascularisable, d'une maladie rénale chronique de stade 3 (débit de filtration glomérulaire 45-50 ml/min) et de fibrillation auriculaire permanente avec fréquence cardiaque >80 battements par minute. Lequel des médicaments suivants n'apporte PAS de bénéfice au patient selon les données actuellement disponibles ?", "full_answer": "L'ivabradine est en fait contre-indiquée chez les patients souffrant de fibrillation auriculaire. Initialement (bien que des preuves soient apparues récemment pour nuancer cette information), l'ivabradine était censée être sélective sur le courant sodique dans le nœud sinusal. Si le patient est en fibrillation auriculaire, il n'est pas logique d'agir sur le nœud sinusal.", "type": "CARDIOLOGIE ET CHIRURGIE CARDIOVASCULAIRE", "options": {"1": "Ivabradine.", "2": "Inhibiteurs de l'enzyme de conversion de l'angiotensine.", "3": "Antialdostéroniques.", "4": "Bêta-bloquants.", "5": null}, "correct_option": 1, "explanations": {"1": {"exist": true, "char_ranges": [[0, 97]], "word_ranges": [[0, 12]], "text": "L'ivabradine est en fait contre-indiquée chez les patients souffrant de fibrillation auriculaire."}, "2": {"exist": false, "char_ranges": [], "word_ranges": [], "text": ""}, "3": {"exist": false, "char_ranges": [], "word_ranges": [], "text": ""}, "4": {"exist": false, "char_ranges": [], "word_ranges": [], "text": ""}, "5": {"exist": false, "char_ranges": [], "word_ranges": [], "text": ""}}} +{"id": 131, "year": 2012, "question_id_specific": 221, "full_question": "Un individu présente une \"faiblesse musculaire\" (hémiparésie spastique) des deux membres droits, avec hyperréflexie et signe de Babinski, ainsi qu'une \"paralysie faciale flasque\" de l'hémiface gauche, avec incapacité de fermer l'œil gauche ou de rétracter le côté gauche de la bouche, en plus d'autres altérations. D'après les données décrites, il s'agit d'une affection touchant, entre autres, les faisceaux moteurs : cortico-spinal et cortico-nucléaire, mais à quel niveau du neuroaxe se situerait la lésion ?", "full_answer": "Il s'agit toujours d'une question concernant le tronc cérébral. Nous avons une hémiparésie droite et une atteinte du noyau facial gauche, puisque les noyaux facial supérieur et inférieur sont touchés ; nous avons donc un tableau clinique croisé, et nous devons penser à l'atteinte du tronc cérébral. Si l'on fait un rapide schéma de l'emplacement des noyaux des paires crâniennes : les 4 premiers au-dessus du pons, les 4 suivants dans le pons et les 4 derniers en dessous du pons, on constate que le noyau facial se trouve dans les 4 noyaux du milieu. Donc noyau facial gauche, pons gauche, donc réponse 4.", "type": "NEUROLOGIE ET NEUROCHIRURGIE", "options": {"1": "Au niveau de l'aire 4 de Brodmann du cortex cérébral droit.", "2": "Dans la capsule interne, bras postérieur du côté droit.", "3": "Dans le pédoncule cérébral gauche.", "4": "Dans la partie médiane du pons caudal du côté gauche.", "5": "Dans le bulbe rachidien avant la décussation du tractus cortico-spinal droit."}, "correct_option": 4, "explanations": {"1": {"exist": false, "char_ranges": [], "word_ranges": [], "text": ""}, "2": {"exist": false, "char_ranges": [], "word_ranges": [], "text": ""}, "3": {"exist": false, "char_ranges": [], "word_ranges": [], "text": ""}, "4": {"exist": true, "char_ranges": [[218, 607]], "word_ranges": [[34, 104]], "text": "un tableau clinique croisé, et nous devons penser à l'atteinte du tronc cérébral. Si l'on fait un rapide schéma de l'emplacement des noyaux des paires crâniennes : les 4 premiers au-dessus du pons, les 4 suivants dans le pons et les 4 derniers en dessous du pons, on constate que le noyau facial se trouve dans les 4 noyaux du milieu. Donc noyau facial gauche, pons gauche, donc réponse 4."}, "5": {"exist": false, "char_ranges": [], "word_ranges": [], "text": ""}}} +{"id": 70, "year": 2012, "question_id_specific": 127, "full_question": "Une femme de 30 ans ayant un antécédent de toxicomanie et suivant actuellement une cure de désintoxication est amenée aux urgences pour une somnolence profonde, sans aucun autre symptôme. Nous apprenons que son traitement comprend de l'olanzapine et des benzodiazépines. Que devons-nous faire en premier lieu ?", "full_answer": "Une surconsommation des médicaments qu'il prend doit être suspectée. L'olanzapine n'ayant pas d'antidote, il convient en principe d'antagoniser l'effet des benzodiazépines. Si le patient reste somnolent et ne réagit pas aux stimuli, il faut procéder à une intubation pour sécuriser les voies respiratoires.", "type": "ANESTHÉSIOLOGIE, SOINS INTENSIFS ET MÉDECINE D'URGENCE", "options": {"1": "Passer à l'IOT.", "2": "Effectuer une radiographie du crâne.", "3": "Administrer du flumazénil.", "4": "Administrer la naloxone.", "5": "Attendez qu'il se réveille."}, "correct_option": 3, "explanations": {"1": {"exist": false, "char_ranges": [], "word_ranges": [], "text": ""}, "2": {"exist": false, "char_ranges": [], "word_ranges": [], "text": ""}, "3": {"exist": true, "char_ranges": [[0, 172]], "word_ranges": [[0, 21]], "text": "Une surconsommation des médicaments qu'il prend doit être suspectée. L'olanzapine n'ayant pas d'antidote, il convient en principe d'antagoniser l'effet des benzodiazépines."}, "4": {"exist": false, "char_ranges": [], "word_ranges": [], "text": ""}, "5": {"exist": false, "char_ranges": [], "word_ranges": [], "text": ""}}} +{"id": 463, "year": 2019, "question_id_specific": 71, "full_question": "Une femme souffrant d'hypotension et d'hypoperfusion tissulaire chez laquelle les paramètres suivants ont été observés après l'insertion d'un cathéter de Swan-Ganz : indice cardiaque de 1,4 L/min/m2, pression de coin capillaire pulmonaire de 25 mmHg et résistance vasculaire systémique de 2 000 dynes/m2. Le type de choc présenté par le patient est le suivant :", "full_answer": "Aucun autre détail sur la patiente n'a été donné, si ce n'est qu'elle était en état de choc (hypoperfusion tissulaire) et qu'en insérant un cathéter de Swan-Ganz, nous avons trouvé un index cardiaque bas (<2), une PCP élevée (25mmHg) et des résistances systémiques élevées (normales : 600-1200 dyne/m2). Ces données sont compatibles avec une origine cardiogénique du choc.", "type": "SOINS CRITIQUES ET D'URGENCE", "options": {"1": "Cardiogène.", "2": "Septique.", "3": "Hypovolémie.", "4": "Anaphylactique.", "5": null}, "correct_option": 1, "explanations": {"1": {"exist": true, "char_ranges": [[0, 372]], "word_ranges": [[0, 58]], "text": "Aucun autre détail sur la patiente n'a été donné, si ce n'est qu'elle était en état de choc (hypoperfusion tissulaire) et qu'en insérant un cathéter de Swan-Ganz, nous avons trouvé un index cardiaque bas (<2), une PCP élevée (25mmHg) et des résistances systémiques élevées (normales : 600-1200 dyne/m2). Ces données sont compatibles avec une origine cardiogénique du choc."}, "2": {"exist": false, "char_ranges": [], "word_ranges": [], "text": ""}, "3": {"exist": false, "char_ranges": [], "word_ranges": [], "text": ""}, "4": {"exist": false, "char_ranges": [], "word_ranges": [], "text": ""}, "5": {"exist": false, "char_ranges": [], "word_ranges": [], "text": ""}}} +{"id": 476, "year": 2020, "question_id_specific": 93, "full_question": "Un homme hypertendu de 58 ans se présente aux urgences pour une syncope de 30 secondes lors d'un jogging, avec récupération spontanée et sans séquelle. La tension artérielle est de 135/65 mmHg. L'auscultation cardiaque montre un souffle systolique fort et intense, réduit par les manœuvres de Valsalva et un second ton atténué. L'ECG montre un rythme sinusal à 72 bpm, avec des critères d'hypertrophie ventriculaire gauche et des ondes T inversées dans la face antérieure. Cochez la bonne réponse :", "full_answer": "Les données fournies sur les caractéristiques du souffle (il DIMINUE avec Valsalva) et les données sur l'hypertrophie ventriculaire gauche, nous conduisent au diagnostic de sténose aortique sévère.", "type": "SOINS INTENSIFS", "options": {"1": "Le tableau clinique suggère une thromboembolie pulmonaire.", "2": "Les données fournies indiquent une cardiomyopathie hypertrophique avec une obstruction sévère de la voie d'écoulement du ventricule gauche.", "3": "Ces données correspondent à une sténose aortique sévère.", "4": "La dissection aortique doit être exclue par une tomographie assistée par ordinateur avec contraste.", "5": null}, "correct_option": 3, "explanations": {"1": {"exist": false, "char_ranges": [], "word_ranges": [], "text": ""}, "2": {"exist": false, "char_ranges": [], "word_ranges": [], "text": ""}, "3": {"exist": true, "char_ranges": [[0, 197]], "word_ranges": [[0, 27]], "text": "Les données fournies sur les caractéristiques du souffle (il DIMINUE avec Valsalva) et les données sur l'hypertrophie ventriculaire gauche, nous conduisent au diagnostic de sténose aortique sévère."}, "4": {"exist": false, "char_ranges": [], "word_ranges": [], "text": ""}, "5": {"exist": false, "char_ranges": [], "word_ranges": [], "text": ""}}} +{"id": 546, "year": 2022, "question_id_specific": 124, "full_question": "Une femme de 82 ans souffrant de fibrillation auriculaire chronique depuis 10 ans et dont la fonction systolique biventriculaire est préservée présente des épisodes symptomatiques de bloc auriculo-ventriculaire complet, pour lesquels il est décidé d'implanter un système de stimulation cardiaque définitif. Quel est le traitement indiqué ?", "full_answer": "Stimulateur cardiaque VVI (ventriculaire à chambre unique).", "type": "CARDIOLOGIE", "options": {"1": "Stimulateur cardiaque DDD (bicaméral).", "2": "Stimulateur VVI (ventriculaire à chambre unique).", "3": "Stimulateur AAI (stimulateur auriculaire à chambre unique).", "4": "Thérapie de resynchronisation ventriculaire (CRT).", "5": null}, "correct_option": 2, "explanations": {"1": {"exist": false, "char_ranges": [], "word_ranges": [], "text": ""}, "2": {"exist": true, "char_ranges": [[0, 59]], "word_ranges": [[0, 7]], "text": "Stimulateur cardiaque VVI (ventriculaire à chambre unique)."}, "3": {"exist": false, "char_ranges": [], "word_ranges": [], "text": ""}, "4": {"exist": false, "char_ranges": [], "word_ranges": [], "text": ""}, "5": {"exist": false, "char_ranges": [], "word_ranges": [], "text": ""}}} +{"id": 313, "year": 2016, "question_id_specific": 177, "full_question": "Un adénocarcinome du côlon de stade T3N1M0 a été diagnostiqué chez un patient de 57 ans. Il a été traité par résection suivie d'une chimiothérapie postopératoire FOLFOX (fluorouracil, leucovorine et oxaliplatine) pendant 6 mois. Lors d'un examen de routine deux ans plus tard, on a constaté une élévation de l'antigène carcinoembryonnaire (ACE) à 30 ng/ml (auparavant moins de 2 ng/ml). Elle ne présente aucun symptôme, l'examen physique est sans particularité, la radiographie pulmonaire est normale. La tomodensitométrie montre une masse de 3 cm dans le lobe hépatique droit, qui est détectée par la TEP. Aucune autre anomalie n'est observée au scanner ou à la TEP. Quelle est, selon vous, l'approche la plus appropriée ?", "full_answer": "Nous sommes confrontés à une situation de métastase hépatique probable d'un adénocarcinome localisé du côlon traité il y a deux ans. Probable car nous n'avons pas de confirmation histologique, mais comme c'est souvent le cas, qu'est-ce que cela pourrait être d'autre ? Comme il s'agit d'une situation d'oligométastase et que les traitements de l'adénocarcinome avancé du côlon permettent une survie prolongée avec une bonne qualité de vie chez ces patients, si rien ne contre-indique la chirurgie, celle-ci devrait être la première option (bonne réponse 3). Je joins l'algorithme proposé par le NCCN dans les situations de récidive (http://www.nccn.org/professionals/physician_gls/pdf/colon.pdf). Dans le cas d'une situation non résécable, un traitement par chimiothérapie serait envisagé avec l'intention de convertir la lésion métastatique unique en une lésion résécable. Étant donné que plus de 12 mois se sont écoulés depuis le traitement par FOLFOX, le même schéma pourrait être utilisé, bien que FOLFIRI+bevacizumab soit également utile. La capécitabine en monothérapie, dans la situation actuelle du patient, ne serait pas non plus une bonne option de départ, car les réponses sont plus faibles par rapport au FOLFOX ou au FOLFIRI plus combinaisons antiangiogéniques. La radiothérapie hépatique pourrait être envisagée, car il existe des techniques de SBRT qui donnent de très bons résultats, mais dans ce cas, il est préférable de commencer par demander une évaluation par les chirurgiens, et si la chirurgie est exclue, la RT serait envisagée.", "type": "ONCOLOGIE", "options": {"1": "Chimiothérapie par FOLFIRI (fluorouracil, leucovorine et irinotécan) plus bévacizumab.", "2": "Monochimiothérapie avec la capécitabine.", "3": "Évaluation par résection chirurgicale de la lésion hépatique.", "4": "Radiothérapie hépatique.", "5": null}, "correct_option": 3, "explanations": {"1": {"exist": true, "char_ranges": [[697, 1043]], "word_ranges": [[98, 150]], "text": "Dans le cas d'une situation non résécable, un traitement par chimiothérapie serait envisagé avec l'intention de convertir la lésion métastatique unique en une lésion résécable. Étant donné que plus de 12 mois se sont écoulés depuis le traitement par FOLFOX, le même schéma pourrait être utilisé, bien que FOLFIRI+bevacizumab soit également utile."}, "2": {"exist": true, "char_ranges": [[1044, 1274]], "word_ranges": [[150, 186]], "text": "La capécitabine en monothérapie, dans la situation actuelle du patient, ne serait pas non plus une bonne option de départ, car les réponses sont plus faibles par rapport au FOLFOX ou au FOLFIRI plus combinaisons antiangiogéniques."}, "3": {"exist": true, "char_ranges": [[269, 557]], "word_ranges": [[42, 85]], "text": "Comme il s'agit d'une situation d'oligométastase et que les traitements de l'adénocarcinome avancé du côlon permettent une survie prolongée avec une bonne qualité de vie chez ces patients, si rien ne contre-indique la chirurgie, celle-ci devrait être la première option (bonne réponse 3)."}, "4": {"exist": true, "char_ranges": [[1275, 1552]], "word_ranges": [[186, 231]], "text": "La radiothérapie hépatique pourrait être envisagée, car il existe des techniques de SBRT qui donnent de très bons résultats, mais dans ce cas, il est préférable de commencer par demander une évaluation par les chirurgiens, et si la chirurgie est exclue, la RT serait envisagée."}, "5": {"exist": false, "char_ranges": [], "word_ranges": [], "text": ""}}} +{"id": 482, "year": 2020, "question_id_specific": 173, "full_question": "Un patient est amené au service des urgences à la suite d'un incendie dans un bâtiment fermé contenant de la mousse de polyuréthane. Il est conscient, mais présente un état mental de plus en plus morose, des céphalées et une dyspnée sévère. La saturation en oxygène par oxymétrie de pouls est de 92% et l'acide lactique capillaire est de 8 mEq/l. Quel traitement spécifique vous semble le plus approprié ?", "full_answer": "La première chose à savoir est que la combustion du polyuréthane à haute température entraîne la libération de cyanure (très intuitif, oui). Sachant cela et qu'il s'agit d'une intoxication au cyanure, il est plus facile de répondre à la question. Parmi les mesures de traitement initial de base, il y aurait l'administration d'une oxygénothérapie avec FiO2 1 (et non 0,5 comme indiqué dans l'option 1), et d'un antidote, dans ce cas l'hydroxycobalamine (cette question a été posée de manière similaire l'année dernière). La ventilation en caisson hyperbare ne serait indiquée qu'en cas d'intoxication coexistante au monoxyde de carbone (CO). Le point soulevé dans l'option 4 concernant la thérapie liquidienne n'est pas indiqué ; en cas de choc associé, le volume serait administré, mais dans tous les cas pas à ce rythme, mais en bolus rapides.", "type": "SOINS INTENSIFS", "options": {"1": "Administration d'oxygène au moyen d'un masque de type Venturi à 50 %.", "2": "Administration d'hydroxycobalamine par voie intraveineuse.", "3": "Ventilation du caisson hyperbare.", "4": "Fluidothérapie avec perfusion de solution saline physiologique à 21 ml/h.", "5": null}, "correct_option": 2, "explanations": {"1": {"exist": true, "char_ranges": [[247, 402]], "word_ranges": [[40, 65]], "text": "Parmi les mesures de traitement initial de base, il y aurait l'administration d'une oxygénothérapie avec FiO2 1 (et non 0,5 comme indiqué dans l'option 1),"}, "2": {"exist": true, "char_ranges": [[247, 452]], "word_ranges": [[40, 72]], "text": "Parmi les mesures de traitement initial de base, il y aurait l'administration d'une oxygénothérapie avec FiO2 1 (et non 0,5 comme indiqué dans l'option 1), et d'un antidote, dans ce cas l'hydroxycobalamine"}, "3": {"exist": true, "char_ranges": [[521, 641]], "word_ranges": [[82, 99]], "text": "La ventilation en caisson hyperbare ne serait indiquée qu'en cas d'intoxication coexistante au monoxyde de carbone (CO)."}, "4": {"exist": true, "char_ranges": [[642, 845]], "word_ranges": [[99, 135]], "text": "Le point soulevé dans l'option 4 concernant la thérapie liquidienne n'est pas indiqué ; en cas de choc associé, le volume serait administré, mais dans tous les cas pas à ce rythme, mais en bolus rapides."}, "5": {"exist": false, "char_ranges": [], "word_ranges": [], "text": ""}}} +{"id": 105, "year": 2012, "question_id_specific": 233, "full_question": "Les antimicrobiens suivants sont contre-indiqués chez une femme de 35 ans souffrant d'un syndrome dépressif sous inhibiteurs de la recapture de la sérotonine :", "full_answer": "Des cas spontanés de syndrome sérotoninergique associés à l'administration concomitante de linézolide et d'agents sérotoninergiques, y compris des antidépresseurs tels que les inhibiteurs sélectifs de la recapture de la sérotonine (ISRS), ont été rapportés.", "type": "PHARMACOLOGIE", "options": {"1": "Doxycycline.", "2": "Amoxicilline-acide clavulanique.", "3": "Daptomycine.", "4": "Linezolid.", "5": "Vancomycine."}, "correct_option": 4, "explanations": {"1": {"exist": false, "char_ranges": [], "word_ranges": [], "text": ""}, "2": {"exist": false, "char_ranges": [], "word_ranges": [], "text": ""}, "3": {"exist": false, "char_ranges": [], "word_ranges": [], "text": ""}, "4": {"exist": true, "char_ranges": [[0, 257]], "word_ranges": [[0, 34]], "text": "Des cas spontanés de syndrome sérotoninergique associés à l'administration concomitante de linézolide et d'agents sérotoninergiques, y compris des antidépresseurs tels que les inhibiteurs sélectifs de la recapture de la sérotonine (ISRS), ont été rapportés."}, "5": {"exist": false, "char_ranges": [], "word_ranges": [], "text": ""}}} +{"id": 73, "year": 2012, "question_id_specific": 62, "full_question": "Un homme de 37 ans, sans antécédents, s'est présenté au service des urgences pour une syncope en marchant. Il avait commencé deux jours plus tôt par une dyspnée d'effort. Examen : poids 75 kg, TA 75/50 mm Hg. SatO2 à 89%. Auscultation pulmonaire normale. Auscultation cardiaque : tachycardie rythmique à 130 bpm. Abdomen normal. Extrémités : pas d'altération. Hémogramme normal. Troponine 1,2 ng/ml. ECG : tachycardie sensuelle à 130 bpm. T inversé en V1-V4. Angiographie par tomodensitométrie thoracique : défaut de réplétion dans les deux artères pulmonaires principales. Une heure après son arrivée aux urgences, la tension artérielle reste à 70/55 mm Hg. Quel serait le traitement initial le plus approprié ?", "full_answer": "Le cas clinique est une thromboembolie pulmonaire typique. Dans ce cas, elle est associée à une hypotension artérielle persistante. L'option 2 serait correcte si le patient n'était pas hypotendu. L'option 3 n'est pas correcte car la confirmation du diagnostic n'est pas nécessaire pour initier un traitement anticoagulant en cas de MTEP sans hypotension. L'option 4 n'est pas indiquée. L'option 5 ne serait correcte que si le patient présentait une contre-indication à la fibrinolyse. L'hypotension persistante en cas d'ETEP est l'indication la plus largement acceptée pour la fibrinolyse, l'option 1 est donc correcte.", "type": "ANESTHÉSIOLOGIE, SOINS INTENSIFS ET MÉDECINE D'URGENCE", "options": {"1": "Héparine non fractionnée, 10 000 U iv en cas de suspicion clinique. Fibrinolyse par tPA 100 mg iv une fois le diagnostic confirmé.", "2": "Enoxaparine : 80 mg par voie sous-cutanée toutes les 12 heures, à partir d'une suspicion diagnostique.", "3": "Enoxaparine ; 80 mg par voie sous-cutanée toutes les 12 heures, à partir de la confirmation du diagnostic.", "4": "Fondaparinux : 7,5 mg par voie sous-cutanée par jour.", "5": "Thromboendartériectomie pulmonaire d'urgence."}, "correct_option": 1, "explanations": {"1": {"exist": true, "char_ranges": [[485, 619]], "word_ranges": [[73, 92]], "text": "L'hypotension persistante en cas d'ETEP est l'indication la plus largement acceptée pour la fibrinolyse, l'option 1 est donc correcte."}, "2": {"exist": true, "char_ranges": [[132, 195]], "word_ranges": [[19, 29]], "text": "L'option 2 serait correcte si le patient n'était pas hypotendu."}, "3": {"exist": true, "char_ranges": [[196, 354]], "word_ranges": [[29, 53]], "text": "L'option 3 n'est pas correcte car la confirmation du diagnostic n'est pas nécessaire pour initier un traitement anticoagulant en cas de MTEP sans hypotension."}, "4": {"exist": true, "char_ranges": [[355, 385]], "word_ranges": [[53, 58]], "text": "L'option 4 n'est pas indiquée."}, "5": {"exist": true, "char_ranges": [[386, 484]], "word_ranges": [[58, 73]], "text": "L'option 5 ne serait correcte que si le patient présentait une contre-indication à la fibrinolyse."}}} +{"id": 538, "year": 2021, "question_id_specific": 2, "full_question": "Une chute avec une fracture-luxation antérieure de l'épaule qui a été réduite aux urgences. Une écharpe a été mise en place et son retrait a été recommandé après 3 semaines. Lors du retrait de l'immobilisation, une zone de dysesthésie a été observée, circonscrite à la région latérale de l'épaule. La patiente est capable d'effectuer une abduction, mais elle n'atteint que 15 degrés. L'IRM montre que la coiffe des rotateurs n'est pas endommagée. Quel est le diagnostic le plus probable ?", "full_answer": "Dysesthésie de la face latérale de la région deltoïde de l'épaule (zone innervée par le nerf axillaire) et paralysie du deltoïde (muscle principal pour l'abduction au-delà de 15º). L'IRM montre que la coiffe des rotateurs n'est pas endommagée, ce qui exclut l'option 1 (qui montrerait une tendinopathie du teres major). La capsulite adhésive se manifeste par une douleur et une raideur de l'épaule, en particulier lors de la rotation externe active et passive. Le nerf suprascapulaire innerve le muscle supraspinatus, qui contribue aux 15 premiers degrés d'abduction de l'épaule (préservés dans la déclaration) et n'innerve pas la peau dans la zone réfléchie.", "type": "TRAUMATOLOGIE", "options": {"1": "Tendinopathie du teres major.", "2": "Capsulite adhésive.", "3": "Neuropathie du nerf suprascapulaire.", "4": "Lésion du nerf axillaire.", "5": null}, "correct_option": 4, "explanations": {"1": {"exist": true, "char_ranges": [[181, 319]], "word_ranges": [[28, 50]], "text": "L'IRM montre que la coiffe des rotateurs n'est pas endommagée, ce qui exclut l'option 1 (qui montrerait une tendinopathie du teres major)."}, "2": {"exist": true, "char_ranges": [[320, 460]], "word_ranges": [[50, 73]], "text": "La capsulite adhésive se manifeste par une douleur et une raideur de l'épaule, en particulier lors de la rotation externe active et passive."}, "3": {"exist": true, "char_ranges": [[461, 659]], "word_ranges": [[73, 102]], "text": "Le nerf suprascapulaire innerve le muscle supraspinatus, qui contribue aux 15 premiers degrés d'abduction de l'épaule (préservés dans la déclaration) et n'innerve pas la peau dans la zone réfléchie."}, "4": {"exist": true, "char_ranges": [[0, 180]], "word_ranges": [[0, 28]], "text": "Dysesthésie de la face latérale de la région deltoïde de l'épaule (zone innervée par le nerf axillaire) et paralysie du deltoïde (muscle principal pour l'abduction au-delà de 15º)."}, "5": {"exist": false, "char_ranges": [], "word_ranges": [], "text": ""}}} +{"id": 139, "year": 2012, "question_id_specific": 166, "full_question": "Un homme de 64 ans, fumeur et grand buveur, sans autres antécédents intéressants. Il a expliqué qu'il souffrait d'une odynodysphagie depuis 3 mois et d'une otalgie gauche. Il a fourni un rapport du dentiste qui a exclu une cause dentaire. Il mentionne également avoir été vu par différents spécialistes et a fourni un scanner de la tête et du cou (sans contraste iodé) qui a été jugé normal. L'examen a révélé une diminution de la capacité de propulsion de la langue ; la palpation de la base de la langue gauche a révélé une induration de consistance pierreuse d'environ 3 cm et la laryngoscopie indirecte n'a pas révélé d'ulcération de la muqueuse. La palpation cervicale ne révèle pas d'adénopathie. Quel est votre diagnostic présumé et la conduite à tenir ?", "full_answer": "On nous présente un patient présentant des facteurs de risque de cancer de l'oropharynx, mais dont l'examen et les tests sont normaux. Que se passe-t-il ici ? Cette question relève de la psychologie inversée : qu'est-ce que l'auteur de la question voulait que je sache ? Cette question a été posée par un oto-rhino-laryngologiste. Il vous dit déjà que le dentiste \"n'a rien vu\", donc il exclut 1. Il ne s'agit pas non plus d'un abcès de la base de la langue, car en trois mois, cela l'aurait tué : il ne s'agit pas non plus du 2. Il nous reste donc l'ankyloglossie, le cancer de la base de la langue et l'inflammation non spécifique. En cas de doute sur la possibilité d'un cancer, il ne faut pas hésiter à demander un nouveau scanner et une biopsie. Les cancers de la base de la langue sont très dangereux et c'est l'une des craintes de tout oto-rhino-laryngologiste lorsqu'un néoplasme est suspecté. Il n'est pas rare qu'ils apparaissent sans masse exophytique ni ulcération de la muqueuse de la langue et ils sont difficiles à détecter par tomodensitométrie en l'absence de contraste. La bonne réponse est 4 et je ne pense pas qu'elle puisse être contestée.", "type": "OTORHINOLARYNGOLOGIE ET CHIRURGIE MAXILLO-FACIALE", "options": {"1": "Cause dentaire / réévaluation par le dentiste.", "2": "Abcès de la base de la langue / débridement.", "3": "Ankyloglossie / libération chirurgicale de l'ankyloglossie.", "4": "Processus malin de l'oropharynx / répétition du scanner et de la biopsie.", "5": "Inflammation non spécifique / traitement aux corticostéroïdes."}, "correct_option": 4, "explanations": {"1": {"exist": true, "char_ranges": [[352, 396]], "word_ranges": [[59, 68]], "text": "le dentiste \"n'a rien vu\", donc il exclut 1."}, "2": {"exist": true, "char_ranges": [[397, 529]], "word_ranges": [[68, 98]], "text": "Il ne s'agit pas non plus d'un abcès de la base de la langue, car en trois mois, cela l'aurait tué : il ne s'agit pas non plus du 2."}, "3": {"exist": false, "char_ranges": [], "word_ranges": [], "text": ""}, "4": {"exist": true, "char_ranges": [[634, 750]], "word_ranges": [[115, 137]], "text": "En cas de doute sur la possibilité d'un cancer, il ne faut pas hésiter à demander un nouveau scanner et une biopsie."}, "5": {"exist": false, "char_ranges": [], "word_ranges": [], "text": ""}}} +{"id": 446, "year": 2018, "question_id_specific": 130, "full_question": "Un homme de 28 ans présente une hématurie macroscopique au service des urgences de l'hôpital. Parmi les anomalies suivantes de l'analyse d'urine, lesquelles permettraient de poser le diagnostic de glomérulonéphrite ?", "full_answer": "La glomérulonéphrite est typiquement associée à des érythrocytes dysmorphiques et à des coulées hématiques dans le sédiment (option 1 correcte). Une protéinurie variable peut également être observée, bien qu'elle ne se présente généralement pas sous la forme d'une microalbuminurie isolée (option 2 incorrecte), ainsi qu'une hématurie avec pyurie, bien que celle-ci ne soit pas caractéristique (option 3 incorrecte). Les caillots se produisent généralement dans le contexte de troubles des voies urinaires inférieures (option 4 incorrecte).", "type": "NEPHROLOGIE", "options": {"1": "Globules rouges dysmorphiques et/ou hématies.", "2": "Protéinurie de 1 g/jour, avec résultat négatif à la bandelette et microalbuminurie supérieure à 300 mg/24 heures.", "3": "Coexistence d'une hématurie avec pyurie sans bactériurie.", "4": "Caillots dans l'urine visibles à l'œil nu.", "5": null}, "correct_option": 1, "explanations": {"1": {"exist": true, "char_ranges": [[0, 144]], "word_ranges": [[0, 20]], "text": "La glomérulonéphrite est typiquement associée à des érythrocytes dysmorphiques et à des coulées hématiques dans le sédiment (option 1 correcte)."}, "2": {"exist": true, "char_ranges": [[145, 311]], "word_ranges": [[20, 43]], "text": "Une protéinurie variable peut également être observée, bien qu'elle ne se présente généralement pas sous la forme d'une microalbuminurie isolée (option 2 incorrecte),"}, "3": {"exist": true, "char_ranges": [[325, 416]], "word_ranges": [[45, 58]], "text": "hématurie avec pyurie, bien que celle-ci ne soit pas caractéristique (option 3 incorrecte)."}, "4": {"exist": true, "char_ranges": [[417, 540]], "word_ranges": [[58, 75]], "text": "Les caillots se produisent généralement dans le contexte de troubles des voies urinaires inférieures (option 4 incorrecte)."}, "5": {"exist": false, "char_ranges": [], "word_ranges": [], "text": ""}}} +{"id": 551, "year": 2022, "question_id_specific": 130, "full_question": "Un homme de 85 ans en fibrillation auriculaire permanente consulte pour une douleur soudaine et une froideur du pied droit depuis 6 heures. L'examen révèle une cyanose plantaire avec perte partielle de la sensibilité et de la mobilité. Le pouls fémoral est palpable, les pouls poplité et distal de cette extrémité sont absents. Laquelle des mesures suivantes est l'approche thérapeutique idéale ?", "full_answer": "Traitement chirurgical urgent par thromboembolectomie.", "type": "CARDIOLOGIE", "options": {"1": "Déconseiller la chirurgie de revascularisation en raison du temps d'ischémie prolongé.", "2": "Appliquez de la chaleur et prévenez le chirurgien vasculaire de garde.", "3": "Traitement chirurgical urgent par thromboembolectomie.", "4": "Traitement médical urgent avec des prostaglandines intraveineuses.", "5": null}, "correct_option": 3, "explanations": {"1": {"exist": false, "char_ranges": [], "word_ranges": [], "text": ""}, "2": {"exist": false, "char_ranges": [], "word_ranges": [], "text": ""}, "3": {"exist": true, "char_ranges": [[0, 54]], "word_ranges": [[0, 5]], "text": "Traitement chirurgical urgent par thromboembolectomie."}, "4": {"exist": false, "char_ranges": [], "word_ranges": [], "text": ""}, "5": {"exist": false, "char_ranges": [], "word_ranges": [], "text": ""}}} +{"id": 383, "year": 2016, "question_id_specific": 150, "full_question": "Un garçon de 12 mois qui, lors des examens de santé effectués depuis sa naissance, présente un testicule droit dans le canal inguinal qui ne peut être descendu dans le scrotum. Cochez la bonne réponse :", "full_answer": "Le testicule rétractile se caractérise par le fait que le testicule se trouve en permanence à l'extérieur de la poche scrotale, qui peut être abaissée manuellement (bien qu'il revienne automatiquement à son emplacement d'origine lorsqu'il est libéré). Ce phénomène est généralement dû à un pédicule court. Dans ce cas, on nous dit que le testicule est toujours dans le canal inguinal (cryptorchidie), ce qui permet d'écarter l'option 1. Le traitement hormonal par HCG est déjà tombé en désuétude, en raison de sa faible efficacité, de la possibilité de réascension et parce que des phénomènes apoptotiques et inflammatoires ont été décrits dans le testicule associé à son utilisation (nous écarterons l'option 3). L'orchidopexie est le traitement de choix de la cryptorchidie, et il est souhaitable de la réaliser entre 6 mois et un an, et ne devrait pas être différée au-delà de deux ans.", "type": "PÉDIATRIE", "options": {"1": "Le diagnostic le plus probable est celui d'un testicule rétractile.", "2": "Il faut attendre l'âge de deux ans pour que la descente spontanée se produise.", "3": "La gonadotrophine chorionique humaine est le traitement de premier choix.", "4": "L'indication d'une orchidopexie ne doit pas être différée.", "5": null}, "correct_option": 4, "explanations": {"1": {"exist": true, "char_ranges": [[306, 436]], "word_ranges": [[46, 68]], "text": "Dans ce cas, on nous dit que le testicule est toujours dans le canal inguinal (cryptorchidie), ce qui permet d'écarter l'option 1."}, "2": {"exist": false, "char_ranges": [], "word_ranges": [], "text": ""}, "3": {"exist": true, "char_ranges": [[437, 713]], "word_ranges": [[68, 111]], "text": "Le traitement hormonal par HCG est déjà tombé en désuétude, en raison de sa faible efficacité, de la possibilité de réascension et parce que des phénomènes apoptotiques et inflammatoires ont été décrits dans le testicule associé à son utilisation (nous écarterons l'option 3)."}, "4": {"exist": true, "char_ranges": [[714, 836]], "word_ranges": [[111, 133]], "text": "L'orchidopexie est le traitement de choix de la cryptorchidie, et il est souhaitable de la réaliser entre 6 mois et un an,"}, "5": {"exist": false, "char_ranges": [], "word_ranges": [], "text": ""}}} +{"id": 614, "year": 2022, "question_id_specific": 118, "full_question": "Un homme de 41 ans consulte pour des gonalgies depuis plusieurs jours. Lors de l'examen, le test de Thessaly (douleur lors des mouvements de rotation interne et externe, genou fléchi) a été réalisé et s'est révélé positif. Parmi les lésions suivantes, laquelle est la plus probable ?", "full_answer": "La manœuvre de Thessalie consiste pour le patient, en appui monopodal et en flexion du genou de 5º puis de 20º, à effectuer des rotations internes et externes de la jambe. Si cela reproduit la douleur rapportée par le patient, le test est considéré comme positif. Il s'agit d'un test utilisé pour le diagnostic de la pathologie méniscale (sensibilité supérieure à 90 % et spécificité supérieure à 95 %) (Réponse 1 correcte).", "type": "TRAUMATOLOGIE", "options": {"1": "Lésion méniscale.", "2": "Rupture du ligament croisé antérieur.", "3": "Rupture du ligament croisé postérieur.", "4": "Lésions dues à l'arthropathie dégénérative.", "5": null}, "correct_option": 1, "explanations": {"1": {"exist": true, "char_ranges": [[0, 424]], "word_ranges": [[0, 72]], "text": "La manœuvre de Thessalie consiste pour le patient, en appui monopodal et en flexion du genou de 5º puis de 20º, à effectuer des rotations internes et externes de la jambe. Si cela reproduit la douleur rapportée par le patient, le test est considéré comme positif. Il s'agit d'un test utilisé pour le diagnostic de la pathologie méniscale (sensibilité supérieure à 90 % et spécificité supérieure à 95 %) (Réponse 1 correcte)."}, "2": {"exist": false, "char_ranges": [], "word_ranges": [], "text": ""}, "3": {"exist": false, "char_ranges": [], "word_ranges": [], "text": ""}, "4": {"exist": false, "char_ranges": [], "word_ranges": [], "text": ""}, "5": {"exist": false, "char_ranges": [], "word_ranges": [], "text": ""}}} +{"id": 499, "year": 2020, "question_id_specific": 182, "full_question": "Une tumeur abdominale liée à la paroi gastrique a été détectée chez un homme de 60 ans souffrant de douleurs abdominales et de saignements gastro-intestinaux supérieurs. L'histologie montre une tumeur à cellules fusiformes avec peu de mitoses, positive pour le CD117. Le diagnostic le plus probable est le suivant :", "full_answer": "La tumeur stromale gastro-intestinale est mieux connue sous le nom de GIST, CD117, ckit, PDGF positif, manuel.", "type": "ONCOLOGIE MÉDICALE", "options": {"1": "Neurofibrome de la paroi gastrique.", "2": "Tumeur stromale gastro-intestinale.", "3": "Léiomyome.", "4": "Sarcome granulocytaire.", "5": null}, "correct_option": 2, "explanations": {"1": {"exist": false, "char_ranges": [], "word_ranges": [], "text": ""}, "2": {"exist": true, "char_ranges": [[0, 110]], "word_ranges": [[0, 17]], "text": "La tumeur stromale gastro-intestinale est mieux connue sous le nom de GIST, CD117, ckit, PDGF positif, manuel."}, "3": {"exist": false, "char_ranges": [], "word_ranges": [], "text": ""}, "4": {"exist": false, "char_ranges": [], "word_ranges": [], "text": ""}, "5": {"exist": false, "char_ranges": [], "word_ranges": [], "text": ""}}} +{"id": 613, "year": 2022, "question_id_specific": 118, "full_question": "Un homme de 41 ans consulte pour des gonalgies depuis plusieurs jours. Lors de l'examen, le test de Thessaly (douleur lors des mouvements de rotation interne et externe, genou fléchi) a été réalisé et s'est révélé positif. Parmi les lésions suivantes, laquelle est la plus probable ?", "full_answer": "Test de Thessalie. Test de provocation réalisé en cas de suspicion de lésion méniscale.", "type": "TRAUMATOLOGIE", "options": {"1": "Lésion méniscale.", "2": "Rupture du ligament croisé antérieur.", "3": "Rupture du ligament croisé postérieur.", "4": "Lésions dues à l'arthropathie dégénérative.", "5": null}, "correct_option": 1, "explanations": {"1": {"exist": true, "char_ranges": [[0, 87]], "word_ranges": [[0, 14]], "text": "Test de Thessalie. Test de provocation réalisé en cas de suspicion de lésion méniscale."}, "2": {"exist": false, "char_ranges": [], "word_ranges": [], "text": ""}, "3": {"exist": false, "char_ranges": [], "word_ranges": [], "text": ""}, "4": {"exist": false, "char_ranges": [], "word_ranges": [], "text": ""}, "5": {"exist": false, "char_ranges": [], "word_ranges": [], "text": ""}}} +{"id": 247, "year": 2014, "question_id_specific": 117, "full_question": "Un homme de 54 ans, diagnostiqué avec un diabète sucré de type 2, s'est présenté à notre clinique. Il a signalé une fièvre de 40 ºC accompagnée de frissons, d'arthromyalgie et de céphalées, pour lesquels il a commencé un traitement au paracétamol. Les symptômes sont apparus 24 heures avant son retour en Espagne. Trois jours plus tard, elle a présenté une éruption maculo-papulaire généralisée qui s'est transformée en pétéchies plus intenses sur les membres inférieurs. Les examens de laboratoire ont révélé une leucopénie à 3 200/mm3 et des plaquettes à 91 000/mm3, ainsi qu'une forte élévation des aminotransférases. La goutte de sang épaisse, le frottis de sang périphérique, la PCR et l'antigène du paludisme sont négatifs. Quel est le diagnostic le plus probable ?", "full_answer": "Il est frappant dans cette question que l'on nous dise que notre patient est retourné en Espagne, mais pas d'où (ce qui pourrait nous aider à faire des digressions sur la cause possible). Cependant, la description du tableau est très exhaustive et typique, ce qui rend la question facile. Il s'agit d'une maladie qui provoque de la fièvre, des arthralgies et un exanthème généralisé avec des pétéchies sur les membres inférieurs. Les analyses sanguines montrent une leucopénie accompagnée d'une plaquettopénie et d'une élévation des transaminases. Le paludisme a été exclu comme cause possible. Toutes les données sont très évocatrices de la dengue, en particulier la plaquettopénie et les pétéchies (évocatrices d'une fragilité capillaire), la réponse 3 est correcte.", "type": "LES MALADIES INFECTIEUSES", "options": {"1": "Infection à coronavirus.", "2": "Chorioméningite lymphocytaire.", "3": "Dengue.", "4": "Infection par le virus du chikungunya.", "5": "Encéphalite de Saint-Louis."}, "correct_option": 3, "explanations": {"1": {"exist": false, "char_ranges": [], "word_ranges": [], "text": ""}, "2": {"exist": false, "char_ranges": [], "word_ranges": [], "text": ""}, "3": {"exist": true, "char_ranges": [[596, 741]], "word_ranges": [[93, 113]], "text": "Toutes les données sont très évocatrices de la dengue, en particulier la plaquettopénie et les pétéchies (évocatrices d'une fragilité capillaire),"}, "4": {"exist": false, "char_ranges": [], "word_ranges": [], "text": ""}, "5": {"exist": false, "char_ranges": [], "word_ranges": [], "text": ""}}} +{"id": 88, "year": 2012, "question_id_specific": 56, "full_question": "Une femme de 78 ans, ayant des antécédents d'hypertension artérielle sous traitement à l'amlodipine, a été admise il y a trois mois en raison de l'apparition d'une insuffisance cardiaque et d'une fibrillation auriculaire. Elle est venue à votre cabinet pour un bilan cardiologique. Le rapport de sortie indique le résultat de l'échocardiographie, qui a montré un dysfonctionnement modéré du ventricule gauche, et le traitement à la sortie : l'amlodipine a été arrêtée et le furosémide et l'énalapril ont été mis en place. La patiente dit se sentir mieux, sans fatigue et avec moins d'œdèmes dans les jambes, mais elle n'est pas encore complètement rétablie. L'examen physique n'a révélé aucun signe d'insuffisance cardiaque, mais sa tension artérielle et sa fréquence cardiaque étaient respectivement de 150/90 mmHg et 120 bpm. L'ECG montre une fibrillation auriculaire à 110-120 bpm. Quel traitement est le plus susceptible d'améliorer les symptômes et de réduire la mortalité due aux événements cardiovasculaires ?", "full_answer": "La patiente est en insuffisance cardiaque avec une dysfonction ventriculaire modérée et présente également une fibrillation auriculaire ; elle a des critères d'anticoagulation, et entre les deux options qui incluent l'acénocoumarol, nous sommes enclins à choisir l'option 4, car elle inclut le carvédilol, un médicament dont il a été démontré qu'il augmentait la survie en cas d'insuffisance cardiaque. Le vérapamil (et les antagonistes du calcium en général) sont contre-indiqués dans l'insuffisance cardiaque systolique : ils n'augmentent pas la survie (ils augmentent même la morbidité et la mortalité) et, en particulier, les non-dihydropyridines (vérapamil et diltiazem) ne doivent pas être associées aux bêta-bloquants en raison du risque accru de blocage du système de conduction.", "type": "CARDIOLOGIE ET CHIRURGIE VASCULAIRE", "options": {"1": "Digoxine et acide acétylsalicylique.", "2": "Aténolol et acide acétylsalicylique.", "3": "Vérapamil et acénocoumarol.", "4": "Carvédilol et acénocoumarol.", "5": "Carvédilol et clopidogrel."}, "correct_option": 4, "explanations": {"1": {"exist": false, "char_ranges": [], "word_ranges": [], "text": ""}, "2": {"exist": false, "char_ranges": [], "word_ranges": [], "text": ""}, "3": {"exist": true, "char_ranges": [[403, 523]], "word_ranges": [[58, 74]], "text": "Le vérapamil (et les antagonistes du calcium en général) sont contre-indiqués dans l'insuffisance cardiaque systolique :"}, "4": {"exist": true, "char_ranges": [[138, 402]], "word_ranges": [[18, 58]], "text": "elle a des critères d'anticoagulation, et entre les deux options qui incluent l'acénocoumarol, nous sommes enclins à choisir l'option 4, car elle inclut le carvédilol, un médicament dont il a été démontré qu'il augmentait la survie en cas d'insuffisance cardiaque."}, "5": {"exist": false, "char_ranges": [], "word_ranges": [], "text": ""}}} +{"id": 536, "year": 2021, "question_id_specific": 122, "full_question": "Un homme de 75 ans, hypertendu sous traitement d'énalapril (20 mg/jour), consulte pour une détérioration de l'état général, des céphalées bitemporales et une claudication mandibulaire depuis 3 semaines. De plus, au cours des dernières heures, il a signalé deux épisodes d'amaurose fugace de l'œil gauche. L'examen physique a révélé un épaississement et une absence de pouls dans l'artère temporale gauche. Analyse : CRP 6 mg/dl (VN <1) ; hémoglobine 10,5 g/dl ; ESR 92 mm. La radiographie du thorax est normale. En tenant compte du diagnostic le plus probable, cochez l'option correcte pour le traitement :", "full_answer": "Cas clinique d'artérite à cellules géantes. La corticothérapie reste le premier choix (réponse 1 incorrecte), à administrer en pulsations (doses différentes selon la littérature, mais au moins 125 mg/j) en cas d'atteinte oculaire (réponse 3 incorrecte). Le traitement actuel de choix pour l'épargne corticoïde est le Tocilizumab (réponse 4 incorrecte), qui a également montré une réduction des récidives (réponse 2 correcte).", "type": "RHEUMATOLOGIE", "options": {"1": "Le rituximab doit être administré, les glucocorticoïdes ayant été relégués en deuxième intention en raison de leurs effets secondaires.", "2": "Le tocilizumab s'est avéré efficace pour réduire les rechutes et la dose cumulative de prednisone chez plus de 50 % des patients.", "3": "En présence de symptômes ischémiques, et afin de ne pas aggraver le risque vasculaire du patient, les glucocorticoïdes à des doses supérieures à 30 mg/jour doivent être évités.", "4": "Associé à des bolus de glucocorticoïdes de 1 g/jour, l'infliximab doit être utilisé en première intention pour l'induction de la rémission.", "5": null}, "correct_option": 2, "explanations": {"1": {"exist": true, "char_ranges": [[0, 109]], "word_ranges": [[0, 15]], "text": "Cas clinique d'artérite à cellules géantes. La corticothérapie reste le premier choix (réponse 1 incorrecte),"}, "2": {"exist": true, "char_ranges": [[353, 425]], "word_ranges": [[50, 61]], "text": "qui a également montré une réduction des récidives (réponse 2 correcte)."}, "3": {"exist": true, "char_ranges": [[110, 253]], "word_ranges": [[15, 36]], "text": "à administrer en pulsations (doses différentes selon la littérature, mais au moins 125 mg/j) en cas d'atteinte oculaire (réponse 3 incorrecte)."}, "4": {"exist": true, "char_ranges": [[254, 352]], "word_ranges": [[36, 50]], "text": "Le traitement actuel de choix pour l'épargne corticoïde est le Tocilizumab (réponse 4 incorrecte),"}, "5": {"exist": false, "char_ranges": [], "word_ranges": [], "text": ""}}} +{"id": 492, "year": 2020, "question_id_specific": 107, "full_question": "Un patient de 18 ans s'est présenté aux urgences pour une épistaxis qui durait depuis plusieurs jours, sans antécédents personnels ou familiaux intéressants. L'examen a révélé un état afébrile, de multiples ecchymoses et l'absence de splénomégalie palpable. Examens de laboratoire : leucocytes 7,2 x103/μL, Hb 12,3 g/dL, plaquettes 6,0 x103/μL. La thrombocytopénie a été confirmée au frottis, où l'on a observé une augmentation de la taille des plaquettes. Les études de coagulation et de biochimie sont normales. Quel est le diagnostic le plus probable ?", "full_answer": "Il s'agit d'un cas clinique facile avec seulement une thrombopénie clinique, des pétéchies et des épistaxis. Le point 1 est faux, il y aurait de la fièvre, des données d'hémolyse, des symptômes neurologiques, des schistocytes dans le frottis. En 2 la coagulation doit être altérée et dans ce cas elle n'est pas mentionnée.", "type": "HÉMATOLOGIE", "options": {"1": "Purpura thrombocytopénique thrombotique.", "2": "Coagulation intravasculaire disséminée.", "3": "Thrombocytopénie d'origine infectieuse.", "4": "Thrombocytopénie immunitaire primaire.", "5": null}, "correct_option": 4, "explanations": {"1": {"exist": true, "char_ranges": [[109, 242]], "word_ranges": [[16, 38]], "text": "Le point 1 est faux, il y aurait de la fièvre, des données d'hémolyse, des symptômes neurologiques, des schistocytes dans le frottis."}, "2": {"exist": true, "char_ranges": [[243, 322]], "word_ranges": [[38, 53]], "text": "En 2 la coagulation doit être altérée et dans ce cas elle n'est pas mentionnée."}, "3": {"exist": false, "char_ranges": [], "word_ranges": [], "text": ""}, "4": {"exist": false, "char_ranges": [], "word_ranges": [], "text": ""}, "5": {"exist": false, "char_ranges": [], "word_ranges": [], "text": ""}}} +{"id": 599, "year": 2022, "question_id_specific": 183, "full_question": "Une femme de 45 ans, sans antécédents particuliers, a consulté pour une dyspnée qui durait depuis environ 4 jours. L'examen a montré une tachycardie rythmique, sans souffle, et l'auscultation pulmonaire était normale. Les gaz du sang artériel montrent une pO2 de 70 mmHg et une pCO2 de 32 mmHg. L'hémogramme et les fonctions rénale et hépatique sont normaux. Le temps de prothrombine est de 90 %, le temps de céphaline activé (TCA) est de 2 par rapport au contrôle (N <1,2). Parmi les diagnostics suivants, lequel est le plus probable ?", "full_answer": "Dans ce cas clinique, nous sommes en présence d'une femme qui présente rapidement une dyspnée avec hypoxémie, un état compatible avec une thromboembolie pulmonaire. Le temps de prothrombine de 90 % et le TCA avec un rapport de 2 indiquent un problème de coagulation ; et les anticorps antiphospholipides peuvent être détectés par l'allongement des tests de coagulation dépendant des phospholipides, réponse correcte 1. D'autre part, bien que le facteur V Leiden ait été identifié comme une cause fréquente de thrombose familiale, il n'altèrerait pas le temps de coagulation. Dans le cas de l'hémophilie, il y aurait un problème de coagulation, mais ce n'est pas une cause fréquente de thromboembolie. Enfin, si la péricardite aiguë peut se manifester en 4 jours et par une tachycardie, les autres symptômes ou paramètres analytiques ne sont pas caractéristiques.", "type": "RHEUMATOLOGIE", "options": {"1": "Syndrome des antiphospholipides.", "2": "Facteur V de Leiden.", "3": "L'hémophilie.", "4": "Péricardite aiguë.", "5": null}, "correct_option": 1, "explanations": {"1": {"exist": true, "char_ranges": [[0, 418]], "word_ranges": [[0, 64]], "text": "Dans ce cas clinique, nous sommes en présence d'une femme qui présente rapidement une dyspnée avec hypoxémie, un état compatible avec une thromboembolie pulmonaire. Le temps de prothrombine de 90 % et le TCA avec un rapport de 2 indiquent un problème de coagulation ; et les anticorps antiphospholipides peuvent être détectés par l'allongement des tests de coagulation dépendant des phospholipides, réponse correcte 1."}, "2": {"exist": true, "char_ranges": [[419, 574]], "word_ranges": [[64, 89]], "text": "D'autre part, bien que le facteur V Leiden ait été identifié comme une cause fréquente de thrombose familiale, il n'altèrerait pas le temps de coagulation."}, "3": {"exist": true, "char_ranges": [[575, 700]], "word_ranges": [[89, 110]], "text": "Dans le cas de l'hémophilie, il y aurait un problème de coagulation, mais ce n'est pas une cause fréquente de thromboembolie."}, "4": {"exist": true, "char_ranges": [[701, 862]], "word_ranges": [[110, 135]], "text": "Enfin, si la péricardite aiguë peut se manifester en 4 jours et par une tachycardie, les autres symptômes ou paramètres analytiques ne sont pas caractéristiques."}, "5": {"exist": false, "char_ranges": [], "word_ranges": [], "text": ""}}} +{"id": 134, "year": 2012, "question_id_specific": 169, "full_question": "Un homme de 35 ans se présente aux urgences pour une douleur de l'œil droit présente depuis 3 jours. L'examen biomicroscopique du segment antérieur montre, après coloration à la fluorescéine, un ulcère cornéen central en forme de dendrite. Quel est le diagnostic ?", "full_answer": "L'ulcération dendritique de la cornée est typique de la kératite herpétique. Elle se manifeste par une douleur peu perceptible et une rougeur de l'œil minime ou inexistante. Certains ulcères ressemblant à des dendrites peuvent troubler le diagnostic. Ces pseudodendrites se retrouvent dans l'herpès zoster et dans les abrasions cornéennes en résolution. Bien qu'en pratique il faille prêter attention à la forme de la dendrite (la vraie dendrite est plus profonde et possède des bulbes terminaux à l'extrémité de chaque branche) et à l'évolution de la douleur (dans une pseudodendrite après une abrasion, la douleur devrait s'améliorer), il est plus simple de répondre à une question MIR : s'il s'agit d'une dendrite, la première chose à laquelle il faut penser est l'herpès. Les kératites fongiques et bactériennes présentent des infiltrats cornéens. La crise glaucomatocyclitique (syndrome de Posner Schlossman) est un type d'uvéite antérieure hypertensive idiopathique qui ne présente pas d'ulcération.", "type": "OPHTHALMOLOGIE", "options": {"1": "Kératite herpétique.", "2": "Abrasion de la cornée.", "3": "Kératite fongique.", "4": "Kératite bactérienne.", "5": "Crise glaucomatocyclique."}, "correct_option": 1, "explanations": {"1": {"exist": true, "char_ranges": [[0, 173]], "word_ranges": [[0, 27]], "text": "L'ulcération dendritique de la cornée est typique de la kératite herpétique. Elle se manifeste par une douleur peu perceptible et une rougeur de l'œil minime ou inexistante."}, "2": {"exist": true, "char_ranges": [[690, 775]], "word_ranges": [[107, 121]], "text": "s'il s'agit d'une dendrite, la première chose à laquelle il faut penser est l'herpès."}, "3": {"exist": true, "char_ranges": [[776, 851]], "word_ranges": [[121, 130]], "text": "Les kératites fongiques et bactériennes présentent des infiltrats cornéens."}, "4": {"exist": true, "char_ranges": [[776, 851]], "word_ranges": [[121, 130]], "text": "Les kératites fongiques et bactériennes présentent des infiltrats cornéens."}, "5": {"exist": true, "char_ranges": [[852, 1005]], "word_ranges": [[130, 149]], "text": "La crise glaucomatocyclitique (syndrome de Posner Schlossman) est un type d'uvéite antérieure hypertensive idiopathique qui ne présente pas d'ulcération."}}} +{"id": 245, "year": 2014, "question_id_specific": 115, "full_question": "Un patient sous chimiothérapie pour une leucémie est admis pour une pneumonie pour laquelle un traitement au céfépime a été prescrit. Une radiographie thoracique/CT scan montre un infiltrat avec un signe de halo et un ménisque en croissant. La lésion est périphérique et une ponction transthoracique est indiquée pour le prélèvement. Dans l'attente des résultats histologiques et microbiologiques définitifs, quel antimicrobien ajouteriez-vous au traitement ?", "full_answer": "Question à réponse directe. Il s'agit d'un patient sous chimiothérapie présentant des symptômes respiratoires et le \"signe du halo\" à la radiographie, très évocateur d'une aspergillose pulmonaire invasive. Le traitement était à base de voriconazole. L'aspergillus est résistant au fluconazole, qui n'est donc pas une option valable.", "type": "LES MALADIES INFECTIEUSES", "options": {"1": "Ganciclovir.", "2": "Caspofungine.", "3": "Fluconazole.", "4": "Pipéracilline-tazobactam.", "5": "Voriconazole."}, "correct_option": 5, "explanations": {"1": {"exist": false, "char_ranges": [], "word_ranges": [], "text": ""}, "2": {"exist": false, "char_ranges": [], "word_ranges": [], "text": ""}, "3": {"exist": true, "char_ranges": [[250, 332]], "word_ranges": [[35, 47]], "text": "L'aspergillus est résistant au fluconazole, qui n'est donc pas une option valable."}, "4": {"exist": false, "char_ranges": [], "word_ranges": [], "text": ""}, "5": {"exist": true, "char_ranges": [[28, 249]], "word_ranges": [[4, 35]], "text": "Il s'agit d'un patient sous chimiothérapie présentant des symptômes respiratoires et le \"signe du halo\" à la radiographie, très évocateur d'une aspergillose pulmonaire invasive. Le traitement était à base de voriconazole."}}} +{"id": 387, "year": 2016, "question_id_specific": 156, "full_question": "Le nourrisson de 5 mois a été nourri exclusivement au sein jusqu'à aujourd'hui, date à laquelle, en raison de son travail, on lui a donné un biberon de lait maternisé en guise d'entrée en matière et, il y a quelques jours, on lui a donné des céréales avec et sans gluten. Antécédents familiaux : mère de 28 ans souffrant d'asthme, père de 32 ans en bonne santé et frère de 5 ans souffrant de maladie cœliaque et de dermatite atopique. Antécédents personnels : grossesse sans histoire et accouchement par césarienne, la mère s'étant vu offrir un biberon de lait maternisé dès le premier jour de vie à la maternité. Depuis quelques jours, elle souffre de distension abdominale, de selles diarrhéiques, d'un refus d'alimentation, d'un érythème périorbitaire qui dure plus longtemps après le biberon et, depuis peu, de vomissements après le biberon, bien qu'elle ait bien toléré l'allaitement. Quel est le diagnostic le plus probable ?", "full_answer": "Enfant ayant des antécédents familiaux multiples d'allergie (asthme, atopie), qui a reçu du lait artificiel dans les premières heures de sa vie (sensibilisation aux protéines du lait de vache) et qui, coïncidant avec la prise de lait artificiel, développe des symptômes digestifs et un exanthème = APLV.", "type": "PÉDIATRIE", "options": {"1": "La maladie cœliaque.", "2": "Gastro-entérite aiguë.", "3": "Allergie aux protéines du lait de vache.", "4": "Allergie à la bouteille.", "5": null}, "correct_option": 3, "explanations": {"1": {"exist": false, "char_ranges": [], "word_ranges": [], "text": ""}, "2": {"exist": false, "char_ranges": [], "word_ranges": [], "text": ""}, "3": {"exist": true, "char_ranges": [[0, 303]], "word_ranges": [[0, 47]], "text": "Enfant ayant des antécédents familiaux multiples d'allergie (asthme, atopie), qui a reçu du lait artificiel dans les premières heures de sa vie (sensibilisation aux protéines du lait de vache) et qui, coïncidant avec la prise de lait artificiel, développe des symptômes digestifs et un exanthème = APLV."}, "4": {"exist": false, "char_ranges": [], "word_ranges": [], "text": ""}, "5": {"exist": false, "char_ranges": [], "word_ranges": [], "text": ""}}} +{"id": 41, "year": 2011, "question_id_specific": 139, "full_question": "Chez un patient en mauvais état général, fiévreux (température axillaire de 39,5°), présentant une tuméfaction sous-mandibulaire bilatérale évoluant depuis six jours, des douleurs buccales et un trismus, laquelle des affirmations suivantes est correcte ?", "full_answer": "Ce patient est dans un état grave. Le trismus et la fièvre suggèrent une infection de l'espace cervical profond et ce n'est pas un jeu car les voies respiratoires peuvent être obstruées. Sachant cela, les options 1, 2 et 5 sont hors de propos et il me reste 3 et 4. Il y a peut-être déjà une médiastinite ; après tout, c'est l'histoire naturelle des infections de l'espace cervical profond, mais la réponse 4 est plus correcte et de plus, nous sommes dans le bloc ORL, la spécialité des voies respiratoires. J'aime cette question parce qu'elle fait appel à des connaissances qui sont importantes dans la pratique clinique.", "type": "OTORHINOLARYNGOLOGIE ET CHIRURGIE MAXILLO-FACIALE", "options": {"1": "La première probabilité diagnostique est un carcinome du plancher buccal avec des métastases cervicales bilatérales.", "2": "La priorité thérapeutique est d'assurer la nutrition du patient.", "3": "Il s'agit très certainement d'une médiastinite.", "4": "Le risque d'obstruction des voies respiratoires doit être considéré comme une priorité.", "5": "En guise de premier diagnostic, une cytologie par aspiration à l'aiguille fine doit être effectuée."}, "correct_option": 4, "explanations": {"1": {"exist": true, "char_ranges": [[0, 242]], "word_ranges": [[0, 44]], "text": "Ce patient est dans un état grave. Le trismus et la fièvre suggèrent une infection de l'espace cervical profond et ce n'est pas un jeu car les voies respiratoires peuvent être obstruées. Sachant cela, les options 1, 2 et 5 sont hors de propos"}, "2": {"exist": true, "char_ranges": [[0, 242]], "word_ranges": [[0, 44]], "text": "Ce patient est dans un état grave. Le trismus et la fièvre suggèrent une infection de l'espace cervical profond et ce n'est pas un jeu car les voies respiratoires peuvent être obstruées. Sachant cela, les options 1, 2 et 5 sont hors de propos"}, "3": {"exist": false, "char_ranges": [], "word_ranges": [], "text": ""}, "4": {"exist": false, "char_ranges": [], "word_ranges": [], "text": ""}, "5": {"exist": true, "char_ranges": [[0, 242]], "word_ranges": [[0, 44]], "text": "Ce patient est dans un état grave. Le trismus et la fièvre suggèrent une infection de l'espace cervical profond et ce n'est pas un jeu car les voies respiratoires peuvent être obstruées. Sachant cela, les options 1, 2 et 5 sont hors de propos"}}} +{"id": 385, "year": 2016, "question_id_specific": 153, "full_question": "Quel est le diagnostic le plus probable chez un nouveau-né présentant une microcéphalie, un retard de croissance intra-utérin, une cardiopathie congénitale, un pied talus vertical et un faciès particulier (microphtalmie, petites fissures palpébrales, micrognathie et oreilles dysplasiques), des mains avec l'index et l'auriculaire au-dessus du majeur et de l'annulaire ?", "full_answer": "Il décrit fidèlement un syndrome d'Edwards. Bien que la maladie cardiaque et le trouble du pied qu'il décrit soient communs à plusieurs chromosomopathies, le trouble du doigt est très caractéristique du syndrome d'Edwards.", "type": "PÉDIATRIE", "options": {"1": "Trisomie 18 (syndrome d'Edwards).", "2": "Trisomie 13 (syndrome de Patau).", "3": "Trisomie 21 (syndrome de Down).", "4": "Trisomie 9.", "5": null}, "correct_option": 1, "explanations": {"1": {"exist": true, "char_ranges": [[44, 222]], "word_ranges": [[6, 33]], "text": "Bien que la maladie cardiaque et le trouble du pied qu'il décrit soient communs à plusieurs chromosomopathies, le trouble du doigt est très caractéristique du syndrome d'Edwards."}, "2": {"exist": true, "char_ranges": [[44, 222]], "word_ranges": [[6, 33]], "text": "Bien que la maladie cardiaque et le trouble du pied qu'il décrit soient communs à plusieurs chromosomopathies, le trouble du doigt est très caractéristique du syndrome d'Edwards."}, "3": {"exist": false, "char_ranges": [], "word_ranges": [], "text": ""}, "4": {"exist": true, "char_ranges": [[44, 222]], "word_ranges": [[6, 33]], "text": "Bien que la maladie cardiaque et le trouble du pied qu'il décrit soient communs à plusieurs chromosomopathies, le trouble du doigt est très caractéristique du syndrome d'Edwards."}, "5": {"exist": false, "char_ranges": [], "word_ranges": [], "text": ""}}} +{"id": 194, "year": 2013, "question_id_specific": 162, "full_question": "Une fillette de 3 ans, originaire du Bangladesh, a consulté pour des épisodes fébriles intermittents d'une durée de 3 semaines, associés à une faiblesse et à une perte d'appétit. L'examen physique révèle une splénomégalie marquée et une pâleur cutanéo-muqueuse. Examens complémentaires : hémoglobine 8,5 mg/dl, hématocrite 26%, MCV 86 fL, MCH 29 pg, leucocytes 2800/ml avec 300 neutrophiles/ml, plaquettes 54000/ml, GOT 85 U/l, GPT92 U/l et hypergammaglobulinémie polyclonale au protéinogramme sérique. Indiquer le diagnostic le plus probable avec les données disponibles à ce moment :", "full_answer": "Des données telles que la pancytopénie et l'hypergammaglobulinémie sont très caractéristiques du kala-azar. L'origine de l'enfant oriente également vers ce diagnostic, bien qu'il s'agisse d'une maladie qui se manifeste également dans la région méditerranéenne. Dans le diagnostic différentiel, il est important de garder à l'esprit la possibilité d'une leucose.", "type": "PÉDIATRIE", "options": {"1": "Leucémie lymphoblastique aiguë.", "2": "Lymphome de Burkitt.", "3": "Leishmaniose viscérale.", "4": "Tuberculose miliaire.", "5": "Paludisme chronique."}, "correct_option": 3, "explanations": {"1": {"exist": false, "char_ranges": [], "word_ranges": [], "text": ""}, "2": {"exist": false, "char_ranges": [], "word_ranges": [], "text": ""}, "3": {"exist": true, "char_ranges": [[0, 167]], "word_ranges": [[0, 21]], "text": "Des données telles que la pancytopénie et l'hypergammaglobulinémie sont très caractéristiques du kala-azar. L'origine de l'enfant oriente également vers ce diagnostic,"}, "4": {"exist": false, "char_ranges": [], "word_ranges": [], "text": ""}, "5": {"exist": false, "char_ranges": [], "word_ranges": [], "text": ""}}} +{"id": 371, "year": 2016, "question_id_specific": 126, "full_question": "Une femme de 59 ans ayant des antécédents personnels de cancer du sein, opéré il y a 8 mois, et d'ostéoporose, fait une chute et se fracture le col du fémur droit. Elle subit une intervention chirurgicale et est admise à l'hôpital où elle reste plâtrée pendant 10 jours. La semaine suivant sa sortie de l'hôpital, elle a soudainement ressenti un essoufflement, ce qui l'a amenée à consulter son médecin généraliste qui a remarqué que son mollet droit était œdémateux et douloureux à la palpation. Quels éléments cliniques ne seraient PAS importants pour déterminer qu'elle courait un risque clinique élevé de souffrir d'une thromboembolie pulmonaire ?", "full_answer": "Les autres antécédents et constatations sont très directement liés au risque accru de phénomènes thrombotiques. L'ostéoporose n'est pas un facteur de risque de thrombose.", "type": "PNEUMOLOGIE ET CHIRURGIE THORACIQUE", "options": {"1": "Antécédents de cancer du sein.", "2": "Antécédents d'ostéoporose.", "3": "Antécédents de repos pendant plus de 3 jours.", "4": "La présence d'un œdème unilatéral du mollet droit.", "5": null}, "correct_option": 2, "explanations": {"1": {"exist": true, "char_ranges": [[0, 111]], "word_ranges": [[0, 15]], "text": "Les autres antécédents et constatations sont très directement liés au risque accru de phénomènes thrombotiques."}, "2": {"exist": true, "char_ranges": [[112, 170]], "word_ranges": [[15, 24]], "text": "L'ostéoporose n'est pas un facteur de risque de thrombose."}, "3": {"exist": true, "char_ranges": [[0, 111]], "word_ranges": [[0, 15]], "text": "Les autres antécédents et constatations sont très directement liés au risque accru de phénomènes thrombotiques."}, "4": {"exist": true, "char_ranges": [[0, 111]], "word_ranges": [[0, 15]], "text": "Les autres antécédents et constatations sont très directement liés au risque accru de phénomènes thrombotiques."}, "5": {"exist": false, "char_ranges": [], "word_ranges": [], "text": ""}}} +{"id": 283, "year": 2016, "question_id_specific": 56, "full_question": "Un patient de 54 ans a été admis pour une fièvre de 38°C (38°C) au cours des cinq jours précédents et une dyspnée au repos (LV) apparue 6 heures avant son arrivée à l'hôpital. Aux urgences, l'examen est compatible avec une insuffisance cardiaque et l'ECG montre un bloc auriculo-ventriculaire complet avec une fréquence ventriculaire d'échappement de 45 bpm. Les signes d'insuffisance cardiaque sont réfractaires au traitement médical et l'échocardiographie transœsophagienne montre une valve aortique avec un orifice régurgitant effectif de 0,5 cm2_ Les cultures en série sont positives pour Streptococcus gallolyticus. Indiquez le meilleur plan d'action :", "full_answer": "Chirurgie cardiaque pour le remplacement de la valve aortique par une prothèse mécanique avec antibiothérapie selon l'antibiogramme.", "type": "CARDIOLOGIE ET CHIRURGIE VASCULAIRE", "options": {"1": "Chirurgie cardiaque pour le remplacement de la valve aortique par une prothèse mécanique avec antibiothérapie selon l'antibiogramme.", "2": "Antibiothérapie selon l'antibiogramme et implantation d'une contre-pulsion par ballonnet intra-aortique et d'un stimulateur cardiaque transitoire pendant 3 semaines, après quoi un stimulateur cardiaque permanent sera implanté.", "3": "Implantation d'un stimulateur cardiaque transitoire, antibiothérapie selon l'antibiogramme et implantation percutanée d'une prothèse valvulaire aortique.", "4": "Implantation urgente d'un stimulateur cardiaque permanent avec une antibiothérapie conforme à l'antibiogramme pendant 6 semaines.", "5": null}, "correct_option": 1, "explanations": {"1": {"exist": true, "char_ranges": [[0, 132]], "word_ranges": [[0, 17]], "text": "Chirurgie cardiaque pour le remplacement de la valve aortique par une prothèse mécanique avec antibiothérapie selon l'antibiogramme."}, "2": {"exist": false, "char_ranges": [], "word_ranges": [], "text": ""}, "3": {"exist": false, "char_ranges": [], "word_ranges": [], "text": ""}, "4": {"exist": false, "char_ranges": [], "word_ranges": [], "text": ""}, "5": {"exist": false, "char_ranges": [], "word_ranges": [], "text": ""}}} +{"id": 109, "year": 2012, "question_id_specific": 149, "full_question": "Une femme asymptomatique de 32 ans se présente pour un contrôle gynécologique car elle souhaite être enceinte. Lors de ce bilan, un myome utérin de 4 cm est détecté, en partie intramural et en partie sous-séreux, situé sur la face antérieure de l'utérus et ne déformant pas la cavité endométriale. Quelle attitude préconisez-vous ?", "full_answer": "La bonne réponse est 5. Un fibrome asymptomatique de cette taille qui ne déforme pas la cavité endométriale ne présente aucun risque de grossesse.", "type": "GYNÉCOLOGIE ET OBSTÉTRIQUE", "options": {"1": "Myomectomie par laparoscopie.", "2": "Myomectomie laparotomique.", "3": "Embolisation de myomes par cathétérisme artériel.", "4": "Traitement avec des analogues de la Gn-Rh pendant les trois mois précédant la tentative de grossesse.", "5": "Tentative de grossesse sans traitement préalable."}, "correct_option": 5, "explanations": {"1": {"exist": false, "char_ranges": [], "word_ranges": [], "text": ""}, "2": {"exist": false, "char_ranges": [], "word_ranges": [], "text": ""}, "3": {"exist": false, "char_ranges": [], "word_ranges": [], "text": ""}, "4": {"exist": false, "char_ranges": [], "word_ranges": [], "text": ""}, "5": {"exist": true, "char_ranges": [[24, 146]], "word_ranges": [[5, 24]], "text": "Un fibrome asymptomatique de cette taille qui ne déforme pas la cavité endométriale ne présente aucun risque de grossesse."}}} +{"id": 92, "year": 2012, "question_id_specific": 235, "full_question": "Patiente de 60 ans chez qui une néoplasie mammaire a été diagnostiquée il y a 10 ans. Elle a été traitée par radiochimiothérapie puis par hormonothérapie pendant 5 ans. Une scintigraphie osseuse réalisée pour une douleur osseuse polytopique a montré la présence de métastases osseuses. Elle est actuellement traitée par des opioïdes mineurs et des AINS avec un bon contrôle de la douleur. Elle a consulté pour des maux de tête qui ne disparaissaient pas avec l'analgésie actuelle et un scanner du cerveau a montré des images compatibles avec des métastases cérébrales. En ce qui concerne le traitement de la douleur, indiquez la réponse correcte :", "full_answer": "Les métastases cérébrales provoquent des douleurs dues à l'hypertension crânienne. Le traitement de choix est l'administration de corticostéroïdes. La dexaméthasone en doses orales de 4 à 16 mg/jour est le traitement de choix.", "type": "SOINS PALLIATIFS", "options": {"1": "Passer à des opioïdes plus puissants.", "2": "Des doses supplémentaires d'opioïdes doivent être administrées si nécessaire.", "3": "Des corticostéroïdes doivent être ajoutés.", "4": "Passer à un opioïde plus puissant et conserver les AINS.", "5": "Le patient doit être admis pour recevoir un traitement intraveineux avec un opioïde majeur."}, "correct_option": 3, "explanations": {"1": {"exist": false, "char_ranges": [], "word_ranges": [], "text": ""}, "2": {"exist": false, "char_ranges": [], "word_ranges": [], "text": ""}, "3": {"exist": true, "char_ranges": [[0, 147]], "word_ranges": [[0, 18]], "text": "Les métastases cérébrales provoquent des douleurs dues à l'hypertension crânienne. Le traitement de choix est l'administration de corticostéroïdes."}, "4": {"exist": false, "char_ranges": [], "word_ranges": [], "text": ""}, "5": {"exist": false, "char_ranges": [], "word_ranges": [], "text": ""}}} +{"id": 342, "year": 2016, "question_id_specific": 158, "full_question": "Une femme de 24 ans, primigeste, fait une fausse couche spontanée à 7 semaines de grossesse. L'étude anatomopathologique des restes avortés indique une maladie molaire. Nous devons l'informer que :", "full_answer": "La maladie trophoblastique gestationnelle doit faire l'objet d'un suivi (que l'évacuation soit complète ou non) et il faut conseiller à la patiente de ne pas retomber enceinte avant au moins 6 mois avec des titres de BHCG négatifs. Le risque de récidive lors d'une autre grossesse est faible (1/55) et 90 % des cas évoluent de manière satisfaisante sans développer de néoplasme.", "type": "GYNÉCOLOGIE ET OBSTÉTRIQUE", "options": {"1": "Le risque d'une nouvelle gestation molaire lors d'une prochaine grossesse est de 50 %.", "2": "Vous ne devez pas tomber enceinte avant d'avoir subi des examens réguliers et d'avoir passé un an avec des taux de BHCG négatifs.", "3": "Aucune surveillance supplémentaire n'est nécessaire si l'évacuation du tissu trophoblastique a été complète.", "4": "Des contrôles réguliers sont nécessaires car dans 40% des cas, une néoplasie trophoblastique gestationnelle se développe.", "5": null}, "correct_option": 2, "explanations": {"1": {"exist": false, "char_ranges": [], "word_ranges": [], "text": ""}, "2": {"exist": true, "char_ranges": [[0, 231]], "word_ranges": [[0, 38]], "text": "La maladie trophoblastique gestationnelle doit faire l'objet d'un suivi (que l'évacuation soit complète ou non) et il faut conseiller à la patiente de ne pas retomber enceinte avant au moins 6 mois avec des titres de BHCG négatifs."}, "3": {"exist": false, "char_ranges": [], "word_ranges": [], "text": ""}, "4": {"exist": false, "char_ranges": [], "word_ranges": [], "text": ""}, "5": {"exist": false, "char_ranges": [], "word_ranges": [], "text": ""}}} +{"id": 357, "year": 2016, "question_id_specific": 232, "full_question": "Une femme de 24 ans consulte après avoir constaté une lymphadénopathie inguinale. L'interrogatoire n'a révélé la présence d'aucune gêne locale ni d'éléments évocateurs d'une infection sexuellement transmissible. L'examen révèle deux adénopathies, une dans chaque aine, de 1 cm de diamètre, molles, mobiles, non douloureuses. Aucune lésion cutanée n'a été observée au niveau des membres inférieurs, de l'anus ou du périnée. Quel examen vous semble indispensable ?", "full_answer": "La présence de ganglions inguinaux d'une taille maximale de 1 à 1,5 cm peut être normale en l'absence de symptômes. Dans ce cas, il s'agit de ganglions très peu spécifiques, sans signes de malignité (mobiles, mous, non douloureux) qui ne permettent pas de suspecter une infiltration néoplasique et l'absence d'autres signes cliniques permet également d'exclure la présence de MST.", "type": "GYNÉCOLOGIE ET OBSTÉTRIQUE", "options": {"1": "Une sérologie pour la syphilis, car il s'agit très probablement d'une infection à Treponema pallidum.", "2": "Un examen gynécologique pour écarter la possibilité d'un cancer de l'ovaire.", "3": "D'après les caractéristiques cliniques, il semble que les ganglions lymphatiques soient normaux et qu'il n'y ait pas lieu de procéder à d'autres examens.", "4": "Un test de Paul-Bunell doit être effectué pour exclure une mononucléose infectieuse.", "5": null}, "correct_option": 3, "explanations": {"1": {"exist": false, "char_ranges": [], "word_ranges": [], "text": ""}, "2": {"exist": false, "char_ranges": [], "word_ranges": [], "text": ""}, "3": {"exist": true, "char_ranges": [[0, 380]], "word_ranges": [[0, 59]], "text": "La présence de ganglions inguinaux d'une taille maximale de 1 à 1,5 cm peut être normale en l'absence de symptômes. Dans ce cas, il s'agit de ganglions très peu spécifiques, sans signes de malignité (mobiles, mous, non douloureux) qui ne permettent pas de suspecter une infiltration néoplasique et l'absence d'autres signes cliniques permet également d'exclure la présence de MST."}, "4": {"exist": false, "char_ranges": [], "word_ranges": [], "text": ""}, "5": {"exist": false, "char_ranges": [], "word_ranges": [], "text": ""}}} +{"id": 596, "year": 2022, "question_id_specific": 121, "full_question": "Une femme de 79 ans est admise pour une fracture ostéoporotique de la hanche. En ce qui concerne la prévention secondaire des fractures de fragilité, cochez la MAUVAISE réponse :", "full_answer": "Cette question correspond à la section sur la traumatologie, mais on peut y répondre en connaissant la rhumatologie et les recommandations sur l'ostéoporose de la Société espagnole de rhumatologie. Dans l'ostéoporose, l'un des principaux risques associés à une augmentation du risque de fracture est la mauvaise observance du traitement, la réponse 1 est donc correcte. La réponse 2 se trouve dans les lignes directrices de la SER, qui confirment que certaines études concluent que les marqueurs du remodelage osseux peuvent être utiles pour le suivi précoce de l'observance et de la réponse au traitement. La réponse 4 est correcte car, toujours dans les lignes directrices du SER 2019, on cite : \"Les preuves scientifiques actuelles nous permettent d'affirmer que ni l'augmentation du calcium alimentaire ni la prise isolée de suppléments de calcium ne protègent contre l'apparition de fractures\". Par conséquent, la réponse correcte à cette question est l'option 3. Les patients sous traitement pharmacologique de la PO devraient utiliser des suppléments de calcium et de vitamine D car pratiquement tous les essais cliniques qui ont démontré l'efficacité des médicaments anti-ostéoporotiques incluent systématiquement des suppléments de calcium et du cholécalciférol (vitamine D3), mais pas en monothérapie.", "type": "RHEUMATOLOGIE", "options": {"1": "Une mauvaise observance du traitement est associée à un risque accru de fracture.", "2": "Les marqueurs du remodelage osseux peuvent être utiles pour le suivi précoce de la réponse au traitement.", "3": "La monothérapie à la vitamine D est efficace pour réduire ces fractures chez les personnes âgées non institutionnalisées.", "4": "L'augmentation du calcium alimentaire ou la prise de suppléments de calcium ne protègent pas à elles seules contre les fractures.", "5": null}, "correct_option": 3, "explanations": {"1": {"exist": true, "char_ranges": [[198, 369]], "word_ranges": [[29, 55]], "text": "Dans l'ostéoporose, l'un des principaux risques associés à une augmentation du risque de fracture est la mauvaise observance du traitement, la réponse 1 est donc correcte."}, "2": {"exist": true, "char_ranges": [[370, 606]], "word_ranges": [[55, 94]], "text": "La réponse 2 se trouve dans les lignes directrices de la SER, qui confirment que certaines études concluent que les marqueurs du remodelage osseux peuvent être utiles pour le suivi précoce de l'observance et de la réponse au traitement."}, "3": {"exist": true, "char_ranges": [[969, 1311]], "word_ranges": [[149, 195]], "text": "Les patients sous traitement pharmacologique de la PO devraient utiliser des suppléments de calcium et de vitamine D car pratiquement tous les essais cliniques qui ont démontré l'efficacité des médicaments anti-ostéoporotiques incluent systématiquement des suppléments de calcium et du cholécalciférol (vitamine D3), mais pas en monothérapie."}, "4": {"exist": true, "char_ranges": [[607, 899]], "word_ranges": [[94, 138]], "text": "La réponse 4 est correcte car, toujours dans les lignes directrices du SER 2019, on cite : \"Les preuves scientifiques actuelles nous permettent d'affirmer que ni l'augmentation du calcium alimentaire ni la prise isolée de suppléments de calcium ne protègent contre l'apparition de fractures\"."}, "5": {"exist": false, "char_ranges": [], "word_ranges": [], "text": ""}}} +{"id": 334, "year": 2016, "question_id_specific": 82, "full_question": "Une femme a été diagnostiquée avec un diabète sucré de type 1 il y a 24 ans. Elle se présente à la clinique avec 3 mois de fourmillements dans les deux pieds, avec une distribution en chaussettes, une douleur paresthésique et une sensation de pieds chauds, principalement la nuit, ce qui interfère significativement avec le sommeil. Lequel des médicaments suivants utiliseriez-vous comme traitement de première intention pour sa pathologie ?", "full_answer": "Duloxétine ; le tableau clinique décrit est celui d'une neuropathie diabétique. Parmi les médicaments proposés, le premier choix serait un double antidépresseur tel que la duloxétine.", "type": "PHARMACOLOGIE", "options": {"1": "Ibuprofène.", "2": "Oxycodone.", "3": "Duloxétine.", "4": "Paracétamol.", "5": null}, "correct_option": 3, "explanations": {"1": {"exist": false, "char_ranges": [], "word_ranges": [], "text": ""}, "2": {"exist": false, "char_ranges": [], "word_ranges": [], "text": ""}, "3": {"exist": true, "char_ranges": [[13, 183]], "word_ranges": [[2, 26]], "text": "le tableau clinique décrit est celui d'une neuropathie diabétique. Parmi les médicaments proposés, le premier choix serait un double antidépresseur tel que la duloxétine."}, "4": {"exist": false, "char_ranges": [], "word_ranges": [], "text": ""}, "5": {"exist": false, "char_ranges": [], "word_ranges": [], "text": ""}}} +{"id": 81, "year": 2012, "question_id_specific": 44, "full_question": "Un patient de 78 ans est atteint d'une cardiomyopathie dilatée idiopathique avec un léger dysfonctionnement du ventricule gauche (fraction d'éjection de 48 %) et une fibrillation auriculaire chronique. Parmi les médicaments suivants, lequel doit être évité dans le cadre de son traitement ?", "full_answer": "La digoxine est utile comme traitement symptomatique (c'est un inotrope positif) dans l'insuffisance cardiaque, bien qu'elle n'améliore pas la survie. Le carvédilol est un aß-bloquant très bien toléré (en raison de son effet anti-a1) qui est associé à une amélioration des symptômes et de la survie. L'acénocoumarol (Sintrom) serait indiqué chez ce patient souffrant de FA chronique afin d'éviter les événements thromboemboliques. L'énalapril et les inhibiteurs de l'ECA en général, en inhibant l'axe rénine-angiotensine-aldostérone, suppriment certains des effets neurohormonaux qui se produisent dans l'insuffisance cardiaque ; ils améliorent la survie des patients présentant une dysfonction systolique. L'ibuprofène et les AINS en général sont contre-indiqués chez les patients souffrant d'insuffisance cardiaque car ils inhibent la synthèse des prostaglandines au niveau rénal, entraînant une augmentation de la résistance vasculaire systémique, une réduction de la perfusion rénale et une inhibition de l'excrétion du sodium et de l'eau, ce qui peut précipiter la décompensation de l'insuffisance cardiaque.", "type": "CARDIOLOGIE ET CHIRURGIE VASCULAIRE", "options": {"1": "Digoxine.", "2": "Carvédilol.", "3": "Acénocoumarol.", "4": "Enalapril.", "5": "Ibuprofène."}, "correct_option": 5, "explanations": {"1": {"exist": true, "char_ranges": [[0, 150]], "word_ranges": [[0, 20]], "text": "La digoxine est utile comme traitement symptomatique (c'est un inotrope positif) dans l'insuffisance cardiaque, bien qu'elle n'améliore pas la survie."}, "2": {"exist": true, "char_ranges": [[151, 299]], "word_ranges": [[20, 46]], "text": "Le carvédilol est un aß-bloquant très bien toléré (en raison de son effet anti-a1) qui est associé à une amélioration des symptômes et de la survie."}, "3": {"exist": true, "char_ranges": [[300, 430]], "word_ranges": [[46, 62]], "text": "L'acénocoumarol (Sintrom) serait indiqué chez ce patient souffrant de FA chronique afin d'éviter les événements thromboemboliques."}, "4": {"exist": true, "char_ranges": [[431, 706]], "word_ranges": [[62, 96]], "text": "L'énalapril et les inhibiteurs de l'ECA en général, en inhibant l'axe rénine-angiotensine-aldostérone, suppriment certains des effets neurohormonaux qui se produisent dans l'insuffisance cardiaque ; ils améliorent la survie des patients présentant une dysfonction systolique."}, "5": {"exist": true, "char_ranges": [[707, 882]], "word_ranges": [[96, 120]], "text": "L'ibuprofène et les AINS en général sont contre-indiqués chez les patients souffrant d'insuffisance cardiaque car ils inhibent la synthèse des prostaglandines au niveau rénal,"}}} +{"id": 386, "year": 2016, "question_id_specific": 154, "full_question": "L'apparition d'une forte fièvre à 39°C chez un nourrisson de 10 mois, qui disparaît brusquement après 3 à 5 jours, suivie d'une éruption morbilliforme céphalocaudale avec un énanthème constitué de papules rougeâtres sur le palais, qui disparaît en une semaine, associée à un bon état général, est généralement due à.. :", "full_answer": "La question des maladies exanthémateuses dont les pédiatres et les candidats à l'examen MIR sont si friands. Les symptômes d'une forte fièvre durant plusieurs jours qui disparaît soudainement avec l'apparition d'une éruption cutanée sont l'exanthème subit, causé par le virus de l'herpès de type 6 (rappelez-vous la règle mnémotechnique de \"l'hexanthème subit\"). Le parvovirus B19 est à l'origine de l'érythème infectieux ou \"cinquième maladie\", dont l'éruption caractéristique se situe sur les joues, donnant l'aspect d'un visage giflé. La coxackie A16 provoque l'herpangine et aussi la maladie mains-pieds-bouche, qui présente une éruption d'abord maculaire, qui ne respecte pas les paumes et les plantes et évolue vers des vésicules qui éclatent. Enfin, l'infection par l'EBV provoque un syndrome mononucléosique avec une forte fièvre durant plusieurs jours, un érythème pharyngé et des exsudats grisâtres sur les amygdales. Si des pénicillines sont administrées pour la traiter (confusion avec une pharyngotonsillite streptococcique), elle peut se manifester par une éruption cutanée généralisée. En outre, elle se manifeste également par de la fatigue, une inflammation du foie avec augmentation des transaminases et une splénomégalie.", "type": "PÉDIATRIE", "options": {"1": "Parvovirus B19.", "2": "Virus de l'herpès de type 6.", "3": "Coxackievirus A16.", "4": "Pré-infection par le virus d'Epstein-Barr.", "5": null}, "correct_option": 2, "explanations": {"1": {"exist": true, "char_ranges": [[363, 537]], "word_ranges": [[52, 77]], "text": "Le parvovirus B19 est à l'origine de l'érythème infectieux ou \"cinquième maladie\", dont l'éruption caractéristique se situe sur les joues, donnant l'aspect d'un visage giflé."}, "2": {"exist": true, "char_ranges": [[109, 362]], "word_ranges": [[17, 52]], "text": "Les symptômes d'une forte fièvre durant plusieurs jours qui disparaît soudainement avec l'apparition d'une éruption cutanée sont l'exanthème subit, causé par le virus de l'herpès de type 6 (rappelez-vous la règle mnémotechnique de \"l'hexanthème subit\")."}, "3": {"exist": true, "char_ranges": [[538, 749]], "word_ranges": [[77, 109]], "text": "La coxackie A16 provoque l'herpangine et aussi la maladie mains-pieds-bouche, qui présente une éruption d'abord maculaire, qui ne respecte pas les paumes et les plantes et évolue vers des vésicules qui éclatent."}, "4": {"exist": true, "char_ranges": [[750, 927]], "word_ranges": [[109, 134]], "text": "Enfin, l'infection par l'EBV provoque un syndrome mononucléosique avec une forte fièvre durant plusieurs jours, un érythème pharyngé et des exsudats grisâtres sur les amygdales."}, "5": {"exist": false, "char_ranges": [], "word_ranges": [], "text": ""}}} +{"id": 239, "year": 2014, "question_id_specific": 146, "full_question": "Un homme de 80 ans a été admis en raison de l'apparition soudaine d'une aphasie et d'une hémiparésie droite. Ses antécédents comprennent une hypertension, bien contrôlée par un régime alimentaire, et une détérioration cognitive au cours de la dernière année, étudiée par son neurologue. Le scanner crânien réalisé en urgence montre un hématome lobaire frontal gauche sans prise de contraste. Quelle est la cause la plus probable de l'hématome ?", "full_answer": "L'angiopathie amyloïde est la cause la plus fréquente d'hémorragie spontanée non hypertensive chez les patients âgés, et est généralement localisée au niveau lobaire (comme dans le cas présent). Elle est souvent associée à la maladie d'Alzheimer (Manuel CTO). La réponse la plus courante qui peut soulever le plus de doutes est la réponse 2, hypertension, mais les localisations les plus fréquentes sont : le putamen, le thalamus, le pons et le cervelet. Il s'agit donc le plus souvent d'hémorragies profondes (Manuel CTO). De plus, en tant qu'indice, cela nous indique que le patient souffre d'une hypertension bien contrôlée sans médicaments, bien que cette donnée ne permette pas d'exclure la réponse. Réponse 1, une malformation est typique des jeunes patients, rare chez les personnes âgées. Elle diminue également le risque de ne pas trouver une malformation avec un scanner avec contraste. La réponse 4, une tumeur cérébrale, ne semble pas probable car la masse n'est pas visible au scanner et, comme dans le cas précédent, il est plus difficile de ne pas trouver une telle lésion après l'administration d'un produit de contraste. La réponse 3 ne semble pas correcte car la vascularite est une entité rare et plus fréquente chez les personnes âgées.", "type": "NEUROLOGIE", "options": {"1": "Malformation artérioveineuse masquée par un hématome aigu.", "2": "L'hypertension artérielle chronique.", "3": "Vascularite isolée du système nerveux.", "4": "Tumeur cérébrale.", "5": "Angiopathie amyloïde cérébrale (angiopathie congophile)."}, "correct_option": 5, "explanations": {"1": {"exist": true, "char_ranges": [[705, 796]], "word_ranges": [[110, 124]], "text": "Réponse 1, une malformation est typique des jeunes patients, rare chez les personnes âgées."}, "2": {"exist": true, "char_ranges": [[361, 523]], "word_ranges": [[56, 82]], "text": "les localisations les plus fréquentes sont : le putamen, le thalamus, le pons et le cervelet. Il s'agit donc le plus souvent d'hémorragies profondes (Manuel CTO)."}, "3": {"exist": true, "char_ranges": [[1138, 1256]], "word_ranges": [[181, 202]], "text": "La réponse 3 ne semble pas correcte car la vascularite est une entité rare et plus fréquente chez les personnes âgées."}, "4": {"exist": true, "char_ranges": [[897, 1137]], "word_ranges": [[140, 181]], "text": "La réponse 4, une tumeur cérébrale, ne semble pas probable car la masse n'est pas visible au scanner et, comme dans le cas précédent, il est plus difficile de ne pas trouver une telle lésion après l'administration d'un produit de contraste."}, "5": {"exist": true, "char_ranges": [[0, 259]], "word_ranges": [[0, 38]], "text": "L'angiopathie amyloïde est la cause la plus fréquente d'hémorragie spontanée non hypertensive chez les patients âgés, et est généralement localisée au niveau lobaire (comme dans le cas présent). Elle est souvent associée à la maladie d'Alzheimer (Manuel CTO)."}}} +{"id": 525, "year": 2021, "question_id_specific": 53, "full_question": "Une femme de 42 ans consulte son médecin de famille pour des gonalgies. La consultation est l'occasion d'évaluer les modes de vie, en particulier le tabagisme. Si vous souhaitez suivre la stratégie d'éducation à la santé basée sur le modèle des cinq \"A\", lequel des éléments suivants n'est PAS inclus dans cette stratégie ?", "full_answer": "Les 5A résument les activités qui devraient être menées par le personnel de santé au cours de l'intervention brève auprès du patient pour l'aider à arrêter de fumer, à savoir : s'informer, conseiller, analyser, aider, accompagner ou accepter. Par conséquent, l'option qui n'est pas incluse dans la stratégie serait l'option 2 : augmenter.", "type": "MÉDECINE PRÉVENTIVE", "options": {"1": "S'informer : poser des questions sur les facteurs de risque et les comportements à risque (demander à la patiente si elle fume).", "2": "Augmenter : accroître la perception du risque pour faciliter le changement (expliquer les conséquences du tabagisme).", "3": "Conseils : donner des conseils clairs, spécifiques et personnalisés (conseils pour arrêter de fumer).", "4": "Accepter : Convenir ensemble des objectifs de changement (évaluer l'état de préparation à une tentative d'arrêt du tabac).", "5": null}, "correct_option": 2, "explanations": {"1": {"exist": false, "char_ranges": [], "word_ranges": [], "text": ""}, "2": {"exist": true, "char_ranges": [[0, 338]], "word_ranges": [[0, 53]], "text": "Les 5A résument les activités qui devraient être menées par le personnel de santé au cours de l'intervention brève auprès du patient pour l'aider à arrêter de fumer, à savoir : s'informer, conseiller, analyser, aider, accompagner ou accepter. Par conséquent, l'option qui n'est pas incluse dans la stratégie serait l'option 2 : augmenter."}, "3": {"exist": false, "char_ranges": [], "word_ranges": [], "text": ""}, "4": {"exist": false, "char_ranges": [], "word_ranges": [], "text": ""}, "5": {"exist": false, "char_ranges": [], "word_ranges": [], "text": ""}}} +{"id": 287, "year": 2016, "question_id_specific": 64, "full_question": "Un patient de 65 ans s'est présenté avec une dyspnée progressive évoluant depuis 5 jours jusqu'au repos, une orthopnée à trois oreillers et des épisodes de dyspnée nocturne paroxystique. L'auscultation a révélé des crépitants bilatéraux, un souffle holosystolique irradiant vers l'aisselle et un rythme de galop sur les troisième et quatrième sons. Cochez la bonne réponse :", "full_answer": "Le quatrième bruit survient généralement lorsqu'il y a un certain degré de sténose de la valve.", "type": "CARDIOLOGIE ET CHIRURGIE VASCULAIRE", "options": {"1": "Le troisième bruit coïncide avec la phase de remplissage rapide de la diastole ventriculaire du cycle cardiaque.", "2": "Le diagnostic le plus probable est l'insuffisance cardiaque.", "3": "Le souffle holosystolique peut correspondre à une régurgitation mitrale.", "4": "Le quatrième bruit se produit généralement lorsqu'il y a un certain degré de sténose de la valve.", "5": null}, "correct_option": 4, "explanations": {"1": {"exist": false, "char_ranges": [], "word_ranges": [], "text": ""}, "2": {"exist": false, "char_ranges": [], "word_ranges": [], "text": ""}, "3": {"exist": false, "char_ranges": [], "word_ranges": [], "text": ""}, "4": {"exist": true, "char_ranges": [[0, 95]], "word_ranges": [[0, 16]], "text": "Le quatrième bruit survient généralement lorsqu'il y a un certain degré de sténose de la valve."}, "5": {"exist": false, "char_ranges": [], "word_ranges": [], "text": ""}}} +{"id": 472, "year": 2020, "question_id_specific": 140, "full_question": "Femme obèse de 65 ans qui est tombée sur sa main avec le coude en extension. Elle présente une douleur au bras avec gonflement, une impotence fonctionnelle du bras et une incapacité à étendre le poignet et les doigts. Elle souffre très probablement de :", "full_answer": "Toutes les données cliniques fournies doivent être prises en compte : après un traumatisme, nous avons une douleur dans le bras avec une atteinte des tissus mous et l'impossibilité d'étendre le poignet et les doigts. La seule option qui inclut toutes les données est l'option 4. L'option 1 n'explique pas l'atteinte du poignet et des doigts. La luxation du coude n'explique pas le manque de mobilité des doigts ainsi que la fracture diaphysaire de l'humérus et du double avant-bras. \"Le mécanisme direct est possible, de même que la transmission indirecte des forces du coude et de la main... l'atteinte du nerf radial est extrêmement fréquente, atteignant une incidence de 10 à 18% des cas\". La fracture-luxation de l'humérus proximal aurait une plus grande implication au niveau de l'épaule, la luxation du coude au niveau du coude, et il n'y a pas de données suggérant une double fracture de l'avant-bras.", "type": "CHIRURGIE ORTHOPÉDIQUE ET TRAUMATOLOGIE", "options": {"1": "Fracture luxée de l'humérus proximal.", "2": "Luxation du coude.", "3": "Fracture diaphysaire de l'humérus associée à une double fracture de l'avant-bras.", "4": "Fracture diaphysaire de l'humérus avec lésion du nerf radial.", "5": null}, "correct_option": 4, "explanations": {"1": {"exist": true, "char_ranges": [[279, 341]], "word_ranges": [[46, 56]], "text": "L'option 1 n'explique pas l'atteinte du poignet et des doigts."}, "2": {"exist": true, "char_ranges": [[342, 482]], "word_ranges": [[56, 79]], "text": "La luxation du coude n'explique pas le manque de mobilité des doigts ainsi que la fracture diaphysaire de l'humérus et du double avant-bras."}, "3": {"exist": true, "char_ranges": [[693, 908]], "word_ranges": [[114, 149]], "text": "La fracture-luxation de l'humérus proximal aurait une plus grande implication au niveau de l'épaule, la luxation du coude au niveau du coude, et il n'y a pas de données suggérant une double fracture de l'avant-bras."}, "4": {"exist": true, "char_ranges": [[70, 278]], "word_ranges": [[11, 46]], "text": "après un traumatisme, nous avons une douleur dans le bras avec une atteinte des tissus mous et l'impossibilité d'étendre le poignet et les doigts. La seule option qui inclut toutes les données est l'option 4."}, "5": {"exist": false, "char_ranges": [], "word_ranges": [], "text": ""}}} +{"id": 173, "year": 2013, "question_id_specific": 51, "full_question": "Une femme de 53 ans souffrant d'asthme bronchique s'est présentée à la clinique d'allergologie pour un bilan de santé. Elle a signalé des exacerbations répétées avec des symptômes nocturnes, une utilisation fréquente de médicaments de secours et une dyspnée à l'effort lors de la marche sur terrain plat. La spirométrie montre un rapport VEMS/CVF pré-bronchodilatateur de 60 % et un VEMS de 55 %. Les tests cutanés sont positifs pour les acariens et les IgE totales sont de 150 UI/ml. Le patient est traité par une combinaison de salmétérol/budésonide à forte dose (50/500 mg : 2 bouffées deux fois par jour), de prednisone orale d'entretien (10 mg/jour) et de théophylline. Parmi les options suivantes, quelle est l'approche thérapeutique la plus recommandable ?", "full_answer": "L'indication de l'omalizumab chez un patient comme celui dont il est question est une indication correcte, bien que dans le cas d'un asthme sévère difficile à contrôler, il existe d'autres mesures préalables qui ne sont pas envisagées et qui doivent toujours être prises en compte avant de recourir à ce traitement. Il est essentiel de s'assurer, premièrement, qu'il s'agit bien d'un asthme et non d'une autre maladie qui simule l'asthme, deuxièmement, qu'il n'y a pas de complications qui aggravent l'évolution, comme la présence d'un reflux gastro-œsophagien, de bronchectasies, la présence de germes opportunistes étant donné le traitement chronique aux stéroïdes, etc. et troisièmement, il est nécessaire de confirmer que le patient suit correctement les traitements, surtout le traitement inhalé, etc. Une fois que tous ces éléments ont été vérifiés, il est temps de commencer un traitement d'essai avec l'omalizumab.", "type": "PNEUMOLOGIE", "options": {"1": "Ajouter du sulfate de magnésium.", "2": "Augmenter la dose de prednisone à 30 mg/jour.", "3": "Ajouter l'omalizumab.", "4": "Prescrire un traitement par nébulisation à domicile.", "5": "Passer à une combinaison à forte dose de budésonide et de formotérol."}, "correct_option": 3, "explanations": {"1": {"exist": false, "char_ranges": [], "word_ranges": [], "text": ""}, "2": {"exist": false, "char_ranges": [], "word_ranges": [], "text": ""}, "3": {"exist": true, "char_ranges": [[0, 106]], "word_ranges": [[0, 16]], "text": "L'indication de l'omalizumab chez un patient comme celui dont il est question est une indication correcte,"}, "4": {"exist": false, "char_ranges": [], "word_ranges": [], "text": ""}, "5": {"exist": false, "char_ranges": [], "word_ranges": [], "text": ""}}} +{"id": 480, "year": 2020, "question_id_specific": 171, "full_question": "Une femme de 67 ans ayant des antécédents de dyslipidémie se présente aux urgences avec une dysurie et une polyurie suivies de fièvre, de frissons et d'une détérioration de l'état général. À son arrivée, elle semble grave, tachycarde, tachypnéique, avec une pression artérielle de 60/40 mmHg et une température de 39°C. Laquelle des mesures suivantes ne ferait PAS partie de la prise en charge INITIALE ?", "full_answer": "Parmi les options présentées, la seule mesure qui ne figure pas dans les lignes directrices de la campagne Surviving Sepsis sur la prise en charge du sepsis et du choc septique (comme le patient auquel nous avons affaire) est l'administration de dobutamine par voie intraveineuse. La procédure appropriée consisterait en l'administration de liquides, la réalisation d'hémocultures, la mesure du lactate sérique et l'administration précoce d'une antibiothérapie à large spectre ciblée sur le foyer septique suspecté. Si la pression artérielle moyenne nécessaire à une perfusion adéquate des organes n'est pas obtenue, il convient de mettre en place une perfusion de noradrénaline, comme indiqué à la question 51.", "type": "SOINS INTENSIFS", "options": {"1": "Perfusion intraveineuse de dobutamine.", "2": "Mesure du lactate sérique.", "3": "Extraction d'hémocultures.", "4": "Gestion des fluides.", "5": null}, "correct_option": 1, "explanations": {"1": {"exist": true, "char_ranges": [[0, 280]], "word_ranges": [[0, 45]], "text": "Parmi les options présentées, la seule mesure qui ne figure pas dans les lignes directrices de la campagne Surviving Sepsis sur la prise en charge du sepsis et du choc septique (comme le patient auquel nous avons affaire) est l'administration de dobutamine par voie intraveineuse."}, "2": {"exist": false, "char_ranges": [], "word_ranges": [], "text": ""}, "3": {"exist": false, "char_ranges": [], "word_ranges": [], "text": ""}, "4": {"exist": false, "char_ranges": [], "word_ranges": [], "text": ""}, "5": {"exist": false, "char_ranges": [], "word_ranges": [], "text": ""}}} +{"id": 58, "year": 2011, "question_id_specific": 199, "full_question": "Une femme de 31 ans (consultante), asymptomatique, s'est présentée avec une grossesse de 10 semaines selon l'échographie (primipare). Son jeune frère (cas index) âgé de 26 ans est atteint d'ataxie et diagnostiqué génétiquement comme porteur hétérozygote d'une mutation expansive de 70 répétitions CAG dans le gène SCA3 (chromosome 14). La patiente est orientée vers un conseil génétique qui évalue la possibilité d'une biopsie des villosités choriales pour étudier le génotype du fœtus. Cette procédure invasive est-elle indiquée comme diagnostic prénatal (DPN) dans ce cas ?", "full_answer": "La SCA3 est l'ataxie spinocérébelleuse de type 3, également appelée maladie de Machado-Joseph. Elle est causée par des mutations dans le gène ATXN3 par le mécanisme d'expansion des répétitions triplets CAG. Les individus atteints présentent entre 52 et 86 répétitions de triplets CAG en hétérozygotie avec une pénétrance complète. Le porteur hétérozygote de la mutation est atteint de la maladie, dont le mode de transmission est autosomique dominant. Dans ce cas clinique, le porteur de la maladie et du gène muté en hétérozygotie est le frère (26 ans) de la femme enceinte. Je pense qu'avant de procéder à une biopsie du chorion, la femme enceinte devrait être testée pour connaître son statut vis-à-vis du gène ATXN3. Elle a 50% de chances d'être porteuse de la mutation et d'avoir la maladie, mais comme elle a 31 ans, je pense qu'elle aurait déjà dû avoir des symptômes. Néanmoins, je ferais le test génétique sur la femme enceinte par sécurité. Si elle sort sans la mutation, il n'y aura plus rien à faire car la maladie ne sera plus transmise. Si elle est porteuse de la mutation, je ferais une biopsie des villosités choriales. Les réponses 1 et 2 sont toutes deux vraies, mais je pense que la réponse 1 est plus vraie que la réponse 2.", "type": "GENETIQUE", "options": {"1": "Elle est indiquée après étude du génotype du consultant et seulement si le consultant est hétérozygote.", "2": "Non indiqué, car l'ataxie SCA3 est à pénétration complète et le consultant est asymptomatique et n'a donc pas hérité de la mutation.", "3": "Il peut être indiqué lors de la grossesse suivante, après le génotypage du premier enfant et la détection de la mutation chez le premier enfant.", "4": "L'ataxie SCA3 se transmet de manière récessive, il n'y a donc pas de risque appréciable de transmission de la maladie et le DPN n'est pas indiqué.", "5": "Il est indiqué quel que soit le génotype de la consultante, car l'ataxie SCA3 est héritée de la mère (transmise par les femmes)."}, "correct_option": 1, "explanations": {"1": {"exist": true, "char_ranges": [[452, 875]], "word_ranges": [[68, 145]], "text": "Dans ce cas clinique, le porteur de la maladie et du gène muté en hétérozygotie est le frère (26 ans) de la femme enceinte. Je pense qu'avant de procéder à une biopsie du chorion, la femme enceinte devrait être testée pour connaître son statut vis-à-vis du gène ATXN3. Elle a 50% de chances d'être porteuse de la mutation et d'avoir la maladie, mais comme elle a 31 ans, je pense qu'elle aurait déjà dû avoir des symptômes."}, "2": {"exist": false, "char_ranges": [], "word_ranges": [], "text": ""}, "3": {"exist": false, "char_ranges": [], "word_ranges": [], "text": ""}, "4": {"exist": false, "char_ranges": [], "word_ranges": [], "text": ""}, "5": {"exist": false, "char_ranges": [], "word_ranges": [], "text": ""}}} +{"id": 51, "year": 2011, "question_id_specific": 230, "full_question": "Une femme enceinte de 37 semaines présente un antigène de surface de l'hépatite B positif. Elle s'interroge sur le nouveau-né et demande si elle peut l'allaiter. Quel conseil vous semble le plus approprié ?", "full_answer": "La bonne réponse est 4. Les enfants nés de mères porteuses du VHB devraient se voir administrer le vaccin contre l'hépatite B et l'immunoglobuline à différents points de ponction au cours des 24 premières heures de leur vie. Bien que le virus soit excrété dans le lait maternel, le risque d'infection par cette voie est très faible et ne constitue donc pas actuellement une contre-indication à l'allaitement.", "type": "PÉDIATRIE", "options": {"1": "Administrer le vaccin contre l'hépatite B à la naissance. Allaitement.", "2": "Administration du vaccin et des immunoglobulines à la naissance. Allaitement à partir d'un mois.", "3": "Immunoglobulines à la naissance et allaitement artificiel.", "4": "Vaccination et immunoglobulines à la naissance. Alimentation maternelle.", "5": "Alimentation artificielle et isolement pendant 4 semaines."}, "correct_option": 4, "explanations": {"1": {"exist": false, "char_ranges": [], "word_ranges": [], "text": ""}, "2": {"exist": false, "char_ranges": [], "word_ranges": [], "text": ""}, "3": {"exist": false, "char_ranges": [], "word_ranges": [], "text": ""}, "4": {"exist": true, "char_ranges": [[24, 224]], "word_ranges": [[5, 38]], "text": "Les enfants nés de mères porteuses du VHB devraient se voir administrer le vaccin contre l'hépatite B et l'immunoglobuline à différents points de ponction au cours des 24 premières heures de leur vie."}, "5": {"exist": false, "char_ranges": [], "word_ranges": [], "text": ""}}} +{"id": 184, "year": 2013, "question_id_specific": 65, "full_question": "Un patient souffrant d'obésité (indice de masse corporelle 38) a été adressé à votre clinique. Dans ses antécédents médicaux, le patient indique qu'il est obèse depuis l'âge de 17 ans (actuellement âgé de 36 ans), qu'il a suivi plusieurs régimes avec des pertes de poids allant de 5 à 10 kg, mais qu'il a repris son poids par la suite. La patiente suit actuellement un régime de 1500 Kcal (auto-administré et sans restriction de graisse) avec un bon suivi de ce régime, en faisant une heure d'exercice aérobique 4 jours par semaine. Elle a perdu 3 kg mais doit encore en perdre 7. Si vous avez la possibilité d'ajouter un médicament contre l'obésité, lequel utiliseriez-vous pour réduire l'absorption des graisses ?", "full_answer": "L'orlistat agit en diminuant l'absorption des graisses ; c'est actuellement le seul médicament approuvé pour le traitement de l'obésité.", "type": "ENDOCRINOLOGIE", "options": {"1": "Orlistat.", "2": "Topiramate.", "3": "Sibutramine.", "4": "Liraglutide.", "5": "Metformine."}, "correct_option": 1, "explanations": {"1": {"exist": true, "char_ranges": [[0, 136]], "word_ranges": [[0, 19]], "text": "L'orlistat agit en diminuant l'absorption des graisses ; c'est actuellement le seul médicament approuvé pour le traitement de l'obésité."}, "2": {"exist": false, "char_ranges": [], "word_ranges": [], "text": ""}, "3": {"exist": false, "char_ranges": [], "word_ranges": [], "text": ""}, "4": {"exist": false, "char_ranges": [], "word_ranges": [], "text": ""}, "5": {"exist": false, "char_ranges": [], "word_ranges": [], "text": ""}}} +{"id": 295, "year": 2016, "question_id_specific": 75, "full_question": "Indiquez la situation clinique en relation avec l'infection par le virus de l'hépatite B chez un patient âgé de 5 ans, originaire du Nigeria, présentant un examen physique normal et les sérologies suivantes pour l'hépatite B : HBsAg + / ANTI- HBs - / HBeAg - / ANTI- Hbe + / ANTI- Hbc IgM - / ANTI- Hbc IgG +/ ADN VHB + :", "full_answer": "Question dans laquelle, ayant un tableau en tête, il sort tout seul, en écartant des options : Option 1 : l'infection aiguë serait HBeAg + et ANTI-HBe - . En outre, les anticorps anti-HBe seraient de type IgM (ce n'est qu'en raison de ce dernier élément d'information que nous devrions l'exclure directement). Option 2 : la bonne. Elle répond à toutes les caractéristiques. Option 3 : le patient vacciné ne peut pas avoir d'ADN du virus (la seule chose qui soit positive chez un patient vacciné est le type d'IgG anti-Bs). Option 4 : l'option qui pourrait susciter plus de doutes. Chez le porteur asymptomatique (porteur chronique / séroconverti), la différence est qu'il n'y a pas d'ADN viral.", "type": "GÉNÉTIQUE ET IMMUNOLOGIE", "options": {"1": "Infection aiguë.", "2": "Infection chronique.", "3": "Patient vacciné.", "4": "Porteurs asymptomatiques.", "5": null}, "correct_option": 2, "explanations": {"1": {"exist": true, "char_ranges": [[95, 309]], "word_ranges": [[17, 52]], "text": "Option 1 : l'infection aiguë serait HBeAg + et ANTI-HBe - . En outre, les anticorps anti-HBe seraient de type IgM (ce n'est qu'en raison de ce dernier élément d'information que nous devrions l'exclure directement)."}, "2": {"exist": true, "char_ranges": [[310, 373]], "word_ranges": [[52, 63]], "text": "Option 2 : la bonne. Elle répond à toutes les caractéristiques."}, "3": {"exist": true, "char_ranges": [[374, 436]], "word_ranges": [[63, 76]], "text": "Option 3 : le patient vacciné ne peut pas avoir d'ADN du virus"}, "4": {"exist": true, "char_ranges": [[523, 694]], "word_ranges": [[91, 118]], "text": "Option 4 : l'option qui pourrait susciter plus de doutes. Chez le porteur asymptomatique (porteur chronique / séroconverti), la différence est qu'il n'y a pas d'ADN viral."}, "5": {"exist": false, "char_ranges": [], "word_ranges": [], "text": ""}}} +{"id": 559, "year": 2022, "question_id_specific": 160, "full_question": "Une femme de 35 ans prenant des contraceptifs s'est présentée au service des urgences avec un syndrome fébrile et des paresthésies dans le côté gauche du corps. Les analyses sanguines montrent une Hb à 7,5 g/dl, des plaquettes à 7 000/microl, des leucocytes normaux avec une numération différentielle normale, une LDH à 1 200 UI/l, des réticulocytes à 10 % (normale 0,5-2 %), une haptoglobine indétectable dans le sérum, un test de Coombs direct négatif et un frottis de sang périphérique avec des schistocytes. La coagulation (temps de prothrombine et TCA) est normale. Lequel des diagnostics suivants est le plus probable ?", "full_answer": "La crainte de tous les hématologues de garde... que vous ayez une PTT. Car, bien que peu fréquente, c'est la plus grande urgence hématologique. Le cas clinique est assez typique. Un jeune homme fiévreux et présentant des symptômes neurologiques se présente au service des urgences (il est en fuite). Les examens de laboratoire montrent une anémie avec des signes d'hémolyse (augmentation de la LDH et du réticulum, présence de schistocytes dans le sang périphérique) et une thrombopénie sévère. En outre, la coagulation est normale. Pour le distinguer des autres éléments (mais il ne faut pas douter...), il vous indique que le coombs est négatif. Pour rappel : TRAITEMENT URGENT : REMPLACEMENTS PLASMATIQUES. La confirmation diagnostique est le déficit en ADAMTS 13, mais le traitement est commencé avant d'avoir cette valeur, ce qui n'est généralement pas fait en urgence. Et, comme nous le savons tous, ces choses arrivent généralement la nuit et/ou le week-end, ce qui rend les choses plus difficiles...", "type": "HÉMATOLOGIE", "options": {"1": "Anémie hémolytique auto-immune.", "2": "Purpura thrombocytopénique thrombotique.", "3": "Syndrome d'Evans (anémie hémolytique immunitaire et thrombopénie).", "4": "Thrombocytopénie immunitaire idiopathique.", "5": null}, "correct_option": 2, "explanations": {"1": {"exist": false, "char_ranges": [], "word_ranges": [], "text": ""}, "2": {"exist": true, "char_ranges": [[145, 648]], "word_ranges": [[24, 104]], "text": "Le cas clinique est assez typique. Un jeune homme fiévreux et présentant des symptômes neurologiques se présente au service des urgences (il est en fuite). Les examens de laboratoire montrent une anémie avec des signes d'hémolyse (augmentation de la LDH et du réticulum, présence de schistocytes dans le sang périphérique) et une thrombopénie sévère. En outre, la coagulation est normale. Pour le distinguer des autres éléments (mais il ne faut pas douter...), il vous indique que le coombs est négatif."}, "3": {"exist": false, "char_ranges": [], "word_ranges": [], "text": ""}, "4": {"exist": false, "char_ranges": [], "word_ranges": [], "text": ""}, "5": {"exist": false, "char_ranges": [], "word_ranges": [], "text": ""}}} +{"id": 558, "year": 2022, "question_id_specific": 160, "full_question": "Une femme de 35 ans prenant des contraceptifs s'est présentée au service des urgences avec un syndrome fébrile et des paresthésies dans le côté gauche du corps. Les analyses sanguines montrent une Hb à 7,5 g/dl, des plaquettes à 7 000/microl, des leucocytes normaux avec une numération différentielle normale, une LDH à 1 200 UI/l, des réticulocytes à 10 % (normale 0,5-2 %), une haptoglobine indétectable dans le sérum, un test de Coombs direct négatif et un frottis de sang périphérique avec des schistocytes. La coagulation (temps de prothrombine et TCA) est normale. Lequel des diagnostics suivants est le plus probable ?", "full_answer": "Le patient présente les critères du purpura thrombocytopénique thrombotique (PTT) : 1. 2. syndrome fébrile (fièvre) 3. anémie hémolytique migroangiopathique à Coombs négatif (présence de schistocytes) 4. thrombopénie", "type": "HÉMATOLOGIE", "options": {"1": "Anémie hémolytique auto-immune.", "2": "Purpura thrombocytopénique thrombotique.", "3": "Syndrome d'Evans (anémie hémolytique immunitaire et thrombopénie).", "4": "Thrombocytopénie immunitaire idiopathique.", "5": null}, "correct_option": 2, "explanations": {"1": {"exist": false, "char_ranges": [], "word_ranges": [], "text": ""}, "2": {"exist": true, "char_ranges": [[0, 216]], "word_ranges": [[0, 28]], "text": "Le patient présente les critères du purpura thrombocytopénique thrombotique (PTT) : 1. 2. syndrome fébrile (fièvre) 3. anémie hémolytique migroangiopathique à Coombs négatif (présence de schistocytes) 4. thrombopénie"}, "3": {"exist": false, "char_ranges": [], "word_ranges": [], "text": ""}, "4": {"exist": false, "char_ranges": [], "word_ranges": [], "text": ""}, "5": {"exist": false, "char_ranges": [], "word_ranges": [], "text": ""}}} +{"id": 227, "year": 2014, "question_id_specific": 72, "full_question": "Femme de 45 ans ayant des antécédents de souffle cardiaque détecté dans l'enfance. Elle a été admise au service des urgences pour des palpitations, une grande fatigue et un œdème malléolaire. L'examen physique a révélé une absence de cyanose. Saturation en oxygène à l'oxymètre de pouls 97% Tension 120/80 mmHg. Rythme cardiaque irrégulier à 100 bpm. Souffle systolique éjectif (2/6) dans le foyer pulmonaire. 2ème bruit déplié, large et fixe. Pas de râles. Légère hépatomégalie (2-3 cm). Léger œdème malléolaire. ECG : arythmie complète due à une fibrillation auriculaire à 100 bpm. QRS +120º. Trouble de la conduction de la branche droite du faisceau de His. Quelle est l'orientation diagnostique ?", "full_answer": "S'il a un souffle pédiatrique, je pense qu'il est congénital. Hépatomégalie, œdème malléolaire et BRD, quelque chose qui surcharge le cœur droit. Nous excluons donc une sténose aortique et mitrale. Une persistance du canalicule donnerait une cyanose : on l'exclut. VSD ou ASD ? Si on me dit qu'il a une FA, c'est parce que les oreillettes sont de taille XL, et qu'un VSD ne produit pas de dilatation atriale.", "type": "CARDIOLOGIE", "options": {"1": "Déficience septale ventriculaire.", "2": "Sténose aortique.", "3": "Sténose mitrale.", "4": "Communication interauriculaire.", "5": "Canal artériel patenté."}, "correct_option": 4, "explanations": {"1": {"exist": true, "char_ranges": [[278, 408]], "word_ranges": [[44, 70]], "text": "Si on me dit qu'il a une FA, c'est parce que les oreillettes sont de taille XL, et qu'un VSD ne produit pas de dilatation atriale."}, "2": {"exist": true, "char_ranges": [[0, 197]], "word_ranges": [[0, 30]], "text": "S'il a un souffle pédiatrique, je pense qu'il est congénital. Hépatomégalie, œdème malléolaire et BRD, quelque chose qui surcharge le cœur droit. Nous excluons donc une sténose aortique et mitrale."}, "3": {"exist": true, "char_ranges": [[0, 197]], "word_ranges": [[0, 30]], "text": "S'il a un souffle pédiatrique, je pense qu'il est congénital. Hépatomégalie, œdème malléolaire et BRD, quelque chose qui surcharge le cœur droit. Nous excluons donc une sténose aortique et mitrale."}, "4": {"exist": true, "char_ranges": [[278, 408]], "word_ranges": [[44, 70]], "text": "Si on me dit qu'il a une FA, c'est parce que les oreillettes sont de taille XL, et qu'un VSD ne produit pas de dilatation atriale."}, "5": {"exist": true, "char_ranges": [[198, 264]], "word_ranges": [[30, 40]], "text": "Une persistance du canalicule donnerait une cyanose : on l'exclut."}}} +{"id": 168, "year": 2013, "question_id_specific": 86, "full_question": "Quel examen conseilleriez-vous en premier lieu à un patient de 82 ans présentant un angor d'effort et un souffle systolique d'éjection d'intensité 3/6 à l'auscultation ?", "full_answer": "En présence d'un souffle éjectif, nous soupçonnons en premier lieu une sténose aortique. L'angine peut être due à la sténose elle-même ou à une maladie coronarienne sous-jacente (probable, compte tenu de l'âge). Avec cette information, nous ne pouvons pas savoir si la sténose est sévère. Par conséquent, la première chose à faire est d'étudier la valvulopathie : nous ne voulons pas faire courir le patient ou le surcharger de dobuta et provoquer une syncope, ni procéder à un cathétérisme sans avoir effectué au préalable un autre test non invasif. Et le cardioTC, chez une personne de 80 ans, va nous dire ce que nous savons déjà : qu'elle a du calcium jusqu'aux sourcils.", "type": "CARDIOLOGIE ET CHIRURGIE CARDIOVASCULAIRE", "options": {"1": "Un test d'effort.", "2": "Une coronarographie.", "3": "Un échocardiogramme d'effort.", "4": "Une tomodensitométrie coronaire.", "5": "Un échocardiogramme Doppler."}, "correct_option": 5, "explanations": {"1": {"exist": true, "char_ranges": [[364, 407]], "word_ranges": [[57, 65]], "text": "nous ne voulons pas faire courir le patient"}, "2": {"exist": true, "char_ranges": [[461, 550]], "word_ranges": [[74, 89]], "text": "ni procéder à un cathétérisme sans avoir effectué au préalable un autre test non invasif."}, "3": {"exist": true, "char_ranges": [[411, 460]], "word_ranges": [[66, 74]], "text": "le surcharger de dobuta et provoquer une syncope,"}, "4": {"exist": true, "char_ranges": [[554, 675]], "word_ranges": [[90, 113]], "text": "le cardioTC, chez une personne de 80 ans, va nous dire ce que nous savons déjà : qu'elle a du calcium jusqu'aux sourcils."}, "5": {"exist": false, "char_ranges": [], "word_ranges": [], "text": ""}}} +{"id": 329, "year": 2016, "question_id_specific": 84, "full_question": "Chez un homme diabétique de 70 ans ayant des antécédents de cardiopathie ischémique, quelle est la cible thérapeutique pour le cholestérol LDL et l'hémoglobine glycosylée (Hb A1c) ?", "full_answer": "Diabète avec ecv ldlc<70.", "type": "ENDOCRINOLOGIE", "options": {"1": "LDLc<15 mg/dL et Hb A1c <6,5%.", "2": "LDLc<100 mg/dL et Hb A1c <7%.", "3": "LDLc<70 mg/dL et Hb A1c <7%.", "4": "LDLc< 115mg/dL et Hb A1c <7%.", "5": null}, "correct_option": 3, "explanations": {"1": {"exist": false, "char_ranges": [], "word_ranges": [], "text": ""}, "2": {"exist": false, "char_ranges": [], "word_ranges": [], "text": ""}, "3": {"exist": true, "char_ranges": [[0, 25]], "word_ranges": [[0, 4]], "text": "Diabète avec ecv ldlc<70."}, "4": {"exist": false, "char_ranges": [], "word_ranges": [], "text": ""}, "5": {"exist": false, "char_ranges": [], "word_ranges": [], "text": ""}}} +{"id": 447, "year": 2018, "question_id_specific": 131, "full_question": "Homme de 25 ans sans antécédents connus. Il souffrait depuis 2 mois d'une urine mousseuse et pleine, sans autre symptôme. L'examen physique n'a rien révélé. Urines élémentaires : pH 5 ; glucose négatif ; hémoglobine ++ ; protéines ++ ; estérase leucocytaire négative ; protéinurie 1,5 g/24 heures ; Na urinaire 60 mEq/L ; K urinaire 30 mEq/L ; Cl urinaire 100 mEq/L. Biochimie sanguine : créatinine 1,6 mg/dl ; urée 80 mg/dl ; Na 140 mEq/l ; K 3,8 mEq/l. Etude immunologique : anti-DNA négatif ; ANCA négatif ; anti-MBG négatif, facteur rhumatoïde négatif ; C3 20 mg/dL (normale 60-120) ; C4 10 mg/dL (normale 20-40). Une biopsie rénale est effectuée. Lequel de ces diagnostics est le plus susceptible d'être trouvé dans la biopsie ?", "full_answer": "De nombreuses données analytiques nous sont présentées, mais nous ne devons en examiner que deux : les niveaux des fractions C3 et C4 du complément sont faibles. Parmi les options proposées, seule la glomérulonéphrite membranoproliférative présente une hypocomplémentémie (option 4 correcte).", "type": "NEPHROLOGIE", "options": {"1": "Glomérulonéphrite membraneuse.", "2": "Glomérulonéphrite à changement minime.", "3": "Glomérulonéphrite mésangiale à IgA.", "4": "Glomérulonéphrite membranoproliférative.", "5": null}, "correct_option": 4, "explanations": {"1": {"exist": false, "char_ranges": [], "word_ranges": [], "text": ""}, "2": {"exist": false, "char_ranges": [], "word_ranges": [], "text": ""}, "3": {"exist": false, "char_ranges": [], "word_ranges": [], "text": ""}, "4": {"exist": true, "char_ranges": [[99, 292]], "word_ranges": [[16, 41]], "text": "les niveaux des fractions C3 et C4 du complément sont faibles. Parmi les options proposées, seule la glomérulonéphrite membranoproliférative présente une hypocomplémentémie (option 4 correcte)."}, "5": {"exist": false, "char_ranges": [], "word_ranges": [], "text": ""}}} +{"id": 72, "year": 2012, "question_id_specific": 52, "full_question": "Homme de 72 ans, ex-fumeur, hypertendu et diabétique. Il a été victime d'un accident vasculaire cérébral avec parésie résiduelle du bras gauche il y a quatre mois. Il s'est présenté au service des urgences d'un hôpital régional avec une douleur précordiale intense évoluant depuis 2 heures ; à l'examen physique, il présentait une peau froide et moite, une tension artérielle de 80 mm Hg ; l'électrocardiogramme montrait une élévation marquée du segment ST dans les dérivations V1-V6, I et aVL. Lequel des traitements suivants pour les répercussions coronariennes est le plus approprié ?", "full_answer": "Plusieurs faits importants sont réunis dans cette question. Tout d'abord, il s'agit d'un patient présentant un IAM antérieur et latéral étendu, avec un choc cardiogénique. D'autre part, les antécédents du patient nous apprennent qu'il a subi un accident vasculaire cérébral il y a quatre mois. Le tableau clinique du patient montre clairement qu'une stratégie de reperfusion urgente est indiquée, ce qui invalide les options 4 et 5. La question fondamentale est de savoir s'il faut choisir la fibrinolyse (qui peut être effectuée dans un hôpital régional) ou l'ICP (pour laquelle le patient doit être adressé à un centre tertiaire). L'option 2 n'a pas beaucoup de sens, car si la fibrinolyse était indiquée, le transfert ne serait qu'une perte de temps. Un antécédent d'accident vasculaire cérébral (probablement ischémique) il y a 4 mois est une contre-indication relative à la fibrinolyse (si le délai est inférieur à 3 mois, il s'agit d'une contre-indication absolue), mais le facteur définitif qui fait pencher la balance en faveur de l'ICP est l'étendue de l'IAM et la présence d'un choc, qui sont des indications pour une ICP urgente. La bonne réponse est donc 3.", "type": "ANESTHÉSIOLOGIE, SOINS INTENSIFS ET MÉDECINE D'URGENCE", "options": {"1": "Fibrinolyse systémique immédiate à l'hôpital régional.", "2": "Transfert immédiat vers un centre tertiaire pour traitement par fibrinolyse.", "3": "Transfert immédiat vers un centre tertiaire pour une procédure de revascularisation coronaire percutanée.", "4": "Stabilisation hémodynamique et procédure de revascularisation différée.", "5": "Traitement immédiat par fondaparinux et abciximab."}, "correct_option": 3, "explanations": {"1": {"exist": true, "char_ranges": [[754, 971]], "word_ranges": [[120, 152]], "text": "Un antécédent d'accident vasculaire cérébral (probablement ischémique) il y a 4 mois est une contre-indication relative à la fibrinolyse (si le délai est inférieur à 3 mois, il s'agit d'une contre-indication absolue),"}, "2": {"exist": true, "char_ranges": [[633, 753]], "word_ranges": [[99, 120]], "text": "L'option 2 n'a pas beaucoup de sens, car si la fibrinolyse était indiquée, le transfert ne serait qu'une perte de temps."}, "3": {"exist": true, "char_ranges": [[977, 1140]], "word_ranges": [[153, 182]], "text": "le facteur définitif qui fait pencher la balance en faveur de l'ICP est l'étendue de l'IAM et la présence d'un choc, qui sont des indications pour une ICP urgente."}, "4": {"exist": true, "char_ranges": [[294, 432]], "word_ranges": [[45, 67]], "text": "Le tableau clinique du patient montre clairement qu'une stratégie de reperfusion urgente est indiquée, ce qui invalide les options 4 et 5."}, "5": {"exist": true, "char_ranges": [[294, 432]], "word_ranges": [[45, 67]], "text": "Le tableau clinique du patient montre clairement qu'une stratégie de reperfusion urgente est indiquée, ce qui invalide les options 4 et 5."}}} +{"id": 505, "year": 2020, "question_id_specific": 85, "full_question": "Un patient de 79 ans est amené au service des urgences à 19 heures en provenance d'une maison de retraite, mais nous ne disposons pas du rapport d'orientation et ne connaissons donc pas ses antécédents. Il est désorienté dans le temps et l'espace. Il a des difficultés à se souvenir d'informations simples. Il devient parfois très nerveux et agité, parce qu'il voit des personnes menaçantes, même lorsqu'il est seul. À d'autres moments, il semble somnolent et inattentif. Les principales données étayant le diagnostic de délire par opposition à celui de démence sont les suivantes :", "full_answer": "Little to comment.... est la définition du délire. Altération de l'état de conscience.", "type": "PSYCHIATRIE", "options": {"1": "La présence d'hallucinations visuelles.", "2": "Symptômes cognitifs.", "3": "Agitation psychomotrice.", "4": "Fluctuation du niveau de vigilance.", "5": null}, "correct_option": 4, "explanations": {"1": {"exist": false, "char_ranges": [], "word_ranges": [], "text": ""}, "2": {"exist": false, "char_ranges": [], "word_ranges": [], "text": ""}, "3": {"exist": false, "char_ranges": [], "word_ranges": [], "text": ""}, "4": {"exist": true, "char_ranges": [[22, 86]], "word_ranges": [[3, 13]], "text": "est la définition du délire. Altération de l'état de conscience."}, "5": {"exist": false, "char_ranges": [], "word_ranges": [], "text": ""}}} +{"id": 360, "year": 2016, "question_id_specific": 93, "full_question": "Une femme de 46 ans ayant des antécédents de valvulopathie rhumatismale a dû subir un remplacement de la valvule mitrale par une prothèse mécanique. L'évolution postopératoire initiale a été favorable. Cependant, après avoir commencé un traitement à l'acénocoumarol, elle a développé une nécrose cutanée affectant la région abdominale et les extrémités. Laquelle des altérations suivantes justifierait cet état ?", "full_answer": "La nécrose cutanée induite par la warfarine est un effet indésirable rare et grave de la thérapie anticoagulante orale, survenant chez seulement 0,01 à 0,1% des patients, et est particulièrement liée à un déficit en protéines de coagulation C et S.", "type": "HÉMATOLOGIE", "options": {"1": "Déficit en antithrombine.", "2": "Facteur V Leiden.", "3": "Hyperhomocystéinémie.", "4": "Déficit en protéine C.", "5": null}, "correct_option": 4, "explanations": {"1": {"exist": false, "char_ranges": [], "word_ranges": [], "text": ""}, "2": {"exist": false, "char_ranges": [], "word_ranges": [], "text": ""}, "3": {"exist": false, "char_ranges": [], "word_ranges": [], "text": ""}, "4": {"exist": true, "char_ranges": [[0, 248]], "word_ranges": [[0, 41]], "text": "La nécrose cutanée induite par la warfarine est un effet indésirable rare et grave de la thérapie anticoagulante orale, survenant chez seulement 0,01 à 0,1% des patients, et est particulièrement liée à un déficit en protéines de coagulation C et S."}, "5": {"exist": false, "char_ranges": [], "word_ranges": [], "text": ""}}} +{"id": 459, "year": 2018, "question_id_specific": 173, "full_question": "Un homme de 36 ans, sans antécédents, présente, après un traumatisme à haute énergie consécutif à une chute de moto, une fracture ouverte médiodiaphysaire du tibia de grade II selon Gustilo. Parmi les manœuvres ou thérapies suivantes, laquelle est la plus efficace pour prévenir l'infection ?", "full_answer": "Je pense que la réponse la plus correcte est 2, car les fractures de Gustilo II sont principalement couvertes de Gram (+) au moyen d'un Cepha de 1ère génération, et il ne s'agit pas d'une question de prévention mais de traitement (il est considéré comme contaminé).", "type": "TRAUMATOLOGIE ET ORTHOPÉDIE", "options": {"1": "Antibiothérapie intraveineuse immédiate à large spectre.", "2": "Débridement approfondi - nettoyage de la plaie au bloc opératoire avec au moins 10 litres de solution saline.", "3": "Stabilisation urgente de la fracture par fixation externe et fermeture rapide de la plaie.", "4": "Fermeture précoce de la plaie à l'aide d'antiseptiques et d'une oxygénothérapie de deux litres par minute.", "5": null}, "correct_option": 2, "explanations": {"1": {"exist": false, "char_ranges": [], "word_ranges": [], "text": ""}, "2": {"exist": true, "char_ranges": [[0, 161]], "word_ranges": [[0, 29]], "text": "Je pense que la réponse la plus correcte est 2, car les fractures de Gustilo II sont principalement couvertes de Gram (+) au moyen d'un Cepha de 1ère génération,"}, "3": {"exist": false, "char_ranges": [], "word_ranges": [], "text": ""}, "4": {"exist": false, "char_ranges": [], "word_ranges": [], "text": ""}, "5": {"exist": false, "char_ranges": [], "word_ranges": [], "text": ""}}} +{"id": 149, "year": 2012, "question_id_specific": 162, "full_question": "Une femme de 52 ans s'est présentée au service des urgences pour une paralysie bilatérale des membres supérieurs apparue brutalement deux heures plus tôt. Lors de l'entretien, elle était souriante et disait ne pas comprendre pourquoi ses proches étaient si inquiets. La famille rapporte que la veille, leur fils a été arrêté par la police pour trafic de drogue et qu'ils n'ont pas encore pu le contacter. Lequel des diagnostics suivants est le plus probable ?", "full_answer": "Le joli mot pour décrire ceci est : \"Belle indifférence\", cette femme montre une élégante indifférence face à une paralysie. Le diagnostic .... permet d'exclure 2,3,4 car le tableau clinique ne correspond pas : pas de délire, pas d'inquiétude, pas de tristesse. Le diagnostic 1 serait s'il faisait du profit, s'il simulait sciemment et pour faire du profit il simulait sa maladie. Ce n'est pas le cas. Conversif serait si le symptôme voulait nous transmettre quelque chose, c'est-à-dire s'il s'agissait d'une métaphore de son monde intérieur... Le plus juste dans ce cas : 5.", "type": "PSYCHIATRIE", "options": {"1": "Trouble factice.", "2": "Trouble délirant.", "3": "Hypocondrie.", "4": "Trouble dépressif majeur.", "5": "Trouble de la conversion."}, "correct_option": 5, "explanations": {"1": {"exist": true, "char_ranges": [[262, 401]], "word_ranges": [[42, 67]], "text": "Le diagnostic 1 serait s'il faisait du profit, s'il simulait sciemment et pour faire du profit il simulait sa maladie. Ce n'est pas le cas."}, "2": {"exist": true, "char_ranges": [[144, 261]], "word_ranges": [[23, 42]], "text": "permet d'exclure 2,3,4 car le tableau clinique ne correspond pas : pas de délire, pas d'inquiétude, pas de tristesse."}, "3": {"exist": true, "char_ranges": [[144, 261]], "word_ranges": [[23, 42]], "text": "permet d'exclure 2,3,4 car le tableau clinique ne correspond pas : pas de délire, pas d'inquiétude, pas de tristesse."}, "4": {"exist": true, "char_ranges": [[144, 261]], "word_ranges": [[23, 42]], "text": "permet d'exclure 2,3,4 car le tableau clinique ne correspond pas : pas de délire, pas d'inquiétude, pas de tristesse."}, "5": {"exist": true, "char_ranges": [[402, 544]], "word_ranges": [[67, 86]], "text": "Conversif serait si le symptôme voulait nous transmettre quelque chose, c'est-à-dire s'il s'agissait d'une métaphore de son monde intérieur..."}}} +{"id": 246, "year": 2014, "question_id_specific": 116, "full_question": "Une tuberculose pulmonaire a été diagnostiquée chez un patient ayant des antécédents de consommation excessive d'alcool, en raison d'une toux, d'une fièvre, d'une expectoration et de l'isolement de Mycobacterium tuberculosis dans les cultures d'expectoration. Le patient a commencé un traitement à base d'isoniazide, de rifampicine, d'éthambutol et de pyrazinamide, avec une tolérance adéquate. Vingt jours après le début du traitement, une résistance à la rifampicine de M. tuberculosis isolé à partir d'expectorations est signalée. Quel schéma thérapeutique choisiriez-vous sur la base de ce rapport ?", "full_answer": "Une autre question qui peut susciter la discussion. Lorsqu'un diagnostic d'infection tuberculeuse active est posé, un traitement à base d'isoniazide, de rifampicine et de pyrazinamide, associé à l'éthambutol, est mis en place en cas de suspicion de souches résistantes aux médicaments. Une fois la résistance exclue, l'éthambutol peut être administré. En cas de résistance à la rifampicine, celle-ci est retirée et les trois autres médicaments sont ajoutés pendant 12 mois. L'option d'ajouter une quinolone pendant les deux premiers mois de traitement (moxifloxacine ou lévofloxacine) est envisagée en cas d'atteinte pulmonaire étendue. Dans ce cas, on ne parle pas d'atteinte pulmonaire extensive mais on nous donne une information \"consommation excessive d'alcool\". Vous savez que dans le MIR aucune donnée n'est gratuite, et si la personne qui a posté cette question a voulu refléter cela, son intention était probablement de nous faire réfléchir. L'alcool a un effet immunosuppresseur direct et l'alcoolisme est associé à une altération de l'immunité dans les voies respiratoires. Le contexte social qui accompagne généralement le patient alcoolique n'aide pas non plus. Tous les facteurs mentionnés ci-dessus nous font pencher davantage vers la réponse numéro 3, même s'il est indéniable que la réponse numéro 1 pourrait également être correcte.", "type": "LES MALADIES INFECTIEUSES", "options": {"1": "Isoniazide, éthambutol, pyrazinamide pendant 12 mois.", "2": "Isoniazide, éthambutol, pyrazinamide pendant 12 mois et streptomycine pendant deux mois.", "3": "Isoniazide, éthambutol, pyrazinamide pendant 12 mois et une quinolone pendant deux mois.", "4": "Isoniazide, éthambutol, pyrazinamide et une quinolone pendant 18 mois.", "5": "Isoniazide, éthambutol, pyrazinamide pendant 18 mois et streptomycine et quinolone pendant 2 mois."}, "correct_option": 3, "explanations": {"1": {"exist": false, "char_ranges": [], "word_ranges": [], "text": ""}, "2": {"exist": false, "char_ranges": [], "word_ranges": [], "text": ""}, "3": {"exist": true, "char_ranges": [[951, 1084]], "word_ranges": [[141, 160]], "text": "L'alcool a un effet immunosuppresseur direct et l'alcoolisme est associé à une altération de l'immunité dans les voies respiratoires."}, "4": {"exist": false, "char_ranges": [], "word_ranges": [], "text": ""}, "5": {"exist": false, "char_ranges": [], "word_ranges": [], "text": ""}}} +{"id": 457, "year": 2018, "question_id_specific": 126, "full_question": "Un homme de 47 ans, aux antécédents d'adénome pléomorphe de la parotide droite, traité par chirurgie (parotidectomie extrafaciale) il y a 6 mois, se présente à notre clinique en raison de douleurs à la mastication accompagnées de sueurs et d'une rougeur de la peau dans la région préauriculaire. Quel serait le meilleur traitement ?", "full_answer": "Ce tableau clinique est appelé syndrome de Frey et consiste en une réinnervation anormale des structures régionales après une parotidectomie due à une lésion du nerf auriculotemporal, de sorte que lors de la mastication, un érythème et une transpiration préauriculaire apparaissent. Le traitement consiste en une injection intradermique de toxine botulique.", "type": "OTORHINOLARYNGOLOGIE ET CHIRURGIE MAXILLO-FACIALE", "options": {"1": "Parotidectomie totale élargie en cas de suspicion de récidive tumorale.", "2": "Prégabaline.", "3": "Injection intradermique de toxine botulique.", "4": "Antibiothérapie à large spectre.", "5": null}, "correct_option": 3, "explanations": {"1": {"exist": false, "char_ranges": [], "word_ranges": [], "text": ""}, "2": {"exist": false, "char_ranges": [], "word_ranges": [], "text": ""}, "3": {"exist": true, "char_ranges": [[0, 357]], "word_ranges": [[0, 51]], "text": "Ce tableau clinique est appelé syndrome de Frey et consiste en une réinnervation anormale des structures régionales après une parotidectomie due à une lésion du nerf auriculotemporal, de sorte que lors de la mastication, un érythème et une transpiration préauriculaire apparaissent. Le traitement consiste en une injection intradermique de toxine botulique."}, "4": {"exist": false, "char_ranges": [], "word_ranges": [], "text": ""}, "5": {"exist": false, "char_ranges": [], "word_ranges": [], "text": ""}}} +{"id": 165, "year": 2013, "question_id_specific": 229, "full_question": "Femme de 58 ans, poids 130 kg, taille 155 cm, indice de masse corporelle > 30 avec hypertension artérielle légère, glycémie 108 mg/dL et absence d'œdème des membres inférieurs. Les analyses sanguines ont montré Cr 2,0 mg/dl, Urée 86 mg/dl, Alb 3,8 g/1, Na 142 mEq/L, K 4 mEq/L. Analyse d'urine : sédiment sans altération et protéinurie sur 24 heures de 6,3 g/24 h. Lequel des éléments suivants est le plus susceptible d'être présent ?", "full_answer": "La bonne réponse est : 2. glomérulonéphrite segmentaire focale. Nous sommes en présence d'un patient en insuffisance rénale dont la manifestation est une protéinurie. Cela nous place dans le tableau d'une GN non proliférative (membraneuse, focale et segmentaire et changements minimes, ce qui exclut les deux autres). Si l'on tient également compte du fait que le patient était obèse et légèrement hypertendu, il s'agit très probablement d'une glomérulonéphrite segmentaire focale.", "type": "NEPHROLOGIE", "options": {"1": "Glomérulonéphrite membraneuse secondaire.", "2": "Glomérulonéphrite focale et segmentaire.", "3": "Néphropathie à IgA.", "4": "Glomérulonéphrite rapidement progressive.", "5": "Néphropathie à changement minime."}, "correct_option": 2, "explanations": {"1": {"exist": false, "char_ranges": [], "word_ranges": [], "text": ""}, "2": {"exist": true, "char_ranges": [[64, 317]], "word_ranges": [[9, 47]], "text": "Nous sommes en présence d'un patient en insuffisance rénale dont la manifestation est une protéinurie. Cela nous place dans le tableau d'une GN non proliférative (membraneuse, focale et segmentaire et changements minimes, ce qui exclut les deux autres)."}, "3": {"exist": false, "char_ranges": [], "word_ranges": [], "text": ""}, "4": {"exist": false, "char_ranges": [], "word_ranges": [], "text": ""}, "5": {"exist": false, "char_ranges": [], "word_ranges": [], "text": ""}}} +{"id": 157, "year": 2012, "question_id_specific": 73, "full_question": "Patient de 75 ans souffrant d'une gonarthrose importante avec \"Genu Varo\" qui entraîne une impotence fonctionnelle marquée, une mobilité limitée et des douleurs permanentes qui l'obligent à prendre des médicaments antirhumatismaux, anti-inflammatoires et analgésiques de façon continue.", "full_answer": "Nous avons affaire à un patient qui a épuisé la voie pharmacologique puisqu'il prend tout et qui a deux problèmes majeurs : un genu varum et une gonarthrose importante. Cela signifie que nous ne pouvons rien faire avec une arthroscopie et encore moins une synovectomie (1 et 5 faux). Le genu varum pourrait faussement nous faire penser à une ostéotomie, mais même si nous la faisions et corrigions le varus, le mal est déjà fait et l'arthrose est sévère. Peut-être qu'il y a quelques années, avant l'apparition de l'arthrose, il aurait été possible de résoudre le problème, mais pas maintenant (4 faux). En outre, pour envisager une ostéotomie, il faudrait disposer de plus de données pour savoir sur quel os appliquer l'ostéotomie, le tibia ou le fémur. Enfin, on peut penser qu'il est préférable d'essayer la rééducation avant d'envisager la chirurgie, mais j'écarte cette option car la question souligne que l'on a épuisé les mesures pharmacologiques (bien qu'elle puisse être envisagée car on n'a pas précisé si elle a été essayée ou non). Quoi qu'il en soit, avec la prothèse du genou, nous corrigeons le problème de la douleur due à l'arthrose et nous pouvons également corriger le varus du genou.", "type": "TRAUMATOLOGIE ET ORTHOPÉDIE", "options": {"1": "Lavage arthroscopique du genou.", "2": "Réhabilitation du genou affecté.", "3": "Arthroplastie du genou atteint.", "4": "Ostéotomie tibiale d'abduction de la supratubérosité.", "5": "Synovectomie du genou."}, "correct_option": 3, "explanations": {"1": {"exist": true, "char_ranges": [[0, 283]], "word_ranges": [[0, 49]], "text": "Nous avons affaire à un patient qui a épuisé la voie pharmacologique puisqu'il prend tout et qui a deux problèmes majeurs : un genu varum et une gonarthrose importante. Cela signifie que nous ne pouvons rien faire avec une arthroscopie et encore moins une synovectomie (1 et 5 faux)."}, "2": {"exist": true, "char_ranges": [[762, 1043]], "word_ranges": [[128, 173]], "text": "on peut penser qu'il est préférable d'essayer la rééducation avant d'envisager la chirurgie, mais j'écarte cette option car la question souligne que l'on a épuisé les mesures pharmacologiques (bien qu'elle puisse être envisagée car on n'a pas précisé si elle a été essayée ou non)."}, "3": {"exist": true, "char_ranges": [[1064, 1203]], "word_ranges": [[177, 201]], "text": "avec la prothèse du genou, nous corrigeons le problème de la douleur due à l'arthrose et nous pouvons également corriger le varus du genou."}, "4": {"exist": true, "char_ranges": [[284, 603]], "word_ranges": [[49, 102]], "text": "Le genu varum pourrait faussement nous faire penser à une ostéotomie, mais même si nous la faisions et corrigions le varus, le mal est déjà fait et l'arthrose est sévère. Peut-être qu'il y a quelques années, avant l'apparition de l'arthrose, il aurait été possible de résoudre le problème, mais pas maintenant (4 faux)."}, "5": {"exist": true, "char_ranges": [[0, 283]], "word_ranges": [[0, 49]], "text": "Nous avons affaire à un patient qui a épuisé la voie pharmacologique puisqu'il prend tout et qui a deux problèmes majeurs : un genu varum et une gonarthrose importante. Cela signifie que nous ne pouvons rien faire avec une arthroscopie et encore moins une synovectomie (1 et 5 faux)."}}} +{"id": 382, "year": 2016, "question_id_specific": 231, "full_question": "Un homme de 33 ans, indépendant dans toutes les activités de la vie quotidienne, ayant des antécédents d'hypertension bien contrôlée, de fibrillation auriculaire antiagrégée et d'adénocarcinome de la prostate à l'âge de 73 ans, actuellement indemne de toute maladie, a été amené au service des urgences pour une aphasie et une hémiparésie droite d'apparition soudaine, 45 minutes plus tôt. Quelle est l'approche la plus correcte ?", "full_answer": "Le patient souffre d'un déficit neurologique soudain probablement lié à un événement vasculaire, et nous devons donc d'abord effectuer un scanner crânien pour exclure un accident vasculaire cérébral hémorragique. Si le scanner ne montre pas d'hémorragie, le patient présente un AVC ischémique aigu, probablement cardioembolique en raison de sa pathologie cardiaque, qui n'est pas anticoagulée. Étant donné que le processus tumoral est sous contrôle, il n'y a pas de contre-indication à l'administration d'une fibrinolyse dans ce cas, car le patient est dans un bon état de base. Bien que le patient soit âgé de 88 ans, l'âge n'est plus présenté comme un critère d'exclusion absolu, mais la situation générale du patient doit être prise en compte, et dans ce cas il est précisé que le patient a un bon état de base.", "type": "NEUROLOGIE", "options": {"1": "Effectuer une tomodensitométrie crânienne et, en l'absence de lésions hémorragiques ou d'autres contre-indications dans l'analyse sanguine, commencer immédiatement une thrombolyse intraveineuse.", "2": "Réaliser d'urgence un scanner crânien et, en l'absence de saignement, commencer l'anticoagulation.", "3": "Effectuer une tomodensitométrie crânienne et admettre le patient dans un centre de réadaptation, car l'approche ne sera pas différente selon que l'étiologie est ischémique ou hémorragique.", "4": "Aucun scanner crânien n'est nécessaire, mais j'arrêterais le traitement antiplaquettaire.", "5": null}, "correct_option": 1, "explanations": {"1": {"exist": true, "char_ranges": [[100, 533]], "word_ranges": [[14, 77]], "text": "nous devons donc d'abord effectuer un scanner crânien pour exclure un accident vasculaire cérébral hémorragique. Si le scanner ne montre pas d'hémorragie, le patient présente un AVC ischémique aigu, probablement cardioembolique en raison de sa pathologie cardiaque, qui n'est pas anticoagulée. Étant donné que le processus tumoral est sous contrôle, il n'y a pas de contre-indication à l'administration d'une fibrinolyse dans ce cas,"}, "2": {"exist": false, "char_ranges": [], "word_ranges": [], "text": ""}, "3": {"exist": false, "char_ranges": [], "word_ranges": [], "text": ""}, "4": {"exist": false, "char_ranges": [], "word_ranges": [], "text": ""}, "5": {"exist": false, "char_ranges": [], "word_ranges": [], "text": ""}}} +{"id": 496, "year": 2020, "question_id_specific": 71, "full_question": "Un patient de 40 ans, dont l'examen physique est normal, chez qui la tomodensitométrie a révélé une masse médiastinale et un nodule hépatique, ainsi qu'une élévation du taux sérique de lactate déshydrogénase (LDH), de bétagonadotrophine chorionique humaine (bêta-HCG) et d'alpha-fœtoprotéine dans les analyses de laboratoire. Quel est le diagnostic suspecté ?", "full_answer": "Très typique également. Dans l'urgence, il peut être diagnostiqué à l'aide d'un test de grossesse (chez les hommes), car il détecte cette HCG bêta dans l'urine. L'hépatocarcinome peut élever (uniquement) l'alpha-fœtus, le séminome la LDH, le lymphome également la LDH.", "type": "ONCOLOGIE MÉDICALE", "options": {"1": "Hépatocarcinome métastatique.", "2": "Séminome extragonadique métastatique.", "3": "Lymphome médiastinal.", "4": "Cancer des cellules germinales non séminomateuses métastatique extragonadique.", "5": null}, "correct_option": 4, "explanations": {"1": {"exist": true, "char_ranges": [[161, 268]], "word_ranges": [[26, 40]], "text": "L'hépatocarcinome peut élever (uniquement) l'alpha-fœtus, le séminome la LDH, le lymphome également la LDH."}, "2": {"exist": true, "char_ranges": [[161, 268]], "word_ranges": [[26, 40]], "text": "L'hépatocarcinome peut élever (uniquement) l'alpha-fœtus, le séminome la LDH, le lymphome également la LDH."}, "3": {"exist": true, "char_ranges": [[161, 268]], "word_ranges": [[26, 40]], "text": "L'hépatocarcinome peut élever (uniquement) l'alpha-fœtus, le séminome la LDH, le lymphome également la LDH."}, "4": {"exist": true, "char_ranges": [[24, 160]], "word_ranges": [[3, 26]], "text": "Dans l'urgence, il peut être diagnostiqué à l'aide d'un test de grossesse (chez les hommes), car il détecte cette HCG bêta dans l'urine."}, "5": {"exist": false, "char_ranges": [], "word_ranges": [], "text": ""}}} +{"id": 14, "year": 2011, "question_id_specific": 213, "full_question": "Un homme de 41 ans consulte pour un nodule dans la thyroïde droite et une aspiration à l'aiguille fine est réalisée. L'analyse cytologique permettra de :", "full_answer": "Je pense que la bonne réponse est 5 ; la FNA ne permet pas une étude histologique si elle n'est pas cytologique, donc les questions 1, 2 sont éliminées. La question 3 semble avoir été placée comme un bouche-trou car elle n'est pas pertinente. La question 4 pourrait soulever des doutes si vous n'êtes pas pathologiste, mais la réponse la plus logique est 5 car le carcinome papillaire est le plus fréquent.", "type": "CHIRURGIE", "options": {"1": "Distinguer l'adénome folliculaire du carcinome folliculaire peu invasif.", "2": "Identifier les zones d'invasion capsulaire ou vasculaire du carcinome folliculaire.", "3": "Reconnaître l'hyperplasie des cellules G associée au cancer familial de la moelle épinière.", "4": "Distinguer une tumeur à cellules de Hurthle d'un adénome folliculaire oxyphile.", "5": "Identifier les caractéristiques cytologiques typiques du carcinome papillaire."}, "correct_option": 5, "explanations": {"1": {"exist": true, "char_ranges": [[38, 152]], "word_ranges": [[9, 29]], "text": "la FNA ne permet pas une étude histologique si elle n'est pas cytologique, donc les questions 1, 2 sont éliminées."}, "2": {"exist": true, "char_ranges": [[38, 152]], "word_ranges": [[9, 29]], "text": "la FNA ne permet pas une étude histologique si elle n'est pas cytologique, donc les questions 1, 2 sont éliminées."}, "3": {"exist": true, "char_ranges": [[153, 242]], "word_ranges": [[29, 44]], "text": "La question 3 semble avoir été placée comme un bouche-trou car elle n'est pas pertinente."}, "4": {"exist": true, "char_ranges": [[243, 317]], "word_ranges": [[44, 56]], "text": "La question 4 pourrait soulever des doutes si vous n'êtes pas pathologiste,"}, "5": {"exist": true, "char_ranges": [[324, 406]], "word_ranges": [[57, 72]], "text": "la réponse la plus logique est 5 car le carcinome papillaire est le plus fréquent."}}} +{"id": 330, "year": 2016, "question_id_specific": 85, "full_question": "Un garçon de 22 ans souffrant d'hyposmie présente une absence de développement des caractères sexuels secondaires et une infertilité. Volume testiculaire bilatéral de 4 ml. Sur le plan analytique, FSH 1,2 U/L (vn 5-15) ; LH 0,6 U/L (vn 3-15) ; testostérone 100 ng/diu (vn 300-1200), prolactine normale. Indiquez le traitement qu'il proposera pour obtenir la fertilité :", "full_answer": "Le syndrome de Kallman ou syndrome de Morsier est une insuffisance congénitale associée à un hypogonadisme hypogonadotrope dû à un déficit de production de GnRH. Des injections répétées de GnRH sont utilisées pour restaurer la fertilité.", "type": "ENDOCRINOLOGIE", "options": {"1": "Pompe à perfusion de Gn_RH.", "2": "Administration intramusculaire mensuelle de triptoréline.", "3": "Administration intramusculaire de FSH et LH une fois par semaine.", "4": "Traitement par la bromocriptine.", "5": null}, "correct_option": 1, "explanations": {"1": {"exist": true, "char_ranges": [[0, 237]], "word_ranges": [[0, 36]], "text": "Le syndrome de Kallman ou syndrome de Morsier est une insuffisance congénitale associée à un hypogonadisme hypogonadotrope dû à un déficit de production de GnRH. Des injections répétées de GnRH sont utilisées pour restaurer la fertilité."}, "2": {"exist": false, "char_ranges": [], "word_ranges": [], "text": ""}, "3": {"exist": false, "char_ranges": [], "word_ranges": [], "text": ""}, "4": {"exist": false, "char_ranges": [], "word_ranges": [], "text": ""}, "5": {"exist": false, "char_ranges": [], "word_ranges": [], "text": ""}}} +{"id": 181, "year": 2013, "question_id_specific": 59, "full_question": "Un patient diabétique de type 1 se présente aux urgences pour dyspnée et malaise général. Les analyses sanguines montrent une glycémie de 450 mg/dl, Na 142 mEq/l, K 4 mEq/l, pH 7,15, bicarbonate 12 mmol/l. Quel traitement ne serait PAS indiqué ?", "full_answer": "Question facile car le traitement de l'acidocétose diabétique est un sujet récurrent dans le MIR. Le bicarbonate est indiqué à un pH < 7.", "type": "ENDOCRINOLOGIE", "options": {"1": "Solution saline physiologique iv rapide.", "2": "Glucose salin à 5 % par voie IV lorsque la glycémie est inférieure à 250 mg/dl.", "3": "Insuline humaine rapide iv.", "4": "Chlorure de potassium 100 mEq/jour dilué dans les sérums.", "5": "Bicarbonate de sodium 1M 100 cc iv en 30 minutes."}, "correct_option": 5, "explanations": {"1": {"exist": false, "char_ranges": [], "word_ranges": [], "text": ""}, "2": {"exist": false, "char_ranges": [], "word_ranges": [], "text": ""}, "3": {"exist": false, "char_ranges": [], "word_ranges": [], "text": ""}, "4": {"exist": false, "char_ranges": [], "word_ranges": [], "text": ""}, "5": {"exist": true, "char_ranges": [[98, 137]], "word_ranges": [[15, 24]], "text": "Le bicarbonate est indiqué à un pH < 7."}}} +{"id": 182, "year": 2013, "question_id_specific": 61, "full_question": "Une femme de 76 ans nous consulte car elle s'inquiète du risque d'accident vasculaire cérébral (AVC), sa mère étant décédée d'un AVC il y a un an. On lui a diagnostiqué une hypertension artérielle et un diabète sucré de type 2. Elle est traitée par glipizide, aspirine, énalapril et atorvastatine. Elle fume 20 cigarettes par jour et mène une vie sédentaire. L'examen révèle une tension artérielle de 150/80 mm Hg. L'analyse de sang révèle un taux d'hémoglobine A1C de 8 % et un taux de cholestérol LDL de 110 mg/dl. Laquelle des mesures suivantes est associée à une plus grande réduction du risque d'accident vasculaire cérébral ?", "full_answer": "La tension artérielle est le facteur cardiovasculaire le plus influent pour les femmes de plus de 70 ans.", "type": "ENDOCRINOLOGIE", "options": {"1": "Obtenir un contrôle optimal de la tension artérielle.", "2": "Atteindre des niveaux optimaux d'hémoglobine A1C.", "3": "Ajouter un antioxydant au traitement.", "4": "Arrêter de fumer.", "5": "Atteindre un taux de LDL inférieur à 100 mg/dl."}, "correct_option": 1, "explanations": {"1": {"exist": true, "char_ranges": [[0, 105]], "word_ranges": [[0, 18]], "text": "La tension artérielle est le facteur cardiovasculaire le plus influent pour les femmes de plus de 70 ans."}, "2": {"exist": false, "char_ranges": [], "word_ranges": [], "text": ""}, "3": {"exist": false, "char_ranges": [], "word_ranges": [], "text": ""}, "4": {"exist": false, "char_ranges": [], "word_ranges": [], "text": ""}, "5": {"exist": false, "char_ranges": [], "word_ranges": [], "text": ""}}} +{"id": 21, "year": 2011, "question_id_specific": 111, "full_question": "Un homme de 32 ans, qui a été récemment libéré de prison, présente un test cutané aux protéines dérivées (PPD) avec une induration de 10 mm. Il y a un an, le PPD était négatif, il est actuellement asymptomatique, sa sérologie VIH est négative et sa radiographie pulmonaire est normale. Parmi les approches suivantes, laquelle vous semble la plus appropriée ?", "full_answer": "En cas de conversion de la PPD dans les 2 ans, et si l'on exclut une tuberculose actuelle, une chimioprophylaxie à l'INH est indiquée, mais je pensais que c'était pour 6 mois et non pour 9. Le reste n'est pas contestable.", "type": "INFECTIEUX", "options": {"1": "Pas de traitement et contrôle annuel par Rx.", "2": "Effectuer une culture des expectorations à la recherche de mycobactéries et ne traiter qu'en cas de résultat positif.", "3": "Isoniazide pendant 9 mois.", "4": "Traitement avec 4 médicaments pendant 4 mois.", "5": "Traitement par RIF et PRZ pendant 2 mois."}, "correct_option": 3, "explanations": {"1": {"exist": false, "char_ranges": [], "word_ranges": [], "text": ""}, "2": {"exist": false, "char_ranges": [], "word_ranges": [], "text": ""}, "3": {"exist": true, "char_ranges": [[0, 133]], "word_ranges": [[0, 24]], "text": "En cas de conversion de la PPD dans les 2 ans, et si l'on exclut une tuberculose actuelle, une chimioprophylaxie à l'INH est indiquée,"}, "4": {"exist": false, "char_ranges": [], "word_ranges": [], "text": ""}, "5": {"exist": false, "char_ranges": [], "word_ranges": [], "text": ""}}} +{"id": 60, "year": 2011, "question_id_specific": 59, "full_question": "Un touriste étranger de 40 ans est adressé au service des urgences pour un faible niveau de conscience, une température de 37,7 ºC et une dyspnée. Il n'a subi aucun traumatisme. Plusieurs examens sont effectués. Le plus immédiat est une EBA sans oxygène montrant un PH de 7,33, une PCO2 de 50, une PO2 de 65, une HCO3 de 27. Diagnostic le plus probable :", "full_answer": "En fait, il peut s'agir de toutes, mais la cause la plus fréquente d'acidose respiratoire aiguë non compensée chez un jeune est la bronchopneumonie.", "type": "ANESTHÉSIOLOGIE ET SOINS INTENSIFS", "options": {"1": "TEP.", "2": "Pneumonie lobaire.", "3": "PAE.", "4": "Hypoventilation alvéolaire.", "5": "Bronchopneumonie."}, "correct_option": 5, "explanations": {"1": {"exist": false, "char_ranges": [], "word_ranges": [], "text": ""}, "2": {"exist": false, "char_ranges": [], "word_ranges": [], "text": ""}, "3": {"exist": false, "char_ranges": [], "word_ranges": [], "text": ""}, "4": {"exist": false, "char_ranges": [], "word_ranges": [], "text": ""}, "5": {"exist": true, "char_ranges": [[40, 148]], "word_ranges": [[8, 24]], "text": "la cause la plus fréquente d'acidose respiratoire aiguë non compensée chez un jeune est la bronchopneumonie."}}} +{"id": 455, "year": 2018, "question_id_specific": 155, "full_question": "Chez un homme de 70 ans, hypertendu, présentant une hémiplégie droite d'apparition brutale avec une légère dysarthrie, sans altération de l'évocation ou de la compréhension du langage, quel type d'accident vasculaire cérébral vous paraît le plus probable ?", "full_answer": "La réponse la plus proche est 1. Si nous sommes stricts, nous parlerions d'un infarctus lacunaire purement moteur (avec une légère dysarthrie). Puisque les trois autres options ne nous donnent pas un tableau clinique comme celui de l'énoncé, les deux options 2 et 3 nous donnent un tableau clinique beaucoup plus florissant et étendu. Les options 2 et 3 donnent toutes deux un tableau clinique beaucoup plus florissant et étendu.", "type": "NEUROLOGIE", "options": {"1": "Un infarctus lacunaire capsulaire gauche.", "2": "Infarctus sylvien fronto-temporal cortico-sous-cortical gauche.", "3": "Un infarctus du tronc cérébral.", "4": "Une hémorragie lenticulaire gauche.", "5": null}, "correct_option": 1, "explanations": {"1": {"exist": true, "char_ranges": [[0, 334]], "word_ranges": [[0, 54]], "text": "La réponse la plus proche est 1. Si nous sommes stricts, nous parlerions d'un infarctus lacunaire purement moteur (avec une légère dysarthrie). Puisque les trois autres options ne nous donnent pas un tableau clinique comme celui de l'énoncé, les deux options 2 et 3 nous donnent un tableau clinique beaucoup plus florissant et étendu."}, "2": {"exist": true, "char_ranges": [[335, 429]], "word_ranges": [[54, 70]], "text": "Les options 2 et 3 donnent toutes deux un tableau clinique beaucoup plus florissant et étendu."}, "3": {"exist": true, "char_ranges": [[335, 429]], "word_ranges": [[54, 70]], "text": "Les options 2 et 3 donnent toutes deux un tableau clinique beaucoup plus florissant et étendu."}, "4": {"exist": false, "char_ranges": [], "word_ranges": [], "text": ""}, "5": {"exist": false, "char_ranges": [], "word_ranges": [], "text": ""}}} +{"id": 1, "year": 2011, "question_id_specific": 35, "full_question": "Un homme de 43 ans, sans maladie particulière, s'est présenté au service des urgences pour quatre selles mélaïques au cours des 12 dernières heures. Il a déclaré avoir pris des AINS les jours précédents. L'examen physique a révélé une peau et des muqueuses pâles. La tension artérielle est de 95/65 et la fréquence cardiaque de 110 bpm. L'abdomen n'est pas douloureux et le toucher rectal révèle des selles mélaïques. Les analyses sanguines ont révélé un taux d'Hb de 8,1 g/dl. Après perfusion de sérum physiologique et début de la transfusion de concentré de globules rouges, la TA était de 120/85 et la FC de 90 bpm. L'endoscopie a révélé des caillots dans l'estomac et un ulcère de 2 cm dans l'antre gastrique avec un fond de fibrine et une petite protubérance blanc grisâtre de 3 mm au centre de l'ulcère (thrombus plaquettaire visible). Quelle est, selon vous, l'approche la plus appropriée ?", "full_answer": "Question sur l'HAD non variqueuse. Jeune patient, sans antécédents, qui après avoir pris des AINS présente une hémorragie avec une instabilité hémodynamique nécessitant une transfusion. La réponse 1 n'est pas conforme à ce qui a été dit, il s'agit d'un patient instable qui a beaucoup saigné, qui a un ulcère de Forrest IIa avec un risque élevé de re-saignement. La réponse 2 parle de la somatostatine, qui est utilisée pour les hémorragies dues à l'hypertension portale. La réponse 4 ne sera envisagée que si le saignement n'est pas contrôlé par endoscopie. Entre 3 et 5, comme il y a un risque élevé de saignement, une thérapie endoscopique est recommandée, de préférence avec 2 techniques, donc la réponse correcte serait 5.", "type": "DIGESTIF", "options": {"1": "Étant donné que le patient est jeune, qu'il n'a pas de maladies associées et que l'ulcère a cessé de saigner, il peut sortir rapidement de l'hôpital avec un traitement par IPP et l'éradication de H. Pylori.", "2": "Retirer l'endoscope, placer une sonde nasogastrique pour une aspiration continue et commencer le traitement avec un IPP et de la somatostatine.", "3": "Effectuer une biopsie pour détecter H. Pylori, retirer l'endoscope en veillant à ne pas déloger le thrombus plaquettaire et poursuivre le traitement pharmacologique avec des IPP par voie intraveineuse.", "4": "Consultation avec le service de chirurgie pour envisager un traitement chirurgical urgent.", "5": "Traitement endoscopique par injection d'adrénaline dans l'ulcère et pose de clips (agrafes métalliques) suivie d'une administration i.v. d'IPP."}, "correct_option": 5, "explanations": {"1": {"exist": true, "char_ranges": [[35, 363]], "word_ranges": [[5, 59]], "text": "Jeune patient, sans antécédents, qui après avoir pris des AINS présente une hémorragie avec une instabilité hémodynamique nécessitant une transfusion. La réponse 1 n'est pas conforme à ce qui a été dit, il s'agit d'un patient instable qui a beaucoup saigné, qui a un ulcère de Forrest IIa avec un risque élevé de re-saignement."}, "2": {"exist": true, "char_ranges": [[364, 472]], "word_ranges": [[59, 76]], "text": "La réponse 2 parle de la somatostatine, qui est utilisée pour les hémorragies dues à l'hypertension portale."}, "3": {"exist": true, "char_ranges": [[560, 728]], "word_ranges": [[91, 120]], "text": "Entre 3 et 5, comme il y a un risque élevé de saignement, une thérapie endoscopique est recommandée, de préférence avec 2 techniques, donc la réponse correcte serait 5."}, "4": {"exist": true, "char_ranges": [[473, 559]], "word_ranges": [[76, 91]], "text": "La réponse 4 ne sera envisagée que si le saignement n'est pas contrôlé par endoscopie."}, "5": {"exist": true, "char_ranges": [[560, 728]], "word_ranges": [[91, 120]], "text": "Entre 3 et 5, comme il y a un risque élevé de saignement, une thérapie endoscopique est recommandée, de préférence avec 2 techniques, donc la réponse correcte serait 5."}}} +{"id": 66, "year": 2012, "question_id_specific": 106, "full_question": "Femme âgée amenée aux urgences par ses voisins qui l'ont vue un peu groggy et avec des taches de selles sur ses vêtements. TA 100/60 mmHg, FC 100 bpm, assise 70/30, FC 105 bpm. PVY normal. Respiration de Kussmaul. Pas de focalisation neurologique. Poids 50 kg. Labo : PH 7,25, PCO2 14, Bicarbonate 5, Na 133, K 2,5, Cl 118, Creat 3,4, NUS 60, Prot 8. Laquelle des réponses suivantes est correcte ?", "full_answer": "Il s'agit d'un cas d'acidose métabolique hyperchlorémique avec un trou anionique normal. Une acidose métabolique sévère provoque une dépression contractile du myocarde, une réduction de l'efficacité des catécholamines, une augmentation de l'incidence des arythmies, une vasodilatation et une augmentation de la perméabilité capillaire, avec une réduction conséquente du flux tissulaire, cérébral (conduisant à la confusion et même au coma), rénal, hépatique... Ces patients présentent souvent une tachypnée intense (respiration de Kussmaul) dans une tentative de compenser l'ACM par une hyperventilation. Nous devons garder à l'esprit que, dans ce cas, le potassium est diminué, de sorte que la cause la plus fréquente (soutenue par les taches sur la robe) serait la diarrhée, de sorte qu'il est plus probable que leur insuffisance rénale (bien qu'ils ne nous donnent pas de données sur les ions urinaires pour le confirmer) soit de type prérénal. Le traitement consisterait en une substitution hydroélectrolytique et, en principe, à moins que l'état du patient ne s'aggrave et que le pH ne soit inférieur à 7,10, nous ne devrions pas administrer de bicarbonate, de sorte que la réponse correcte est 5.", "type": "ANESTHÉSIOLOGIE, SOINS INTENSIFS ET MÉDECINE D'URGENCE", "options": {"1": "Le trouble acido-basique qui se manifeste est l'acidose respiratoire.", "2": "La compensation pour corriger l'acidose n'est pas suffisante.", "3": "Cet examen a permis d'écarter la possibilité d'une déshydratation.", "4": "Il ne faut en aucun cas ajouter du bicarbonate.", "5": "Il souffre d'une insuffisance rénale aiguë d'origine prérénale."}, "correct_option": 5, "explanations": {"1": {"exist": false, "char_ranges": [], "word_ranges": [], "text": ""}, "2": {"exist": false, "char_ranges": [], "word_ranges": [], "text": ""}, "3": {"exist": false, "char_ranges": [], "word_ranges": [], "text": ""}, "4": {"exist": false, "char_ranges": [], "word_ranges": [], "text": ""}, "5": {"exist": true, "char_ranges": [[605, 947]], "word_ranges": [[80, 140]], "text": "Nous devons garder à l'esprit que, dans ce cas, le potassium est diminué, de sorte que la cause la plus fréquente (soutenue par les taches sur la robe) serait la diarrhée, de sorte qu'il est plus probable que leur insuffisance rénale (bien qu'ils ne nous donnent pas de données sur les ions urinaires pour le confirmer) soit de type prérénal."}}} +{"id": 521, "year": 2021, "question_id_specific": 76, "full_question": "Une fillette de 7 ans vient à la clinique parce que ses parents remarquent que son visage est rouge depuis la veille et que, depuis quelques heures, ils ont remarqué des taches rouges sur son corps. À l'examen, elle est afébrile, ses amygdales et ses oreilles sont intactes et l'auscultation cardio-pulmonaire est normale. Elle présente un exanthème maculo-papuleux confluent sur les deux joues, y compris la lèvre supérieure, ressemblant à un \"visage giflé\". Sur le thorax, il y a une éruption réticulaire, en dentelles, non prurigineuse. Quel est le diagnostic clinique le plus probable ?", "full_answer": "Bien que cette question soit à cheval entre la dermatologie et la pédiatrie, nous souhaitons apporter notre pierre à l'édifice. Il s'agit de l'érythème súdux ou cinquième maladie (également appelé mégaloérythème), causé par le parvovirus B19. Il est typique qu'après un épisode de forte fièvre, un érythème apparaisse sur les deux joues sous la forme d'une \"gifle\" qui a généralement une extension crânio-caudale dans les jours qui suivent.", "type": "DERMATOLOGIE", "options": {"1": "La varicelle.", "2": "Rubéole.", "3": "Erythème infectieux ou 5e maladie.", "4": "Exanthème soudain ou 6ème maladie.", "5": null}, "correct_option": 3, "explanations": {"1": {"exist": false, "char_ranges": [], "word_ranges": [], "text": ""}, "2": {"exist": false, "char_ranges": [], "word_ranges": [], "text": ""}, "3": {"exist": true, "char_ranges": [[128, 440]], "word_ranges": [[20, 68]], "text": "Il s'agit de l'érythème súdux ou cinquième maladie (également appelé mégaloérythème), causé par le parvovirus B19. Il est typique qu'après un épisode de forte fièvre, un érythème apparaisse sur les deux joues sous la forme d'une \"gifle\" qui a généralement une extension crânio-caudale dans les jours qui suivent."}, "4": {"exist": false, "char_ranges": [], "word_ranges": [], "text": ""}, "5": {"exist": false, "char_ranges": [], "word_ranges": [], "text": ""}}} +{"id": 20, "year": 2011, "question_id_specific": 110, "full_question": "Patient âgé de 40 ans, fumeur depuis l'âge de 18 ans et sans antécédents particuliers. Il consulte après 24 heures de frissons, de fièvre thermométrique à 39ºC et de toux avec expectoration. La radiographie du thorax a montré une densité accrue avec un motif alvéolaire dans le LSD et le patient a été diagnostiqué avec une PAC. Quel est l'agent étiologique le plus fréquent et donc celui qui doit être couvert lors du choix du traitement empirique de l'ATB ?", "full_answer": "Il s'agit d'une question d'épidémiologie assez facile : chez les adultes, en l'absence d'autres données, le pneumocoque occupe la première place.", "type": "INFECTIEUX", "options": {"1": "Mycoplasma pneumoniae.", "2": "Chlamydia pneumoniae.", "3": "Streptococcus pneumoniae.", "4": "Legionella pneumophila sérogroupe 1.", "5": "Haemophilus influenzae."}, "correct_option": 3, "explanations": {"1": {"exist": false, "char_ranges": [], "word_ranges": [], "text": ""}, "2": {"exist": false, "char_ranges": [], "word_ranges": [], "text": ""}, "3": {"exist": true, "char_ranges": [[56, 145]], "word_ranges": [[8, 21]], "text": "chez les adultes, en l'absence d'autres données, le pneumocoque occupe la première place."}, "4": {"exist": false, "char_ranges": [], "word_ranges": [], "text": ""}, "5": {"exist": false, "char_ranges": [], "word_ranges": [], "text": ""}}} +{"id": 422, "year": 2018, "question_id_specific": 88, "full_question": "Quel est le traitement de choix pour un homme de 38 ans chez qui on a diagnostiqué une maladie de Cushing en rapport avec un macroadénome hypophysaire de 22 mm de diamètre ?", "full_answer": "Bien que la plupart des adénomes produisant de l'ACTH récidivent, la chirurgie est toujours la première option thérapeutique. Les médicaments disponibles (kétoconazole, analogues de la somatostatine) visent à diminuer les effets de l'excès de cortisol ou à réduire la libération d'ACTH, mais ne sont pas curatifs.", "type": "ENDOCRINOLOGIE", "options": {"1": "Dans un premier temps, il faut essayer un traitement médical et, s'il n'est pas efficace, recourir à la chirurgie.", "2": "Surrénalectomie.", "3": "Résection chirurgicale sélective de l'adénome.", "4": "Radiothérapie pour tenter de réduire la taille de la tumeur.", "5": null}, "correct_option": 3, "explanations": {"1": {"exist": false, "char_ranges": [], "word_ranges": [], "text": ""}, "2": {"exist": false, "char_ranges": [], "word_ranges": [], "text": ""}, "3": {"exist": true, "char_ranges": [[0, 125]], "word_ranges": [[0, 18]], "text": "Bien que la plupart des adénomes produisant de l'ACTH récidivent, la chirurgie est toujours la première option thérapeutique."}, "4": {"exist": false, "char_ranges": [], "word_ranges": [], "text": ""}, "5": {"exist": false, "char_ranges": [], "word_ranges": [], "text": ""}}} +{"id": 348, "year": 2016, "question_id_specific": 160, "full_question": "Une femme de 45 ans, mère de trois enfants, se présente à une consultation de diagnostic précoce. La cytologie cervicovaginale est compatible avec une lésion intraépithéliale malpighienne de haut grade. Parmi les options suivantes, laquelle choisiriez-vous ?", "full_answer": "La colposcopie est indiquée pour toutes les patientes présentant une cytologie anormale (lésion intraépithéliale ou suspicion de cancer invasif ou d'infection à HPV), en tant que deuxième niveau de dépistage. Si des lésions sont observées lors de cet examen, une biopsie de la lésion doit être effectuée. Il est inutile de répéter la cytologie (car il s'agit d'un test de dépistage et non d'un test de diagnostic), il n'est pas non plus indiqué de pratiquer une hystérectomie pour une lésion intraépithéliale qui n'a pas été confirmée par une biopsie, et le curetage de la cavité endométriale ne fournit pas d'informations puisque la lésion provient du col de l'utérus.", "type": "GYNÉCOLOGIE ET OBSTÉTRIQUE", "options": {"1": "Répéter la cytologie dans un mois.", "2": "Colposcopie avec éventuellement une biopsie.", "3": "Hystérectomie avec salpingectomie bilatérale et conservation des ovaires.", "4": "Curetage fractionné.", "5": null}, "correct_option": 2, "explanations": {"1": {"exist": true, "char_ranges": [[305, 414]], "word_ranges": [[47, 67]], "text": "Il est inutile de répéter la cytologie (car il s'agit d'un test de dépistage et non d'un test de diagnostic),"}, "2": {"exist": true, "char_ranges": [[0, 304]], "word_ranges": [[0, 47]], "text": "La colposcopie est indiquée pour toutes les patientes présentant une cytologie anormale (lésion intraépithéliale ou suspicion de cancer invasif ou d'infection à HPV), en tant que deuxième niveau de dépistage. Si des lésions sont observées lors de cet examen, une biopsie de la lésion doit être effectuée."}, "3": {"exist": true, "char_ranges": [[415, 550]], "word_ranges": [[67, 89]], "text": "il n'est pas non plus indiqué de pratiquer une hystérectomie pour une lésion intraépithéliale qui n'a pas été confirmée par une biopsie,"}, "4": {"exist": true, "char_ranges": [[555, 669]], "word_ranges": [[90, 108]], "text": "le curetage de la cavité endométriale ne fournit pas d'informations puisque la lésion provient du col de l'utérus."}, "5": {"exist": false, "char_ranges": [], "word_ranges": [], "text": ""}}} +{"id": 411, "year": 2018, "question_id_specific": 52, "full_question": "Un couple ayant un premier enfant de sexe masculin (cas index) atteint de mucoviscidose (mucoviscidose) se présente pour un conseil génétique. Le diagnostic moléculaire indique que le cas index est homozygote pour la mutation F5O8del et que ses parents sont porteurs hétérozygotes. Le généticien clinique les informera sur le mode de transmission de la maladie. Quelle est la bonne réponse ?", "full_answer": "Les deux parents sont porteurs d'une mutation (parce qu'ils ne sont pas malades et qu'ils ont déjà eu un enfant homozygote), et la progéniture suivante a 50 % de chances de recevoir la mutation de chaque parent, de sorte que la probabilité de la recevoir des deux parents est de 50 %x50 %=25 %.", "type": "BIOSTATISTIQUE", "options": {"1": "La probabilité que la progéniture suivante soit affectée est de 25 %.", "2": "La probabilité que la progéniture suivante soit affectée est de 50 %.", "3": "Ayant eu un enfant affectueux, la progéniture suivante sera également affectueuse.", "4": "Après avoir eu une descendance homozygote F5O8del, les descendants suivants seront porteurs hétérozygotes (67%) ou homozygotes sans la mutation (33%).", "5": null}, "correct_option": 1, "explanations": {"1": {"exist": true, "char_ranges": [[0, 294]], "word_ranges": [[0, 54]], "text": "Les deux parents sont porteurs d'une mutation (parce qu'ils ne sont pas malades et qu'ils ont déjà eu un enfant homozygote), et la progéniture suivante a 50 % de chances de recevoir la mutation de chaque parent, de sorte que la probabilité de la recevoir des deux parents est de 50 %x50 %=25 %."}, "2": {"exist": false, "char_ranges": [], "word_ranges": [], "text": ""}, "3": {"exist": false, "char_ranges": [], "word_ranges": [], "text": ""}, "4": {"exist": false, "char_ranges": [], "word_ranges": [], "text": ""}, "5": {"exist": false, "char_ranges": [], "word_ranges": [], "text": ""}}} +{"id": 590, "year": 2022, "question_id_specific": 64, "full_question": "Une femme de 68 ans porte des lunettes de +4,00 dioptries pour la vision de loin dans les deux yeux. Elle consulte car elle ressent depuis quelques semaines des douleurs oculaires accompagnées de maux de tête et d'une vision floue le soir lorsqu'elle regarde la télévision dans une lumière faible. Parmi les pathologies suivantes, laquelle est la plus probable ?", "full_answer": "Nous sommes en présence d'un patient de 68 ans. À cet âge, bien que le cristallin ne présente normalement pas de cataracte à opérer, il s'est généralement épaissi. Cela signifie qu'il projette l'iris vers l'avant et réduit la profondeur de la chambre antérieure. Dans un œil dont la profondeur de la chambre antérieure est normale, cela ne pose généralement pas de problème. Mais les hypermétropes ont déjà une chambre antérieure étroite. C'est la combinaison habituelle que l'on trouve dans la fermeture angulaire primaire : l'hypermétropie et l'âge de plus de 50-60 ans. Les jeunes hypermétropes n'ont généralement pas encore ce problème. Il en va de même pour les personnes plus âgées qui sont emmétropes, myopes ou qui ont subi une opération de la cataracte. Avec cet angle étroit, c'est la mydriase qui déclenche la fermeture angulaire. En d'autres termes, au crépuscule, la pupille se dilate et l'iris s'épaissit dans sa partie périphérique, où il est déjà très proche de l'endothélium cornéen dans le cas d'une chambre étroite. La mydriase déclenche alors la fermeture angulaire, qui s'accompagne d'une douleur oculaire pouvant irradier vers l'arrière (céphalée). Enfin, l'augmentation de la pression intraoculaire entraîne un œdème cornéen provoquant une vision floue. L'option 1 est donc correcte. La migraine n'est pas nécessairement liée à la pénombre, et les douleurs oculaires et la vision floue ne sont pas particulièrement typiques. La kératite actinique est causée par l'exposition aux rayons ultraviolets (tels que le rayonnement solaire) et n'est pas liée à l'hypermétropie ou à la pénombre. En outre, les antécédents d'exposition sont généralement clairs. Enfin, la sécheresse oculaire ne produit pas non plus ces symptômes : il s'agit plutôt de brûlure, de sensation de corps étranger, de lourdeur, etc. Elle n'est pas non plus liée aux autres éléments : pénombre, hypermétropie, etc. Petit détail, j'aurais un peu mieux formulé la question. Il est entendu qu'on nous demande quelle est la pathologie la plus probable par rapport aux symptômes et à l'anamnèse. Mais telle qu'elle est formulée, on nous demande quelle est la pathologie qu'elle a le plus de chances de présenter, comme ça. En réalité, une femme de 68 ans a de fortes chances d'avoir une sécheresse oculaire, même si elle n'est pas à l'origine des symptômes qu'elle présente.", "type": "OPHTHALMOLOGIE", "options": {"1": "Glaucome angulaire primaire.", "2": "La migraine.", "3": "Kératite actinique.", "4": "Syndrome de l'œil sec.", "5": null}, "correct_option": 1, "explanations": {"1": {"exist": true, "char_ranges": [[842, 1306]], "word_ranges": [[136, 203]], "text": "En d'autres termes, au crépuscule, la pupille se dilate et l'iris s'épaissit dans sa partie périphérique, où il est déjà très proche de l'endothélium cornéen dans le cas d'une chambre étroite. La mydriase déclenche alors la fermeture angulaire, qui s'accompagne d'une douleur oculaire pouvant irradier vers l'arrière (céphalée). Enfin, l'augmentation de la pression intraoculaire entraîne un œdème cornéen provoquant une vision floue. L'option 1 est donc correcte."}, "2": {"exist": true, "char_ranges": [[1307, 1447]], "word_ranges": [[203, 225]], "text": "La migraine n'est pas nécessairement liée à la pénombre, et les douleurs oculaires et la vision floue ne sont pas particulièrement typiques."}, "3": {"exist": true, "char_ranges": [[1448, 1609]], "word_ranges": [[225, 250]], "text": "La kératite actinique est causée par l'exposition aux rayons ultraviolets (tels que le rayonnement solaire) et n'est pas liée à l'hypermétropie ou à la pénombre."}, "4": {"exist": true, "char_ranges": [[1675, 1904]], "word_ranges": [[258, 296]], "text": "Enfin, la sécheresse oculaire ne produit pas non plus ces symptômes : il s'agit plutôt de brûlure, de sensation de corps étranger, de lourdeur, etc. Elle n'est pas non plus liée aux autres éléments : pénombre, hypermétropie, etc."}, "5": {"exist": false, "char_ranges": [], "word_ranges": [], "text": ""}}} +{"id": 86, "year": 2012, "question_id_specific": 51, "full_question": "Un homme de 26 ans, asymptomatique et sans antécédents, se présente à l'examen avec un souffle mésosystolique au niveau du foyer aortique et du bord parasternal gauche, qui augmente lors de la manœuvre de Valsalva. La tension artérielle et les pouls périphériques sont normaux et l'ECG montre des signes d'hypertrophie marquée du ventricule gauche et des ondes T négatives très profondes en V3, V4, V5 et V6. Quel est le diagnostic le plus probable ?", "full_answer": "Un jeune homme présentant un souffle systolique aortique qui augmente lors de la manœuvre de Valsalva et des signes d'hypertrophie du ventricule gauche est pratiquement un \"portrait\" des résultats typiques que l'on peut trouver dans la cardiomyopathie hypertrophique. Question très facile.", "type": "CARDIOLOGIE ET CHIRURGIE VASCULAIRE", "options": {"1": "Murmure fonctionnel.", "2": "Sténose aortique.", "3": "Insuffisance aortique.", "4": "Cardiomyopathie hypertrophique.", "5": "Déficience septale ventriculaire."}, "correct_option": 4, "explanations": {"1": {"exist": false, "char_ranges": [], "word_ranges": [], "text": ""}, "2": {"exist": false, "char_ranges": [], "word_ranges": [], "text": ""}, "3": {"exist": false, "char_ranges": [], "word_ranges": [], "text": ""}, "4": {"exist": true, "char_ranges": [[0, 267]], "word_ranges": [[0, 38]], "text": "Un jeune homme présentant un souffle systolique aortique qui augmente lors de la manœuvre de Valsalva et des signes d'hypertrophie du ventricule gauche est pratiquement un \"portrait\" des résultats typiques que l'on peut trouver dans la cardiomyopathie hypertrophique."}, "5": {"exist": false, "char_ranges": [], "word_ranges": [], "text": ""}}} +{"id": 498, "year": 2020, "question_id_specific": 129, "full_question": "Un homme de 65 ans a consulté pour une dysphagie aux solides depuis deux mois. L'oesophagoscopie a révélé une tumeur à 30 cm de l'arcade dentaire, partiellement sténosante, avec des résultats pathologiques de carcinome épidermoïde. Une échographie endoscopique et un PET-CT ont été demandés, mais aucune adénopathie pathologique n'a été observée. Laquelle des options suivantes serait la plus correcte ?", "full_answer": "Chimioradiographie préopératoire dans toutes les lignes directrices.", "type": "ONCOLOGIE MÉDICALE", "options": {"1": "Oesophagectomie transhiatale.", "2": "Oesophagectomie selon la technique Ivor-Lewis.", "3": "Chimiothérapie plus radiothérapie préopératoire.", "4": "Chimiothérapie néoadjuvante.", "5": null}, "correct_option": 3, "explanations": {"1": {"exist": false, "char_ranges": [], "word_ranges": [], "text": ""}, "2": {"exist": false, "char_ranges": [], "word_ranges": [], "text": ""}, "3": {"exist": true, "char_ranges": [[0, 68]], "word_ranges": [[0, 7]], "text": "Chimioradiographie préopératoire dans toutes les lignes directrices."}, "4": {"exist": false, "char_ranges": [], "word_ranges": [], "text": ""}, "5": {"exist": false, "char_ranges": [], "word_ranges": [], "text": ""}}} +{"id": 31, "year": 2011, "question_id_specific": 233, "full_question": "Un patient souffrant d'asthme bronchique et traité par des stéroïdes inhalés à faible dose (200 microgrammes de budésonide toutes les 12 heures) et des médicaments de secours (salbutamol à la demande) se présente pour un suivi clinique régulier. Le patient indique qu'il n'a pas eu d'exacerbations au cours des trois derniers mois et qu'il utilise le salbutamol une fois par jour. Il ne souffre de dyspnée que lorsqu'il monte les escaliers et, certaines nuits, son sommeil est irrégulier à cause de la toux. Parmi les options thérapeutiques suivantes, laquelle serait considérée comme le traitement de choix ?", "full_answer": "Algorithme de traitement de l'asthme. Nous avons exclu de poursuivre le même traitement puisque le patient est symptomatique.", "type": "NEUROLOGIE ET CHIRURGIE THORACIQUE", "options": {"1": "Augmenter la dose de corticostéroïdes inhalés à 800 microgrammes/jour.", "2": "Ajouter du tiotropium.", "3": "Maintenir le même régime pharmacologique.", "4": "Passer à une thérapie combinée avec des corticostéroïdes inhalés à faible dose et des bêta2-adrénergiques à longue durée d'action.", "5": "Ajouter les leucotriènes."}, "correct_option": 4, "explanations": {"1": {"exist": false, "char_ranges": [], "word_ranges": [], "text": ""}, "2": {"exist": false, "char_ranges": [], "word_ranges": [], "text": ""}, "3": {"exist": false, "char_ranges": [], "word_ranges": [], "text": ""}, "4": {"exist": true, "char_ranges": [[0, 125]], "word_ranges": [[0, 18]], "text": "Algorithme de traitement de l'asthme. Nous avons exclu de poursuivre le même traitement puisque le patient est symptomatique."}, "5": {"exist": false, "char_ranges": [], "word_ranges": [], "text": ""}}} +{"id": 618, "year": 2022, "question_id_specific": 119, "full_question": "Femme de 13 ans, sans antécédents pertinents, avec ménarche depuis 3 mois, suivie depuis l'âge de 10 ans pour une scoliose idiopathique qui s'est aggravée. À l'examen physique, elle présentait une bosse de 7 degrés au test d'Adams et, à la scoliographie, une courbe thoraco-lombaire T4-L1 de 35 degrés de Cobb et une courbe de Risser de 0 :", "full_answer": "Le traitement avec une orthèse de Risser 0 à 2 peut être envisagé, car il y a encore de la croissance à réaliser et la correction peut avoir lieu. Son utilisation est envisagée pour des courbes comprises entre 30 et 40º.", "type": "TRAUMATOLOGIE", "options": {"1": "Recommander la natation et réexaminer la situation dans trois mois.", "2": "Prescrire une orthèse.", "3": "Consulter un physiothérapeute pour l'élasticité de la colonne vertébrale.", "4": "Révision dans 6 mois avec une nouvelle radiographie.", "5": null}, "correct_option": 2, "explanations": {"1": {"exist": false, "char_ranges": [], "word_ranges": [], "text": ""}, "2": {"exist": true, "char_ranges": [[0, 220]], "word_ranges": [[0, 41]], "text": "Le traitement avec une orthèse de Risser 0 à 2 peut être envisagé, car il y a encore de la croissance à réaliser et la correction peut avoir lieu. Son utilisation est envisagée pour des courbes comprises entre 30 et 40º."}, "3": {"exist": false, "char_ranges": [], "word_ranges": [], "text": ""}, "4": {"exist": false, "char_ranges": [], "word_ranges": [], "text": ""}, "5": {"exist": false, "char_ranges": [], "word_ranges": [], "text": ""}}}